Está en la página 1de 337

MATEMÁTICA

ALFONSO ROJAS PUÉMAPE


,
MATEMATICA
PRIMER GRADO J>E SECUNDARIA
ALFONSO ROJAS PUÉJVIAPE

Impreso en Perú Printed in Peru

© Derechos de Autor reservados


ALFONSO ROJAS PUÉMAPE

© Derechos de Edición reservados


EDITORIAL SAN MARCOS E.I.R.L.

© Derechos de Artes Gráficas reservados


ALFONSO ROJAS P. - ANÍBAL PAREDES G.

Este Libro no puede ser en parte o totalmente


reproducido; memorizado en sistemas de archivo o
transmitido en cualquier forma o medio electrónico,
mecánico, fotocopia o cualquier otro sin la previa
autorización del AUTOR Y EDITOR.

Edición revisada por:


Dr. Carlos Chávez Vega

Director del Instituto de Investigación de Ciencias


Matemáticas de la Universidad Nacional Mayor de
San Marcos.
Presidente de la Sociedad Matemática del Perú.
MATEMÁTICA
COLECCIÓN SKANNERS

PRIMER GRADO
DE
SECUNDARIA

ALFONSO ROJAS PUÉMAPE


PRESENTACIÓN

La moderna Pedagogía fomenta el diálogo, la comunicación más fluida, la discusión en grupos


y el razonamiento, {ksterrando paulatinamente la sola memorización.

La COLECCIÓN MA TEt-. lÁTICA S KANNERS, pretende servir de apoyo a todas las personas
inmersas en el quehacer educativo nacional en lo que a las ciencias matemáticas concierne, para lo cual
preSC!].tamos un diseño de texto que permitirá contar con un material que cmplc~do de acuerdo a cada
circunstancia o realidad regional nos ayudará a conseguir los lineamientos arriba mencionados.

Nuestros estudiantes pueden contar altUí con las siguientes ventajas:

* Lcng11~1jc sencillo.
* Exposición conversacional.
* Introducci6n animada en casi todos los inicios de tema.
* SKANITO, un personaje que representa a un voluntarioso alumno, siempre dispuesto a
ayudar a esclarecer puntos claves y situaciones en h<; que el alumno-lector requiere de apoyo
moral para su solución.
* Exposición teórica con ejemplos en los que además de recurrir a algunas reglas prácticas, el
estudiante es inducido a RAZONAR.
* En el desarrollo de la teoría hemos empicado una técnica usada en Programación de Com-
putadoras. recurriendo a unos cuadros a los que se accede por medio de "llamadas" en
cualquier instante de la exposición, en los que se destacan rcitcrativamcntc conceptos o
reglas prácticas más importantes, las que una vez leídas o recordadas obligan a regresar a
donde se interrumpió la lectura.
* El desarrollo de un problema resucito tiene la siguiente estructura:

E!',;UNCIADO

Soluciím: O RECUERDA QUE:

Comentario acerca de la forma


como vcmws a razonar para
resolver el problema.

• 1 Explicación de

1 cada paso que ÓO


• 1 se da.

l _ Cálculos Matemáticos
RESPUESTA.-
Interprc~1ción del resultado.
* "APRENDER-HACIENDO DE LO FÁCIL A DIFÍCIL" es la mejor fonna de lograr el
dominio del curso a fin de conseguir lo siguiente:
- Mucha habilidad para resolver situaciones de la vida diaria.
- Aptitud acad~mica para continuar estudios superiores.
- Aprohar d rnrso sin ningún conLr:.iticmpo.
"APRENDER HACIENDO DE LO FÁCIL A LO DIFÍCIL" implica un sistema de
PRÁCTICA DE MATEMÁTICA c¡ue en la COLECCIÓN SKANNERS tiene 4 secciones:
SKANIPRÁCTICA RAZONEMOS, LAS MATEMÁTICAS DE TODOS LOS DÍAS y
RAZONAMIENTO MATEMÁTICO, donde el estudiante puede introducirse con suavidad
a dificultades mayores en forma gradual.
NucsLros proh.'-.ures pued<.'n contar nm las o;igwernes vcnt.:.1Jas:
* Al inicio de cada capítulo, hemos definido O13JETIVOS y ESTRUCTURA del mismo, que
permitirán al profesor tener siempre presente la mcl:.l a lograr, así como la pcrmancnLc
organización de su trabajo (tiempo de dictado, tiempo que rest.:.1 en el período respectivo,
disLrihución de trabajos, seguimiento del sistema de evaluación y complementación, cte.)
* El desarrollo de todo el texto para primer grado de secundaria tiene como base primordial
el actual PROGRAMA OFICIAL DEL MINISTERIO DE EDUCACIÓN. Esto nos llevó a dividir
todo el texto en siete grandes capítulos en el siguiente orden: NÚMEROS NATURALES,
NÚMEROS ENTEROS, PROPIEDADES DE LOS NÚMEROS, NÚMEROS
RACIONALES, REPRESENTACIÓN DECIMAL DE LOS NÚMEROS RACIONALES,
ECUACIONES E INECUACIONES DE PRIMER GRADO Y SISTEMA INTER-
NACIONAL DE UNIDADES (SI). Cabe destacar que hemos alterado el orden tradicional
entre NÚMEROS ENTEROS Y PROPIEDADES DE LOS NÚMEROS porque hemos
introducido algunos criterios de divisibilidad que ya emplean a los números enteros. Ade-
más, a lo largo de todo el texto hemos aplicado muy sutilmente el proceso de solución de una
ECUACIÓN en forma razonada, lo cual prepara las condiciones intelectuales de nuestros
alumnos a una exposición más fundamentada sobre el tema en el texto del siguiente grado.
* La parte PRÁCTICA de c~1da capítulo la hemos dividido en 4 secciones que cubrirán los
aspectos OPERATIVO y RAZONADO en lo l¡uc a ejercicios y problemas concierne:
()) SKANIPRÁCTICA, es una sección dundcsc incidccspccialmcntccn la parteOPERA-
TIV A dd curso.
(2) RAZONEr--IOS, es una sección dividida en BLOQUES de problemas, cada uno de los
cuales cubre aspectos específicos dd capÍlulo respectivo, con grado de dificull:.ld de
menor a mayor.
(3) LAS MATEMÁTICAS DE TODOS LOS DÍAS, es una sección donde se plantean
situaciones prácticas que pueden muy bien acontecer en la vida diaria. .
(4) RAZONAr-.llENTO MATEMÁTICO; hemos agrupado alJUÍ, problemas de razona-
miento lógico, curiosidades matemáticas y problemas lJUe sólo exigen una dosis de con-
ccnLraóún para su solución.

Estimados colegas, estamos modestamente dispuestos a colaborar con ustedes en su diario y


abnegado Lrabajo; muy gustosos recibiremos sugerencias en pro de la mejora de esta obra por el bien
de nuestros alumnos.

EL AUTOR.
1 MATEMÁTICA
1

C \Pll'l I O 1
NÚMEROS NATU RALES

EL f'O'\JL !'\TO DE LOS :\Ul\lEROS NATI RALFS l IN 1 •••••••••••••••••••••••~............................. 15


1.1 Correspondencia Biunívoca .............................................................................................. 16
1.2 Conjuntos Coordmables .................................................................................................... 17
1.3 Número natural ............... .... ......... .. ............. ..................................................................... 17
1.4 Representación gráfica de IN ............................................................................................ 17
1.5 Comparación de números naturales .................................................................................. 18
1.6 Propiedades del conjunto de números naturales................................................................ 20
SKANIPRÁCTICA - 1...................................................................................................... 21

.\DICIO'\;) SI '> rR'\CCION DEN( MEROS NATURALES ................................................. 23


2. 1 Adición en IN .................................................................................................................... 23
2.2 Propiedades de la adición de números naturales ........ .. ............................................. .. 23
2.3 Técnicas operativas de la adición de números naturales ................................................... 25
2.4 Sustracción en IN .............................................................................................................. 26
2.5 Propiedades de la sustracción de números naturales......................................................... 26
2.6 Operaciones combinadas de adición y sustracción en IN ................................................. 28
2.7 Aplicaciones prácticas con la SUMA y DIFERENCIA de números naturales........ .......... 28
SKANIPRÁCTICA - 2 ...................................................................................................... 31
RAZONEMOS - 1 ............................................................................................................. 33

l\1l Ll"ll'L IC.I\CION) POTENCIACION DE NUI\IEROS NATl'RAI ES .............................. 39


3.1 Propiedades de la multiplicación de números naturales .................................................... 40
3.2 Técnicas operativas de la multiplicación en IN ................................................................ 41
3.3 Operaciones combinadas de multiplicación. adición y sustracción en IN ........................ 41
3.4 Potenciación de números naturales ................................................................................... 42
3.5 Potencias de exponente I y O ......................................................................................... 42
3.6 Potencias de hase 10 .......................................................................................................... 42
SKANIPRÁCTICA - 3...................................................................................................... 43
1
DIVISION DC Nl \ll ROS NATURAi rs ············································--·································· 45
4. 1 Di visión exacta .................................................................................................................. 46
4.2 Propiedad fundamental de la división exacta .................................................................... 46
4.3 División inexacta o Euclidiana.......................................................................................... 46
4.4 Propiedades de la división inexacta .................................................................................. 47
4.5 Operaciones combinadas de multiplicación y división ..................................................... 48
4.6 Multiplicación y división de potencias de bases iguales................................................... 48
SKANIPRÁCTICA - 4...................................................................................................... 49

RAI? Cl 1 \ORAD\) OPERACIONES COMBINI\DAS EN IN ............................................. 51


5. 1 Raí.,, cuadrada de números naturales ................................................................................. 51
5.2 Operaciones combinadas................................................................................................... 51
1 AlFO:\SO ROJAS PUEMAPE 1
5.3 Signos de colección ........................................................................................................... 52
SKANIPRÁCTICA- 5 ...................................................................................................... 53
RAZONEf\-10S - 2 ............................................................................................................. 55
LAS MATEMÁTICAS DE TODOS LOS DÍAS.. ............................................................ 64
RAZONAMIENTO MATEMÁTICO ............................................................................... 66

C \PÍTU O 2
NÚMEROS ENTEROS

EL CONJLINTO DE NÚMEROS ENTEROS ( íí: l ................................................................... 69


1.1 ¿Por qué es necesario un conjunto más amplio que IN ? ................................................. 69
1.2 Representación de íl en la recta numérica ....................................................................... 71
1.3 Comparación de números enteros ..................................................................................... 71
1.4 Números opuestos ............................................................................................................. 72
1.5 Valor absoluto de números enteros ................................................................................... 72
1.6 Relación de Igualdad de Números Enteros - Propiedades ................................................. 74
1.7 Relación de Desigualdad de Números Enteros - Propiedades ........................................... 74
SKANIPRÁCTICA - 1 .••••.••.••••.•••.••..••.•••.•••...•.•..•..•.•.•...••.••••••••.••••••••••••••••••••••••• ••••••••....•• 75

-\DICIOt\J Y SL STRACCIOI\I El\ :&......................................................................................... 77


2.1 Interpretación de la adición de números enteros ............................................................... 77
2.2 Regla de signos en la adición de números enteros ............................................................ 78
2.3 Propiedades de la adición en íl ......................................................................................... 78
2.4 Sustracción de números enteros ........................................................................................ 80
2.5 Operaciones combinadas de adición y sustracción en íl .................................................. 80
2.6 Uso de signos de colección ............................................................................................... 81
SKANIPRÁCTICA - 2 ...................................................................................................... 83

I\ILI flPLICAC IÓ\I Y POTENCIACION FN íí: ...................................................................... 87


3.1 Multiplicación de números enteros. Regla de signos ....................................................... 87
3.2 Propiedades de la multiplicación de números enteros ....................................................... 89
SKANIPRÁCTICA - 3 ...................................................................................................... 91
3.3 Potenciación de números enteros ...................................................................................... 95
3.4 Signos en la potenciación en íL....................................................................................... 95
3.5 Multiplicación de potencias de bases enteras iguales........................................................ 96
3.6 División de potencias de bases enteras iguales ................................................................. 96
3.7 Potencia de una multiplicación indicada de números enteros ........................................... 97
3.8 Potencia de una división indicada de números enteros ..................................................... 98
3. 9 Potencia de Potencia ......................................................................................................... 98
SKANIPRÁCTICA - 4...................................................................................................... 99

-1- DIVISION Y RADICACIO:'\ EN íí: .......................................................................................... . IOI


4.1 División exacta .................................................................................................................. 101
4.2 Regla de signos de la división en íl .................................................................................. 102
4.3 Propiedades de la división exacta de números enteros ...................................................... 103
4.4 Radicación en í2 ............................................................................................................... 105
4.5 Modo de hallar la raíz cuadrada de un número entero ...................................................... 106
4.6 Raíz de una multiplicación indicada de números enteros .................................................. 106
1 MATEMÁTICA

4 .7 Raíz de una di visión indicada de números enteros ....................................................... . 106


4.8 Raíz de una Potencia ................................. ..................................................................... 106
SKANIPRÁCTICA - 5 .................................................. .. ................................................ 107
PROBLEMAS RESUELTOS SOBRE LAS PRIMERAS CUATRO OPERACIONES
ELEMENTALES CON NÚMEROS ENTEROS ............................................................ 111
RAZONEMOS.. ............. ......................... .... .................. .......................... ........................ 1 17
LAS MATEMÁTICAS DE TODOS LOS DÍAS ............................................................ 125
RAZONAMIENTO MATEMÁTICO·································--···--·····------··----····--··--·--··--··· 127

PROPIEDADES DE LOS NÚMEROS

OIVI<:;IHILIDAD ----------------··--···--·--·--···--·--·········--········--·························································· 131


1.1 ¿Qué es divisibilidad? ····--················--·--··-- --·----·--···------·----·····----······················--·····--····-- 131
1.2 ¿Cuándo un número es divisible por otro?...................................................................... 131
1.3 Múltiplo de un número ·····························--··························--····------·····------·------------···---- 132
1.4 Divisor de un número --··------··----··--·--·······-- ········----·······--·--···--······················--··--· ············ 132
SKANIPRÁCTICA - 1 ·--····················--·················------·····--···--··--·--······················--········· 133

CRITERIOS DE DIVISIBILID \D ·············----····--··--································································ 135

2.1 Divisibilidad por 2" ·······························································································----····· 135


01
2.1 Divisibilidad por 5 ·--·----················································· ···········--·······--···--··················-- 136
2.3 Divisibilidad por 3 ····--··································--····--··--········----····················--···················· 136
2.4 Divisibilidad por 9 ·······················································--···--·······--····--·--···------····--··········· 137
2.5 Divisibilidad por 6 ···························----··--·--····----··--·------······--·--··--·································· 137
2.6 Divisibilidad por 7 .......................................................................................................... 138
2. 7 Divisibilidad por 11 ........................................................................................................ 138
SKANIPRÁCTICA - 2 ·························--·······--····--·----···--················································ 139
NÚI\IEROS PRIMOS Y Nl1MEROS COMPUESTOS ------··--································· ················· 143
3.1 Número Primo ··········································--··--········ ··----··················································· 143
3.2 Número Compuesto ··------······--·····--·······································------···--·--···--···--·················· 144
3.3 Números Primos Entre Sí (PESÍ) ·············--······----·------·--···------······························------···· 144
3.4 Criba de Eratóstenes ············--·····--······--··--···--······································--········--····--···--····· 144
3.5 ¿Cómo averiguar si un número dado es primo? ···························--·····--········--················ 145
3.6 Descomposición de un número en una multiplicación indicada de sus factores primos
(Teorema de Gauss) ··············--····--······----····--·································--······························· 146
3.7 Divisores de un número ·····----·--·--·----·----··--··························--···--··························--·--··--· 146
SKANIPRÁCTICA - 3 ................... ·--········--··--························································--··"· · 147
MÁ>-11\10 COMÚN DIVISOR (MCD) ······················--·······················--·····--···························· 149

4.1 Divisores comunes ········--·----· ···································------··--············································· 149


4.2 ¿Qué es el MCD? ···········--························--··--····································--··························· 149
4.3 Procedimiento para hallar el MCD ··············································----····--·--·· ····· ............... 150
Por "golpe de vista"; por descomposición en factores primos; por el método abreviado;
por el método de las divisiones sucesivas (Algoritmo de Euclides)
SKANIPRÁCTICA - 4 .................................................................................................... 153
1 ALFO:'.SO ROJ.\S PUEMAPE 1 ii:!'Mi 9Ji
i l\1Íi'\I\IO COMCN !\IÚLTIPLO (mcm l .................................................................................... . 155
5.1 Múltiplos comunes.. .......................................................................................................... 155
5.2 ¿Qué es el mcm? ............................................................................................................... 155
5.3 Procedimiento para hallar el mcm .................................................................................... 156
Por ..Golpe de vista", por descomposición en factores primos, por el método abreviado.
por aplicación de la propiedad que relaciona dos números con su MCD y su mcm.
SKANIPRÁCTICA - 5 ...................................................................................................... 159
RAZONEMOS ..................................................................... .'............................................ 161
LAS MATEMÁTICAS DE TODOS LOS DÍAS .............................................................. 169
RAZONAMIENTOMATEMÁTlCO ............................................................................... 171

CAPÍTUI O 4
NÚMEROS RACIONALES

FRACCIO:...ES ............................................................................................................................ 175


1.1 ¿_Qué es una fracción? ....................................................................................................... 176
1.2 Comparación de una fracción con una unidad .................................................................. 177
SKANIPRÁCTICA - 1 ...................................................................................................... 179
1.3 La fracción como elemento de un producto cartesiano ............................................... ..... 181
1.4 Signos en una fracción ...................................................................................................... 181
l .5 Fracciones equivalentes ..................................................................................................... 181
1.6 Simplificación de fracciones ............................................................................................. 183
SKANIPRÁCTICA - 2...................................................................................................... 185
1.7 Clase de equivalencia ........................................................................................................ 187
1.8 Número Racional ... ................................................. ..... ..... .......... .............................. ..... 187
1.9 Relación entre IN. Z y Q ............................................................................................... 188
1.10 Representación de Q en la recta numérica....................................................................... 188
1.11 Transformación de fracciones a denominador común....................................................... 189
1. 12 Comparación de números racionales ................................................................................. 192
SKANIPRÁCTICA - 3 ...................................................................................................... 193
RAZONEMOS - 1................ ........................................................................................ ... 195
LAS MATEMÁTICAS DE TODOS LOS DÍAS .............................................................. 205

\lllCION Y SUSTRACCJON EN <T'.................................................................................... 206


2. 1 Adición de fracciones de igual denominador.................................................................... 206
2.2 Adición de fracciones de diferente denominador.............................................................. 207
2.3 Propiedades de la Adición de Números Racionales .......................................................... 21 O
2.4 Sustracción de Números Racionales ........................... ,..................................................... 211
2.5 Operaciones combinadas de Adición y Sustracción en Q ................................................ 212
SKANIPRÁCTICA - 4 ...................................................................................................... 214

I\IL'I TIPLICI\CION Y POTrl'\CIACIOI\/ 11\J () ...................................................................... 218


3.1 Propiedades de la multiplicación de números racionales .................................................. 220
SKANIPRÁCTICA - 5 ...................................................................................................... 222
3.2 Potencia de una fracción ................................................................................................... 224
3.3 Signos de una potencia de base racional ........................................................................... 224
3.4 Multiplicación de potencias de bases racionales iguales................................................... 225
l\H.TEMATICA
1
3.5 Potencia de potencia de base racional ............................................................................... 226
3.6 Potencia de una multiplicación ......................................................................................... 226
SKANIPRÁCTICA - 6 ...................................................................................................... 228
-l DIVISIÓN Y RADICACIÓN EN Q .......................................................................................... 230
4.1 ¿Cómo dividir fracciones? ................................................................................................ 230
4.2 Otra forma de presentar una división de fracciones .......................................................... 231
4.3 División de potencias de igual base racional ..................................................................... 232
SKANIPRÁCTICA - 7 ...................................................................................................... 234
4.4 Radicación en Q ................................................................................................................ 236
4.5 Signos de radicación en Q ................................................................................................. 237
4.6 Exponente fraccionario ..................................................................................................... 237
4.7 Raíz de una multiplicación ................................................................................................ 237
4.8 Raíz de Raíz ...................................................................................................................... 238
4.9 Potencia de una Raíz ......................................................................................................... 238
4.1O Operaciones combinadas ................................................................................................... 238
SKANIPRÁCTICA - 8 ...................................................................................................... 240
RAZONEMOS - 2 ............................................................................................................. 242
LAS MATEMÁTICAS DE TODOS LOS DÍAS .............................................................. 253
RAZONAMIENTO MATEMÁTICO ............................................................................... 255

CAPÍTU LO .5
~---------------------------
REPRESENTACIÓN DECIMAL DE LOS NÚMEROS RACIONALES
NÚMERO DECIMAL ................................................................................................................ 259
1.1 Fracción ordinaria y fracción decimal ............................................................................... 259
1.2 Número decimal ................................................................................................................ 260
1.3 Tabla de Valor de posición de las cifras de un número decimal ....................................... 260
1.4 Propiedades de los números decimales ............................................................................. 261
1.5 Comparación de números uccimalcs ................................................................................. 261
1.6 Clasificación de números decimales ................................................................................. 262
SKANIPRÁCTICA - 1...................................................................................................... 265
FRACCION GENERATRIZ ....................................................................................................... 269
2.1 Generatriz de un número decimal exacto .......................................................................... 269
2.2 Generatriz de un número decimal periódico puro ............................................................. 270
2.3 Generatriz de un número decimal periódico mixto ........................................................... 270
SKANIPRÁCTICA- 2...................................................................................................... 271
OPERACIONES CON NÚMEROS DECif\li\LES .................................................................... 273
3.1 Allición y sustracción de números decimales .................................................................... 273
3.2 Multiplicación y potenciación de números decimales ....................................................... 275
3.3 División de números decimales ......................................................................................... 278
3.4 Radicación de números decimales ..................................................................................... 278
SKANIPRÁCTICA - 3 ...................................................................................................... 279
RAZONEMOS .................................................................................................................. 281
LAS MATEMÁTICAS DE TODOS LOS DÍAS. ............................................................. 288
RAZONAMIENTO MATEMÁTICO ............................................................................... 290
1 .\I.FO,so ROJAS Pt:DIAPE 1

e \PI n ·1 o 6 ,--------------------------.
ECUACIONES E INECUACIONES DE PRTMER GRADO EN Q

I\JTRODl!CCIÓN AL ALGEBR-\ ............................................................................................. 293


1. 1 Expresión Algebraica ........................................................................................................ 294
1.2 Término Algebraico .......................................................................................................... 294
1.3 Elementos de un Término Algebraico ............................................................................... 294
1.4 Términos Semejantes ........................................................................................................ 295
1.5 Reducción de Términos Semejantes .................................................................................. 295
1.6 Polinomios en Q ................................................................................................................ 296
SKANIPRÁCTICA - 1....•.........•...•..•..•..•..••••.••.••••.•..•.•.••..•.•.•.•..•..•.•..••••...••...•••..•...••..•..•.• 297

2 ECUACIOi\ES DE PR IMER GRADO COi\; LINA INCÓGNITA ............................................. 299


2. 1 ¡.Qué es una ecuación? ...................................................................................................... 299
2.2 Ecuaciones Equivalentes ................................................................................................... 299
2.3 Propiedades de las ecuaciones ........................................................................................... 299
2.4 Procedimiento práctico de resolución de una ecuación ..................................................... 300
SKANIPRÁCTICA - 2 ...................................................................................................... 301

SISTEMAS DE ECUACIONrs DE PR IMER GRADO ............................................................ 303


3. 1 Método de Reducción ....................................................................................................... 303
SKANIPRÁCTICA - 3 ...................................................................................................... 305

4. INECUACIONES DE PRIMER GRADO CON UNA INCÓGNITA ......................................... 307


4.1 Desigualdad ....................................................................................................................... 307
4.2 Tipos de Desigualdad ........................................................................................................ 307
4.3 Propiedades de la Desigualdad .......................................................................................... 308
4.4 Inecuaciones de Primer Grado y su Resolución ................................................................ 31 O
SKANIPRÁCTICA- 4 ...................................................................................................... 311
RAZONAMIENTOMATEMÁTICO ............................................................................... 313

C<\PÍ I"UI O 7
SISTEMA INTERNACIONAL DE UNIDADES (S.I)

1. Revisemos algo de historia.. ......................................................................................... 317


2. El Sistema Internacional (S.I). Sus unidades. ............. .................................................. 317
3. La Unidad de Longitud ................................................................................................. 318
4. La Unidad de Masa ....................................................................................................... 320
5. La Unidad de Tiempo..... .............................................................................................. 322
SKANIPRÁCTICA - 1 ................................................................................................. 323
6. La Unidad de Área ...................... ................................................................................. 325
7. La Unidad de Volumen ................................................................................................ 326
SKANIPRÁCTICA - 2 ................................................................................................. 327
RAZONAMIENTO MATEMÁTICO. ......................................................................... 329
CLAVES DE RESPUESTAS DE LAS SERIES DE
RAZONAMIENTO MATEMÁTICO .................................................. ........................ 331
1 I\IATEl\lATICA
1

CAPÍTULO 1 •

-,

r ..,

!
NUMEROS
¡

1 NATURALES 1

ll l
~
~~~~~ 1:
1

.;:;::::g;

OBJETIVOS:

• Reconocer el número natural como representante de la clase de


conjuntos coordinables entre sí.
• Reconocer las propiedades aritméticas aplicadas al conjunto de
los números naturales.
• Aplicar las propiedades y técnicas operativas de los números
naturales en la resolución de problemas.
1 AIFO~SO ROJAS PlEM\PE 1
ESTRUCTURA DEL CAPÍTULO;

l. rL CO'.\.ll'"l;TO ur I os Nl '\lE- 3.3. Üpl·raciones combinadas de l\lul-


l{OS !\: \ Tl IUI FS ( IN ). ti1>licacilÍn, AdicilÍn y Sustrac-
ciún en IN.
1.1. Correspondcncia Biunívoca.
3.4. Potcnciacic,n de números nalura-
1.2. Conjunlos Coordinables. lcs.
1.3. N11mcro Nalural. 3.5. Po1encias de exponente I y O.
1.4. Representación gráfica de IN 3.6. Pmcncias de hase 10.
1.5. Co111par:1l·ión de Números Nalurales SKANIPRÁCTICA 3
1.6. Pmpiedades del eonjunlo de los
Números Naturales. -'· UI\ ISIO'\ DE 1\1 1\IEIH)S N \Tl :-
U.\LES
SKANIPRÁCTICA 1
., -
-\l>H 10, \ SI ,rn. \( T IO:\ 1>1·
. -L 1. Divisi6n exacla.
'\l \IHHl".' \ l l l~\U "i . 4.2. Propiedad fundamcnlal ck la divi-
sión cxacla.
2.1. Achciún en IT'J
4.3. Divisi6n inexacta o Euclidiana.
2.2. Propiedades de la Adición de Nú-
meros Naturales. 4.4. Propiedades de la di\"isión inexac-
2.3. l l·u11rns o¡x:rali\"as de la Adición. ta.

2.-l. Suslracción en [N 4.5. Operaciones comhin~ulas de mul-


Liplicación y división.
2.5. Propiedacks de la Sustracción de
números nalur..1ks. 4.6. Mulliplicaci6n y División de po-
tencias de bases iguales.
2.6. Opcrac1ones combinadas de Adi-
ción y Sustracción en IN
2.7. Aplicaciones práclicas con la suma SKANIPRÁCTICA 4
y dikrl'lll"ia ck mimeros naum1lcs.
5. U \1/ n \l>I~ \I> \ \' Ol'EI~ \CIO-
SKANIPRÁCTICA 2 '\ES C0\111" \U \S._., · 1

5.1. Raíz cuadrada de números nalu-


RAZONEMOS -1 raks.
5.2. Operaciones combinadas.
_\. \ ll 1 1 1I' 1 H \ C IC ) ' \ 1•o 1 1: , _ 5.3. Signos de colcl:ción.
< 1 ,e ·u,, lll· , , ,,n~os , , n -
SKANIPRÁCTICA 5
R \I t-: S
RAZONEMOS -2
3.1. Propiedades de la l\lulliplil·acilÍn
LAS MATEMÁTICAS DE TODOS
de mímcros nalllrales.
LOS DIAS
3.2. T ~cnicasopcraL1vas dc la mullipli-
caci6n en [N . RAZONAMIENTO MATEMATICO
1 M,\TEMÁTICA
1

e
1. CONJUNTO DE WS NÚMEROS NATURALES (IN))
SKANITO
, cuanto · h
brc. tienes
en tu b1bl,o-
leca 1Ulfl Tendna que contarlos, nunca j
l 10 hice

Rl·sponda a prq.!1111tas como: ¿Cuántos ... fueron?. ¿cuán-


tos ... tienl·s'!, ¿cuántos ... l·amhiaste?, fue desde la aparinun
del i1omhre, un pruhlrma que siempre tuvo solución de la
manera m.ís natural posible; esta solución inicial es lo que
ahora conocemos con el nombre de PROCESO DE CON-
TAR.
Imaginemos un poco ... algunos miles de años atnís:
En una fría mañana de invierno, muy temprano un hombre Í O IMPORTANTE:~
sale de caza y porcada animal que mata, traza una marca en
"Unc: ...·etc:. unamarca
un tronco de árhol; ya cansado, regresa donde su familia, no
antiguo PROCESO DE
fue un día tan malo. , . ¡,C11á111ns animales cazó'!, hastaha
CONTAR
relacionar los anímate~ cazados con las marcus hechas en el
tronco: '•Cada animal, una marca" ... ¡estaba CONT AN-
DO!~ O
- ¡,Existcalguna propiedad quescaconuínal conjunto de ani-
males cazados y al conjunto de marcas en el árJ)()I? ... claro
que SÍ. ..
1 AlFO:\SO ROJAS PUÉU-\PE 1
... pero es una propiedad que no tiene forma, ni color, ni olor. .. es
una propil·dad que sólo llUeda como idea:
la IDEA DE NÚl'vlERO, que por ser um amigua y tan cspon1ánca se
le ha llamado NÚMERO NATURAL. C::: O
Entonces:

U1ili1amos los Nlll\lEROS NATURALES para CONTAR


los elementos de un conjunLO.
l:2;3;-l;5;6; ...

P11sll'riormente estos números fueron c:-.111diados y agrupa-


dos en un SISTEl\lA DE NÚMEROS NATURALES. que
para su comxto func ion.un icnlo considera al CERO como su O ; ATENCION !
primer elcmen10.
El número es una IDEA por
Rl·presemamos a los NÚMEROS NATURALES como el esodec1mosquef!sunENTE
CONJUNTO [N , que por la no1ación lk conjun1os lo ABSTRACTO al que repre-
escrih1111os así: sentamos pc,t simbo/os
llamados NUMERALES.

1 [N = {O; l;'.!;3;-l;5;6; ... j Así:


2 es el numeral que repre-
Para dar una deliniciún más formal de NÚl\lERO NATU- senta a la IDEA DE NÚME·
RAL \'l'amo:-. algunos concep1os previos: RO DOS; fue creado por los
árabes.
11 es el numeral que repre-
senta a la misma IDEA DE
José, Francisco y Esteban trabajan en un centro de cómputo, NÚMERO DOS; fue creado
donde sólo hay tres computadoras, de modo que cada uno se por los romanos.
sienta frente a la suya: tal situación puede ser representada Enla práctica a los simbo/os
del siguiente modo: o numerales les llamamos
NUMEROS por comodidad
en los usos que le demos.

A cada uno, le 1oc.1 una.


A i:ada ckmrnlO dl'I conjunto A, le CORRES PON DE 1111 clc-
me1110 del con.1111110 H y \'iceversa, es lkcir: A cada ekmcnto
del rnnjumn Ble CORRESPONDE un elemento del conjun-
to A.
1 1\-lATEMÁTICA
1
Esta cnr,r,·,pondencia recibe el nombre de CORRESPON-
DENCIA BIUNÍVOCA. ~ O

1.2 CO:\Jl"'l;TOS COORDl~ ,\BLES.-


Si entre loselementosdcdosconjuntosexistc una CORRES-
O ¡ CUIDADO!
PONDENCIA IllUNÍVOCA, tales conjuntos son COOR-
DINABLES. Correspondencia UNO A
UNO en alJlbos sentidos.
Así por ejemplo: 1CORRESPONDENCIA
BIUNÍVOCA!
A={a.r.c}
B = {Ana, Rosa. Carlos}

Podemos establecer una CORRESPONDENCIA BIUNÍ-


VOCA entre los elementos de A y de B y enL<mces afirmar
que ambos conjuntos son COORDINABLES.

Le llamamos NÚMERO NATURAL a la pro-


piedad común que existe en todos los CON-
JUNTOS COORDINABLES entre sí.

Ejemplo:
00 RECORDAR QUE:
"Representar /,\'
"Operar en /,\
"Resolvér en IN

La propiedad común en estos CONJUNTOS COORDINA-


BLES es el NÚMERO NATURAL TRES que lo repre,cnta-
___
s1gnif1ca que estamos tratando
sé'o con el CONJUNTO DE
\..NÚMEROS NATURALES

mos así:
SISTEMA ARÁRIGO: 3
SISTEMA RO~IANO: 111

Nos ayudará mucho, representar al conjunto de los números


naturales en una línea n:cla, del modo siruiente:
* Fijamos en dicha línea recw un p111110 O al que haremos
corresponder el mí mero O y hacia la derecha otro punto
A al que le hacemos corresponder el número 1.
1 -\lFff\SO ROJ-\~ PUE\l-\J'E 1

Con la medida OA hacia la derecha de I fijamos otro


punto B al que le hat-cmos corresponder el número 2, y
así sucesivamenle: C::: O
\.~
---- . _.. -•--~,.-~
" 2
B
J
C
~- "
------•>---~►~
l> 1:. ,
O IMPORTANTE
Tendr1..•mos entonces de este modo la REPRESENTACIÓN A ·:a nea recta donde
GRÁFICA DEL CONJUNTO DE LOS NÚMEROS NA- '------- e res
TURALES . PI "tC'~ C"'l 1

I '"larr: ; RECTA NU-


C::: > Algo que notamos en esta recta numérica es que: MERICA
( 1) No a Lodos lo., punLos de la recta numt'rica les '- - --~
corresponde un número nmural.
(2) La recta numérica tiene principio en el punLo corrc ...-
pomlienle al número CERO, pero a la derecha de ial
pulllo no Li1..·ne fin. porque el conju111o [}J es inliniln.
C::: 00

1.5 l'O\JP \I{ \CI<)'\ DE \l \IEROS '.'\ .\ rt R \l I.S.-


Dado... llo'- 11umcros naluralcs a y b, ésto-. pueden ,cr Í!!ualcs
o no; si no lo son, tenemos que empicar los símbolos > 6 < 00 RECORDAR QUE :

Si a es menor que h, escribimos: a<h En un CONJUNTO FINITO


podemos terminar de contar sus
Si a es mayor lJUl' h. esaihimos: a>h elenenr--

En la recta numérica: Ejemplo:


Si a< h P = (1, 2 3, 4 10 J

emonl·l.', a ,•._t.á a la izquierda de b En un CONJUNTO INFINITO es


1mpos1ble terrmnar de contar sus
o h ~lá a la derecha de a elementos
E1empi~
N = { O. 1. 2, 3. 4.
[_11..111plc ..,

Es dec ir:

En la recta numérica. los números "crecen" de i:ll¡uicnla a derecha.

Además:

si a dos números naturales les corresponde el mismo


pulllo en la recia numérica, tales números son iguales.
1 MATEMATICA
1
Si dos números naturales tienen diferente número de
cifras, será menor. al)uél que tenga la mcnorrnntidad de
cifra~.
Así:
...__,,__.
7-Ui < ...__,,__.
7748
3 cifras 4 cifro1s

Si dos números tienen la misma cantidad de cifras. se


comparan una tras otra desde los órdenes mayores hasta O ¡IMPORTANTE!
el orden de las unidades.
El número natural 73 482
Por cjl·mplo: t1enecincocifras. Cada una.
expresa las unidades de
TVi-l<io D n:wm cada orden. Así:

¿Qué símlx1lo cs~11bi111os en el recuadro?


Para conocer la n.:spucsta, comparamos la primera
cifra de la izquierda en cada número hasta dar con
una diferente. la lJUC nos füciliwrá el símholo a
7us3t~:=
LDecenas
Centenas
colocar en el recuadro. . _Unid. de Millar
Dec. de Millar
Luego: q_-(5460 ~ 7 3,249~ por ser
1n Jtt
5 mayor que 2.
Estos sím hotos> ó < tam hién los cm picamos cuando cs-
tahleccmos suhconjuntos de [N.
lüemplos:
( I J Dc·terminar 1wr extemión d siKuiente conjunto:
~
C=~ ;X< 51 Q 00
Solud,ill:
• En la recta 111wu:rica:
00 RECUERDA QUE:
xlx se lee 'x tal que xn
xlx E IN se lee· "x tal que x
pertenece al conjunto IN
• La infor111adán q1«· st' nos da en el ejemplo es qu<' C
c·s un conjunto de números naturales f.'1ENORES que
5 ( ¡NO INCLU/,\ 10S a 5! ), por lo que en ,•I punto de
la recw que representa ll 5 . dibujamos una e.tfc-rita
hueca. entmu:e.f los únii:os números naturales que
nunplen con x < 5 .wn O; I : 2: 3 ; 4.
Lue¡;o: C={0:!;2;3;4} Rpta.
•1 AlFOi\SO ROJAS P( l[\I \PE 1

Dl'taminar por e,tl'llsaón el sigu1e111l'


conjunto: O ¡ CUIDADO /
M = (x E
. -
[N /<x es par" 5 < x < 91 5 < X<9
Valores naturales de x
Su/uciú11:
comprendidos entre 5 y 9.
y
¡Sin ll]CIUtr a 5 a 9!
El símhulo I\ lo lccmn., comn '·y'". ---o---
l 11.\ <'lt·mc-1110.\ de /11 .\tm lns '!_IÍmeros ¡>ares 55x59.....
y lldl'mcís est<Ín comprc·ndidos entre· 5 _v 9.
Valores naturales de x
comprend1do&g_esáe5hasta
• Dih1t/wnos la rl'clll n1t111<;rtn1 l'n [N el9.
,Incluyendo a 5 y 9!
_..•,..__. . .___••_._____ L.-~_.- .
O 2345 6 7

8
t t
• SI' nn.\ pule m• inclwr a los c·x1rc•1tws 5
y Y de lll parte .wm1hrcada en d ~rú{ico M = { 6; 8 } ¡Sólo pares!
C::o
l'or lo tantn. el coniunto ,W determina- /
do por 1·xtl'11sián wra: ___/

1.6 PROPIEDAD ES DEL CO:\Jl ':\TO I>E LOS i\l~;\IEROS


'.',; r\ TLR ALES.

l. El conjunto de los numcros naturaks es infinito. No


existe un mímero natural que s~1 menor que CERO ni
tampoco existe un número natural l(lll' sea el mayor de
todos. porque siem¡m· habrá uno mayor que él.

19\
1\-0,ol Entonces la recta numérrca por el lado derecho 00 ¡ATENCIÓN!
•1 .• . '10 tiene cuando acabar ,Es inhnitaI
_\.. ,_/'v'.-.. El cero es una e)(cepción a
-l\,:r~
':::f
esta af1rmac,on. ya que tiene
un numero natural mayor que
,
.r.. t?I . PERO NO TIENE UN
El ron_junto de los mímcm.., naiurale, e.., un conjunto NVMERO NATURAL ME-
ordenado. NOR QUE ÉL.

Fs dec ir:
D.idos do ... números naturales difcrL'llle.., ¡... iempre!
11110 l'°' menor que el otro.

o también:
--... 1

Dado un mím~ro nawral. siempre habrá un númao natural


111c1111r qu.-: ~I y otro mayor que él. ~ 00
1 l\1\TE!\1ATICA
1

l. Dad11' hl', ,ip11,·n1,·-.conjun10-..l 1Hn pk1.1rcl c11adro,:snihi,·ndod 111ím.:-m lk ,·km,:nio-.


lk cada conj11n10 r,·,p,·ctivo:

Nº Con¡unto
EIE
~. 1
Nº Conjunto N· de
Elem ntos
- - - - .___
-
1 A = {x,x E [N" x $7)
-

6 F = {x1x E [N " 7 $ x < 17}


. -

:i /\
2
3
B=
C=
l X X'

\X le

{x X ~
J., X es
X< 99)

par •. X < 18J


11 201
7 G = \X x E

H= p xE
[N " · 2 < x :$ 25j

[)'; "· x :$ 47}


[N " X < 47)
-...- -

4 í) = ~\j ,-, e X < 1= {XX E


--~
5 E = ix x f "\J ,- 23 , X < 881 u J = lx X E [N ,-, 1 :$ X $ 19}
1

11. R,·ph·,enta ,·11 la 11·.·1..1 llllllk'ric.1l.1da 111111 lk los si~111,·111,·-. c11nj11111os:

( 1) \ 1 = i \/ \ €e J·\l ,l 2 · \ < 111} (6) ,\ = 1,/x E [N /\ 5 < X :S 15 }

L ~l Ci = 1-..1, --= -"- /\ 7 < , < 13 l 17) B = I -..,~ E [N /\ 1 < x :S 11)


'~) 11 .= 1X/ \ C ~l\i /\ 2 $ ; ,s 1 11 1,8) C = {x/x E [N /\ 1(l :S "< < -12}

(-1' T = 1x/ x , [N /\ 11 :$ X< 19} (9) D = 1x/x E [N /\ 25 < x s 261


{)) S = 1\J\ E [N/\ X< 17 1 1 IO) E = { >./x E [N /\ 1 17 < x < 118}

111. Fmpkando la rl'l:t~, n11111,;m·a ,·n IN dl"l.·1Uar l.h siguicnt,:s op,·racioncs de conjuntos:
11 l Dados: 12) Dados:

,\ = 1\/_\ E: fN /\ 2 < X < 9} _1\1= {x/xE [N/\3<x<72}

B = 1x/, E [N " 5 < x :S 1-11 S = { \/X E [N /\ 1() < X < 2-1}

llalbr: 1lallar:
.11 A ..J B el A - B aJ tl.l u S cltl.1-S
hlA n B di A .\ B h1MnS d) M~S

1V. Conll·,1~1 a b, ,1~11i,·n1,·, pr,·pm1~1,:


11 1; l'm q11,· k lla111a1111i-; RECT -\ '\:l ' \IÉRIC.Y!
12 1f ,nih,: ,·1 ma~ rn nmn,•n, natural qu,· ,"\i,ta.
131 ¡ A qu,· ,,. k llama NUMERAL?
1-l¡¡_A qu,· k lla111amos Nt°' \ll:RO ;\;ATL' RAL'! ,1
\51 A c;1da punto d.: b rcct.1 numi:n,:1 k corr,·spomk 1111 NÚtl.1ERO l'\ATURAL. ¡_Es
,·s10 corr,·c10'! ¡,Por qu~"! L
1 ALFONSO ROJAS PUÉ\1APE 1

Aquí mi s operaciones de SKANIPRÁC:TICA(j) :


1 MATEMATICA
1

2. ADICIÓN Y SUSTRACCIÓN DE NÚMEROS


NATURALES

.\l>ll"I<)'\ E'\ IN
Es una operación que hace corresponder a cada par de
números m, n E IN otro número natural llamado suma y
denotado por m + n.
/Jl'm¡,lo: ~ + 3 =5 opcra, ·1,in Adir:irín
operador +
S/l/tlllfl{/()J 2 y 3
S/llfl{I : 5
El a\'ancc lk SKANI I U en la prin1..-ra ilustración pmkmos
graficarlo en la recta num0rin1 así:

~ ..-----....,.
•O •1 •2 •J •4 •5
2 + 5
, , PROPIEl> .\l>ES l>F 1 .\ .\DICl<l'\ l>I•: '.\l :\IEROS '.\ .\ ITl{ .\LFS.
l. PIH>PIEl>A I> l> E CLA USLIRA.-
" Si sumamos dos o mús números naturaks, el resultado
tamhil'n es otro número natural".
1 ALFONSO ROJAS PUÉMAPE 1
Si a E [N y h E [N cmonccs (a+ b) E [N

Ejemplo:
Si: 7 E [N y 8 E [N entonces (7 + 8) E [N

2. PROPIEDAD CONl\ll'TATl\'A.-

"El orden de los sumandos no altera la suma".


Es decir: a+b=h+a
Ejcm1>lo: 7+5=5+7
12 = 12

l
3. PROPIEDAD ASOCIATIVA.-
"La forma como agrupamos los sumandos no altera la
suma". ( COMPLETA. .

(a+ b) + c =a+ (h + e) .en el cas,llero el numero que


falta
E,iemplo: (4+ 11)+2=4+(1I +2)
(1) ::] +2:7 + 5T2
15+2=4+ 13
(2) 7• L-, + 6 = 1 T 7• 9
17 = 17
(3) 5+3• e=.,. 7 + 3
ELEI\IENTO NEUTRO.- (4) 15+ 0 +2=7+15
"Si sumamos cualquier mímero nalural con d cero. d (5l r_J • 11 • g = 11 • 12
resultado s igue siendo el mismo mírnero natural".
a+O=a
~--~J
El cero es d elemento n('lltro de la ADICIÓN .

5. PROPIEDAD UE LA I\IONOTONÍA.-
"Si a ambos miembros lk' una igualdad k sumamos un
mismo número natural. entom:es resulla otra igualdad '".
Ejemplo: 5+8=13
5+8+7=13+7
20 = 20

6. PROPIEDAD CANCELATIVA.-
"Si en ambos miembros de una igualdad cxis;tc un
mismo sumando. podemos supnnurlo, siendo la expre-
sión obtenida otra igualdad".

Ejem1>lo: 7+/-+1=3+/+5
7+ 1 =3+5 6 8=8
1 MATEMÁTICA
1
~-J TÉC:\ICr\ S OPE l{r\TIV r\S DE Lr\ r\DICIO~.-
Veamos algunos ejemplos:
( l) E{t'ílttar: 738 + 473 + 58 + 1 357 = 2 626
/ º Forma.

Sumamos 11{)r columnas em¡>czando por /tu c{fras de las


unidade.~. 1mdicndo escrihir lns sunumdos cnfnrma verti-
rnl (uno hajn el otro_) n c•n fomw hnriwntal (uno al cosuuio
ael ntro). E., la tt;cni,:a que aprendimos desde ¡>ri11wria.
, ,,. Forma.

Por partes; cm¡>l<'ando la PROPIEDAD ASOC/ATl\'A:

738+473+58+1 357
1 211 1 41 S = 2 626

.. p,,cf,·mos l'tl-'ll}<.lr ,·uma.,· de 11úm,·ros 110 muy grande., nnpl,·,uulo


l,1 r•r11pil'llacl asm·ia1iv,1 con el .J111 d,· ro11.~,.·~uir hahiliclad ,·11 d
cal.-ulo mc,u,il.

(:!) E/i.•t:ltwr: 73 + 37
\ ·oy " ir cks1:rihi<'fulo ttn pron·so scncilln que siilo elche
¡>rm:tit:11rsc en la mcnte:

¡
Dcsco111ponc111os en decenas y unidades:
7 decenas y 3 unidw..'!!s; 3 decenas y 7 unidades
S11mt1111os Las decenas: 7 + 3 = JO decenas
<Í /00 unidades 1Suma Total'
S1111w111os las unidades: 3 + 7 = JO unidades
El resultado final será: I JO unidades
CÁLCULO MENTAL
(3 J Efel'luar a simple vista : 52 + 89
/ \ /\ Efectuar-
so2 80 9 (1)85 + 17
~ (2) 13+ 92
(3) 72+ 89
Suma de decenas : 13 rí 130 unicliule., (4)63+38
S11111a d<' unidad<'.\.' 2 + 9 = / I (5)61 +57
Suma 10111/: 130 + 11 = /-11 (6)39+ 87
(7)37+52+ 17
t-1) E{l't'ltlllr a simplr vista: (8) 13+28+ 11 + 15

77 + 58 + 13
❖ 1 ALFO~SO ROJAS PUEM.\PE 1
- decenas : 7 + 5 + l = 13 á 130 unidades }
- unidades :7 + 8 + 3 = 18 unidades _ __,
- resultado : 13() + 18 = I.J8

:!A Sl STR ,\l:CIO'\ E'\ lN .


Es una operación en la que dados dos números m, n E IN
llamados MINUENDO (M) y SUSTRAENDO (S)
respectivamente, con m ~ n,se obtiene un número DE IN tal
que n +D=m.

Esta diferencia se presenta así:


O IMPORTANTE
La SUSTRACCION NO
SIEMPRE ES POSIBLE EN 1

l::jcm¡,los: IN, por esta razón aparece


un nuevo con¡unto de
f I J Efe elitar: 15-7=8 numeras llamado CON-

t 1~ Diferencia
Sustraendo
Minuendo
JUNTO DE NÚMEROS
ENTEROS que lo estu-
.1 d1aremos en el s1gu1ente 1

Aqui comprol>amos fúcilmente que:


\..capitulo. J
7 + 8 = 15
Es decir: SUSTRAENDO + DIFERENCIA = MINUENDO
(2) Efectuar: 6-9=;,'!
1Cu1dadol Esta SUSTRACCIÓN es imposible de efectuar porque
NO EXISTE un número natural que sumado con el
SUSTRAENDO 9 dé como resultado r,_
Por esta razán afirmamos que la SUSTRACCIÓN EN [N
NO SIEMPRE ES POSIBLE. C:' O

2.5 Pl{OPIED.\DES DEL\ Sl'STK\CCI<>:X l>E I\TI\IEROS


"\ ,\TLl{ .\I ES.-

1. Si sumamos o rcsLamos un mismo número natural al


MINUENDO y al SUSTRAENDO, la diferencia NO
SE ALTERA.
Ejc•m¡,lo:
15-8=7
1 l\lATEMATICA
1

1
Sumemos 3 a cada l~rmino de la sustracci<Ín: ~

í
O
C15 + 3) - C8 + 3) O ¡ ATENCION !
'----r---'
1 La Misma
18 11 = 7 01ferenc1a 1 Términos de la sustracc,on
MINUENDO
"' S1 sumamos o rcs1amos un mismo número natural y
SC>LO AL !\IINlJE;'iDO, la DIFERENCIA queda SUSTRAENDO
aumentada o disminuida en esa misma cantidad.
1~1c•mplo:
15 8 = 7
t 1La diferencia quedó
15 + 3
r10 aumentada en 3!
18 8 =
3. Si sumamos o r~·stamos un mismo mímero nalllral
SÓLO AL SUSTRAENDO, la DIFERENCIA queda
disminuida o aumentada en esa misma cantidad.
/~j('lll¡J/o:
15 8 = 7
l
15
(8+3)

11 =
i
4
¡La diferencia
quedo disminuida
en 31

-l. Si sumamos el SUSTRAENDO con la DIFERENCIA 00 ¡ CUIDADO !


ohtcncmns como rc,ullado el minuendo.
Despues de la palabra DE o
1~1c•mp/os: DEL se escribe el minuen-
( I J Si: 15 - 8 = 7 do.
Despues de la palabra
Entonres: 8 + 7 = 15
RESTAR se escribe el
(2 j La diferencia de las edades de Afarilza y sustraendo.
Carmen es 5 años. Si la mcnnr de ellas ti<·nc Así:
11 mios, ,;cuál es la edad de la mayor? DEaRESTAR b
So/uciú11: Esto es
a-b
Veamos los dalos: DIFERENCIA -- ➔ 5
LA MENOR -- ➔ 11
Como: La f\la} or - La f\lcnor = Diferencia
Entonces: La Mayor = Diferencia + La \knor
La f\layor = 5 + 11 = 16 ai'los
5. Si del tvllNUENDO restamos la DIFERENCIA, ohtc-
ncmos el sustraendo. ~ 00 ¡ATENCIONI
(iem¡,/os: ,Por que la sustracc1on no
( I J Si 15 - 8 = 7 siempre es posible en [)\J ?
Enwru:es: 15 - 7 = 8
+ 1 ALFONSO ROJAS PUEMAPE 1
(2) El mayor ele dos hermanns tiene 17 años de•
edad. Si el mayor l'Xcede al menor c·n 5 añns:
¿Qué edad tiene el menor?
Soluciá11:
• Examinemos los datos: El MAYOR ---) /7 años
La DIFERENCIA ---) 5 años C:::.> O

• Corno La MAYOR - La MENOR = DIFERENCIA


pnr propiedad: O ¡CUIDADO'

La A1AYOR - DIFERENCIA = La MENOU ·mexcedea t en d"s1g-


n1f1ca que de m a t hay
17 5 = 12 años un EXCESO o D~E-
RENCIA
.:!.6 OPERACIO'.'\ES CO\IBl:'\ADAS DE AIHCION Luego:
\ Sl"STRA( '( 'IÚ~ EN IN m-t• d

* Si hay operaciones enLre parénLesis, operamos primero


<.~sLas, para suprimir dicho paréntesis.
Ejc•mp/o: 7 + 2 + (5 - 1)

• f'riml'm el interior del pan1ntcsis: 7+2+4


• Lue~o sumamos lns tres números: 7 + 2 + -l = 13
* Si no hay parénLesis procedl'mos a operar de izquierda
a derecha:
Ejc•mp/os:

( l ). 28 - 12 - 5 - 2 = 9
L ~:r TT
L__J
9

19
.:!.7 \PLICACIO, ES PRACTICAS COi\ L\ SL\I.\
Y IHFER EI\CIA DE l\Úl\lEROS ~ r\'ITRALES.
Dados la SUl\lA (S) y la DIFERENCIA (D) de dos números
naLuralcs a y b donde a> h, el número mayor a se calcula
00 ¡ ATENCION !
como la semisuma de S y D C::::' 00
Semisuma es la MITAD DE
Es decir: LA SUMA o la suma dividida
entre2
1 MATEMATICA

Ackmás:
El número menor b se calcula como la scmidiferl·ncia de S
y D.

Es decir:

¿Me ayudas a resolver a•gunos e¡emplos?

Veamos:

(1) Las edades de Sergio y su p.1pá suman


54 años. Si la edad del papá excede en
30 años a la del hijo: ¿C'uüntos m1os
tendra S..-rgio tknLio dl~ 5 ar1os?

a)12 b)l4 c)17 d)l6 e)l8


Su/udó11:

)a ,e n11, tl,11111<1 .\l ' \I,\ tic t/11, r,m1ul,11h·., . ¡ IMPORTANTE!


I'ªº fu /J/F(N(\C/ \ ,e"º·' tia u1 la /rt1,l' .
cxn tic l'fl ,Es cierto que
R, r·ul·rcla qu, S1 a ex, ,·ele ('11 e mi11., a h. 15excedea 10en5?
ou11ru ,,_, a - b = e· Sí. ya que
15 - 10 = 5
Se nos pide la edad del menor, para lo cual
necesitamos saber la SUMA (S) y la DIFE- , Es cierto que:
15 excede en 5 a 10?
RENCIA (D) de ambas edades; si Icemos el
problema encontramos que: s,. ya que
15- 10 = 5
* SUMA= S = 54
* ... "laedaddd papáexcedl·en JO años
a la edad del hijo'· ...• es decir:
DIFERENCIA= D = 30
S-D
Con la SUMA (S) y DIFERENCIA (D) ya Menor Edad = - - -
2
podemos hallar la menor edad empicando la " 54-30
SEMIDIFERENCIA de S y D. es decir: _ J ' Menor Edad =
2
'l4
- Operando: Menor Edad = -=-- = 12 1Edad actual
2 de Serg,01
Nespuesta:
Dentro de 5 años, Sergio tendrá ( 12 + 5), es decir 17 años.
(2) Calcular a + h + e + d - e en la siguiente operación:
♦ 1 ALFONSO ROJAS PUEMAPE 1
Solución:
,·, ,,,,,,1. dt. · ""'' '""''' d1. · ~ IIIUILl 'TtJ.\ d, f ,,,, IJ f O RECUERDA
, i/r,n , ,1.la """· a /,1 </lit' lun " ,¡11,· " '1:UIY, I
r ,l\trt /' ¡,ard el, l c r11u11..tJT lo."í ,uurtt.·ro , t./llL'
QUE:
/c1/t,111 ,.,, /,,, cuc1dn111, Al efectuar opera-
c,ones combinadas
Al sumar c•mpezamos por las deADICIÓNySUS-
unidades¿ verdad?, enlnnr:es,¡," qué TRACCIÓN sm pa-
número esr:rihimos en lugar de b rentes1s • se opera
para que al .rnmarsc• con 8 nos dé :1' conforme al orden
otro mímero que termine' en .1? _/ □ 9 O O 3 indicado de 1zqu,er-
da a derecha
tal mímero es 5. que al
sumar con 8 nos da 13 del
que ·· e.,,:rihi111os 3 y llt•va-
mos /" a la .úguil'fll<' r:o - :1'
lumna : __/ □ 9 O O 3

Enformasimilar r11zonmnos para 1 1 1


<·011seguir los mímeros que escri- 35GJ_1_w+
hiremos <'11 los demás cu11dritos -l [}J 2 [Ij 8
ohtl'niendo fi1111lmt'nte :
ITJ9 O O 3
Nes¡,un·ta:
La c·x¡1resirí11 flt'did11 será:
a+b+c+d-e=7+5+_"~+7-7=15 ~ O 00 RECUERDA QUE:
Si al MINUENDO le qu,-
(3) La diferencia de dos números es 465. Si al
tamos cierta cantidad.
tnl'nor k aumentamos 36 y al mayor le quita-
entonces la DIFEREN-
mos 42,¡,rnál es la nueva diferencia?
CIA queda DISMINUIDA
en esa misma cantidad.
Soluciú11: Si al SUSTRAENDO le
1
aumentamos una cierta
,,;7
/'c1~a, ,1/, ul,ir la 11141''<1 t/1/,·r, ·11< 1<1 h ,I\ </11< ' r,•,
ca11t1dad, la DIFEREN-
tlar la, pru¡•wd,u/, ·, ,/,• la ,u,1ra,, wn quL' /u -¿ra
h,111,1/t,·r,1r/a/J/Fl:Nl:VC/,\. C: 00 CIA queda DISMINUIDA
__j en esa misma cantidad.
Representemos la citmla diferencia: a-b = 465
Si al menor le aumentamos 36, enLon-
ces 465 disminuye en 36 : - - ► a - (b + 36) = 465 - 36
Si al mayor le quitamos 42, a la dife -
rencia también hayquequitarle42,es
decir: (a - 42) - (b + 36) = 465 - 36 -t 42
La nueva diferencia de los nuevos 465 - ]6 42
números es:
YJ
L ~ Respuesta
I
" 1 J\IATE!\lr\TICA
1
SK \ '.\ 1PR \CTIC.\ 'l
_::_,/

l. Ekctuar a "-;11nplc vista.. las siguientes opcrac1ones:

(1) 13 + 17 (6) 67 + 91 (11) 87 + 63 (16) 70-10 (21) 65-17


(2) 15 + 85 (7) 85 + 17 (12) 162 + 35 (17) 40-20 (22) 85-29
(3) 23 + 9 (8) 93 + 81 (13) 115 + 73 (18) 72- 10 (23) 76-45
(4) 27 + 38 (9) 66 + 28 (14) 79 + 65 (19) 72 - 14 (24) 99- 52
(5) 56 + 61 (10) 71 + 68 (15) 68 + 112 (20) 78-27 (25) 87-46

11. Completar d siguiente cuadro escribiendo la propiedad rcspcctiva:

PROPIEDAD PROPIEDAD
EXPRESIÓN EN IN APLICADA EXPRESIÓN EN IN APLICADA

3+0=3 Elemento Neutro 7=4+3 ➔ 7+5=4+3+5

3+5=5+3 6+0=6

7+(2+5)=(7 +2)+5 10+8=6+4+8 ➔ 10=6+4

3+5=2+1+5 -+ 3=2+1 32+ 17=(30+ 10)+(2+7)

7+5=12 25+ 19=(20+ 10)+(5+9)

111. Electuar las siguientcs operaciones:

(1) 672+115 (11) 73-18 (21) 26-8-(4+2)


(2) 3173+5 317 (12) 68-29 (22) 518+6-58-(12+ 7)
(3) 5 412+6 311 (13) 106-37 (23) 619+(37-17)+ 1-(16+5)
(4) 7 813+2 515 (14) 716-613 (2~) 712+614-311-17+2
(5) 8180+4 239 (15) 819-507 (25) 5 134+2 817-1 832-601-( 107+18)
(6) 5 317+3111 (16) 7 517-2 314 (26) 7 416+7 818-2 31 4+707+805-16
(7) 62 013+37 005 (17) 3108-1515 ·(27) 8 515+6 313-(2 016+918)-(36+24)
(8) 51 617+68 316 (18) 2 512-1 052 (28) 25 317+ 13 619-5 318-3 211-612
(9) 316 509+97 515 (19) 73 015-12111 (29) 725 619-301515+27613-3 405+ 13
(10) 618 516+317 601 (20) 578 612-327 619 (30) 5 212 316+2 303 601-566 316-8 105

IV. Contest.a las siguientes pregu111.:.1s:


( 1) ¿Por qu.: no es posible una sustracción donde el SUSTRAENDO sea m,1yor que el
minul'llllo'! ,
(2) ¿Por qué no pode1J1os aplicar la propiedad conmutativa a la SUSTRACCION'!
(3) Si en urKt ADICION de Lres números. triplicamos éstos,j,qué ocurre con la suma?
(-l) Al sumar l\llNUENDO, SUSTRAENDO y DIFERENCIA,¡,qué se puede afirmar
acerca dd resultado obtenido'!
(5) Si se nos dacomlHlatos la SUl\tA y 1.1 DIFERENCIA dedos números naturales. ¿es
posible hallar uno de ellos'!, ¿cu:.íl es el procedimiento a seguir'!
1 ALFONSO ROJAS PUÉMAPE 1 ifaJ@41

A411í mis operaciones de SKANIPRÁCTICA @

1J:.-··q.
~

J'J.
1 MATEMÁTICA
1
, .RAZONEMOS - 1 1
Instrucciones:
A conti 1111ació11 le presento bloques de ejercicios para afianzar lo
leido hasta aquí. Marca la respuesta correcta y compáraln con
la CLA VE DE RESPUESTAS que aparece al fi,nal; sin embargo,
te recomiendo que expliques en tu cuaderno las RAZONES de
tu resultado. No tomes este trabajo como una pesada carga .<:ino
como un DES AFÍO a tu intehgencia


BLOQlFI (8) / En un juego, un apostador gana SI. 26, luego
♦ pierde SI. 15, después gana SI. 8 y por último
(1) 1 JuangaslóSl.12encomprarunlibro,S/.6más ♦ vuelve a ganar SI. 1O. ¿Cuánto g11ño o perdió?
en comprar unos lentes y SI. 46 en una camisa:
Luisa gastó SI. 32enuna blusa y SI. 8 menos en ♦ (9) En una batalla, un ejército tenía 2 342 soldados
una falda . ¡,Quién gastó más'! y otro 1 936 soldados. Después de la batalla el

primer ejército quedó con 835 soldados y el
(2) / Luis comienza a hacer una zanja y después de ♦ segundo con 91 Osoldados.¿ Cuántos murieron
6 horas se retira; Carlos sigue la obra por 8 en la batalla?
horas más y se retira: por último Javier trabaja ♦
tantas horas como Luis-y Carlos. ¿Qué tiempo ♦ (IO) Si Julián nacióelañodelsesquicentenariode la
demoró hacer la zanja'! independencia del Perú. ¿en qué año cumplió

18 años de edad?
(3) Un niño ve tres programas de TV seguidos. El

primero duró 25 minutos.el segundo90minutos (11 ) / Una persona tiene SI. 250enel Banco;deposita
yel tercero tanto como el primero.¿Qué tiempo ♦ SI. 80; luego deposita nuevamente SI. 60; poste-
estuvo viendo TV el niño?
♦ riormente retira SI. 150. ¿Cuánto le queda en el
(4)t Rosa gastó SI. 30 en una blusa y SI. 5 más en
Banco'!

una falda. Entonces, en I blusa y 2 faldas gastó:
♦ (12/ Unalumnodebesumarcomomínimo42 puntos
en sus notas de los cuatro bimestres. Si el
(5) Una persona recorrió cierto día 12 km; al día
♦ primer bimestre sacó 15, el segundo 10 y el
siguiente 8 km más que el día anterior y al tercer
día recorrió 4 km. menos que el primer día. tercero 13; ¿cuál será su nota el cuarto bimestre?

¿Qué distancia total recorrió'!
♦ (13) 1 Un profesor reparte caramelos entre sus
(6) Liliana se pone a dieta. El primer mes bajó ♦
alumnos. Si cada uno recibe I caramelo más
· 80..Q_gr·; el segundo mes bajó 100 gr má<; que el que el anterior y hay 12 alumnos: ¡,cuántos
mes anterior; el tercer mes subió 500 gr. ♦ caramelos recibirá el último si el primero recibió
¿Cu.íntos gramos bajó Liliana al tercer mes? 5 caramelos'!

(7) Un Sr. entra a un restaurante y consume un ♦ (14) Enunacarreradepostasde4x400m,¿cuántos


ceviche que le cuesta SI. 7: luego un arroz con metros en total recorrieron entre los 4
pollo que le cuesta SI. 9 ydepostreunagelatina ♦ participantes'!
de SI. 2. ¿Cuánto recibió de vuelto, si pagó con
un billete de SI. 50 y dejó SI. 1 de propina al ♦ (15) Un depósito con agua tiene un agujero por el
mozo'! ♦ cual se va saliendo el agua. La primera hora
♦ 1 ALFONSO ROJAS PUÉMAPE
1 f,4.,@§¡fj
salió 48 litros, la segunda hora 6 litros menos ♦ (25) ¿Cuántas cifras se emplean en la numeración
que la hora anterior y la tercera hora 2 litros de un libro, desde la página 996 hasta la página
menos que la hora anterior. Si aún quedan 230 ♦ 1 004, inclusive?
litros, ¿cuántos litros habían inicialmente en el ♦
depósito? (26) Las notas de Gustavo, Silvia, Jorge y El isa son:
♦ 16, 18, 14 y 15respectivamente. Sielprofesor
(16) Un chocolate cuesta SI. 4, una bolsa de ♦ lesaumentó2puntosacadauno,¿cuálesahora
caramelos SI. 2 y un paquete de galletas cuesta la suma de sus notas?
SI. 3. Si Esteban tiene SI. 5, ¿qué dulces podrá ♦
comprar? ♦ (27) Susana y PilarvandecornprasconSI. l 30cada
una. Susana gastó SI. 60 y Pilar SI. 20 menos
(17) Lourdes vendió un televisor en$ 430 perdiendo ♦ que Susana.¿ Cuánto dinero les queda a las dos
$ 80. ¿Cuánto le costó a ella el televisor? ♦ juntas?

(18) Un negociante compra un equipo de sonido en ♦ (28) ¿Cuánto su man todos los puntos de las caras de
Sl.430yloquierevenderganandoSl. l20.¿En ♦ un dado?
cuánto debe vender el equipo de sonido?
♦ (29) Cuatro amigos van de compras pensando en
( 19) f Osear compra un horno en$ 330 y lo vende en ♦ gastar SI. 80 cada uno. Si los 2 primeros gastaron
$ 250. ¿Cuánto perdió en este negocio? SI. 20 más, y los 2 últimos gastaron SI. 10
♦ menos. ¿cuánto gastaron en total los 4 juntos?
(20) Un artículo cuesta SI. 235 en mayo y cada mes ♦
aumenta SI. 2 su valor. ¿Cuánto costará en (30) Un ascensor funciona corno máximo c on
octubre? ♦
800 kg. Si suben 5 personas de 65 kg. 70 kg.
♦ 58 kg. 50 kg y 72 kg. respectivamente, ¿fun-
(21) En una balanza de platillos, se colocan en un cionará el ascensor'!
platillo pesas de l kg. 5 kg y lO kg. ¿Cuántos ♦
kilos debe pesar la pesa que se debe colocar en ♦ (31) Una piscina tiene 5 tuberías; la primera vierte
el otro platillo para que esté equilibrada la 18 litros por minuto; la segunda vierte 24 litros

balanza? por minuto; ta tercera viene30 litros por minuto;
♦ la cuarta vierte 15 litros por minuto y la quinta
(22) En una carrera, el que llegó en primer lugar viene 18 litros por minuto. ¿Cuántos litros
demoró 242 minutos; el segundolugar3 minutos ♦ habrán en la piscina en un minuto, si se abren
después del primero; el tercero 2 minutos ♦ las 5 tuberías, estando vacía la piscina?
después del segundo yel cuano llegó 6 minutos
después del tercero. ¿Qué tiempo demoró el ♦ (32) De lunes a viernes se registró la temperatura en
cuarto? ♦ cierto lugar; las medidas fueron 18º. 20º, 16°,
♦ 21 º, l 8°. ¿Entre qué días se registró la mayor
(23) Un depósito tiene 180 litros de agua; se sacan variación de temperatura?
35 litros; luego se agregan 42 litros; después se ♦
vuelve aagregar50 litros; por último se extraen (33) Rodolfo tiene 18 años y María nació en l 970;

25 litros. ¿Cuántos litros hay ahora en el si María es 4 años menor que Rodolfo, ¿en qué
depósito? ♦ año nació Rodolfo?

(24) Un gasfitero cobró SI. 25 por hacer una
reparación en una casa; además cobró SI. 36 ♦
por hacer reparaciones en otra casa. Si gastó

SI. 12 en materiales en la primera casa y otro
tanto en la otra casa; ¿cuál fue su ganancia? ♦
1 MATEMÁTICA
1
HLOQUEII ♦ (8) Dos depósitos juntos tienen 86 litros de agua. Si
uno de ellos tiene 14 litros más que el otro.
(1) .,.. La suma de dos números es 24 y su diferencia ♦ ¿cuántos litros tendría el que contiene menos
es 8. ¿Cuál es el menor de dichos números? ♦ agua si le agrego dos litros más?

a) 6 b) 16 c) 8 ♦ a) 36/ b) 35 i c) 37 /
d) 22 e) 4 <l) 38 / e) 39/

(2) Al sumar dos números se obtiene 40. Si el ♦ (9) Se reparteunaherenciade S/. 300000entre dos
mayor excede al menor en 12, ¿cuál es el personas. ¿Cuánto recibe la más afortunada, si

número mayor? se sabe que tendría S/. 48 000 más que la otra?

a) 24 b) 26 c) 28 a) S/. 170 000 b)S/. 182000
d) 27 e) 25 ♦
c) S/. 174 000 d) S/. 186 000
♦ e) SI. 172 000
(3) Manuel y César tienen juntos SI. 300. ¿Cuánto
dinero tiene César si se sabe que tiene SI. 40 ♦
(10) Al dividir una regla de 80 cm en dos pedazos.
menos que Manuel? "l ♦ resulta uno 12 cm más grande que el otro.
¿Cuánto mide el pedazo más pequeño?
a) S/. 130 b) S/. 100 c) S/. 170 ♦
d) S/. 160 e) SI. 180 a) 30 cm b) 28 cm c) 32 cm

d) 34 cm e) 31 cm
(4) La suma de las edades de Víctor y Elizabeth es ♦
66. ¿Qué edad tiene Víctor si dice ser 18 años ♦ (11 ( Hallar las cifras que debemos escribir en los
mayor que Elizabeth? casilleros para que la operación sea correcta.
♦ Dar la suma de las cifras halladas.
a) 36 b)26 C) 52
d)42 e)44 ♦
7 2 60 +

(S) / Si sumarnos las edades de Rocío y Walter
obtenernos 78 años. Si hace JO años la diferen- ♦
7
□ 6

cia de sus edades era 2 años, ¿qué edad tiene ♦


□ o s
Rocío? a) 27 b) 26 c) 21
♦ d) 18 e) 19
a) 36 b)40 c) 28
d) 34 e) 30 ♦
(12) ¿Qué ocurre con la suma de las edades de tres
♦ hermanos si triplican dichas edades?
(6) Dentro de 7 años mi edad será 8 años más que
♦ a) La suma se triplica
la de Ricardo. Si actualmente nuestras edades
suman 56 años, ¿cuál es la edad de Ricardo? ♦ b) La suma queda aumentada en 3
c) La suma queda disminuida en 3
a) 22 b) 20 c) 21 ♦ d) La suma queda multiplicada por 9
d)23 e) 24 ♦ e)N .A.

(7) En el año 2 000. la edad del señor Femández ♦ (13) ¿Qué ocurre con la diferencia de dos números
excederá en 7 años a la edad de su esposa.¿ Cuál ♦ si al mayor de ellos le aumentamos 125 y al
es la edad del señor Fernández, si en la actualidad menor lo disminuimos en 125?
su edad sumada con la de su esposa da 7 S años? ♦
a) La diferencia no se altera

a)4I b) 38 c)42 b) La diferencia queda aumentada en 125
d)4S e) SI ♦ c) La diferencia queda aumentada en 250
1 ALFO~SO ROJAS PUÉMAPE 1
d) La diferencia queda disminuida en 125 ♦ 9 6 [Jo 3 -
e) La diferencia queda disminuida en 250

□6 2 o o
4 0]
5
(14) Dar la cifra más grande que se obtiene al ♦ 4 8
completar los siguientes casilleros para que la ♦
a) 13 b) 15 c) 19
suma sea correcta:
d) 17 e) 21

♦ (20) En una operación de sustracción. la suma del
minuendo con el sustraendo y la diferencia es
♦ igual a 8 668 . Calcular el minuendo.

a) 4 b) 6 c) 5 a) 4 352 b)4 334 c) 2 157
d)7 e) 9 ♦ d) 4 278 e) 4 338

1(15) La diferencia de <los número~ es 24. Si al BLOQUE 111
minuendo y al sus1raendo le aumentamos 5, ♦
¿cuál es 1~ nuern diferencia? ♦ (1) ¿Cuanto le co~IÓ a Susana lo que al vender en
SI. 23 762 le deja una pérdida de SI. 1 603?
a) 29 b) 19 c) 24 ♦
d) 14 e) 34 a) SI. 25 366 b) SI. 24 365 c) SI. 26 535

dl SI. 25 365 el SI. 23 465
1(16) Juanes mayor que Jorge por6años. Si a ambas ♦
edades le aumentamos 6 años, ¿cuál es la nueva (2) La suma de las edades de Tom y Jerry es 84

diferencia de sus edades? años; si .lerry es menor que Tom por 18 años.
♦ ¿cuál es la edad de éste último?
a) 3 años b) 9 años c) 6 años
d) 18 años e) 12 años ♦ a)49 b) 50 c) 51

d) 52 e) 53
(17) En la siguiente operación, ¿cuál es la menor de
las ci íras a colocarse en los casi llcros para que ♦ (3) ¿,A cómo debemos vender lo que costó
la diferencia sea correcta? ♦ SI. 13 615 si deseamos obtener una ganancia de
SI. 6019?
7 [B 3 5J 5 ♦
,1)Sl.18314 b)S/. 19534 c)Sl. 19634
3 5 4
□2 ♦ <ll S/. 19 034 e) 19 114

□ o 7 8 ♦
♦ (4) Necesitamos saber el peso total de 5 cajones.
a) 5 b) 3 c) 1 sabiendo que el primero pesa 713 kilos; el
d) 2 e) 4
♦ segundo pesa 17 kilos menos que el primero; el
tercero pesa 18 kilos más que el primero; el
(18) La diferencia de dos números es 860. Si al ♦ cuarto pesa tanto como el primero y el segundo
menor le restamos 22 y al mayor le aumentamos ♦ .1untos,y el quinto pesa 2 kg menos que el
35. ¿cuál será la nueva diferencia? cuarto.

a)803 b¡847 c)873 a) 4 9561..g b) 4 7561..g c) 4 598 1-..g

d)9l7 e)890 d) 4 656 kg e) 4 296 kg

(19) Dar la suma de las cifras que debemos escribir ♦ (5) Si mis ingresos al mes fueran de S/. 255 más,
en los casilleros en blanco para que l.1operación podría gastar SI. 300 en alimentos, SI. 350 en
sea correcta: ♦ alquiler. SI. 420 en ropa y SI. 200 en otros
1 MATE!\-L-\.TICA
1
gastos, qued.índome de ahorro SI. 385. ¿Cuál ♦ SI. 76 867, un segundo pago de SI. 52 300. un
es mi ingreso mensual? tercer pago de SI. 36 762 y un cuarto pago de
♦ SI. 45 680; si le sobran SI. I I 756 teniendo la
a) SI. 900 b) SI. 1 400 c) SI. 1 200
♦ factura del 5° pago. ¿a cuánto asciende éste.
d) SI. 1 100 e) SI. 1 300
sabiendo que antes de efectuar cualquier pago
♦ tenía SI. 31 O000?
(6) Habiendo comprado un auto usado por
SI. 7 685 y una camioneta por SI. 9 568, deseo ♦ a)Sl. 17568 b)Sl. 26715 c)Sl. 86635
saber la ganancia total que he logrado, si el auto ♦ d) SI. 85 818 e) SI. 52 314
lo vendo luego en SI. 8 432 y la camioneta en
SI. 10769. ♦ (13) El papá de Luis se ha propuesto comprar un
♦ auto nuevo cuyo costo al contado es de
a) S/. 1 947 b) SI. 1 945 c) SI. 1 948
SI. 18 000; el vendedor le ofrece congelar este
d) SI. 1 998 e) SI. 1 748 ♦ precio si lo compra a lo más de aquí a seis
(7) Si a. b y e son números naturales, ¿cuál es el ♦ meses. El primer mes este señor ahorra
SI. 2 457 para tal fin. el segundo mes ahorra
menor valor de a + b + e si se sabe que a> 5, ♦
SI. I 862. el tercer mes SI. 3 569. el cuarto
b>9. c> 19?
♦ ahorra Sl.4315 y el quinto SI. 1 219. ¿Cuánto
a) 32 b) 33 c) 38 le falta para poder efectuar ya la compra?

d) 42 e) 36
a)Sl.4578 b)Sl.3792 c)S/. 4818

(8) Si x e y son dos números naturales y adem,ís d) SI. 2 194 e) SI. 3 699
y> 5 y x < 17, ¿cuál es la mayor diferencia ♦• (! ) Para irde una ciudad .. A,. a otra ciudad .. B .. un
4
natural de x e y?
señor viaja 147 km a caballo. 2 km a pie, 259
a)8 b)l2 c) I0 ♦ km en tren y 78 km en bicicleta. Si luego hace
d) 13 e ) 14 un recorrido en auto de tantos kilómetros como
♦ a caballo y en bicicleta viajó, faltándole aún 38
(9) Siendo m > 25. n < 15; calcular la menor ♦ km para llegar; ¿qué distancia existe entre
diferencia natural sabiendo que m y n son Ay B?
también números naturales. ♦
a) 739km b) 730km c) 715km
12 ♦
a) b) 14 c) 13 d) 708 km e) 749 km
d) 15 e) 16 ♦
♦ (15) Al sumar tres números naturales obtenemos
(10) Si A,B.C.D IN y además A<7, B<ll,
E 618 por resultado; siendo 322 el mayor y la
C<9.D>6:calcularel mayorvalorde "E",si: ♦ diferencia entre los otros dos números es igual
A+B+C - D = E a 126. ¿Cmíl es el número menor'l

a)15 b)l7 c)27 a) 78 b) 68 c) 72
d) 28 e) 29 ♦
d) 85 d) 84

(11) RecibíSl.453con losquecomprétrescamisas. (16) Al sumar el minuendo. el sustraendo y la
sobrándome S/. 378. ¿Cuánto me costó cada ♦ diferencia de una sustracción obtenemos 8 356
camisa si las tres son de la misma talla y ♦ como resultado. Si el minuendo es el doble del
calidad? sustraendo. ¿cuál es el sustraendo?

a) SI. 35 b) SI. 20 c) SI. 25 a) 2 059 b) 2079 c) 2089
d) SI. 18 e) SI. 27 ♦ d)2019 e) 2 029

(12) Un industrial debe efectuar cinco pagos. De ( 17) Al intentar calcular dos números naturales
pronto observa que hizo un primer pago de ♦ conociendo la suma y la diferencia se comete
t 1 ALFONSO ROJAS PUÉMAPE
1 1~:~:•~1~¡'I
un error al considerar la suma con 12 unidades ♦
menos. dándonos por númeromayorel 88. Si la CLAVE DE RESPUESTAS
diferencia de los números está comprendida ♦ RAZONEMOS - 1
entre 70 y 80 siendo la suma de sus cifras igual ♦
a 15, ¿cuál es la suma de los números BLOQUE 11
verdaderos? ♦
(1) e (6)e (11) e (16) e
a) 108 b) 110 c) 112 ♦
(2) b (7) a (12) a (17) d
d) 115 e) 118 ♦ (3)a (8)d (13)c (18} d
(4) d (9) e (14) b (19) d
(18) Compré un disco duro para computadora en ♦
(5) b (10) d (15) e {20) b
SI. 725. que luego vendí en SI. 813; volví a ♦
comprar el mismo disco y lo volví a vender esta BLOOUEIII
vez en SI. 872; si por última vez volví a comprar ♦
otro disco duro (que siempre me lo venden al ♦ (1) d (6) e (11) e (16)c
mismo precio) pero al venderlo sólo conseguí (2) e (7) e (12)c (17) b
que me pagaran SI. 707, ¿cuál fue mi ganancia ♦ (3) e (S)c (13) a {18)e
total? ♦ (4) a (9) a (14) e
(5) b (10) b (15) d
a) SI. 117 b) SI. 127 c) SI. 147 ♦
d) SI. 207 e) SI. 217

1 MATEMATICA 1
3. MULTIPUCACIÓN Y POTENCIACIÓN DE
NÚMEROS NATURALES

, Va se y tu o...,e,us saxr
c..,.;""_S CuilClln'lOS en !Ola•
r,•~dS,1.:-0s para 36 ai... m

~:.~:~L~ª ª ' ~g
donac,On de~
11
<>
¡
.,... ,-
. '
--, ros , vcroa_o_?
,,.-- .- -
_ _ _ __

cu ad erres
Oeb,enoo 111-
c,b,r caca
r
alumno so,o - - 1 -----
8 r

Mo oa:uce
que tcnor,a
que sumar 36 Í
veces e
1UFF• '

Podemosafirmau¡ue en la práctica, la MULTIPLICACIÓN


es una operaci6n que abrevia la suma.
En nuestra ilustraci6n ¡no es necesario 4ue Luchín opere
tanto! ya que la siguiente igualdad es correcta:

8 + 8 + 8 + .... + 8 = 8 X 36 = 288
36 sumandos donde: 8 Multiplicando} O ,ATENCIÓN'
36 Multiplicador ¡Factores!
Para 3 ó mas factores
X Operador se opera de dos en
dos (operac,on bma-
La multiplicación es una operación en la que a cada par de r,a) as,:
números m, ne IN (llamados factores) hace corresponder 2 " .l "' -t "' 5 = 120

~
otro número natural m . n 6 mn llamado PRODUCTO.

Ejemplo:
J X 5 = IS 120

Factores Producto
-UFO~SO ROJAS PUE\1-\PE 11k%d41
J. I Pl{OPIEl>,\l>ES l>E L\ :\ll'LTIPLIC,\CIO:\ E:\ IN

l. l'IUll'IEl>:\I> UE CL.\lºSUU.- 5. ELEMENTOABSORBENTE.-


"EI produclo de dos mímeros naturales es "Cuando uno de los factores de es CERO, el
siempre otro nínnero natural'". producto también es CERO. Luego CERO es el
elemento absorbente de la multiplicación".
Si a) h E (N
axbxcx0xdxe =O
Emonces aXbr IN
Ejemplo:
EJ1.:ml'lo: 5 X 7 X )5 X 0 X 8 X 32 = Ü
5EIN ;f>EIN
Lur~o: 5 " t, = 311 donde 10 E IN 6. PIUll'IED:\U l>ISTIUlllºTl\'A.-
"Si 1111 número na111ral mulliplica a una suma o
diferencia, ~e dis1rih11ye como factor en cada
2. PIHll'IEUAU ('O~~llTATIV,\.- elenwnto de la suma o diferencia".
··Et orden de los fal·tores no altera el pro- Ejnn¡,lo.~:
d11c1t>00.
(1) 5(3+ 2 )=5 x 3+5x 2

Ej('lfl/'lo: Coml'rub,·mos:
3x7=7x3 5x5=15+10
21 = 21 25 = 25
(2) 3(7 - 2) = 3 "7 - 3 "2
3. PIUll'IEU,\U ASOCl ,\Tl\',\.- Com¡,r,,/Jcmos:
"La forma como se agnipen los fal·lores. no ) x5 =21 -fl
00 15 = l:'i
ahera el produc10 .
a" h" e= a" (h" e)= (a" h)" c
Ejt'll1¡•lo: 7. l'IUll'IEl>AU DE ~IO~OTO~Í:\.-
Si 2 "3 "5 = 30 .. M11hiplici1.11doaamhos miemhrnsdeun:1 igual-
lh1d por un m 1smo mímero muural. resulta otra
TwrJJié11 {'oclrmo.~ C1/ir1ruu que:
igualdad".
2 "(3 "5) = (2 "3) "5 Ejr,n¡•lo:
2" 15 = t, "5 6+4=X+2
30= 30
5(t, + 4) = 5(X + 2)
I'º' cli.~lri/Jutiviclml:
4. ELD1E~TO ~El.TIW.- 5x f>+5x-l=5xX+5x2
""La muhiplicaci<ín tiene un elemelllo NEU - 30 + ."!O = 40 + IO
TRO lllle es 1... 511=50

¡Cualquier número nall1- X. l'IU>l'IEU.\U <".\~CEL.\Tl\",\.-

-
--✓

• ••• ,1
H:!'
;;...
rnl muhiplic.1do por I da
como producto el mismo
número na1ural!
•·si un mismo factor no nulo, aparece 11111h1pli -
cando a .unhos miembros de una igualdad,
po<lemos ll>Uprimi rlo conscn·.ín<lose la iiual -
dad'º.

Ejemplos: F.jcml'lo:
7 ><IX - 2)= 3 x 2" 7
(1) 5x1=5 X-2=3x2
(2) 25 X 1 = 25 6 = t,
1 l\tATtMATICA
1
n :C:\IC.\S OPEl{ATl\'.\S DEL\ :\ll' LTIPI ICACH):\ E~ IN

* Repasemos la t.:cnica müs conocida:

o tamhién 735x42
1470 - - - - _ . 1 4 7 O - - - • 2 unidades por 735 e) Productos
2940 - --- _ . 2 lJ 4 O - - - • 4 dcccn..1s t)(>r 735 Parciales
30870 3 O8 7 O O Producto Total

* Al muhiplicar la unidad seguida de ceros por un número


n.uur..11, cscrihimos este númem y le agregamos tantos O I CUIDADO!
ceros como hayan después de la unidad. S, empleamos numeras.
Ejemplos: ( 1) 56 " 1O = 560 indicamos la -mult1pl1c,1c,on
(:!) 72" 100 = 7200 por cualqu,era delas s,gu,en-
tes formas
(3) -t?" HXlO = 47000
12 x 3
o 12x (3)
* Podemos aplicar cálculo mental en muhiplicacioncs o 12(3)
se11l·illas, aplirnmlo la PROPIEDAD DISTRIBUTI- o (12}{3)
VA.
S, empleamos /erras md,ca-
Ej'-·mplos: (1)8xJ3 =8><(10+3) mos la mu//,pl,cac,ón as,:
= 8>< 10+8><3 = 80+ 24 = 104 x,y
1Puedes ensayar este Ó X, y)
cálculo menta!mentel <2) 12 " 15 = 12( 1O + 5) C:::: O 6 x{y)
= 120 + 60 = 180 ó (x){y)
(3)73 " 12 = 73( IO + 2) ó KY
= 730 + 146 = 876

J.J OPER\('JO:\ES ('(>:\IBl:\ .\UAS HE .\ ll"LTIPLI C AC I<):\.


\l>ICH>, Y SLSTR\CCI<>:\ E:\ ~
Si no l1.1y par.:ntesis. se sigue el siguicmc orden:
l. Efcnumnos las f\tl lLTIPLICACIONES.
2. Ekctumnos las ADIC'IONES y SUSTRACCIONES -;i-
gui'-·mlo l'I orden indicado.
Si hay par.:1fü•-;is. d'-·cuiamos primno las opcr..ll"ion'-·s indi-
cadas '-'11 el i111crinr de éste. siguiendo el mismo orden
Sl'ñalado .1111aionll\.·n1e.
E_jt-mplos:
( I) E{<·i:11wr 7-2>< ."\+lJ- ➔ (2) 12 + 3 - (8 + 2" 5 - 3) + 6(7 - 5)
'----r-' '----r-'
=7 - 6+9-4
..__,__..., = 12+3-(:~+. 1<_1-3)+6(2)
= 1+9--l = 15-( 18 -3)+12
= W--l= ~ = 15- ~12=0+ 12=[u]
♦ 1 AlFOMO ROJAS PUÉMAPE 1
.,A POT E:\CL\C H)'.\ DE l\(Jl\ JEIH>S ;\ AlTRALES
O ¡ATENCIÓN!
Dados a e IN, ne IN, entonces la potencia a" se define así: Tanto a como n y P son
numeros naturales
a1 = a
2
a =ax a
a3 = a x a x a = a 2 x a
a4 =a x a x a x a = a3 x a ("'10 ¡CUIDADO!
En general: Cualqu,er número natu-
ral diferente de CERO.
a" = a x a x a x ... a ó~Oo elevado al exponente
n veces•
CERO da co"lo resulta-
do 1.

donde: aes la base


E1m.
sr.,
27'=,
n es el exponente
s, la base fuera CERO y
p es la potencia el exponente TAMBIÉN.
el result,1do NO ESTA
DEFINIDO.
/~j(-111¡,!tJ.\: <f = NO ESTÁ
(1) 2i Icemos '\los clcv.idn al exponente dos" DEFINIDO
o "dos elevado al cu.ulrado"
o "dos al 1:umlrmlo"
signifirn: 2 2 = 2 )( 2 = 4
(2) , 3 Icemos: "dos elevado al cxponenLe tres··
o "dos elevado al n1ho"
o "dos al 1:uho"
signi fi1:a: 23 = 2 )( 2 ><2 = 8

Ust·mos la rcprese111ación gent·ral: a"= P 0(."'10 IMPORTANTE

* Si 11 =O y a * O
En operaciones combina-
* Si n = 1 das donde tambie>n
aparezcan POTENCIAS
J.(, POT E:\ CI. \ S l> E B ASE IO efectuamos primaro éstas,
luego lds mult,plicac,ones y
Cualquier potcnl·ia de hast' I0es igual a la unidad seguida de f,nalmenteadiciones y sus-
la Gllllidad de CEROS que imliquc el cxponeme. tracc,ones S, hay paránte-
s,s, operamos pr1mero el
Hjem¡1!tJs:
interior de éstos.
I0 2 = IOx 10 = 1110 ¡ 1 seguido de dos ceros!
105 = IO x 111 X 10 X 10 X 10 = l(X)(XI0 ¡ 1 seguido de 5 ..:eros!
1 MATEMATICA
1

l. ComplcLa el siguiente cu.1dro escrihicndo la propiedad corrcspondicnlc:

Operación Propiedad Operación Propiedad Operación Propiedad


Indicada Indicada Indicada

5(7+1)=5-8 ➔ 7+1 = 8 7+8=15 7>8=8-7


8-2=6 ➔ 3(8-2)=3-.6 12(5+3)=12 -5+ 12 -.3 3(~6)~:3-2+3..:6
7, 6,18-0...:9:0 3(7+ 1+2)=3>.7+3 •1 +3x2 5,.1=5

6-(3,5)=3-(6 , 5) (2-9) -3=(3·9)"-2 42,3:3,42

7-29=29 , 7 12764..:64:64x12764 3,0x17x5s0

17, 1=17 32.7 . 5 o.9..:3:0 2Y1z2

11. Efrctuar:
(1)72x59 (5) 1 315 X 203 (9) 51 019 X 720 (13) 725 612 X 312 519
(2) 423 X 75 (6) 3 758 X 7 002 (10) 45 712 X 73 418 (14) 923 018 X 517 617
(3) 978 X 57 (7) 6 866 X 5 569 (11) 105117 X 68 513 (15) 387 615 X 528 619
(4) 757 X 323 (8) 7619 X 3008 (12) 708 996 X 305 519 (16) 1 515 619 X 728 008

111. Empicando la propi~·dad di~1rihuti\"a, opcrJ mc111almt·111c:


¡Comprueba tus
Multiplicación Producto Multiplicación Producto Multiplicación Producto respuestas•

3 , 17 12 " 13 11 X 27 1Preg. Nv I Apta.


8 )' 23 13 v. 25 21 ,, 19 1 36
2 52
6 x 19 15 • 12 17 " 36 3 35
4 118
5 61
IV. Electuar las siguicmcs op~•r,Kion~·s comhin,1das: 6 247
7 100 390
(1) 3+7•5·2 (11) 25°, 2 + 32 + 4-5-6+7 3 8 107
(2) -S-:-S +13-9 (12) 57 -11,3 + 25 -17,2 + 24 9 324
10 1770
(3) 32 -4+5·2·3 (13) 48~6 · 17 + 5" • 3- 19 11 368
(4) 52 +7-9+3 - 10 (14) 27x9 • 25 + 3-17 • 5 12 31
13 215
(5) 18 • 2 • 5 -2) + 5 (15¡ 117 + 27 "62 • 3• 14 257
. (6) 24 • 5 + 7 · 2 + 33 • 6 (16) 78-17 + 35 • 4,19 • 687° 15 1 710
16 1 492
17) 72 ,32 + 10 2, 103 • 3-17 (17) 104 ><13·2 • 52 • 3 • 6 • 57° 17 259 965
1_8) 12 -22 ,,32 .4 + 5- 6- 7 (18) 29 + 19x7 • 82 + 52 • 9 + 11 -13 18 257
19 11 613
(9) 5, 3• • 7835°, 72 • 9' (19) 103..< 11 -62 + 51,13-19 + 5' 20 27033
(10) 1+2+3-.-4 . 5' + 62 x72 • 8 (20) 32 + 10' • 17 + 17° + i7 1 + 17x103
1 ALFONSO ROJAS PUÉMAPE 1

Aquí mis opcracirnws d~ SKANIPR.·\CTICA 0:

..
1 MATEMÁTICA
1

( 4. DIVISIÓN DE NÚMEROS NATURALES)

¡_C11an1a, \",·e,•,; con111.·n,· -l2 a 7".' O ¿Cuantas veces ..


6 ¿verdad? o expresado de otra manera: contiene:

¡_cu;íl \.'S d lllÍifü'ro na111ral por d q11,· lkh,·mos m11h1plicar 24 a 6? D


a 7 para l(Lh.: nos lk~ -l2 lk rcsulwdo"! .. ..... ¡ ¡ ¡6! ! ! 27 a 9? D
Es decir: 7 • 6 = -l2 L; o 49 a 7? D
6 r,·cil~ el nomhr1.• lk COCIENTE lk dividir -l2:7
56 a 8? D
donde
42 r,·cilx: d nomb re dl' Dl\'IDENDO
78 a 2? D
7 r\.'l·ilx: l.'I nombre de DIVISOR
,.
DIVIDIR t•,; calcular el mí mt·ro de \'Cccs que co111i1.·nc 1111
mímero llamado DIVIDEN DO (D) a otro llamado DIVI-
SOR Id). Esle .. n1ínwro de wc,·s·· r1.•cih1.· l.'I nombre de
COCIENTE (q). Además D, d y q E IN
1 ALFONSO ROJAS ruhtAPE 1
~. I IH \ ' ISIO'." EXACT .\

Si el DIVIDENDO (D) contiene una cantidad EXACTA de


\'CCes al DIVISOR (d).entom:cs tenemos una DIVISIÓN
EXACTA.
O Otras Formas
Eslll di\'isión se represenlll así:
. de represent,1r la d,v,:
1 D:d =q 1 ~ "> O de tal modo que se cumpla s,on exacta son

que: j d x q = D 1 /d ,;c. O o
d =q
Ejemplos: Ot d = q

(1) 1S : 3 = S porque 3 x 5 = 15 o l5!_


(2) 125:5=25 porque 5 >< 25=125 C::::- 00 o q

~--2 Pl{OPIED \I> l·T:'\l> .\\IE:'\Tr\L l>E L\ I>l\ ISH>~ EXACTA

Si multiplicamos o dividimos el DIVIDENDO y el DIVI-


SOR por un mismo número natural distinto de cero, el CO-
CIENTE no se altera.
;.Un ejemplo'!
Dividamos 48 entre 16
( s: por favor )
'v · 48 : 16 = 3 ¡es el cociente!
Ahora multiplicamos 48 y 16 por el número
f;
·~·-
~ natural 2.
48 X 2: 16 X 2 00 j MUCHO CUIDADO!
.;,
1!:HI
i:: 96: 32= W
1EI COC1ente sigue siendo el mismol
... con las siguientes divisiones:
y si 4~ y 16 son divididos entre 2 tendremos:
48/2: 16/2 • 24 : 4 = 6 ya que 4 x 6 = 24
• 24 : 1= 24yaque 1x24= 24
24 : 8 = [}] 1El cociente tampoco se alteral
• 24 :24= 1yaque24x 1= 24
• O : 24= 0yaque24x0= O
~.J l>I \ "ISl<>:'i l:'\EX ,\CI A O El 1CLIOIAl'\A • 24 : O= ABSURDO
porque no existe nmgún
Si el DIVIDENDO {D) no contiene una cantidad exacta de número natural que mul-
veces al DIVISOR (d), la DIVISIÓN es INEXACTA. tiplicado por CERO dé 24
• O : O= INDETERMINADO
Ejt."mplo: porque CUALQUIER NÚ-
¿Cuántos conjuntos de 7 manzanas cada uno podremos for- MEROMUL TIPLICADOpor
CERO DA CERO.
mar con 45 mm11anas'!
Luego HA Y QUE EVITAR

45 = 7 + 7 + 7 + 7 + 7 + 7 + 3 ¡Compruébalo! Q. y _Q_
o o
6 \\. 1.'.eS 7
0
1 MATEMATICA

Podemos nowr 4ue formaremos 6 conjuntos de 7 manzanas


cada uno sohr.índonos 3.
Es decir:
El número 7 est..í contenido 6 veces en 45, pero <.·on un
sobrante o RESIDUO 4ue es .'\.

Luego: DIVIDENDO (D) e:> 45


DIVISOR (d) e:> 7
~>()
COCIENTE (4) e:> 6 ·• • O RECUERDA QUE:
RESIDUO (R) e:> 3 Esta división lambien la re-
presenramos as,:
donde: 45 = 7x6 + 3 ¡,De acuerdo'! 45 Ll__
En forma general escribiremos así: -42 6
3 Lcoc1enle
d*O
0< R<d L_Residuo

-&A PIH>PIEl) .\l>ES D EL\ l>l \ 'IS IO:\ l:\E X ,\ l ºT \

1. Si multiplicamos o dividimos el DIVIDENDO (D) y el DIVI-


SOR (d) por un mismo número natural distinto de cero, el
COCIENTE NO SE ALTERA, pero el RESIDUO t¡ueda
l\lUL TIPLICADO o DIVIDIDO por dicho mímero natural.
Ejemplu ...:
( I) En la división 4:! : 9 el cociente es 4 y el residuo es W
ó 42 = 9 x 4 + 6
tv1ultipliquemos DIVIDENDO y DIVISOR por 2:
X-t:IH ~ 00

Ahora, el crn:ienlc SIGUE SIENDO 4 pero ¡Es lll-l'ir, el residuo ini-


cl residuo SUl3I(> a ~ "27
L...'..::.J
____ l·ial l¡u,·dlí MULTIPLI-
CADOpor2!
- - - -- ---..
84=18x4+12
OCiPUEDES...
(2) En la di\·isiún 5~ : 14, el cocie1lll' es 3 y el residuo es ~
ó 52 = 14 x .'\ + Jq·si dividimos el DIVIDENDO (52) y el . . . comprobarlo:

lJIVISOR (14) entre 2. la nueva divisi,ín será: 8J L!_!l_


-72 4
26: 7 26Ll_ "°i2'
Lcoc,ent~

¡ El residuo llUCdlí
rn
__J L
3
¡ El coci,·ntc es el
L__ Res1duo

DIVIDIDO cmrc 2! mismo!


1 ALFOMO ROJAS PUÉM.\PE 1
, El RESIDUO en una división inexacta es siempre
MENOR que el DIVISOR. Además, tal RESIDUO es la
diferencia entre el DIVIDENDO y el resultado de
multiplicar el divisor por el cociente.
l~em¡,lo:
Al efectuar 17 318 : 53, se obtiene 326 como
cociente (q) y 40 d~ residuo (R) C:::: ' O O DIVIDAMOS...
17318~
Por la propiedad dcscriw: R<d
-159 326
en este caso:
Además: R = D - dq
40< 53
----¡¡-¡-
:_!_Qi_
L q
R = 17 318 - (53) . (3'.!6) 358
-318
R=l7318-17278
40
R =-H> L___R

4.5 OPER .\CIO,ES CO'.\IBl~ .\D .\S l>E


\ll ' LTIPLICACI():\ \' 1>1 \ ' ISIO:'\
¡,Nl--ccsitas efectuar .U x 2: -l : 2 x 5 '!
'" 00 EN GENERAL ...
Emom:cs 0p('ramos en el onkn mclicado ele ill¡uicnla a ~:
d.:recha, así:
f ... podemos afirmar que.·
[
64:4:2><5
f am X él' = am+n

16 : 2 x 5
I; donde:

8 >< 5
...._____,
i, :~: n son números natu-

8ill ~~':;;.-:.»~x:::;~~:m::~=:=~~::::::~::::::=:::;:~:=:~::-w

4.6 \ll l TIPLIC ,\CH):\ \' l>I\ ISI<):'\ IH:


POTE:'\CL\S l>E B.\SES J(;l'ALES
• Ayúdame a efectuar: 31' x 34 {f 000 EN GENERAL . ..
lt .. .af,rmamos lo s1gwente:
ó veces
•---
____ 4 vece~ ,
:,.,.
f
m
~=am -n: a -:1:-0
l(l \ 'Cl'l'S i¡:: a
Luego: 111
3'' x 3• = .\ lÍ 3" • J' = 3 1
'.. C:::: 00 { donde:

• f a, m, n son números natura-


¡,Y sí di\'idimos POTENCIAS de BASES IGUALES'!
6
i~ les.
3
=
3x3x3x3x'.\x3 ~?(::~::-:-:,.;;,..:❖:❖:-:-:-:-.-·•: :.- :-:-;-:-·-:❖:-: 0:.;.:.:.;.·-:,;_-:-:w·-:-· •

t,
1 t,- 4
~=3
3
1 MATEMATICA
1
SK .\:\IPR ,\CTICA 0
l. Escribir en el casillero correspondiente el número que falta para que la igualdad sea
correcta:
(1) ~ ,._ D =XI (6) T!, -,, □ =7008 (11) 7)7 X 0 = M2432
(2) 5x D = 125 (7) 86 x □ = 4 816 (12) 985 X D = 105395
(3) 7x O =8J 4 (8) 116 < □ =9628 (13) 1 013 x O =41J 304
1-I l 13 ><0 = 234 (9) 158 x O = 11 850 (1-H 3 006 x □ = 2 690370
(5) 21 ,., □ = 1 869 ( 10) 206 X □ = )9 776 (15) 5 118 x O = 5 466 408

H. ¡,Cuál es el número natural que debemos aumen1ar al dividendo para que la divisi6n sea
exacta y el cocit·nte q se transforme en (q + 1) '!
(1) 18 : 5 (6) 1 019 : 39 (11) 213 615: 318
(2) 73 : 16 (7) 6 518 : 57 (12) 517 819: 607
(3) 117 : 32 (8) 72 bl3 : 8l) (13) 725 763 : 3 208
(4) 819 : 43 (9) 98517 : 97 (14) 515319:2704
(5) 909: 77 (111) 971119 : 12-~ (15) 612 51 1 : 3 507

IIL Ekcluar las siguientt'S operaciones combinadas:



(1) 3 x6: 2 (6) 128 : 2 : 2 :2 X 3 ( 11) 72 : 2x6x5:5
(2) 16 : 2x5x2 :4 (7) I0 : 5x7x2 (12) 2x3x4x5x6 : 12 : 2 : 15
(3) 18x3x2 : 4 (8) 4x36 : 2x5 (13) 80 : 5x2x4 : ,1-6x3 : 8
(4) 52 : 4 : 13 X 17 (9) 6x8x2:4x7 ( 14) 48 : 4 X 5 X 2 : 20 X 7 : 21
(5) 45: 5 X2 X 3 (10) 5x8xl0 : 5 : 2 ( 15) 75 X 2 : 50 X 60 : 36 : 5

IV. Ekuuar la, ,1guicntcs di\ isioncs comprohamlo el rt·sul1adu en e.ida una:

Divisi,ín <I R D=d,¡+R División q R D =dq+R

40 : t, (,: .:1 40 = t,x6 + -1 = 40 117 : 28

70 : 12 329 : 75

57 : 10 7b'!o : 98

7X: 15 lbl2 : 67

22 : -~ 2315 : 86

V_ Cm11cs1~1 las siguit·111cs prcgu111as:


1. ¡,Que on1rre con el residuo de la d1v isiün 142 : 16 si se lriplirn sus términos'!
2. ¡_Que ocurre con el co,-icnte de la misma divisii'in anlerior si ~e triplica slílo el dividendo?
3. En la divisi,ín .:18 : 12 dividimos amhos 1érminos entre 4,¡,se modifica el cociente"!
_-1. Multiplicamos amhos ténninos de la división 2t,8 : 15 por 2_¡_Qué relaci,ín nistc entre el
residuo inicial y el nuevo residuo·! ¡_Se altern el n>ciente'?_ Demostrar.
1 ALFONSO ROJAS PUÉMAPE 1

Aquí mis operaciones de SKANIPRÁCTICA©:

.r

..
..-.- .. '. ·- --:- . .-.... ·-· ·-
IV.W :cfl 1 1 MATEl\lATICA
1

5. RAÍZ CUADRADA Y OPERACIONES


COMBINADAS EN IN

5.1 RAÍZ Cl 1 \l>l~r\l> .\ DE '.\('.\IEROS :\ .\ ITR ,\LES


¡,Cuál es el resultado de efectuar 52'!
O t,RECUERDAS?
Vimos anteriormente que esta operación se llama POTEN-
CIACIÓN y que el resultado de dicha operación se llama PO- En POTENCIACION el ex-
ponent@rndica las veces que
TENCIA:
mult1pl1camos la base por
5 2 = 1 25 1 e: , o ella misma

L Potencia Asi

Ohserva que los datos fueron la BASE 5 y el EXPONENTE


2. Supongamos ahora que los datos son el EXPONENTE 2
y el re~ultado 25, en este caso la operación que nos permite
calcular la BASE, se llama RADICACIÓN.

El número natural que elevado al cuadrado nos dé 25 como


resultado, se llama raíz cuadrada. En este caso, la raíz
~uadrada de 25 es 5 que la representamos así:

~ =5 ya ltue 52 = 25 00 ¡IMPORTANTE'

donde: 25 es el RADICANDO o CANTIDAD Sólo cuando el INDICE es 2


podemos d@¡arde escr,b,r este
SUB RADICAL
y sobre@ntendemos que se
2 es el ÍNDICE C · 00 trata del calculo de la RAÍZ
5 es la RAÍZ CUADRA DA o RAIZ DE INDI-

,r es el operador RADICAL
CE 2

V25 = "\./25
En general:
1 v'm =r --+- r
2
=m V25=5
OPEl~ .\CIO'.\ES CO'.\IIU'.\r\D ,\S

En cjcrciciosdomk aparecen algunas o todas las operaciones


estudiadas cJectu..1rnos 6tas según el siguiente orden:
Operamos las POTENCIAS y las RAÍCES.
.,l. Luego operamos las I\IULTIPLICACIONES y DIVISIONES .
3. Fin..1lrncntc operarnos las SUMAS y RESTAS.
1 ALFONSO ROJAS PUÉMAPE 1 liSifüiíl!ti
Acompáñame a examinar alguno" ejemplos ¡,sí'!
O SINOS ...
(1 ) Efectuar: ... quedamos sólo con SUMAS y
2 RESTAS. operamos en el orden ,n.
X5+3 --~X l-42:2
dicadode llOUIERDA a DERECHA.
Ejemplo:
So/ucitin:
72+ 10-27-15-2
82-27 - 15-2
,\',, lo ,,¡.,¡,t,·, : d 11r.lr11 d, · ,.,,,·r,1n,;11 ,., _. 55-15-2
,- .
f'/WIF./W --> f' \ R 40-2
su;u.,·1,0 _ _, M \ 1, 38
/ I.RCF./UJ - -> S \ N

2 _ ~ 3
• Efet:ttumws I'OTENCIAS ,. RAÍCES: 85 + 3 - V -N x 2 - 42 : 2
X5 + 9 - ....,.....,
7 x X - '--r---'
-l::! : 2
• Lucgodectm11nos /\IUC/1I'LICACIONES
y DI\ '/SJONES : X5+9-56-21

• Por fin, SUMAMOS y RESTAMOS 85 + 9 - 56- 21


e;·•, .o '-----y-----'

9-l

Rcspuc.,·t<t
La cxprcsirín dada es igual a 17. 17

_(
5.3 SIGNOS DE COLECCIÓN c.,o ¡CUIDADO!
=': Si en los SIGNOS DE
Los más usados son: PARÉNTESIS ( ), LLAVES { } y CORCHE- COLECCIÓN ocurre que
TES 1 ]. Si en un ejercicio de operaciones combinadas aparece un hay 11nos al interior de
SIGNO DE COLECCIÓN, efectuamos primero las operaciones al otros. empezamos efec-
interior de ésH.'. C:::: 00 tuando operaciones al ,n-
t,;mor de los primeros.
Efectuar:
}
(7x5-3 2 ) + 2[4 2 + (6 : 3+ 5-{9)+5 x 3
2 2
-.J64]
Su/udú11:
,\/ "f'' r.1r ,1/ 1111, ru,r de , ,1d,1 .\IC ;,\'o /JT:. COLT:.C- 3~
C/()l\' . d onl,•11 el,· f•('r rt1C1 t11/f ., , ·., d mi '11Lu l{ll<' d
t1f1li, ac/,, ,·11 d •'J •·m¡1f,, t I 1

• Efectuamos opcnu.:iones al interior de


los paréntesis: --------t► ~6 + 21-l 2 + -l + 5.1., V · W 1
• Ahora operamos dcmro del corchete: 26 + 2116 + -l + 25x9 - 8]
26 + 2116 + 4 + 225 - XI
26 + 212371 = 26 +-l7-l = ~ Rpta.
1 MATEMATICA
1
SK .\:\IPIL\CTICA ®
l. Efcc111ar haci,·mlo la comprohaciún rcspccLiva:
(Si -\/i'ii'" = r, se lkhc comprobar que r 2 = m)

(1) 0 (5) V¼ (9) -vioo Vi%


(13) (17) V:\24
(::!i V9 (6) ~ ( 10) '\.Íl2I (1-l) 025 (18) '\D6I
Cl)V16 (7) V64 onviM (IS)V256 (19) V400
(--0 v'25 (H) VXl (12) V16Q (16) V289 (20) ~

11. A "simple ,·isw•· ¡_cuál lkh,·rfa ser el valor a cscrihir,c en el cuadriLO, a fin de que la
i!!ualdad ,,·a rorrccL:.1'!

(1) 6+ '-\ = 10 (11) 3-2+5+'V25=6+ 0

(2) =-
(7. 2 · 1) + ~~ = 17 (12) □ +V9 .-8·6 = 14

+ ,/16 · 2 3" ~
3
(3J L"\ - 2 = 7·,5 - 1 (13) ._ 2 + 15 : 5 - 2 = 25
(4) 52 : 5 + V49 , Ci'. = 19 (14) 36 : 9-5-~=12
(5) 2( [~ + 1) + ; 2' '= 40 ¡15¡ 100 s- + 3 . LiJ = 25
(6) 16-8-4 -3 - [L\ =º (16) 6 -9 : 2 + VS1 = [ii; + 8
-
(7) 64 : 2 : 2 · 2 ~ ~ = 10 (17) 2
15 : 5+3 =2-[5] +2
(8) V9 .3 : 1 , ::.t.QJ - 3 = 7 (18) 3+(72 -1)-[TI =50

(9) 72 •2+5 -3= [iJ +100 (19) 9 + (33 + 18 : 6) = [SJ + 30


2
(10) 16 : 4 + 5 - [ = 6 (20) 10 + V100 X 2: 5 = [5] + 14
111. EkcLUar:

(1) 52 +2'3 2+7 Rp@J (11) 5 + {7 -8 · (5' • 2 8-5) + (3• 1)} Rp. ~
[iiJ
(2)
(3)
°Vm+4' 32 - 2
15 3 - 4 +6-
Rp
Rp [ill
(12)
(13)
3--1~ ,4
2. .J,02• 3
+

2
.
4 4} . (6• 5-6)
2 3 -
Rp.@J
(5'-2 - 7•3) Rp[IJ
(4) 2 .3. ,: 6 + ~ R::iDij (14) 10'' : 10'-" - 823" ,, + ,:• ,3' - (Vl 15. 3' · 2) Rp. j 7749 I
(5) 10' 2' - 5' +y;;- Ro~ (15) 10 +(7"~5' _5••· +3"' , 3":3") + ~
2 03
Rp.~
(61 (7" + 7. 2) - 1-v'ioo + 2') Ro. @] (16) ~ . (7' - (4': 2 + 10º) + (7·<8 - 3') +4} Rp.~
(7) (1 3' - 5, 8)+V225 Rp~ (17) 5',2 + 12•·,-2" . 2;' - (10'•2 5') + 7'} l I
Rp. 593
(8) ~ : (52 - 23) + 7 • 2' R::i~ (18¡ 4• · <:' 23 : V4 + (5'v2 · 10•5>-3) Rp. @IJ
(9) 3 X [4; - V2SG] x 3 ' Rp !21216 \ 119) 7' ;: 2 {W,+(10" 'v10"':10 -6 ,3 )+5,:4 10 2 2}
l
Rp 46651
-w3 2 ~) 2
(10) [3' - 2' - 3880] . 2' + 1 Rp.[L) (20) -6 x3 · 9 + 8 4+5 x3+11x2 Rp. (ili]
1 ALFONSO ROJAS PUÉMAPE 1

Aquí mis operaciones de SKANIPRÁCTICA 0


1 MATEMÁTICA
1
RAZONEMOS - 2 1
lnslrrtcciunes:
A continuacüin te presento bloques de ejercicios para
afianzar lo letdo hasta aqu{. Marea la respuesta correcta
y compárala con la CLAVE DE RESPUESTAS que
aparrcealfinal;sinrmbargo, terrcomiendoqueexpliques
en tu cuaderno lns RAZONES de tu procedimiento y
resultado. No tomes este trabajo como una pesada carga,
sino como un DESAFÍO a tu inteligencia.

♦ (7) Un edificio tiene 48 ventanas y 80 puertas.


¿Cuántas puertas más que ventanas tendrán 5
(1) En un salón hay 24 alumnos y en otro salón hay ♦ edificios?
31 alumnos. Si a cada alumno del primer salón ♦
se le obsequia 12 caramelos y a cada alumno (8) En un edificio, las escaleras que hay entre piso
del otro salón se le obsequia 4 caramelos me- ♦ y piso, tienen 15 peldaños. Una persona suhe
nos, ¿cuántos caramelos se van a repartir en ♦ del primerpisoalquimoy luego hajaal segundo
total? piso. ¿Cuántos peldaños subió y bajó?

(2) Dentro de una caja roja se meten 5 cajas azules; ♦ (9) Un dentista extrae 3 muelas pur hora; a cada
en cada caja azul se meten 3 cajas rojas y en paciente le extrae máximo 2 muelas. Cada
cada caja roja se meten 8 cajas blancas. ¿Cuántas ♦ paciente tiene entre 20y 24 muelas. ¿Cuál es el
cajas hay en total? ♦
mayor tiempo que podrá emplear en
15 pacientes?
(3) Un auto viaja a 68 km/h. En cierto momento ♦
está en el km 87. ¿En qué km estará luego de ♦ (lO) Un tío tiene una bolsa de caramelos que quiere
6 horas de viaje? repartir entre 5 sobrinos; cada uno recibe
♦ 8 caramelos y todavía quedan 3 en la bolsa.
(4) Tengo SI. 171 y compro 3 camisas de SI. 27 ♦ ¿Cuántos caramelos hay en la bolsa?
cada una. ¿Cuántos pañuelos podré com- ♦ (11) El papá de Luchín gana SI. 800 mensuales y
prar con lo que me queda, si cada uno cuesta gasta SI. 650. ¿Cuánto ahorrará en I año?
SI. 6? ♦
♦ (12) En un salón de clase, por cada hincha del
(5) El conductor de un camión repartidor, tiene Alianza hay 3 de Universitario y 2 del Cristal.
instrucciones de dejar 15 cajas de leche en cada ♦ ¿Cuántos son hinchas de Universitario si en el
hospital y 10 cajas de leche en cada colegio .. Si ♦ salón hay 30 alumnos'?
en una mañana visitó 5 hospitales y 8 colegios,
¿cuántas cajas de leche repartió? ♦ (13) Rigoherto compra un artículo pagando con 4
billetes de S/. 5, 3 billetes de SI. IO y algunos

(6) Una lavandera puede lavar8 pantalones en una billetes de SI. 20. ¿Cuántos billetes eran de
hora; otra lavandera puede lavar el doble que la ♦ SI. 20, si el artículo costó SI. 150?
primera en una hora. Entonces, las 2 lavanderas ♦
juntas en I hora, ¿cuántos pantalones pueden (14) Un señor reparte propina entre sus 2 hijos y 3
lavar? ♦ hijas, de tal manera que cada hijo recibe el
♦ 1 ALFONSO ROJAS PUÉMAPE 1
doble que cada hija. Si las hijas recibieron ♦ (23) Para entrar al cine hay 2 boleterías; en la
juntas SI. 24, ¿cuánto recibieron los hijos primera atienden a 6 personas en 72 seg. y en la
juntos? ♦ segunda atienden a 15 personas en 3 minutos.
♦ ¿En cuál de las boleterías conviene colocarse,
( 15) Si al multiplicando de la operación 345 x 648, si en cada una hay la misma cantidad de perso-
se leaumenta5,¿en cuántoaumentael producto? ♦ nas?

(16) Un auto recorre 40 km por cada galón de ♦ (24) ¿Cuántas cajas se necesitan para 18 000
gasolina; si recorrió 120 km y cada galón caramelos, si cada caja tiene una docena de
cuesta SI. 4, ¡_cuántos kilómetros recorre el ♦ cajas pequeñas y en cada u nade estas cajas hay
auto con SI. 1 de gasolina? una decena de bolsas y en cada bolsa hay 20
♦ caramelos?
(17) En el consultorio de un médico, por cada ♦
2 pacientes que van con dolor de cabeza, hay ♦
(25) Se disponen 6 filas de 10 alumnos cada una. Si
3 con dolor de estómago y 5 con dolor de se retiran los alumnos del perímetro, ¿cuántos
espalda. Si en la sala de espera hay 30 pacientes, ♦
alumnos quedan?
¿cuántos hay por dolor de estómago?
♦ (26) Cuando iba caminando por cierta avenida,
observé que en cada paradero bajaban de un
(18) En Puno. una persona gasta SI. 6 diarios en ♦
micro, 4 personas y subían 5. Si en cada paradero
alimentación; en cambio en Lima una persona ♦
paraban 4 micros y pasé por 6 paraderos,
gasta SI. 9 diarios por el mismo concepto.
¿cuántas personas bajaron en total?
¿Cuántomásgastaráunafamiliade6miembros, ♦
en Lima que en Puno? (27) En el problema anterior. ¿cuántas personas

subieron en todo el trayecto"!
( 19) La mecha de un cartucho de dinamita demora ♦
150 segundos en provocar la explosión . Si se ♦ (28) En un colegio vanarepartircanastasdeNavidad
prende la mecha a las 3 hrs 17 min, ¿a qué hora a cada profesor. Cada canasta consta de 4 latas
explotará el cartucho? ♦ de leche. 3 tabletas de chocolate. 2 latas de
♦ conserva de durazno y I panetón. Si en el
(20) Una persona tiene SI. 150 en el Banco. colegio hay 35 profesores, ¿cuántas lacas de
Mensualmente gana SI. 900 y gasta SI. 750; lo ♦ leche menos que tabletas de chocolate se han
demás lo ahorra en el Banco. ¿Cuántos meses ♦ repartido en total?
deberán transcurrir, para que tenga en el Banco,
tanto como lo que gana? ♦ (29) Por la instalación de una red de 8computadoras,

un técnico cobró 3 200 dólares. Si en cada una
(21) Un edificio tiene 20 pisos; en cada piso hay gastó 180 dólares en material, ¿cuál fue su
12 departamentos, 4 de ellos con vista a la calle; ♦ ganancia en cada computadora?
cada departamento tiene 8 ó 12 focos. ¿Cuántos ♦
focos hay en el edificio, si los departamentos (30) Un avión lleva 275 pasajeros. En primera clase
que no tienen vista a la calle tienen más focos ♦ viajan 80 personas y cada una pagó SI. 250; en
que los otros? ♦ segunda clase viajaron 125 personas y cada una
pagó SI. 210. ¿Cuánto pagó cada persona que
(22) En una tienda se venden licuadoras a$ 80 cada ♦ viajó en tercera clase si la recaudación total fue
una; planchas a $ 30 cada una y lustradoras a ♦ SI. 58 850?
$ 120 cada una. Si al final del día se vendieron
5 licuadoras, 8 planchas, pero no se sabe cuántas ♦ (31) En una fábrica trabajan 25 personas, siendo la
lustradoras. averigue Ud . el número de ♦ nómina diaria de SI. 550. Si cada persona gasta
lustradoras vendidas, sabiendo que en total se SI. 3 en refrigerio, ¿cuánto le queda a cada
recaudó$ 1 000. ♦ persona, al final del día?
1 MATEMA1ICA
1
(32) En un hospital nacen diariamente 12 niños y ♦ (36) Si Rodolfo recibe el doble de su dinero y luego
mueren diariamente 5 personas; en otro hospi- SI. 20 más, entonces tendría SI. 65. ¡,Cuánto
tal nacen diariamente 8 niños y mueren ♦ tiene Rodolfo?
1Opersonas. ¿Cuántas personas más nacen por ♦
semana en amhos hospitales, que las que (37) Tengo SI. 48 y quiero dar de propina SI. 9 a
mueren? ♦ cada uno de mis 6sobrinos. ¿Cuánto dinero me
hace falta?

(33) En una fiesta hay 64 personas. Si en determi- (38) En cierto lugar se observa que en cada minuto

nado momento todos están bailando excepto pasan 5 autos Toyota, 4 Datsun y 3 Volvo.
8 damas, ¡,cuántos caballeros hay en dicha ♦ ¿Cuántos Datsun menos que Toyota, pasan en
reunión? 1/4 de hora?

(34) Se sabe que la velocidad de la luz es ♦ (39) En un edificio hay 3 pisos; en cada piso hay
300 000 kms por segundo aproximadamente. 5 departamentos y en cada departamento viven
♦ 4 ó 5 personas. ¿Cuántas personas como máxi-
¿Cuánto recorre la luz en un año?
♦ mo viven en dicho edificio?
(35) Jorge tiene 32 caramelos; Pepe tiene ♦ (40) En la multiplicación 345 x 33, un alumno se
22 caramelos y Jaime 30 caramelos. ¿Cuántas equivocó y colocó los 2 productos parciales,

veces contiene Jorge, a lo que le falta a Pepe, uno sobre otro, sin mover a la izquierda el
para tener igual que Jaime? ♦ segundo. ¡,Qué resultado obtuvo?

HLO(.) l 1E 11
(1) Uncomcrciantc m,1yori~ta po~cc 17 c,1miune!>q11ccndctcrminado
momento llevJn un,1 carga de 100 cajonc~ de tomatl'~ cada uno.
Si cada cajón puede alm,tccnar 85 tomate~, ¿cuánto:, :,e llegan J
vender si al descargar lm, camiones !.e encuentra que 6 de c<1d.i
c,tjón se malogrJron por el c,1lur durante el vi.tje?

a)l34000 b)l34300 c)l31100 d)137600 e)l32500

Sol11ció11·
O RECUERDA QUE:
¿Cuántos tomat<'S buenos _y malogrados
Al multiplicar un número natural k por la
cargan los 17 camiones? ¿Cuantos tomates
UNIDAD seguida de m ceros, escnbtmos
malogrados hay en el total de la carga t
el número natural k segwdo de m ceros
Restando las respuestas de ambas preguntas
obtendremos el número de tomates vendidos. Así. 17 x 100 = 1 700

• Calculamos el número total de cajones en los


l7camiones: Qo - - - - - - - - - - 17 camiones x 100 cajones por camión
• Cada c.tjón contiene 85 tomates, entonces el = l 700 cajones.
total de tomates será: 1 700 cajones x 85 tomates por cajón.
• Calculamos el número de tomates = 85 X 1 700 ............. (1 )
malogrados: 6 tomates malos por cajón x 1 700 eaj.
• Si restamos( l )-(2)conseguiremoselnú- = 6x 1 700 ............... (2)
mero de tomates buenos que se vendieron: -----.. 85 X ) 700 - 6 X ) 700 = ) 700 X (85 - 6)
1 700 x 79 = 134 300 tomates.
1 ALFONSO ROJAS PUÉMAPE 1
Respuesta.-
Se llegan a vender 134,300 tomates

Otra forma:
• Podemos establecer el número de
tomates buenos por cajón: 85 - 6 = 79
• El número de cajones es: 17 X 100 = 1 700
• Luego, el total de tomates buenos será: 79 X 1 700 = 1134 300 1 Rpta.@

(2) Deseamos repartir 342 tizas entre un grupo de


estudiantes, tocando a cada uno 7 y quedando de la O ¡MUY IMPORTANTE/
repartición un sobrante de 27 tizas. ¿De cuántos En una división exacta
alumnos estuvo formado dicho grupo?
o-= q equwatea -= d y
o
a) 62 b) 43 c) 45 d) 47 e)38 db . q .
am as expresiones son eqwva-
lentes a:
Solución:
D = dq
Como es un problema de REPARTICIÓN, donde Así:
existe u11 SOBRANTE, entonces tencmoR que
co11sidera,· ww DIVISIÓN INEXACTA, la miRma 24 = 3 ó 24 = B d
que quedará tran.<:{ormada en DIVISIÓN EXACTA 8 J
si al DTVI/JENIJO le quitamoR el RESIDUO 24 = 3x8

• Identificamos los términos de la


división: ------------► DIVIDENDO : 342
DIVISOR : no se sabe
¡es lo que se pide!
COCIENTE : 7
• Si quitamos el RESIDUO al
RESIDUO : 27
DIVIDENDO la división se
transforma en exacta: - - - - - - - - 342- 27 = 315
• Al dividir ahora315 entre el DIVI-
SOR (d) buscado, el cociente exacto
315 = 7 ó 315 = d
es 7: - - - - - - - - - - - - - - -

• Dividendo 315 : 7
d
45= d
7
ºº
Respuesta.-
El grupo estuvo formado por 45 estudiantes. Rpta.@

(3) En la carpintería EL SERRUCHO S.A., 52 carpinteros confeccionan


74 IOO carpetas en 75 semanas. ¿Cuántas carpetas confecciona un
carpintero en una semana?
a) 15 b) 17 c) 18 d) 19 e) 20
1 MATEMÁTICA
1
Solución :

Como no hay más datos debemos suponer


condiciones perfectas; es decir los 52 carpinteros
confeccionan la misma cantidad de carpetas
todas las semanas y todos demoran el mismo O RECORDEMOS QUE·
tiempo en hacer una carpeta .
En una DIVISIÓN EXACTA
• Las 74 100 carpetas 74100175
fueron hechas en 75 - 675 9 88 carpetas D=qó D=d
d q
semanas, luego en 660 ó D= dxq
1 semana se confec- -600
cionaron: ~o
-600
000
• 988 carpetas fueron 988152
hechas por 52 car- -52 19carpctas Qo
pinteros en I sema- 468
na, luego I carpinte- -468
ro en I semana habrá 000
confeccionado:

Respuesta.-
Un carpintero en una semana confecciona 19 carperas. Rpta. @

(4) El papá de Jaimito paga una deuda con 17 ♦ (6) De un cargamenco de 76 cajones de cuadernos,
billetes de SI. 20 cada uno. 19 billetes de SI. 1O un comerciante se comprometió a obsequiar
cada uno y 29 billetes de SI. 5 cada uno. ¿A ♦ 615 cuadernos para promoción de ventas . Si en
cuánto ascendía la deuda? ♦ cada cajón hay 18 cajas chicas con 24 cuadernos
cada una, debiendo venderlos a SI. 3 cada uno;
a) SI. 650 b) SI. 635 c) SI. 680 ♦ ¿cuál será la cantidad recaudada por esta venta?
d) SI. 515 e) SI. 675
♦ a) SI. 97 672 b) SI. 95 713 c) SI. 96 651
(5) Una empresa decide obsequiar una chompa a ♦ d) SI. 92 653 e) SI. 91 322
cada alumno de un colegio en el que hay 7 ♦ (7 ) Se efectúa una compra de 24 cajas conteniendo
secciones de primer grado de 37 alumnos cada 50 tizas cada una. Si todas son distribuidas
una; 5 secciones de segundo grado de 39 ♦ entre 16 profesores, ¿cuántas recibe cada uno?
alumnos cada una; 6 secciones de tercer grado ♦
de 41 alumnos cada una; 4 secciones de cuarto a) 68 b) 65 c) 55
grado de 36 alumnos cada una y 4 secciones de ♦ d) 85 e) 75
quinto grado de 37 alumnos cada una. Si el ♦ ( )
8 Felipe compra 342 champas a SI. 17 cada una
colegio sólo tiene alumnos de secundaria, y obsequia 19. ¿Cuál es su ganancia si las
¿cuántas champas obsequió dicha empresa? ♦
restantes las vende a SI. 28 cada una?

a)992 b)892 c)878 a) SI. 3 130 b) SI. 3 025 c) SI. 2 682
d)862 e)972 ♦ d) SI. 3 230 e) SI. 3 002
1 ALFOi\'SO ROJAS PUÉM,\PE 1
(9) Silvanacompra46camisasaSI. 18cadauna;si ♦ a) Queda multiplicada por 3
sufre un robode9camisasy vende las restantes b) Queda multiplicada por 6
aSl.25cadauna,¿cuáleslagananciaobtenida? ♦ c) Queda multiplicada por 9
d) Queda dividida por 3
a) SI. 107 b) SI. 97 c) SI. 103 ♦
e) Queda dividida por 9
d) SI. 93 e) SI. 88 ♦
(16) Un cerco cuadrado. rodea una superficie de
(10) Uncomerciantecompra78pantalonesaSl. 29 ♦ 121m 2 • ¿Cuál es la longitud de uno de sus
cada uno; si decide obsequiar uno a cada ♦ lados?
integrante de un equipo de fútbol que cuenta
con 5 suplentes, ¡,a cuánto debe vender cada ♦ a) 10 m. b) 9 m . e) 11 m .
uno de los pantalones restantes paraqueobtenga d) 8 m . e) 12 m .
una ganancia total de SI. 156? ♦
(17) En un rectángulo. uno de sus lados es el doble
a) SI. 37 b) SI. 35 c) SI. 39 ♦ del otro. ¿Cuál es la medida de la mitad de su
d) SI. 36 e) SI. 41 ♦ área. si el lado más pequeño mide 43 m'?
a) 1 849 m2 h) t 847 m2 e) 1 843 m2
(11) Compré 98 ventiladores a SI. 45 cada uno. ♦
d) 1 649 m 2 e) 1 749 m 2
¿Cuál debe ser el precio al que los debo vender ♦
para obtener una ganancia total de SI. 1 176? ♦ ( 18) El lado mayor de un rectángulomide40m . Si el
a) SI. 52 b) SI. 57 c) SI. 58 lado menor mide 24 m. menos, ¿cuál es el área
d) SI. 59 e) SI. 55 ♦ de dicho rectángulo si las longitudes de sus
lados son duplicadas?
( 12) Trece generadores de luz cuestan en total ♦
a) 2 370 m2 b) 2 660 m2 e) 2 532 m 1
SI. 16 276; si somos materia de un robo donde ♦ 2
d) 2 560 m e) 2 564 m 2
se perdieron 4 generadores, ¿cuál deberá ser el
precio de venta de los que quedan si nos ♦ ( 19) En un negocio de electrodomésticos, uno de los
proponemos ganar de todos modos SI. 599 por ♦ vendedores gana SI. 96 por cada computadora
todo el rubro de generadores? que vende cuyo costo es de SI. 1 930; además
♦ por cada TV a color de SI. 676, el vendedor
a) SI. 1 875 b) SI. 1 765 c) SI. 1 810
recibe SI. 33. Después de haber vendido 13
d) SI. 1 815 e) SI. 1 825 ♦
computadoras y 19 TV a color, ¿cuánto dehe
(13) Víctor pone a la venta 8 autos avaluados en ♦ recibir dicho vendedor?
SI. 1 560 cada uno. Al venderlos a cambio ♦ a) SI. 1 870 h) SI. 1 725 e) SI. 1 865
recibe 7 TV a color de SI. 620 cada uno, 8 d) SI. 1 855 e) SI. 1 875
computadoras de Sl.3 61 Ocada una, dos casas ♦
de SI. 13 528 cada una. todo esto además de un ♦ (20) Un comerciante compra 16 máquinas de coser
minicomponente. ¿Cuál es el costo de este a SI. 408 cada una; 27 equipos de sonido a
último artefacto? ♦ SI. 732 cada uno y 19 lavadoras a SI. 652 cada
a) SI. 186 b) SI. 204 c) SI. 2 IO una. ¿Cuánto pagó por todo?

d) SI. 240 e) SI. 290 a) SI. 38 650 h) SI. 38 550 e ) SI. 38 620

d) SI. 38 680 e) SI. 38 665
( 14) El lado de un terreno cuadrangular mide 78m. ♦
i.En cuánto aumenta el área de este terreno si la (21) En el problema anterior, si el comerciante desea
medida de sus lados aumenta en l 3m? ♦ ganar SI. 65 por cada máquina de coser que
venda. y SI. 82 por cada equipo de sonido;
a)2047m 2 b)l968m 2 c)2197m 2 ♦
¿cuál deberá ser el precio de venta de una
d) 1 976 m2 e) 2 167 m2 lavadora, si pretende que su ganancia total sea

(15) Se tiene un terreno de forma cuadrada de 42m. ♦
de SI. 4 736?
de lado. Si setriplicalalong1tuddecada uno de a) SI. 630 b) SI. 730 e) SI. 738
sus lados, ¿qué ocurre con el área inicial? ♦ d) SI. 715 e) SI. 682
1 MATEl\L\TICA
1
(22) Necesitamos repartir 1 473 hojas entre los + B que se encuentra a 558 km del punto de
alumnos de un colegio; si cada alumno recibe 6 salida. ¿A qué hora llegará al punto B. ~i por
hojas sobrando en la repartici11n 183 hojas, ♦ cada hora que transcurre recorre 62 km?
¿cuántos alumnos hay en el colegio? ♦
a) 2 pm. b) 5 pm. e) 4 pm
a) 117 b) 215 c) 222 d) 6 pm. e) N.A.

d)217 c)208
♦ (29) En el problema anterior. si al cabo de un poco
(23) Un cierto benefactor desea repartir SI. 855 ♦ más de dos horas <le viaje. el auto sufre un
entre un grupo de excursionistas. cuidando que desperfecto perdiendo dos horas mientras este
todos reciban la misma cantidad. Si cada ♦ se rep,1ra; ¿a qué hora llegaría Ricardo a su
excursionista recibe SI. 17 pudiendo el bene- destino?
factor cancelar una deuda de SI. 22 con lo que ♦ a)6pm. b)4pm . . c)5pm.
sobró; ¿cuántos excursionistas habían?
♦ d) 7 pm. e) N.A.
a)49 b) 47 c) 39 d) 41 e) 43
♦ (30) Si la diferencia entre el dividendo y el residuo
(24) Ante un fenómeno climatológico donde una + de una división es 2 613. calcul,ir el divisor si
ciudad quedó seriamente afectad,1. se envió un el cociente es 67.
lote de 356 calaminas, tocándole a cada poblador ♦ a) 37 b) 35 c) 41 d) 39 e) 43
afectado 15 calaminas. Para no de~pcrtar ♦
resentimientos, cada beneficiado recibió la
♦ HLOQUEIII
misma cantidad de calaminas empicando las 11
sobrantes en un destruido local comunal. ♦ (1) La suma de dos números es 7 843 256 y la
¿Cuántos eran los pobladores afectados'! diferencia de los mismos es 3 25--1- 784. Si
♦ dividimos el menor de estos números cmrc
aJ21 b)l3 c)19 d)l7 e)23 538, ¡_cuál e~ el residuo obtenido?

(25) Para la electrificación ex terna en una zona dela a) 102 b) 204 CJ 202
ciudad. un grupo de vecinos aporta cada uno ♦ di 206 e) 201
SI. 215 faltando todavía SI. 27 para cancelar ♦
dicha deuda vecinal. ¿Cuántos vccmos integran (2) La suma de la~ edades de Coco y Kiko es
dicho grupo si se sabe que la deuda total era de ♦ 22 aiios. Si Kiko es mayor que Coco por
SI. 15 507? 2 años. ¿cuál e~ la ed.id de Kiko'!

a¡ 70 t,J 68 c) 72 d) 75 e) 76 ali! bll3 c)IO d)l2 c)l4

(26) Al dividir 1 016entreotronúmcronatural A.se ♦
(3) Con dos pcda,os de madera alineados uno tras
otro podemos medir longitudes de hastJ 80cm.
obtiene 27 como cociente y 17 como residuo.
¿Cuál es el número natural A que hace aquí las ♦ Si una de ellas es m,h larga que la otra por
veces de divisor? 20 cm, ;,cuál es la longitud de la más corlcl?

a) 40 cm. b) 20 cm. c) 25 cm.
a) 33 b) 34 e) 39 d) 37 e) 35 ♦ dJ35cm. e)30cm.
(27) Ungalóndegasolina le rinde a un automovilista ♦
(4) Una computadora}' un curso para su manejo
27 km de recorrido. Si este señor, recorre ♦ cuesta SI. 1 600. Si la computadora cuesta
durante 7 horas en una carretera dcspej.ida a SI. 1 500 má!, que el curso. ¿cuál es el cmto de
una velocidad de 81 km por cada hora que ♦ la computadora?
transcurre. ¿cuánto!, galones de ga!.olina
necesitará para llegar a su destino? ♦ a) SI. 1 550 bl SI. 1 570 e) SI. 1 560
d) SI 1 510 e) SI. 1 580
a)27 b)l9 c)21 d)l8 c)23 ♦
♦ (5) Un bu:my un parde1apatillascuc~tan SI. 90. Si
(28) Ricardo sale del punto A de una carretera en el par de zapatillas cuesta SI. JU má, que el
línea recta a las 7 a.m . y se dirige hacia el punto ♦ bu7o, ¿cuánto cucst,1 el bun1?
~ 1 All-01\SO ROJAS PUÉMAPE ¡
a) SI. 35 b) Sl.28 c) SI. 32 ♦ actuales es de 46 años, ¿cuál sería la edad de
d) SI. 40 e) SI. 42 Fernando dentro de 4 años?

Una calculadora cuesta SI. 4 más que un libro. ♦
a) 21 años b) 23 años c) 29 años
(6)
Si Lucila recibe de propina SI. 36 y decide d) 25 años e) 31 años
gastar todo en ambas cosas. ¿cuánto cuesta el
libro? ♦ (13) En 1 982 Jéssica era 7 años mayor que Susana.
♦ Si dentro de 3 años la suma de ambas edades
a) SI. 14 b) SI. 16 c) SI. 15 será 29 años, ¿cuál es la edad actual de Jéssica?
d) SI. 17 e) SI. 18 ♦
a) 8 b) 15 c) 7
(7) Se tiene 260 litros de agua repartidos en dos ♦ d) 16 e) No se puede
tanques. Si quitamos 19 litrosa uno y agregamos
.35 litrosal otro tanque. entonces ambos tendrían ♦ (14) Un comerciante compra 350 lavadoras a
la mi~ma cantidad. ¡,Cuántos litros h,1hía en ♦ SI. 567 cada una, de las cuales vende algunas a
uno de ellos inicialmente? SI. 620 cada una, recibiendo SI. 44 640 por
♦ estas ventas. ¿Cuál fue su ganancia por las
a) 102 b) 101 c) 103
mismas?
e) 105 ♦
a)Sl.3816 b)Sl. 3716 c)Sl.3516
(8) En dos toneles tenemos almacenados 68 litros ♦ d)Sl. 3016 e)Sl.3416
de vmo. Si se nos ocurre añadir 16 litros a uno ♦
de ellos y quitar 8 litros al otro. los volúmenes ( 15) En el problema ( 14 ), ¿cuánto dinero todavía
en ambos serían los mismos. Calcular la cantidad ♦ falta recuperar de la inversión inicial?
de litros de vino que tenía inicialmente el tonel ♦
con más vino. a) SI. 152 613 b) SI. 152 810
c) SI. 162 720 d) SI. 117 315
a) 68i b) 24i c) 46i d) 26' e) 48i ♦
e) SI. 153 810

(9) En el ,tUditorio de un colegio los alumnos están ♦ (16) En el mismo problema( 14),¿cuál debería ser el
sent.idos en carpetas largas de 9 persona~ cada precio al que se debe vender las lavadoras
una. Si los hacemos sentar en carpetas largas de restantes para que la ganancia total sea
sólo 8 personas. todos ocuparían 2 carpetas ♦ SI. 12 990?
más. ¿Cuántos alumnos habían en el auditorio? ♦
a) SI. 550 b) SI. 580 c) SI. 600
a) 72 b) 36 c) 288
♦ d) SI. 590 e) SI. 610
d) Faltan datos e) 144
♦ (17) Una Empresa que comercializa ropa.efectúa la
(10) En un sa16n de clase de primer grado, lo~ compra de un lote de 580 camisas, todas de la
alumnos se sientan cómodamente en carpetas ♦ misma talla y calidad, por un valor de SI. 9 280.
de 7 cada una (no sobran ni faltan asientos). Si ♦ El público se aglomera y tienen que vender una
los trasladamos a otro salón donde sólo hay cantidad inicial a SI. 24 cada una, obteniendo
carpet..is de 6 alumnos cada una, ocuparían una ♦ por esta venta de apertura SI. 3 528. ¿Cuál es
carpeta más. ¿Cuántos alumnos había en dicho ♦ esta cantidad inicial de camisas vendidas?
s,1lón de clase?
a) 145 b) 149 c) 151 dl 147 e) 119

a) 42 b)41 c)40
d)45 e) Faltan datos ♦ ( 18) En el problema ( 17), ¿cuál es la ganancia
obtenida en esa primera venta?
(11) Cuando nació Silvana. su papá tenía 31 años. ♦
a) SI. 1 176 b) SI. 1 236 c) SI. 1 315
¿Cuál es la edad actual de Si lvana, si la suma de ♦
d)SI. I 058 e)SI. I 213
ambas edades es 43 años?
a) 4 b) 6 c) 8 d) 10 e) 12 ♦ (19) En el mismo problema (17), si se desea que la
♦ ganancia de todo el lote de 580 camisas sea de
(12) Hace 15 años las edades de Fernando y Carlos SI. 5 506, ¿a qué precio deberían vender lo que
se diferenciaban en 4. Si la suma de sus edades ♦ queda de camisas para obtener dicha ganancia?
1 l\L\TE!\-IÁTICA
1
a) SI. 24 b) SI. 27 c) SI. 28 a) SI. 4 500 b) S/. 3 800 c) SI. 7 500

d) SI. 26 e) SI. 29 d) SI. 5 000 e) SI. 8 000

(20) El dueño de una librería compró 1700 ♦ (27) Jéssica desea saber cuánto costó el auto de
ejemplares de una determinada obra a S/. 14 Mónica, si ésta le ha dicho que al venderlo por
cada uno. Si en el transcurso del traslado sufre SI. 29 000, ha ganado SI. 1 000 más el triple de
un robo en el que se pierden 358 ejemplares, ¡,a ♦ lo que le costó a ella.
qué precio deberá vender cada libro de los que ♦
quedan para que su ganancia total sea de a) SI. 10 000 b) SI. 7 000 c) SI. 14 000
SI. 4 382 a pesar del robo? ♦ d) S/. 15 000 e) Faltan datos.
a) SI. 19 b) S/. 21 c) SI. 27
♦ (28) Compré una computadora en SI. 1 200. ¿En
d) SI. 25 e) SI 24
cuánto debo venderla, si deseo ganar en el

(21) Un ganadero compró 75 vacas por un costo negocio el dohle de lo que me costó aumentado
total de SI. 275 625; por una rara plaga murieron ♦ en S/. 300?
algunas de ellas vendiendo las que quedaron en ♦ a) SI. 2 700 b) S/. 2 900 c) SI. 3 900
st6 125 cada una. ¿Cuántas vacas murieron d) SI. 3 100 e) S/. 3 300
sabiendo que al quedarse el ganadero sin ♦
ninguna de las 75 vacas iniciales, no le queda ♦ (29) Pagué por un buen libro escaso en la ciudad
tampoco ganancia? SI. 270. ¿Cuánto lecostóa la persona que me lo
a) 30 b) 32 c) 28 d) 27 e) 35 ♦ vendió si me enteré que ganó SI. 20 más el
cuádruplo del precio de costo?
(22) Un importador adquiere 50 impresoras offset ♦
todo por SI. 785 000, si porrazoncsdeembalaje a) SI. 50 b) SI. 60 c) SI. 70
quedan 7 máquinas inservibles. ¿a qué precio ♦ d) SI. 80 e) Sin solución en IN
deberá venderlas demás para teneruna máxima · ♦
pérdida de SI. 6 700? (30) Un auto va a 40 km/h y lleva una ventaja de
75 km a otro que va a 65 km/h . ¿En cuánto
a)Sl.17100 b)S/.17900 c)SI. 18050 ♦
tiempo alcanzará éste al primero?
d) SI. 18 225 e) SI. 18 100 ♦
a) 2 horas b) 6 horas e) 5 horus
(23) En el problema anterior, ¿a qué precio se ♦ d) 4 horas e) 3 horas
tendrían que vender las máquinas buenas si se
quisiera tener de todos modos una ganancia de ♦
SI. 53 500? ♦ CI.AVE I>E RESPUESTAS
RAZO;-.;E.MOS - 2
a)S/.18500 b)S/.17800 c)S/. 19500 ♦
d) SI. 19 200 e) SI. 19 700 BLOQUE 11
a (25) e
Un criador compró cierto número de caballos ♦
(1) b (9) b (17)
(24) (2) e (10) e (18)d (26) d
porS/. 114000;vendióunaparteporS/. 80000 ♦ (3) d (11) b (19)e (27) e
a S/. 3 200cada uno,ganandoen e:..taoperaci,·m (4) e (12) a (20) d (28) e
SI. 5 000. ¿Cuántos caballos compró inicial- ♦ (5) a (13) b (21) b (29) a
mente'! (6) e (14) e (22) b (30)d
♦ (7)e (15) e (23) a
a) 40 b) 35 c) 39 d) 42 e) 38 (8)d (16) e (24) e

(25) Compré cierto número de libros por SI. 600. BLOQUE 111
Vendí 40, perdiendo SI. 2 en cada uno y recihí ♦
(1)b (9) e (17) d (25) e
un total de SI. 320. ¿A cómo tengo que vender ♦
(2}d {10) a (18) a (26) d
los restantes si quiero ganar SI. 60"! (3)e (11) b (19) d (27) b
a) S/. 14 b) S/. 15 c) SI. 17 ♦ (4)a (12) e (20) b (28) e
d) SI. 19 e) SI. 20 ♦
(5)d (13) b (21) a (29) a
(6)b (14) a (22) e (30)e
(7) e (15) e (23) e
(26) Si vendo un departamento por SI. 15 800, gano ♦
(8)c (16) e (24) e
el doble del costo, más SI. 800. ¿Cuánto me ♦
costó el departamento?
1 ALFO:\SO ROJAS PUEMAPE 1

LAS i\lATE:\IATICAS DI-: TODOS LOS DÍ.\ S

( 1) Un librero encarga a dos vendedores 60 libros iguales a cada uno. Uno de ellos debe vender
l por S/.30 y el mro 4 por S/.30. Creyendo hacer más sencillo su Lrabajo de vcnw,dccidcn
jumar Lodos los libros y vender 7 por SI 60. ¿Gana o pierde el librero? ¿Cuánto?
Solud(m:
e

Rpta.

(2) Supongamos que las vent.as anuales de cienos periódicos de la ciudad, se rcgisLran en el
1 1

siguienLc cuadro:
Segun estos datos:
AÑOS
(a) ¿Cuantos e1emp1ares se vendieron en 1'987?
Periódicos 1987 1988 1989 1 990 1991 1992
(b) ¿Cu:;ntos ejemplares se vendieron en 1 992?
(e) ¿CL .ntos ejemplares de LA RAZÓN y
Clann 87300 72600 95713 81 400 93913 91 517 VERDADESsevend1eronentre 1 988 y 1 992?
La Razón 47500 52618 46 712 51 418 50317 49615 (d) ¿Cuántos ejemplares de toe--~ los penód1-
Verdades 38317 62 512 77513 58817 69511 70619
cos se vendieron de 1 987 a 1 992?
(e) ¿Cuál de los 5 periódicos :-,vo récord de
Actualidad 13517 23615 32 712 45517 48 719 51 313 ventas entre 1 989 y 1 992?
El Qiservador 27618 25213 32501 43 713 52819 53317 (1) ¿Cuántosejemp •• csdeELOBSERVADOR
se vend 1eron entre 1 987 a 1 992?

Solución:

Respuestas·

(a) 1
(b)I

(c>I

(d)I

(e) 1
(1) 1
1 l\L\TEMATICA

(3) Un l.'.omerciame tiene los siguientes datch rdac ionados con las ventas de sus a ne factos
en un dderminado día:
Considerando que la ganancia del co-
merciameen un anefactoes igual a la di-
ferencia entre el PRECIO DE VENTA y
Prec,o de Prec,o de el PRECIO DE COSTO:
Unidades Costo por Venta por
ARTEFACTO Vendidas unidad unidad (a) ¿Cuánto ganó en la vc1fü1 de lavado-
(SI.) (Si.)
ras'!
lavadoras 7 329 415 (b) ¿Cuánto gan6 en la venta de televi-
sores?
Máquinas de coser 13 429 519
(c) ;,Cuál es su ganancia en los 5 artefac-
Equipos de sonido 9 315 378
tos'!
Telev1sores 27 509 627 (d) ¿Cuánto tiene que pagar a sus pro-
Computadoras 19 1 518 1 715 veedores por todos los artefactos ven-
didos'!
(e) Si en un mes vende 37 lavadora~. 48
computadoras y 115 televisores.
¡_cuál será su ganancia en estos arte-
factos'!.
Solud{m:
1

Rt·spuestas:

(a)
1 ]
(bJ
1 ]
(c)
1 ]
(d)
1 l1
(e)
1 ]
1 ALFONSO ROJAS PUÉMAPE 1

,....
__ __ -----
~AZONAMIENTO MATEMÁTICO
....,
1

(1) ¿Cuántos triángulos hay en la figura mos- (7) El papá de Fclipcclcciclcdistracrse un poco,
trada? para lo cual quiere saber lo que le resulta
más caro: Si llevar dos veces a su hijo
a) 6
Jal·into al cinc donde la entrada cuesta S/.3
b) 16
o llevar una ve,. a Jacinto y a su amigo
c) 11
Vicente al circo donde la entrada cuesta
d} 13
S/.4. ¿Cuál es la afirmación correcta, sa-
e) 7
biendo que al cinc y al circo el valor de la
. (2) ¡,Cuántas figuras de cuatro lados hay en el entrada es único para niños y adultos'!
siguiente gráfico? a) Ir al cinc resulta más barato

a) 10 b) Ir al circo resulta más barato


b)4 c) En ambos casos el ga,¡to es igual
d) Faltan datos
e) 12
d) 6 c)N.A.
e)H (8) Ana, Cecilia y Zoila fríen en tres minutos
tres huevos. ¿Qué tiempo empica una de
(3) ¿Cuántos triángulos hay en la siguiente ellas para frcir un huevo?
figura'!
a)lmin. b)2min. c)3min.
a) 7
d) 4 min . e) Faltan datos
b)4
c) 12 (9) Si x L\ y= x2 + y 2 + xy
d) 13 Calcular a en la siguiente igualdad:
c)9 (3 L\ 2) + a + (5 L\ 1) = 7 L\ 4

(4) ¿Cuántas figurasdc4 lados o cuadriláteros a) 41 b) 43 c) 27 d) 32 e) 39


hay en el siguiente gr:ífico?
(10) rOs=5rsi 12<r<23
a¡8
b) 4 y r Os = S + 3 para otros casos.
c) 10 Si la opcracilÍn r Os está definida cn[l\J,
d) 6
e)N.A. calcular (6018)08

(5) Cortamos cuatro esquinas de la pizarra del a) 105 b) 101 c) 97


salón de clase. ¿Cuánta<; esquinas quedan? d) 91 c)95

a) Ninguna b)4 c) 6 (11) Si a :S b se cumple que a# b = 3a - 1


d) 8 e) 7 y si a > b se cumple que a # b = 1 + a + a2
Según esto, calcular (6 # 3) # 63
(6) Sobre la superficie de una mesa hay 57
moscas. Si matamos 19, ¡,cuántas quedan? a) 128 b) 126 c)256
el) 112 e) 86
a) 13 b) 38 c) 23 d) 19 e)46
1 MATEMATICA 1

CAPÍTULO 2

- --

(f'
'1
1 .,
I!1
NUMEROS
ENTEROS
1
,.:. .•✓.

. ,.__

OBJETIVOS:

• Establecer el conjunto Z como una extensión del conjunto IN.


• Establecerlas relaciones de igualdad y desigualdad entre números
enteros.
• Operar con números enteros.

\
1 AlFOI\SO ROJAS PUEMAPE 1

ESTRUCTURA DEL CAPÍTULO:

l. FL CON.JUNTO DE NÚJ\lERO~ 3.3 Potenciación de NÚMEROS EN-


ENTEROS ( 7L ) TEROS.
3.4 Signos en la potenciación en 7L.
1.1 ¿Por qué es necesario un conjunto 3.5 Multiplicacióndepotenciasdebases
más amplio que IN? enteras iguales.
1.2 Representación de 7L en la recta 3.6 División de potencias de bases
numérica. enteras iguales.
1.3 Comparación de números enteros. 3.7 Potencia de una, multiplicación
indicada de NUMEROS EN-
1.4 Números opuestos.
TEROS.
1.5 Valor absoluto de números enteros.
3.8 Potencia de una división indicada
1.6 Relación de igualdad de Números de NÚMEROS ENTEROS.
Enteros.- Propiedades. 3.9 Potencia de Potencia.
1.7 Relación de desigualdad de Nú-
SKANIPRÁCTICA 4
meros Enteros.- Propiedades
SKANIPRÁCTICA 1 4. [)JVISIÓN Y RADICACION EN 7L
4.1 División exacta.
2. \[)JCION Y Sl'STRACCI6N EN 7L
4.2 Regla de signos en la DIVISIÓN
2.1 Interpretación de la ADICIÓN DE en ?L .
NÚMEROS ENTEROS. 4.3 Propiedades de la división exacta
2.2 RegladesignosenlaADICIÓNDE de NÚMEROS ENTEROS.
NÚMEROS ENTEROS. 4.4 Radicación en ?L.
2.3 Propiedades de la ADICIÓN en 7L 4.5 Modo de hallar la RAÍZ CUA-
2.4 SUSTRACCIÓN de NÚMEROS DRADA de un NÚMERO EN-
ENTEROS. TERO.
4.6 Raíz de una multiplicación indicada
2.5 Operaciones combinadas de ADI-
de NÚMEROS ENTEROS.
CIÓN y SUSTRACCIÓN en 7L.
4.7 Raíz de una división indicada de
2.6 Uso de signos de colección
NÚMEROS ENTEROS.
SKANIPRÁCTICA 2 4.8 Raíz de una Potencia.

3. l\ll:LTIPLICACION Y POTEN- SKANIPRÁCTICA 5


CI \CIÚN EN Z. PROBLEMAS RESUELTOS SO-
BRE LAS PRIMERAS CUATRO
3.1 Multiplicación de NÚMEROS
OPERACIONES ELEMENTALES
ENTEROS. Regla de signos.
CON NÚMEROS ENTEROS.
3.2 Propiedades de la MULTI -
RAZONEMOS
PLICACIÓN de NÚMEROS LAS MATEMÁTICAS DE TODOS
ENTEROS. LOS DÍAS.
SKANIPRÁCTICA 3 RAZONAMIENTO MATEMÁTICO.
1 MATEI\L.\.TICA
1

(1. EL CONJUNTODEWSNÚMEROSENI'EROS(ZJ)
SKANITO. tenia
==r=
---
yo aqu, 47 man-
zanas y me aca-
ban de comprar
19:<,Cuántas me Solo 11er,es que
quedan y cómo lo restar 47 - 19 de-
compruebo? biendo quedar 28
manzanas Para
comprobar solo
sumas 19+28 y te
CC~8_!8SL 1t~r 4
_7_ .

<.,;i;,. -~
No n posible, •
porque no hay _
numero natu• -
r•I alguno q ue ·
sumado con
47 de como re-
sultado 19

Porque en el CONJUNTO IN no s iempre es ¡xlsible la


operación SUSTRACCIÓN. C O
Así por e_jcmplo:
O ¡IMPORTANTE'
* 15 - -~ = I2 porque 3 + 12 = 15
En IN no siempre es posible la
* 25 - 20 = 5 porque 20 + 5 = 25 . pero: SUSTRA CCIÓN. Esta d1hcultad
* 10-IX=¡,? 18 + □ = 10 la resolvemos en el presente
Esdecir,cn este últimoe_1emplo, noexistecn el CONJUNTO cap1tuloamµl1andolN aotrocon-
Junto llamado CONJUNTO DE
DE NÚMEROS NATURALES (IN) un número que sumado
NÚMEROS ENTEROS ( -:Z. ).
con 18 dé como resultado 10 ¡_verdad?.
Este último ejcmplito equivale a resolver la siguienw ecua- Pero en O, y en Z no siempre
ción: es posible la DIVISIÓN EXACTA
IH+x=lll Esta nueva dd,cuftad la resolve-
mos en el capitulo 5 estudiando
De la cual podemos afirmar lo siguiente: un nuevo con1unro mas amplio,
Si no hay un NÚMERO NATURAL que reemplazado :imado CONJUNTO DE NÚME-
en la ecuación, resulte mra igualdad; la ecuación NO ROS RACIONALES ( 1))
TIENE SOLUCIÓN EN IN . \." ' .
1 AlFOl\SO ROJAS PUÉMAPE 1
Ante esll1 diricu I~1d, se hace n.::ccsario ampliar el conjulllo de
los 11tí1nt·ros naturales o [N a otro llamado CONJUNTO OE
l\lll\lEROS ENTEROS que lo representaremos por Z.
Este nuevo conjunto empica el concepto de NÚf\lEI<OS
POSITIVOS y NEGATIVOS . ~ > O
Esto último puede ser cntemlido en forma práctic.1 si me
acompañas a examinar el siguiente ejemplo, ¡,de acuerdo?
... veamos:
Es un honito día soleado, en el que mucha gente va y viene por
una avenida grande, entonces ubicamos a 1111csLro amigo
SKANITO, parado al pie de un poste... ¡,espera a alguien'! ..
. no sahemm, . .. ; pero lo que si sahcmos es que ~I tiene Lrcs ~ O ;ATENCIÓN'
posibilidades en algún momento del día:
Los NUMEROS POSI-
* o se queda parado en el mismo sitio donde está ahora, TIVOS son los m1sm05
* o se desplaza una c icr~1 d is~mcia hacia la derecha del poste, numeros naturales a los
* o se lksplan1 una cil'rta distancia hacia la izquierda del que se les antepone el
poste. s,gno .

Todo esto a tra\'és de la vereda. Los NÚMEROS NEGA-


TIVOS son los mismos
Las dos últimas posibilidades permiten que pensemos en numeros naturales a/os
NÚMEROS POSITIVOS yNEGATIVOSscgúnelsemidodcl que se les antepone el
desplazamiemo reati;,.ado por nuestro amigo. signo -

Vamos a con\'enir en lo siguiente:


Si SKANITO se lksplaza 4 metros hacia la derecha em-
picamos el signo+ delante de 4 así: +-1
S1 SKANITO se despl.v.a 4 metros hacia la il'.llllicrda cm-
¡lleamos el signo - de lame de 4. es decir: --1
El poste s61o scría un PUNTO DE REFERENCIA al que
\'amos a asignar el númcro CERO. ~ - 00

oo 1CUIDAD0
1

EL CERO NO TIENE
SIGNO+ ó -
MATEMÁTICA
1
Entonces, el conjunto de números enteros (Z) está definido así:·

Z = { ... ;-2;-1;0;+1;+2; . .. } Q O

donde:
- n significa que n + (-n) = O ; n E IN
1.2 REPRES EI\TACI() N DE Z EN LA RECTA
:\lJI\IÉIUC A
O IMPORTANTE:
Sobre una línea recta, uhiquemos un punto de referencia al
que le hacemos corresponder el número CERO. Por convención se usa n en
lugar de + n para n > O.
A partir del CERO, empicando una unidad de medida (puede
ser 1 cm), uhicamos puntos hacia la derecha y hacia la Por e1emplo:
izquierda, haciendo corresponder a cada uno los NÚMEROS 4 en lugar de + 4
ENTEROS POSITIVOS y los NÚMEROS ENTEROS NE-
GATIVOS respectivamente, así:

-b -'i -4 -1 2 11 +1 +2 +J +4 +:'í +6

Esta es la Recta
Numérica
ººº
El punto que corresponde al número
CERO se le llama tamhién ORIGEN.

1.3 ;,C()i\ 10 COl\1PARAi\lOS NÚI\IEROS EI\TEROS?


Si tenemos dos números enteros, será MAYOR el que esté
00 ,CUIDADO'
representado a la DERECHA del otro número en la RECTA
NUMÉRICA, así: En el con1unto Z podemos
+1 +6 d1stingu,r los siguientes sub-
6 es mayor que 1 porque:
l) +4
conjuntos:
4 es mayor que O porque: ENTEROS POSITIVOS: Z +
.3 (1
ENTEROS NEGATIVOS· z_-
O es mayor que -3 porque:
-5 .¡ TODOSLOSENTEROSCON
-1 es mayor que -5 porque:
EXCEPCIÓN DEL NÚMERO
6 l
-2 es mayor que -6 porque: CERO: Z*

,Osea que en laRECTANUMÉ- m > n si m - n > O


RICA los números enteros ·cre- para todo
cen· de izquierda a derechal
m,nE Z
Es decir:
.. -4 ~ -2 11 +l +2 + :I +4

e===================~> Oc i1__quicrda a derecha o


de menor a mayor.
ALFO~SO ROJAS PUÉMAPE

1.-1 I\C:\IEROS OPUESTOS


Antes de reconocerlos, vamos a cswblcccr la DISTANCIA de
un punto cualquiera al origen, en la RECTA NUMÉRICA:

·-- -4 -3 -2 -1 O+) +2 +3 +4 . ..

De +2 a O hay una distancia de 2 unidades


De -3 a O hay una distancia de 3 unidacks
De -4 a O hay una distancia de 4 unidades
De +4 a O hay una distancia de 4 unidades O ;ATENCIÓN!
En los dos últimos ejemplos, las DISTANCIAS SON IGUA- En la RECTA NUMÉRICA
LES, luego -4 y +4 son dos NÚMEROS OPUESTOS. podemos distinguir que:

~
Entonces: Si dos puntos de la recta (t) IN e Z
numérica tienen la misma
(2) Z es mf,mto
! ~r. .---:.. distancia al origen, los númc-
,~ ros correspondientes a dichos (3) "Z es ordenado
1 ~ puntos.son OPUESTOS.

1.5 VALOR ABSOLUTO DE NÚI\'1EROS ENTEROS

El valor absoluto del número entero a se representa por la 1


y se define así:

El valor absoluto de a expresa en la recta numérica la


distancia siempre positiva del número a al origen O.

Ejemplos:

( 1) Hallar el valor absoluto de - 3. 1IMPORTANTE!

Se nos pide hallar 1- 3 1 Podemos decir también que el


conjunto Z está formado por
Como - 3 < O entonces por definición: 1- 3 1= - (- 3), lo
IN y los numeras enteros NE-
que significa que 1- 3 1es igual al opuesto de - 3 es decir GA T/VOS.
3.
z
Puedes comprobar en la recta numérica que la distancia IN
de - 3 a Oes efectivamente 3 unidades.

-4
(2) El valor absoluto de+ 7 ó 1+71 es igual a 7 .

(3) El valor absoluto de - 10 ó 1- JO I es igual a 10.


1 MATEMÁTICA
1
(4) Cakularcl valor que puede tener
a si se sabe que:
lal + 8 = 40

Sulul'iú11:

Si lal = h
em,mcl.',; a ptll'lk :-.a +h o -h O ¡CUIDADO !

E_1emplo: /a l
Si lal= 10 ~ O Se lee:
ll puúk ser igual .i - 10 VALOR ABSOLUTO
dea
porque: 1- 101 = IO
Jk'íll lamhii:n :

a pul·1k Sl'í igual a + 10


porqul': I+ 1OI = 1O

• Escribimos 40 como 32 + 8: lal +X= :n + 8


T T
• Si los segundos sumandos son iguales
los primeros también lo serán: lal = 32
• Por la definición de VALOR ABSO-
LUTO: a puede ser 1 +32 <l -321 Rpta.
(.5) Efectuar:

1- 71+ 1- 181-1- 31
1- 61 + 1- si
So/11dó11: ¡IMPORTANTE!

Una /11rm,1 ¡1rtícli, a para ·· d1·.11111ari•- S1 /al= b


n ·r· las harra., de'i \ .-\LO!< rWSO -
Ll '"/O . es ·· d,·.,aparcn·r· t11111hit;11 los Entonces apuede ser igual a
signo., + o - del 111í11wro intl'rior.
+b o : b
1- IXI = 1:-i

• Desapar<!l"l!trlOS las harras del 7 + 18 - 3


valor ahsoluto junto i:011 los six- 6 + .5
nns interiores:
Efectuamos operaciones:
1 ALFOSSO ROJAS PUÉMAPE 1
l.<, l<ELACH):\ UE H;L' \LUAD DE ~Úl\iEROS E;\;TEROS

Dos números entl.'ros son iguales si:


ticn1..'n el mismo signo+ o -
tienen d mismo \'alor absoluto ~ - O
Ejemplo: -7 = -7 o GRÁFICAMENTE.
PROPIEIJADES DE LA l(;UALDAD: En la RECTA NUMERICA.
dos numeros enteros son
(1) Propiedad Rcflcxi\'a.-
,gua/es, s, les corresponde
.. Todo número cillero es igual a sí mismo"
el m;smo punto de la recta.
(::!J Propiedad Sim~trica.-
"Si un número cillero es igual a otro, cmonces este
es igual al primero"
(3) Propiedad Transitiva.-
·'Si un número l.'ntero es igual a un segundo núme-
ro y este es igual a un Lacero, entonces el primero
es igual al tercero ..
Si a = h y h = e entonces a = e
l. 7 REL\CI<):\ DE DESIC , 1 \ 1.1> \I> DE :\(\IEIH)S
E:\'ITROS.

Dos 111ímeros l.'ntcros son desiguales si su rcpres1..·111ación en


la RECTA NUr-.lÉRICA son dos puntos distintos.

a a < h L;Oo h

·s, b esta a la cerecha de a


entonces b es mayor que a ó a 00 RECUERDA QUE:
es menor que b"
El símbolo < s1gmf1ca
MENOR QUE'"
PIH>PIED .- \l>ES DEI.A DESIGUALDAD: El número hac,a donde
se d,rige la "punta" del
(1) Propiedad Cone,a.-
símbolo es el MENOR.
"Dados dos 111ímnos cilleros a y h distilllos, puede
Así: 2 < 7 se lee:
ocurrir que el primero sea menor que el segundo o
"2 es menor Que 7"
que el primero sea mayor que el segundo".
que tamb,en se escribe
a<b o a>h as,:
(::!) Propiedad Transiti\'a. -
"Si un número entero es mayor q111..· un segundo y
éste a su vez es mayor que un tercero entonces el
primero es mayor que el tercero".
Si a> b r-.. b > c elllonecs a> e
1 MATEMATICA
1
L 'i K ,\ :\ 1PI{ ,\ C'TIC A _Q)______r1"'-_:,n_ _ _ _ _ _ _ ____
l. Ordl·na los siguientes númaos enteros en la recta numl;rii.:a:

( 1 ) -7; +6. 1 º· - (M -111; +2; +5; - 1 (11) 100;-I0; -1;0;+5;+1; -2


11) - 111. -12; -13 (7) +15; -13; -1-l; O < 121 7. •1; O; -X: +1: -1: -fi
0) -20; -lll; -fi (X) - 17;-. lfi; - 15; O el 3) + 1; - 1; O; +7; -6; + 12; - 11
C-ll-15; -::!;O 1')) +8; -5: --l; +3 11-l)+-4; h; -5; -7;+1;0; -U
(5) 27; -11; 1 t 10) -2; -5; -li; + to (15) -7; 1;11; -10: +3;-5: +11

11. Completa el siguiente cuadro escribiendo cada conjunto por extensi6n:

CONJIINTO POI{ EXTENSl<>N

A=h/xE [)'..J Ax<8 }


B = {x/xE ZAx<8}

D = {x/xE Z "- IO<x<lO}


E= {x/ x E -Z " - l 3c xd 3 }

111. Completa el s1guil·nte cuadro:

lxl OPUESTO X >Ó< y OPUESTO IYI MAYOR DE


DE X DE y LOS OPUESTOS

7 +7 -7 < -1 +I 1 +7
- u
¡ i? ./- Je -111
-
.,,...
o
-)
o
{
--f, n

' -1 +I
.¿ ,.( o{- j

~ ,_. ') +1 ✓ -5 -rs s 'f' .:;

) - ,l +I 7 -IO .J- JO JO -' )D


J ~ 1-
15 -15 ,!_ -13 -+ I ?, , 3 ., ~ s
.t, + l,
"t< f- ,I -1 7 f- '- L
f f? -X ,e_ -2 .,_, ~
~ -i t
I e 'L -r I 6 ,_ -102 7 +101 - I r: 'l. h'l -rl o 2.
JoS ,_ ,1 c5 -1115
"'- 1
-1
+- .A ,,( .¡ /O 5
1V. Contest~, las siguientes prcgu111as:
( 1) ¡Cual es el miml·ro Cllll·rn nc¡:a1ivo m:íximll"!
(1) ¡_l'odrías csnchir el minll·ro cmcrll negati,'ll mmimo! ¡,Por qu~·!
(3) ;_Es i¡:ual el llpucsto de un mimero que su valor .1hsol111ll·! ¡Cu:íl es la diferencia"!
1-l l ;.Ptll·den dlls minll·rn~ de difcr,·ntc signo pero de igual valor ab,llhllo ser i¡:ualcs"! ¡,Por ,¡ué'!
(51 ¡_C:11:ík~ son lll~ mimcrns cm eros mayores ,1ue -111 y llll'llores t¡Lll' + 1O"!
1 ALFOi"';SO ROJAS PUE)tAPE 1

l Aquí mis operaciones de SKANI PRÁCTICAG)

l ~ _.
.::e-~"
;;.~ -- I

!I
,-~
,.l
1 l\lATEl\lATI CA
1

( 2. ADICIÓN Y SUSTRACCIÓN EN Z)

.!. I l:\TEl{PRET.\CI<):\ DEL-\ ADICIC>:"i DE ~ÚI\IEIH>S E:\TEROS


Como en los negocios se dan siLuacioncs de GANANCIAS y PÉRDI-
DAS, podremos imcrpreLar la ADICIÓN de números cilleros, asignan-
do números pos1Livos a las ganancias y números ncgaLivos a las
pérdidas.
Veamos:

* En un negocio gano S/.4 y en OLro gano S/.6


¿Cu;.'ínLo gano cn ambos negocios'! ... S/.10 ¿verdad?
Como se trata al final dc una ganancia, entonces: (+4) + (+frl = + 1O

* En un negocio picrdo S/.4 y en otro pierdo S/.6


¿Cu.ímo picrdo en ambos negocios'! . . . S/.10 ¿De
acuerdo'!
Como se Lrala al final de una p.:rdida, emonccs: (-4) + (-6) = -JO
* En un negocio, gano S/.4 y en otro pierdo S/.6.
¿Gano o pierdo al final y cufüuo'!
Como loquepicrdocs m;.'ísdc lo que gano,cuandosaco miscuemas
salgo pcnlicndo, ¿cu;.'ínto'!: S/.2, es decir: (+4) + (-6) = -2
1 ALFONSO ROJAS PUEMAPE 1

* En un negocio pierdo S/.-t y l'n otro gano S/.6


¡,Gano o pierdo al final'!, ¿cuánto'?
Como lo que gano es m~ís que lo que pierdo. al final de
cucnt~1s rl·sultú ganando S/.'2 ¡_Vl·rdad'!, es dcrir: (-4) + (+6) = +2
Resurmcndo estas operaciones:
(+4) + (+6) = +to o ,CUIDADO1
(-➔) + (-6) = -JO
Oper,1c,on · ADICIÓN
(+4) + (-6) = -2
Operador . +
(-➔) + (+6) = +2
Elementos · SUMANDOS
Oh,;ervando este resumen podemos establecer la siguiente .. . Resultado SUMA

2.2 1u:ca.,, DE su;~os E:\ LA ADICH>:\ l>E


:\ é \IEl{OS E:\TEl{OS
1. Si se trata de números cilleros del MISMO SIGNO. su-
mamos los valores ahsol u tos, y el signo del resultado es
el mismo de los sumandos.
/~jc111¡1/o.~:
(1 ) (-12)+(-.~ )=-15 00 ¡ MAS PRACTICO '
(2) (+18)+(+12)=+30 C:: · 00 Borramos los paréntesis y el
, Si se trata de números enteros de DISTINTO SIGNO,
operador +, y tendremos·
R'St~1mos los \'.llores absolutos (el mayor l\lENOS el - 12-3=-15
menon y al rl.'sulLado le agregamos el signo del número +18+12=+30
lk l\1A YOR va lor absoluto. +7+2=-5
+ 10-4 = + 6
Ejem¡,lol:
(1) (-7)+(+2)=-5
(2 ) (+ 10) + (-4) = +6
,
2.J Pl{OPIEl> .\l>l·:S DEL\ .\DICIO:\ a-:, Z
l. PlH>PIEDAI> I>E CLAL'SLIRA.
Si a E Z y h E Z entonces (a+ h) E Z
"La suma de dos números enti:ros es otro número entero".
Así:
-7 E Z; +5 E Z luego (-7) + (+5) = -2 E Z

:?.. PIH>l'IEDAD CONl\ll'TATIVA.


a+h=h+a
" El orlkn de los sumandos no alk·ra la suma"
Así:
(-15) + (+101 = (+IOJ + (-15)
1 MATEMATICA
1

3. PROPIEDAD AS()('IATl\'A.

(a+ h) +e= a+ (h + e)
"La forma corno se agrupen los sumandos no allcra la suma"
Asi:
(-6) + (-2) + (-4) = -12
1(-6) + (-2)1 + (-4) = (-6) + [(-2) + (-4)1
(-X)+ (-4) = (-6) + (-6)
(-12)=(-12)

-l. ELEMENTO NELITRO.

Es el CERO.
"Si sumamos cualquier mímcro cmero a con el dcmenlo
nculro, el resultado Larnbién es a".
a+0=a
Así:
(-8) +O= -X

5. ELEl\lE;'l;TO 01•L 1 ESTO o Sll\U~TIUCO.

"Un número cillero es el OPUESTO dc olro, si sumados


dan como result;.1do CERO". a+ (-a)= O
Así: (+ 10) y (-1 O) son números opu~·s1os
t
-
porque(+ 10) + (-10) = O

<,. l'IH>PIEIJAD DE l\lONOTONÍA.-

"Dada una igualdad, podemos sumar a ambos miembros,


un mismo mímcm cmern. resultando entonccs olra igual-
dad" Si a = b emonces a + e = b + c.
Así: (+5) + (-8) = (-3)
(+5)+(-8)+(-10)=(-3)+(-I0)
-13 = -U

7. PROPIEDAD CA!\CELATl\'A.

"Dada una igualdad, si hay un mismo sumando emcro en


ambos nuembros, !X)demns cancdarlo. oblcnicndu cn-
LOnccs otra i!!ualdad". Si a+ e= b + e en~onces a= b.
Así: (-l)+(-12)=(+.,J+(-4)+(-12¡
-1 = -1
1 ALFO'iSO ROJAS PLÉMAPE 1
2A SL:STRACCH>N DE NÚJ\IEROS ENTEROS
Dados dos números cmaos, hallamo..; ,u DIFERENCIA
transformando la SUSTRACCI(>~ l'n una ADICI<iN dd
minul'nclo con el opul'sto cll'I sustraendo. a - b =a+ (-b)
l:)emplos:
(1) Efectuar:
i
(-7) - (- 2)
t. .___
~-- - - - - Minuendo
Sustraendo
O RECUERDA QUE:
Sumamos números enteros
así.

El opuesto tkl sustraendo: +2 Si los SIGNOS SON


La SUSTRACCIÓN convenida en ADICIÓN: IGUALES·
Sumamos los valores
<-7) +<+2)=-s e o absolutos ..
• S, los SIGNOS SON DIS-
OPERACIO'.\'ES CO:\IHl'.\'Al> .\S DE TINTOS.
Al>ICH>, Y Sl 'STRACCION EN Z Restamos el mayor valor
absoluto menos el menor.
Efectuar: En ambos casos. el s,gno del
P = (+ 7) + (-2) - (+-l) + {+ 10) - (-3) resultado es el del mayor
VALOR ABSOLUTO.
a) - 1-t h)+l2 c)-1:! d) + 1-t e) -1

So/uáti11:

( >pa.1rn111 d 1111111,·ro 11.1111ral 7 o .:011 d m1111,·111 ,·m,·n,


~ 7 es 1~11.,l.p,1n¡ue ,.: lr,11.1 de num,·nh .1 <1111ene,
corre~ponded mi~mo pumo en l., KECT A NI 1!\.1fl{I -
C A. por lo 1.111111. en c~1.1, oper.1uone, podemos ,up1 i-
mir el ,1¡!1111 + ~ 1.1111h1en to~ p.ire111c,1,.
Ad,·111 •.- las SI ' STKACCIO\:ES ta, 1r.11i...f11nn.111111,
en .\DICIO"\LS empk.1mlo d Ol'l'LSTO.

• Transformamos las S USTRACCI< >NES


en ADICIONES por d OPUESTO: V P = (+ 7) + (-:!) + (-4) + (+ IO) + (+3)
Escribimos los ENTEROS POSITIVOS
rnmo números NATURALES: ◊ r = (7) +e-~>+ (-41 + oo) + <3>
Suprimimo~ los paréntl.'sis: V P=7-:!--t+l0+3
Agrupamos lo,; mímeros positivos y los
números Ik'gativo,:
vr = 1 + 10 + 3 - 2 - 4

Sumamo-; los positivos y los ncgaLivos


por s~·parado: V P = +:!O- 6 = 6 Respuesw@
1 MATEMATIC.A
1
Efectuar:
E= (-10) + (-1)- (+6)- (-8) + (-5)

a)-12 ~ -14 c)-10 el) +12 e) +2

Solución:
O ;CUIDADO'

PR/,\IT.RO · Cn11va11mo.~ St.:SfRt\CC/ONES EN Para esto solo cam-


J\IJ/CIONF.S C:· O biamos el sustraendo
por su OPUESTO.
S T.Gl'.VIJO : fa.-rihimw lo., T ."{[ F.ROS POS/ /J\ OS
Ejemplo:
como N,\rl 'll,\LF.S

I T.RC FRO · .'ittf'ri,mmo~ f'tlré11IL'., i., (+5) • (+8)


(+5) • (-81
Cl 1;\R/ O · Swnamo., lo .,pos1tivo.,·y neRal1V1J.\por
.,q,arw l o

(!l '/.\J U · [ J,·c1tumw~ l,1 Sl 'S /1UCCl<J,\ ' Ji,wl

• SUSTRACCIONES c-rt ADICIONES: E= (-10) + (-1) + (-6) + (+8) + (-5)


• f'OSff/\ 'OS como NA/l 'RALES: E= (-10) + (-1) + (-6) + (8) + (-5)
• Su¡,rimimns paréntcsi.'i." E=-10-1-6+8-5
• Suma de r1ositivo ., y IIL'g11tivns por
sc·poradn : E = +8 - 10 - l - 6 - 5
- 22

• Sustrcu:cián final : E= ~ RespucsL.1@

:!.6 su;~os DE COLECCl()I\;

Son los mismos que usamos al operar en IN. 00 ¡IMPORTANTE!


Es lkcir: parémesis, corchetes y llaves Sidosomasenterossondel
=·:.
Para suprimirlos procedemos así: mismo signo solo sumamos
todos y el resultado lleva el
* Si hay unos dentro de otros, suprimimos primero los signo de los sumandos.
más inlernos.
Para suprimir paréntesis (o corchetes o llaves) efectua- -a-b-c = -(a +b+c)
*
-3-4-5 = -(3+4+5)
mos las operaciones indicadas al interior. transforman-
,., -12
do en un solo número.
o también:
Podemos suprimir paréntesis antes de operar el interior
así:
(al Si delante del paréntesis está el signo(+), suprimimos el
parénlesis y los signos del interior no se alter.in.
Ejemplo: +(-7 + 3 - 5) = -7 + 3 - 5
1 ALFONSO ROJAS PUÉMAPE 1
(b) Si delante del paréntesis esLá el signo(-), suprimimos el pa-
réntesis pero los números del inLerior los reemplazamos
por sus opuestos, es decir: los signos del interior CAM- o ¡CUIDADO!
BIAN. Paréntesis con signo +
delante:
Ejemplos: (1) -(-7+3-5)=+7-3+5
+(a • b + e) .. +a · b + c
(2) Efectuar: f Paréntesis con signo •
\ delante:
F = -2 + { -1 + 3 - l-8 - (2 - s + 1)l - 1J
:,: -(a · b + c) • -a +b • c
a) 5 b)4 e) 3 d) 2 e) 1
Soludó11:

• Suprimimos primero el paréntesis


interior: e::> O F = -2 + {-1 + 3 - [-8 - 2 + 5 - 1l - 1)
• S1wrimimos el corchete: F = -2 + { -1 + 3 + 8 + 2 - 5 + 1 - 1)
• Suprimimos la llave : F = -2 - 1 + 3 + 8 + 2 - 5 + 1 - 1
• Cancelamos los número., opuestos:
C· oo F=;-I+3+8-5
• Operamos los positivos y negativos
por sc·parado: F =.____,____,
+3 + 8 '--------.----'
-5- 1
F= +11 - fi
• E.fec:11u11nos la .rnstracciánfinal: F= +5
ó F=5 RespuestaG)

OTRA FORMA:
00 RECUERDA OUE:
Antes ele suprimir el paréntesis efl'Ct1u11nos las opera-
ciones del interior; lo mismo hacemos con el corc-Ju•te La suma de un número en-
-~=
y la llave. teroconsuOPUESTOsiem-
pre es CERO.
F = -2 + f -1 + 3 - 1-8 - (2 · 5 + 1) 1- 1 l +a -as O

-2
-6
000 iA TENCIÓNI
F= -2+ 1-1 +3-1-6)-1)
+3 -(-6}
F = -2 + (-1 + 3 + 6 - 1}4> 000
Transformando a ADICIÓN:
+7
+3 + {+6} o
F = -2 + 7 +3 + {6J ó
Respuesw@ +3+ 6
F="+ S ó !F=5I
l~ '1~~1~l~i'I
1
1 MATI:M\TI CA 1 ♦
SK ,\ :\ 1PI{ ,\( 'T ICA tí 1

l. Efectuar las siguientes sumas:

(1) (-5) + (-2) + (-1) (11) (-7) + (-6) + (-2) + (-3) + (-10)
(2) (-7) + (+2) + (+8) (12) (-12) + (-18) + (-1) + (-7) + (+28)
(3) (-10) + (-2) + (-7) (13) (-25) + (+25) + (-5) + (-11) + (+7)
(4) (-12)+(-11)+(+10) (14) (+8) + (-13) + (-12) + (-17) + (+3)
(5) (-6) + (-3) + (-2) (15) (+ 17) + (-11) + (-14) + (-25) + (+22)
(6) (-5) + (+8) + (-3) (16) (-118) + (+36) + (-12) + (+118) + (-1)
(7) (-4) + (+7) + (-1) (17) (-222) + (+ 113) + (-103) + (-46)
(8) (-9) + (-10) + (-11) (18) (+52) + (-50) + (-2) + (-117) + (-1)
(9) ¡.. 51 + ,.31. (+2) (19) (-38) + (-51) + (-13) + (-2) + (+ 1)
(10) (-10) + (-3) + (-18) (20) (-54) + (-18) + (-11) + (-117) + (-425)

11. Comph.-1.1 el ~i!!UÍ\.'llLC cuadro:

a >Ó< b SIGNOS lal >Ó< lbl la+bl SIGNO DEL RESULTADO


IGUALES RESULTADO DE a+ b

-15 < +2 NO 15 > 2 13 - -13


-7 < -5 SI 7 > 5 12 - -12

+5 "":> -18 . ~

-13 +1 rr .c.

-101 -99 r .
+17 ¡.19 '_.A

+15
-6
- -23
+19
-10 - -12
-16 -9
+7 7 -15
+112 '> 118
-22 +14
-1 o
+10 ~ -12
/
♦ 1 ALFONSO ROJAS PUÉMAPE 1

III. Efectúa las siguientes sustracciones:

(1) (+8) - (-5) (11) (+15)-(-15) (21) (-101)-(+102)


(2) (-7)- (+2) (12) (-12)- (+12) (22) (-101)-(-101)
(3) (-1 ) - (+1) (13) (-15)-(-15) (23) (-142)-(+ 140)
(4) (+3) - (+ 10) (14) (-12)-(-12) (24) (-215) - (-210)
(5) (-9)-(-1) (15) (+15)-(+15) (25) (-52) - (+46)
(6) (-2)-(+7) (16) (+ 12) - (+ 12) (26) (-17) - (-19)
(7) (-5)- (+ 17) (17) (-1) - (O) (27) (-29) - (-9)
(8) (-2) - (+ 19) (18) (O) - (-3) (28) (-100)-(-110)
(9) (-1)-(-12) (19) (+15) - (-13) (29) (-615) - (-317)
(10) (-19)- (-13) (20) (+ 12) - (-5) (30) (-142) - (-673)

IV . Completa el siguiente cuadro escribiendo la propiedad de la adic i6n de números enteros


aplicada:

EXPRESION PROPIEDAD EXPRESIÓN PROPIEDAD


CON ENTEROS APLICADA CON ENTEROS APLICADA

(+7) + (-1) = (-1) + (+7)


(-5) + (-3) = -8
Conmutativa [(+1) + (-1)) + (+2) = (+1) + ((-1) + (+2))
(-3) + (+8) = (+5) + (+3) + (-3)
-.~ _, Cc.T.,
.,,,_,v....
.
(-10) +O= -10 (-1) + (-10) = (-1) + (-8) + (-2)
' .
(-7) + (+7) = O (-3) + (-5) = (-9) + (+4) + (-3)
(+12) +(O)= +12 (-27) + (+27) = O l"l U A J..,.
(-8) + (-1) = ( 1) + (-8) (-2) +(O)= -2 '
' .....
.__,_ ..__
(+101) + (-101) = O (-6) + (+1) = (+1) + (-6) kM
O+ (-7) = -7 (-10) + (-16) = (-10) + (-11) + (-5)
~>vv
(-6) + (+6) = O (-13)+(+13)=0
' ' 1 ..
(-1) + (-3) = (-3) + (-1) (-23) +(O)= -23
~~
V. Hallar los números enteros a colocar en los casilleros, emplcamlo las propiedades
estudiadas:
(1) 0 +(-15)=(-15) (8) hu+ (-8) = -19
(2) ~ + (+76) = O (9) (-42) + □ = -25
(3) (-15) + □ = (-15) (10) (-63)=~+(-13)
(4) (-2) + (-8) + E3j = (-7) (11) (-75) + □ + (-13) + (-15) = -17
(5) (-10) + (-5) - (-1) + □ = (-2) (12) (+19) + (-2) +Q-(-12)=(+15)
(6) (+ 12) - (-3) + (-6) =+~ + (-9) (13) (-11)+(-6)-(-9)=0+(-11)
(7) (-1) + (-2) + □ = (-17) (14) (-29) - (-7) ~@ = (-12) + (-6)

-··-·-··.·.·-·-·-·· -·-·-•-·..•. ·-·-·-·--··"';';•:"!" .......•·.•-···•:•.-, .. _._. ··-------··-·-


1 l\lATEMATICA
1

VI. El resultado de sumar: ( + 7) + (+ 5) e,;; + 12


Suma que también se escribe así:+ 7 + 5 = 12 lo cual es más práctico; del mismo modo:
* -7 - 5 = -12.¡,Por qué?
* -7 + 5 = -2.¡_Pnr qué?
* + 7 - 5 = +2.¿Por qué,'!
Según esto efectuar las siguientes operaciones combinadas:

(1) -1-2-3 (23) -17 + (45-19)


(2) -6-7-8-9 (24j -225- (48 - 22- 15) + (-17 + 2)
(3) -9-10-11-12 (25) +8 - 3 + (17 - 42) - (51 - 7 - 8)
(4) -1+3-5+3 (26) -6 + {-5-(48-17-1)-6}
(5) +6 + 7 + 8- 2 (27) -2 -{-3- [+6 + 8 + (-3- 7 - 1)) + 2}
(6) +2 - 3 + 5 + 3 - 8 - 9 - 1 (28) -10 - [-5 + (8 - 6 - 7 + 1)) + (-73 - 8)
(7) -5-3-1 +2+ 1-8-6 (29) -46- {-1 + (-17 + (-6 - 9 - 1)1}
(8) -19-11 + 13-15+ 16 (30) +5 + 7 - 1 + (-62 - 4 + 68) - (-17 + 1 - 6)
(9) -31 .. 33 - 5 + 8 - 19 (31) -15 + {-61 - 55- [-17 -(-29 + 1 + 3))- 3}
(10) +17-15+ 13+ 15+8 (32) -62 - {-17 - 6 - [-1 + (6 - 9 - 11) - 1)}
(11) -6 + 18 - 25 - 17 + 16 (33) -18 + l-9 - 6 - 7 - (6 - 7 - 8 - 1O) - 1]
(12) -25 + 32 + 55 - 59 + 32 (34) {-62 - [17 + 8 - 29) - (63 - 75)}
(13) -37 - 48 - 17 + 19 + 45 (35) -16 -{-16 + 16- [16 + 16-16) + 16}
(14) -17 - 46 + 25 - 18 - 17 (36) -9 - {9 - 9 + 9 - 9 - (-9 + 9 - 9 - 9) - 9}
(15) 65+3-19-28-52 (37) -16 -{+8 - 5- [16- 46- 8 - (-15-1)- 61}
(16) -43 - 72 - 115 + 223 (38) -6 ~ 9 + {6 -17- [65- 13- (-16 + 8) - 11}
(17) -16 + 15- 19- 25- 142 (39) -42 - 55 - { +8 - 5 - [16 - 46 - 8 - (-15 - 1) - 61}
(18) -117+ 185-242-315 (40) -69- 17 + {-19- 6 -(17 + 18 - 46) + (51 - 76)}
(19) -6 - 15- 13 + 2 - 55 (41) -8 + 12 - {-4 - (-6 + 1 - (-6 + 3 - 2)1}
(20) -62 + 57 - 13 + 2 - 99 + 56 (42) -16+15+(15-6-29)-(17-46-1)
(21) -13 + 2 - 19 + 29 --58 (43) +78-715-{-919+(617-815-1)-(-171 +2)}
¡22) -1 + 19-27 +56-17-2 (44) -16-{-48 - (-57 + ( 15- 16-17)- \ 69 - 48))- 1}

l:='=·=
-
RESPUESTAS
(1) -6 (8) -16 (15) -31 (22) +28 (29) -12 (36) -18 (43)+312
(2) -30 (9) -14 (16) -7 (23) +9 (30) +35 (37) - 47 (44)...- -63
(3) -42 (10) +38 (17)-187 (24) -251 (31)-142 (38) -85
(4) o (11) -14 (18)-489 (25) -56 (32) -55 (39) -128
(5) +19 (12) +35 (19) -87 (26) -47 (33) -22 (40) -125
(6) -11 (13) -38 (20) -59 (27) +2 (34) -46 (41) +8
(7) -20 (14) -73 (21) -59 (28) -82 (35) -16 (42) +9
1 ALFONSO ROJAS PUÉMAPE 1

Aquí mis opl'racioncs de SKANIPRÁCTICA 2


MATEMATICA
1

( 3. MULTIPUCACIÓN Y POTENCIACIÓN EN Z)
I

·r f
rl:,,;.--
!?
l··0-~ffi
, '"'Q·
,. \
l \
'J
1

----- - --
En cada uno de los 5
laréos de telas he per-
d,éo
puedo Sexpresar
c,da total?
75. ,como
la per-

Las perdidas se cons,- · ~


derar, negativas, enton- _,.,.--,
"' ,..,... , . ,-15, r -
!r:~?=-·. .
·)
~
~ •
,
;\ L JJ~ :-\ ~ EJ'I

,-4
i·l
1 .el

~=================·~-..
J. I \ILLTIPLICACIC), DE 1'Ú:\IEROS E'\TEROS

5 x (- 75) equivak a escribir: (-75) + (-75) + (-75) + (-75) + (-75)

5 veces
Por lo que hemos estudiado hasta aquí. esta ADI-
CIÓN de sumandos iguales tiene como SUMA: - 375
otamhién: 5 X (-75) = -375

Es decir: Si multiplicamos dos enteros de diferente signo. el signo del resultado es NEGATIVO.
En general, podemos est.ahlcc:er la siguieIHc ...

* ... REGLA DE SIGNOS PARA LA l\ll'. LTI PLI CACI<>N DE NÚ I\I EROS ENT EROS:

• Si dos números enteros tienen el MISMO SIGNO su producto tendrá S IG NO POSITIVO


• Si dos numeros enteros tienen DISTINTO SIGNO. su producto tendrá SIGNO NEGATIVO.

Así: ( 1) (+5)x(+3)=+15
(2) (-5) x (-3)=+15
(3) (-1) " (+8_) = -56
n
,..---
-T'v
-/. \ ' ,

...
Como veras pr,mero nemas
mul\11)1,caoo ~ ~
tos de los números da<los y
luego hemos aplicado la RE
(-l) (+6),., (-5) = -30
,,,
=i=r ~ ;
GLA DE SIGNOS.
De la REGLA DE SIGNOS para la multiplicación se
desprende lo siguiente al mulliplicar dos o más factori.::s: O IMPORTANTE

Si todos los factores tienen SIGNO POSITIVO, el Una MULTIPLJCACIÓN


PRODUCTO tmnbién es positivo. como: (+4)><(+7)• +28
t t
Ejemplos: Factores
j
(1) (+3) (+2)(+5) = +10
También puede ser escr,ta
(2) (+4)(+7)(+1)(+2)=+56 C:::: O
así:
(+4)(+7) • +28
Si algunos de los factore s son de SIGNO NEGATIVO
tendremos en cuenta la cantidad de estos factores: · -:-.-:-:-;-:-:-:-:-:-;-:,:-:-:-.-:-:-:--.-:-:-.-::-:·· . ·:-:•

(a) Si dicha cantidad de factores es PAR, el producto


total es de signo POSITIVO.
Ejemplos:

ll > (-5)(-2><-3) (-1 >= +30


(2) (+7)(-1)(+4)(+2)(-2¡ = +112

(h) Si dicha cantidad de factores es IMPAR, el pro-


ducto tcllal es de signo NEGATIVO.
Ejemplos:

(1) (-2)(-3)(-4) = -24


(2) ( +5)( +3)(-1)(+2) = -30 00 ¡ATENCIÓN'
Una mult,plicac,ón puede
N" de factores m:ga1ivos: 1 ¡IMPAR! interpretarse también como
NQ de factores positivos: ¡ No importa cuámos sean! "· una SUMA ABREVIADA.
así:
(3) Efectuar: a+a+ . +a=nxa
P = 1(-2) + (-2) + ... + (-2)1 (-1)(-3) n yeces
16 veces

Solución:
• En el interior del corchete hay una
suma qul' puede abre\' iarse por medio
de la multiplicación: C:::: 00 ◊
• Como hay tres factores negativos el
producto tamhién es negativo: ◊ P=-16x2 x 1 >-- 3
1P = -961 Rpta.
1 MATEMATICA
1
.t2 PROPIEDADES DE LA MULTIPLICACIÚN DE NÚMEROS
ENTEROS

l. PROPIEDAD DE CLAUSURA.-
"EI resultado de multiplicar dos números enteros es otro número también
entero". a; b e Z entonces ah e Z
Ejemplo:
( -2 ) E "7L. y ( + 7) E "7L.
er.~onces (-2)(+7)=-14 donde(-14)e Z

2. PROPIEDAD CONMUTATIVA.-
"El orden de los factores, no altera el producto". ah = ba
Ejemplo:
(-24) (+3) = (+3)(-24)
-72=-72

3. PROPIEDAD ASOCIATIVA.-
"La forma como se agrupen los factores, no altera el producto". (ah) e= a (be).
Ejemplo:
( - 5 ) ( + 2 )( - 3 ) = + 30
[(-5)( +2)] (-3) = (-5) [( +2)(-3)]

(-lOJ(-3) =(-5)(-6)
+ 30 = + 30

4. ELEMENTO NEUTRO.-
Es+ 1.
Cualquier número entero multiplicado por el ELEMENTO NEUTRO da como
producto el mismo número entero. a x 1 = a pam todo a e Z
Ejemplo:
(-7)(+1) = - 7

5. ELEMENTO ABSORVENTE.-
Esel CERO.
En cualquier multiplicación de dos o más factores, si al menos UNO DE
ELLOS es CERO. entonces el producto es CERO. a x O= O para todo a e Z

Ejemplo:
(-717)(+215)(-3)(-2)(0)(-3) = O
1 ALFONSO ROJAS PUÉMAPE 1

6. PROPIEDAD DE MONOTONÍA.-
Si multiplicamos ambos miembros de una igualdad por un mismo número
entero, obtenemos otra igualdad. Si a = b entonces ac = be.
Ejempló:
(-2)(-7) =+ 14 -

Múltípliéamci_s ambos miembros pot (+3):


(~2) (-7) (+3) '= (+14) (+3)
+42 = +42

, 7. PROPIEDADCANCELATIVA.-
Si en ambos miembros de una igualdad, aparece un mismo número entero
diferente de cero como factor, éste puede cancelarse o suprimirse.
Si ab = ac y a -:t:- O entonces b = c.
Ejemplo:
(-8)(+2)(-3) = (-4)(+4)(-3)
(-8)(+2) = (-4)(+4)
- 16=-16

·=-· 8. PROPlEDAIJ UlSTRlHUTlVÁ.-


a
_Si un número entero multiplica una ADICIÓN, resulta la suma de los
prodúctos de cÚcho número enteró por cada uno de los ~u mandos.
1

Ejemplo:

Efc;ctu~)ndo en ambos rniémbros para comprobar:


. °(-2)[-21::; (tlO) + (~6)
. +4=+4 . . .
·nngcncrnl:

o también:

Éjcmplo:
Efectuar:
Un füctor que i.ipilrcc~ en amba" ~ uitíplícac iones ind icadus es (-6), 1uego este
faGtor recibe el no1:nbrc dcFACTO:R COM0N; ...
Ei'llünccs: E =(a6)((~4) + (+5)1
=
E (-6)1 +11 E = -6
MATIMATICA

SKA:\IPRACTIC,\ (J)
l. Compklar d siguicmc cuadro c kcluando las muhiplicaciones indicadas:

X 1 -➔ 1 +3 1 -5 1 -2 1 + 11 1 - 13 1 -8 1 +6 1 -1 () 1
,
-- ~e -e .,. J (' t'-\ -~ '"l. "' 'J 't \ r. - ,\ o. .--,i e

,s ~s
~· .
~
' -t55 l, <, ... ¼. o Se..
+5 -1.c (.;
1 e,

:7· ~. <.
-t-3,P t- J '( -11 "T,
1 -,..cq> - ~ ~ -\ 7 o
+8 'l ? ':, 1; - ... _ J (:; !'-
~
l ( \\ ·- ~ l, {- l f -~ ('
,
-9 r ~

, J
-<¡ 1 I_ fl
-t , - '1 q +. J1 {- 'J -,of \ n ()

-10 ~'-\e ·1,_, f5c "f 1, 1 t"' ~o ' p., -{.,, "t- e (.

- L~ - ~ e; 1. \'1 -1- G s _.. '- b -i ~J '( ... "G o4 1~ 'f- Je


.11 ·i- '-4 '-\ -3) ,e f.") ~ -- • :, I J- 4. -f 8t> t. (, lt
1' o
+15 - t~s - 1-
(. e, ;z
fJ -~ - (' t; - { q()
' 'c..
-1 7 ·t" e;. e- -s ,l r1°} ~ tj'I, . J ~ r ti 9-' 4J ~" •t oi -.... , 1 e
11. Completar el siguiente cuadro escribiendo las propiedades de la multipl icación de
números enteros aplicadas en cada expresión dada:

EXPRESIÓN PROPIEDAD
l_ EXPRESIÓN
--
PROPIEDAD

(-2)( +5) = (+5)(-2) 7 ,kl;;l,,'f' (-3)(-1)(+2) = (-3)(-1 )(+2) = +6 "1-(r t ' ~~


(-2)(+1) = -2
;,· .... ,(.
-.. \·8)1-9)(+2) = (-9)(+16)1-1) • .., ""':,.r
,-

(-52)(-8) = (-8)(-52) _ ~ . w.; (-7)(+1 ) =-7 .-ntL ,..JW


(-7)(0)(-2)( +3) = O r":c ,/{,, .-.A ...
), {+3)(0)(-8)(+10)(-3) = O
~~
(- 10){+3) = -30
- U-<. >t u.. (-10(-6)( +2) = (-12)(-10) ~-u---
(-27)(+ 1) = -27 ,ia, 1 ,., (-2)[( +1)+(-3)) = (-2)( +1)+(-2)( -3) ....... 1 / ' , ~

(0)(-1)( +1)(-2) = O r _. .r (+3)(-5)+(+3)(-2) = (-3)((-5)+(-2))


'
l.- ' •

(-3)(-2)(-4) = {+81(-3) {-5){+2)(0)(-3)(0) = O e,(;. '-" -1., 1,,-C


~?C.,,,t"\.-é~

(+42)(-52) = (-52)( 42) (-8)(+ 1) = -8 .,,~,t"'V


J ri-J.4J·
(-12)1+ 13)!-1 5)(-17)(0) = O r,-_b, • ~,__A,.._;: (-101(-2) = (+51(-2)(-2)
•c.., • V

,,
- ..,,,._,...,J-.
(+1)(-1 215) = 1 215 (-6)(+3) = (+3)(-6)
. .__I, .
(-10)(+20) =(-5)(-10)(-4) (-5)(0)(-5) = O
,?l{Y1'( ,kn.,, okv'>tJ--- I
1 AlF01'S0 ROJAS PUEMAPE 1

111. Efcc1uar:

(1 l (-2)(-3)(-4) (.35) (+5)(+ 1)(-7)(-3)(+2)(-3)


(2) (-5)(+2)(-6) 1.36) (-3)(-3)(+2)(-2)(-4i(+ I 1
(J) (-7)(+1)(-2) 1.HJ ( +4 )( -3 )(+ 1)l - 1 )( -3 J( + 2)
1-U (-2)(-2)(-2)(-2) (."\8) (-2)(+ 1)(-3)(+5)(-3)(-11
(5) (-6)(+3)(- l )(-3) (.WI ( +5 )(-2)(-1 )(-3)( +2)(-4)
(6) (+8)(+2)(+3)(-1) 1-'0) (-7)(0)(-2)(-3)(0)(0)
(7) (-3)(-1)(-2)(-3) (-')) (-17)(+ 3 )( - 1)( + 1)( -2)
un (-175)( +372)(-57)(0) H.2) (-6)(-2)(-1){+ 1)(-3)(+2)
(9) (-1)(-2)(-3)(-4) (-'3) (-1)(-2)(-3)(+1){+2)(+3)
( 10) (-8)(-2)(-5)(+2) ( "'"') (-5)(-2)(-10)(+5)(-1)
( 11 l (-1 )(-1 )(-1 )(-1 ) (-'5) (-3)(+5)(-2)(-12)(-11)
(1.2) (- 1)( - 1)( - 1)( + 1 )
el .3) (-1 J(-1 1(-1 )(OJ(-1)
( "''·) (-112)(-3)(-2.,)(+2)
1-'71 ( +225 )(-3)(-35)(-5)(-2)
0-'l (+7) (-5)(+2)(-3) 1-'8) (- 135)( + 32)(-437)(-1)
(15) (+8)(-ó)(- I )(-2) (-'9) (-432)( -24)( + 3)(-2)
(16) (-1)(+1)(-1)(+1) 150) (-3 l42)(+17)(-2)(-1)(+1)
(17) (-2)(-2)(-2)(-2)
(181 (-2)(+2)(+2)(-2)
(ll)) (+2)(+2)(-2)(-2) ES IMPORTANTE TENER PRESENTE LO
(.20) (+2)(+2)(+2)(-2) ;. SIGUIENTE:
::::··
(211 (-2) (-1)(-l)(+Hl) La ley de signos de la ADICIÓN NO es la
(22) (-1 )(-1)(-1)(+ HI) misma que de la MULTIPLICACIÓN de
(2.3) (-2)(-2)(-:!)(-2)(0) números enteros:
(2-'l (-6)(-11(-1)(+2) EN LA ENLA
(25) (-2)(-2)(-2)(-2)(0) ADICIÓN· MULTIPLICACIÓN.
(26) (-3)(-4)(-5)(-Ó) (-0)+(-0)= - 0 (+0)(+0)=+0
(+Ol+l+Ol=+O !-OH- Ol=+O
(27) (+5)(-5)(+5)(-5)
(+Ol+Ol=} O l+Ol(- Ol= - O
(:?8)
(.291
(-2)( + 1)(-:; )(+4 l
(-2)(+3)(- I )(+ 1)(-1 )(-3) (-Ol+(+Ql = lº
(- 0)(+0)= - O
(JO 1 ( +3)(-8)(-10)( -25)(-72)(0)(-3) Esle s,gno del resullado
es el del sumando do
l .H 1 (-1 J( - 1 1( - 1)( - 1 )( - 1)( - 1)( -1 )( - 1)( -1 ) ' MAYOR VALOR ABSO
\:
(."H) (-2)(+3)(-7)(+ 1 )(-1 )(-2)(+ 1 J LUTO

(3.3) (+2)(+2)(+2)(+2)(+2)(-2)
13-'1 (-3 )(-5)(-2)( + 1)( +2)
IM'MJJl~----~-~--- ~---~~ -----MA_T_E_MA_T_IC_A____I
IV. En operaciones combinadas de ADICIONES y MULTIPLICACIONES empezamos a
operar por estas últimas.
Ejemplo:
Efectuar: (-2X+3) + (-7)(-2)

Primero las multiplicaciones: (-6) + (+14)

Luego la adición: +8

Según esto efectuar los siguientes ejercicios:

(1) (-1)(+5) + (-3)(-2) (21) (-2)(+3) + (-1)(-2) +(-5)(+6)


(2) (-6)(+4) + (+ 1)(+5) (22) (-4)(+3) + (-5)(-6)- (-1)(+2)
(3) (-7)(+2)+(-3)(+1) (23) (-1)(+2)(-5) +(-2)(-3)-(+3)(-8)
(4) (+ 1)(-8) + (-2)(-4) (24) (-3)(-5) + (-3)(-5)(-7) - 8
(5) (-5)(-6) - (-5)(+2) (25) (+7)(-2)- (-1)(+3)-(-6)(-9)(-1)
(6) (+2)(-1) - (-10)(+2) (26) (-10)(+3)(-5)-(-2)(+3)
(7) (-6)(-2)(+ 1) + (-2) (27) (-6 )(-2)(-3) + (-1 )(-9)
(8) (-12)(-13) (-1)-(-68) (28) (-8)(-3) - (-10)(+6)(-2)
(9) (-6)(-2)(+2) + (-4) (29) (-9)(-20) + (-5)(-2)(-3) - 4
(10) (-3)(-8) + (-1)(+2)1·3) (30) (-7)(-100) + (-2)(-50) - 6
(11) (-2)(+3)(-8)+(-14) (31) (-2)(-6) + (-6)(+8) - (-12)(+3)
(12) (-4)(-6)(+3)(-5) - 3 (32) (-1)(+1)(-2)"-(+7)(+3)(-1)-(+2)(-5)
(13) (-10)(-100) + (-15)(+18) (33) (-3)(-6)(+ 1) +(+4)(-2) - (-7)(+3)
(14) (-14)(-13) +(-6)(-8) (34) (-2)(+5) + (-4)(-5) + (-6)¡-2)(+ 1)
(15) (-6)(+2)(+ 1)- (-3) (35) (+4)(-2)(+5)- (+2)(-6)- 10
(16) (+ 12)(-2) - (-5)(-3) (36) (-8)(+2) + (-3)(-2) + 1-5)(-1)- (-2)(+3)
(17) (+15)(-4) + (-3)(-10) (37) (+5)(-2)(-1) - (-1)(-3)-\·2/\+5)(-1)
(18) (-12)(-8)-(-5)(-1)(-6) (38) (-2)(-7)(-1 )( +1)(-3) + (-3)(-2)( +1)(-6)
(19) (-15)(+4)-1 (39) (-1 )(-2)(-3)( -4) - (+ 1)(+2)(+3)(+4)
(20) (+20)(-2) + (-4)(+3) (40) (-3)(+ 1)(+2)(-1)(-4)(-7)-100

Y. Completar en los recuadros los números enteros que fülian para que se verifique la
igualdad:

(1) (-1)(+7) + ~ = -5 (6) (-17) + (0 3)'(-8) +El] = (-2)(-6) + (-17) + 10

(2) (-9)(-2) - IS] = +21 (7) (-1)(+1)(-5) +f ü = (-3)(-1) +(-2)

(3) (-1oJ+(-11(+3J=f i l -18 (8) (-7)(-2) + (+41(-3) =@ + (-3)(-5)

(4) <-6)(+3) + K1 = -24 (9) (-5)( ~ J +(-7)(-2) = (-6)(+3) +(+22)

(5) (-3)(+2) =f ü - 5 (10) (-48) + (-2)( ~ ) · (-1)(+5) = (+37)(-1)


1'
~
1 ,UFOM,O ROJAS PUÉMAPE 1

Aquí 1111s op~racioncs de SKANIPRÁCTICA (])


111:'Mhfl 1 MATEMATICA
1

Es una operación en la que dada una BASE ENTERA


(número cillero) y un c,ponente natural (n}, hallamos la O ,CUIDAD01
POTENCIA P. C: O
POTENCIA no es lo mismo
que EXPONENTE.
Así: POTENCIA es el resultado de
laoperac,on POTENCIACION
El EXPONENTE NATURAL n, indica la cantidad de veces
que se repite la base entera a como factor, es decir:

aª = a x a "a x .•..• x a
nvecesa

Ejemplos:
Electuar:
( 1) (+3) 2 =(+3)(+3) ◊ (+3f =+9
2 veces
(2) (+3)3 = (+3){+3)(+3) ◊ (+3) 3 = +27

3 veces

(3) (-W = (-3)(-3)


..______,_.._.. ◊ (-3)2 =+9

2 VCCl!S
(-3)3 =-27
(4) (-3)3 = (-3)(-3)(-3)

3 veces

JA su;:\oS UE POTF:\CL\CI{):'\ E:\ z 00 ¡IMPORTANTE!


Numero par es aquel número
De los cuatro últimos ejemplos plxlcmos deducir la siguiente entero que al dividirloentre 2, da
REGLA DE SIGNOS PARA POTENCIACIÓN en Z : como COCIENTE exacto otro
numero entero.
(+a)"Mo1~1PAH = +P
(-a)PAH =+P
(-a)'MPAH = _p
ººº
Nota.- Si a = 1 entonces P = 1 para cuak¡uier exponl!llle natural.
Ejemplos:
(1) (+2)S =+32 (4) (-1)103 =-l (7) (-4)2 = + 16
(2) (-4) 3 = -64 (5) (-5)3 =-125 (8) (-9)2 = +81
(3) (- l)ti4 = + 1 (6) (-10) 2 =+ 100 (9) (+3)4 = +81
1 ALFO~SO ROJAS PUEMAPE 1 lfr3Miúi
].5 :\ll"LTIPLICACH):\ DE POTEI\CIAS DE BASES
El\TERAS IGU \LES
Veamos un ejemplo:
Ercnuar: (-3)4 X (-3)2
4 veces 2 \ICL"C~

• Si desarrollamos cada una


(- 3)(- 3)(- 3)(- 3)x (- 3)(- 3)
de las potencias indicadas
tendremos: 6 vece,

• Como verás, ahora el factor


(-3) ,;e repite 6 veces o
también: (-3)4 X (-3) 2 = (-3)6
O ¡MUCHO CUIDADO'
• ¡,Qué (Kurrió con los expo- (-3)4 "F- -34
nentes 4 y 2? . . .Se sumaron
En el lado izquierdo el ex-
¡,verdad'! ahora el ex ponen- ponente 4 afecta a la
te de la misma hase es 6. base-3.
En el lado derecho el expo-
Luego:
nente 4 afecta sólo a la
base 3.
Para multiplicar POTENCIAS de BASES
enteras IGUALES. se esc:rihc la misma base
y ~e SUMAN LOS EXPONENTES.

En General:

J.6
Veamos un ejemplo.¡,Sí'!
Efectuar 1-.W: (-5) 3 00 ¡ IMPORTANTE!
7
(- 5) Las s,gwentes d1v1-
Esta división también puede ser ---, siones son equiva-
escrita así: ( - 5)
lentes
(- 5)(-5) (-5) (- 5)(-5) (- 5)(-5)
Desarrollemos las potencias: (axm) . (b x m)
(-5) (- :5) (- 5) a:b
• Suprimicndo factores comunes: (-5)(-5)(-5)(-5) = (-5}4 C: 00 Escr1to de otro modo·
axm a
4 veces --=-
7 b xm b
Es decir: ( - 5) ..
---, = (- 5)
(- S)°
• ¡,Qué ocurrió con los exponentes 7 y 3? ... se restaron ¡,ventad?
Ahora el exponente de la misma base es .¡_
1 MATEMÁTICA
1
Luego:
Para dividir POTENCIAS de BASES enteras
IGUALES, cscrihimos la misma hase y RESTA-
l'vlOS los cxponcmes.

En General : m
!!..._ == am - n /a ct-. O, a E Z
n m, n, E [N
a

Recuerda que m y n son números naturalc,.


Acompáñame a ver un CASO PARTICULAR(k la DI-
VISIÓN de hascs enteras igualcs:
m
Si m =n tendremos a
-;;¡-=a
m - m

1 =aº C::: o
aº =I 1 / a ct-. O
¡_¡_·: si misma es igual a la
unidad.
Esto significa que: "Toda hase c111cra distin1~, (lc CERO t
elcvada al exponente CERO da como
res ultado I " (EXPONENTE N ULO) ttb::;;::;;;:::s,.;,:,:,=:;;:;:;:;::,,,=:¡;¡:;:;,-,==::::,=i==··.

Ejemplos: C::: 00
O) (-52)º = I (3) ( - 1 fün )'' = 1
(2) 0° = NO DEFINIDO l.¡) (-7 + 7)º = NO DEFINIDO

3.7 POTENCIA DE UNA MUL TIPLICACH>N INDICADA


DE NÚMEROS ENTEROS
11 E [N
1a x b I" = a" x b" aybE Z
¿Me permites comprobarlo en un ejemplo? En una potencia de EXPO-
NENTE NULO, donde la
Ekctuar: 1(-3)(+5)13 base también es CERO el
resultado NO ESTA' DEFI-
Soluci(m: NIDO, entonces hay que
evitar una situación como
Podríamos operar el interior del corche- ésta.
le, pero esla vez vamos por oiro lado,
desarrollando la potencia cuya base es el
corchele para después aplicar la propie-
dad asocialiva de la multiplicación.

• Desarrollando la potencia cuyo expo-


ne nte es 3 y cuya base es el con:heLC: 1(-3)(+S)ll(-3)(+5)11(-3)(+5) 1
♦ 1 ALFONSO ROJAS PUÉMAPE 1
• Por la PR< >PIEDAD ASOCIA TI VA de [(- 3) (- 3)(- 3)1 ll+ 5)(+ 5)(+ 5)1
la multiplicación:

• Por EXPONENTE NATURAL en cada


corchete: (-3) 3 (+5) 3
Es lkcir: 1(-., )(+Sll' = (- .1 )'(+5) 1 O O

3.8 POTENCIA DE UNA DIVISI()N INDICADA DE


NÚMEROS ENTEROS
nE [N
1a : h I" =a" : h" ayhE Z
o tamhi<'n: OjlMPORTANTE!
[ ;1)1]" an
= -n Esta "distribución" del
h
exponente ocurre cuan-
do en el interior del cor-
Ejemplos: chete hay una multipli-
cación o una division
indicada.
u tamhi.:n: Esto es correcto:
(a X bf "'él' X b"

Esto es ir1correcto:
(a + bf • él' + b"

3.9 POTENCIA DE POTENCIA

Iali 1m = ª u ....
oo ¡CUIDADO/

Ejemplos:
( 1) Ekctuar: 1(-S>'F
Soluci<ín:

Desarrollamlo la potcnna
cuya hase es d corchctl!:
• Aplicamos I\IULTIPLICA-
CIÓN de potencias de h.t-
scsentcras i~uaks: C:::: 00 1(-5)'F =(-5/"

• Ptll'lks ohscr\'ar qui! los


exponentes 2 y 3 quedaron
multiplicados.
1 \lATEl\L-\TICA
1
SKA:\IPl{ .-\CTICA (D
l. Completar el número que falta en el casillero correspondiente:

( 1) (-2f =
D D
(9) (+5)2= (17) (-1)13 =
D
(2) (-3) = 2
D <JO) (+1>17 = D (18) (-1 )14 =
D
(3) (-5) = 3
D (11) (-7) = 3
D (19) (-))º=
D
(4) (+ 7)2 =
D (12) (+12) 2 = □ c20) <-75)° = □
<5> <-1)715 = D (13) <-11>2 = D (21) (-7+7)°= □
(6) (+10) 3 =□ (14) (-2)4 = D (22) (-8)3 = D
(7) (-9)2 =
D (15) (-3)5 = D (23) <-1)12= D
(8) (-4) 3
= D (16) (-4) = D4
(24) c-10>1 = □

Completar los casilleros para que se verifiquen las siguientes igualdades:

(1) (-5)7(-5)2(-5)3 = (-5)1:CJ

(2) (-19) 176 (-19)m= (-19)0 (10) {((-73)'1º}42 + {[(-11nss}'ª= ( □)


10 7 177
(3) ( - 25) ( - 25 ) = ( _ 25 ) 0 (11) (- 7) (-7) □
8 97
1- 25 l (- 7)

(4) (-8)3(+4}ª(-8) 5(-2)ª = (0) □ (12) (·2)H (-18)9 (+9)15 (•18) 6 • ( □ )□


(5) (-5)2(-6)< s.s, .. □ (13) [((-621 ]6)□ r2 = (-57)7· 7
(6) [(-9)(-6})5 = (-9Px (-6)□ (14) [(-9)3 f (-1) 19 (+2)6 =. ( ~ )Gi]
9 □
(7) [' - 15)] = (- 15)
( +17) ( + 17) □

(8) {((-19)3)2}4 = (- 19) o


3. Contesta las siguientes preguntas:

( 1) ¿Cuál es la diferencia entre POTENCIA y EXPONENTE?


(2) ;.Qué signo tiene el resultado de operar (-47)4 8 ? ¿Por qué'!
(3) ¡_Por qué 0° NO ESTÁ DEFINIDO'!
(4) ¿En qué situaciones la potencia de números enteros no es negativa? ¿Por qué'!

: ... ;-...-.-.--· . : . :-:-::: .-:-:-~:-:- .-: :-·-:-: .-:-:-.-.-: -•-·-.-:-: ;._.:-:-:-:-..-. ·-.-: :-·-.. : :-:-· -.-...--:-:-: :• ·-·
1 ALFO\'SO ROJAS PUE\l.\PE 1

Aquí mi, op~rac10111:s lk SKANIPRÁCTICA @


1 MATEMATICA
1

( 4. DIVISIÓN Y RADICACIÓN EN z)
¿Recuerdas la
deuda que te- 1 creo que
nemos por la eran SI 300
compra de ' entre tú y yo.
nuestra compu-
tadora?

~ COMPU í ADORAS
-, J ~ <.. Un momento perlo , _
~ lmenosd1mecuándo •
,,-~--~ ~.,_¡. ~·•
_
- ,,·
J - ,
~~~~~:~~~~~en~:
corrr--,ionde ,--- °\
¡Huy . olvidé ._ : ( 300J 2 ~
que tenía algo -f_t .; t.-=:_-:_-_-:__-_¡-~,.______-----1
quehacer ., {

Ya ,ah..:mos q11l~ una forma de explicar el uso de los núm..:ms


l'tlll'ms "Z ~ C') consiste en asignar los mí meros positi\"OS
a las ganancias en 1111 negocio y los mímcms n..:gatinls a las
p~nlidas o ckudas. En ..:,1..: caso. la tkuda IOl~tl ..:s tk S/300:
e,tocs ---~OO. Si di\"idimos l'lllre dos pasonas_ ll'lldremos que

l
cada una DEBE S/. 1.'ill, esto l'S -150; es <kcir la opcraciún
l(Ul' h..:mos l'kc111ado e, la siguiente:

\ {
o RECUERDA QUE:
Podemos ir ohsa\"ando que cuando se divitkn mi1fü•ros
EL CONJUNTO DE NÚME-
enteros de di,tinto signo, d cocil·nte es negativo.
ROS ENTEROS Z está for-
mado por-
-ti Dl\'ISI();\ EX ACTA.
Los números eateros 1

La Dl\' JSlf>N EXACTA, es una 11peraci611 en la cual halla- NEGATIVOS


1110, un factor llamado COCIENTE (l)J , que 1111, mdica el E/cero
número <k \\.'.Ces qu..: otro bctor NO NULO d..:nominado Los números enteros
DIVISOR (dJ e,tá cu1lll'nido l' n OLro al que llamamos DI VI - POSITIVOS
DEN DO (DJ_
1 ALFONSO ROJAS PUÉ.\UPE 1
Es ,kcir:
D=d x q .... . . (1)
Ejemplo:
¡_Cuál es el mímero enLero (q) que multiplicado por (+2) da
como res u hado (-300)?

~:tr.:-::,r
O 1ATENC/ÓN'
Si empicamos (IJ tcmlrcmos:
Las reglas de signos de la mu/·
(-3(X))=(+2)xq ot.:.11nhifo: (-300):(+'.:!)=q (

~
t1pl1cac1on y la div1s1ón de núme·
Por nuestro ejemplo inicial, vimos que: q = -150 ,, ros enteros son s1m1/ares
Operando con signos ,guates

-L! 1n:c;1,,\ DE Sl(;:\:OS E:\ LA IJl\'ISl()N (+) X(+)=(+)


F'.\ &' (·l (-) = H
(+) : {+) = (+)
(·) : (·) = (+)
(+): (+) = (+J
(-) : (-) = (+) Operando con signos distintos

"Al dividir números cnleros del mismo signo, el cocicnle (+) X(·) = (·)
(·) . (+) = (·)
obtenido es de SIGNO POSITIVO".
(+) . (·) = (·)
(+): (-) = (-) (·) : (+ ) = (·)
(-) : (+) = (-)

"Al dividir números ent.:ru:-. d.: di:-.LinLo signo, el cocieme


ohLenido es de SIGNO NEGATIVO" C O

Obserrnciún:
Si d no está cu111e1mlo un número exaclu de veces en D, la división es l:\EXACTA;
en tal caso aparcl:c un residuo o rc"'Lo R.
Por ejempl,,: Si 14 trompos se rcpanen enlre 3 muchachos, a cada uno sólo le
puc·d,·n Locar 4 1rompos y sohrarían 2.
Esto lo expresamos así: 14 = 3 x 4 + 2 RESUMIENDO
_ , 1 Divisií,n l11exac1a
0
1_
1) - ' 1 x q + 1" _ por DEFECTO
• D,v,sión Exacta
o l) ~
I{ q O= d, q
Veamos olm e_jemplo: \ecesilamos guardar XO hotellas de gaseosa encajone~ de • D1v1s1on Inexacta
24 cada uno. ¿Cuántos cajones ncccsita,nos'! por Defecto
Si usamos 3 cajones guardaríamos 24 x 3 = 72 hotellas, ¡se per,lcrían
X hotella,! D=d-q+R
l'or lo cual debemos Lomar 1 cajón más. • D,v,s,on Inexacta
S1 la <livis1ón fuese por DEFECTO, escrihiríamos así :

por Exceso
:-o = 24 "3 + X
D = d . q' - R'
Pero co,no el cociente ya no e~ 3 si no -1,cntonccs tcndrc,nos :
'-0=~4 " 4 - 16
ó_ _D=dxq·- Division lnc'.'lacla
..._ _ _ _ _r·_, por EXCESO
donde q· = q .. 1

En el ejemplo dado: 4 =J .. 1
iii:'M!:iitl 1 MATEMATICA
1
-1.3 PROPIEDADES DE LA DIVISIÓN EXACTA O o
Veamos las 111ás imponantl's: O ¡CUIDADO!
1º. ELEMENTO ABSORBENTE.- Es el CERO como La DIVISIÓN ENZ:
dividendo. • No es Conmutativa
"El cociente de dividir CERO entre cualquier número (a:b)"'" (b:a)
NO NULO siempre es CERO".
• No es Asociativa
Ejemplo: O: ( + 5 ) =O
(a:b):c"'" a:(b:c)
En general: 1 O: a = O 1 /a"# O

-, o ELEl\1 ENTO NEl ITRO.- Es el UNO como divisor.


" El cociente de di\'idir cualquier número entero enLrc
UNO es el mismo 111í111cro entero".
Ejemplo: (-7) : ( 1) = -7
En !!Cncral: 1 a : 1= a

., . . PROl'IEIJ .\D DISTRJHl 'TIVA.-


"EI cociente de dividir una suma indicada de varios
números enteros entre un divisor no nulo es igual a la
suma de los cocientes de cada sumando entre el mismo
divisor".
Ejemplo:
1(- 10)+(-8 )-(-4) ]:(-2)=(- l 0):(-2)+(-8 ):(-2)-(---l ):(-2)

En general:
00 ¡ ATENC/ON !
1 (a+ b + c) : (d) =a : d + b: d +e: d La PROPIEDAD DISTRI-
BUTIVA también puede ser
4'º. ALTERACIÓN DEL DIVIDENDO.- aplicada en sentido inverso.
"Si al di\'idcndo (O) lo multiplicamos o lo dividimos Esdec,r:
por cualquier número entero sin alterar el di\'isor, el Cl'-
a ·d+b:d = (a+b) : d
ci..-rlll' (ql t.amhién quedará multiplicado o di\'idido por
d icho número entero··. Ejemplo:
(-40):(-4)-1-(-1-36) (·4)=
EJemplo: =[(-40) + ( +36)]: ( ·4)
(- 1O¡ : ( +2) = -5 que es el cocienLe q .
Multipliquemm, el dividendo por (+3):
(-10) (+3) : (+2J
di vi:-.i lÍn que tamhil' n se escribe así:
(- 30) : (+2)=-15
¡el cociente q de la drvrs iún inicial quedó multiplicado por (+3).
•$ 1 ALFONSO ROJAS PUÉMAJ>E 1
5 . ALTERACH>N DEL DIVISOR.-
"Sí al divisor, lo multíplic.:amos o dividimos por un
número entero. distinto ckc.:ero. sin alterar el dívickndo, RESUMIENDO
el cociente quedará dividido en el pri1fü•r c.:aso o multi-
plicado en el segundo caso por el mismo número". Si al dividendo lo mu/ti•
f
Ejemplo: ,::· plicamos por K el co·
t ciente queda multip/i•
(-36¡: (-9) = +4
l\1uhipliquemos el divisor por (-2): $
El coaente m1oal
quedó dividido
entre (·2) 1
f
cadoporK.

Si al Divisor lo multip/i•
{-36) : (-9)(-2) l'.<J ~ r
*- camospork, e/Cocien•
(-36): {+18) = -2 .., ' ' } ( te queda dividido por
~

¡,Qué ocurrió con el c.:ocíente'! __/ K.


:.:::.::

~l{ \ =;::~::-;::•:::=.::--::~-~=~======~====~7.=;::=:=:~::::===·=·····

6''·.- ALTERACI<>N DE DIVIDENDO Y DIVISOR.-


"Si al dividendo y divisor se le mulliplica (o divide) por
un mismo NÚMERO ENTERO NO NULO. el CO-
CIENTE NO SE ALTERA" . ~ -- O
Ejemplos:
O IMPORTANTE
(1) (-48): (+16) = -3
Multiplíqm.·mos DIVIDENDO y DIVISOR rcspcctíva-
mellle por (-2):
(-48)(-2): (+16)(-2)
{+96): (-32) = -3 ¡el co.:íc111e no se altera!
0:d x q
(2) (-36): (-4) = +9 Asi:
Dividamos el DIVIDENDO y el DIVISOR respcc.:tí,·a- (+8) : (·2)" .4
menLe por (+2): porque (·2)(·4) " +8
Si la división fuera:
(-18): {-2) = +9 ¡el cociente tampoco se allera!
0:0
Obsen·ac iún: fi= entonces CUALQUIER
t= NÚMERO multiplicado por el
En u11a DIVISIÓN INEXACTA. si al di\ idl'ndo y al J=i_l. divisorCEROnos da CERO.
divisor los multiplicamos {o dividimos) por un mismo , Poreso:
NÚMERO ENTERO distinto de c.:cro. d COCIENTE ii: 0 : 0 No está definido
NO SE ALTERA pero. ...... EL RESIDUO QUEDA I; ¡HAY QUE EVITARLO!
MULTIPLICA DO (o di,·idído) por dicho número ·t:
?:::::.
cmero.

Ejemplo:
En la dí\'ísíón I 8-l: 31 el cociente es 5
y el residuo es 29.
1 MATEMÁTICA
1

esckcir: 18-l=3I - 5+29 ~ O


() 18-t ll!__ O RECUERDA QUE ·
29 5
\ . En una DIVISIÓN INEXACTA
t-.lulLipliqul·mos ahora Dh·icll'ndo y l>i\"isor por 2: se cumple que:
18-t ,<?:31 >< 2 368 : 62

ahora el COCIENTE sigue siendo 5 ¡ NO SE ALTERA!


y el RESIDUO es 58 ¡Es el residuo amerior mulLiplica-
do por 2!

4.4 RADICACIÓN EN Z
Es una operacicín in\'Crsa de la potenciación.
Dada una expresión como r" = a ; la POTENCIACIÓN nos
pcrmiLc hallar a d:ulos r y n; la RADICACIÓN nos pcrmiLc
hallar r dados a y n.

Va =r --·♦ r"=a

donde: r es la RAÍZ; r E Z
a es el RADICANDO: a E Z
n es d ÍNDICE: n E IN . n ~ 2 00 ¡ATENCION/

V cs el 01x-r.1dor RADICAL imP"! , -


V+ a +r
Ejemplo: ~ = - 2 porque (-2) 3
= -8
un~ = -r
Observaciones.-
'y:;:-; = +r
(1) El símbolo ya para a ~ O expresa un número entero
positivo r tal que r2 = a.

Ejemplo:

V16 = 4, donde 4 es llamada raíz cuadrada principal


o raíz aritmética de 16.

(2) Si n es par y a< O entonces Va no está definida en Z .


1 ALFONSO ROJAS PUÉMAPE 1 iiiMMIY¡fj
Es decir:
Así :
P~ no está definida en Z .
V-25: no l'XÍsll' 1111 nú11ll'm enll·rn r, que eh:vado
(
o i ATENCION ! l
Podemos comprobar
al cuadr~tdo d~ como resultado -25 .
la raíz cuadrada que
hemos calculado así:
4.5 MODO DE HALLAR L,\ RAÍZ CUADRADA
(271)2 + 17= 73458
DE UJ\T NÚMERO ENTERO ~ - 73441 + 17= 73458
1
Veamos el pmcedimil'nto l'll 1111 ejl'mplo:
¡cs._~-(_- 1\ 73 458 = 73 458
'--lt'(J-->
E:x1raa la raíz cuadrada de 73 45X ~1
Sol11ciún:
,,
'\iN
S ...•p;1ran1os el 11tí1ncrucn grup(.,, de dos l:ifra~ ,1 p;tnar de la dcn.·,JM. :\o 71-l5S :!
1111p1,.,n.a s1 en la 11q111 L· nb queda 'iúlo una c1fr:.1. para lue!!P l:.1111c-ar la nu,. . .¡
2x:! =--l
<.,uadrnd.1 ,na, i:erG1na dd pnrner grup, 1 de bt 11quh.: nla . la 4uc lue go dl· _l 1'-l
dl.'\ ~u ~11 cuadrado '-l" re,1.1 de d1l:hu prin1cr gn11l\,: .¡ □• □ =
SL" b.tja el ~•g.uil·nll.' grupo (.~4) al Clhl ,ulo de la dil e n·nt·i~1 (3) anh.-:n1.,r 1

,l·p.ir;nhio la tih1111 .1 l ifrn de l"!-.IC ntinu·ru qut·Lbrufu a b 1.1quu:nb JJ:


,1 la ,:e, dupli'- ;nll•l' b pri1ncrn ra1.1.) c,cnhnth.tS C!',,('-• ..a .11 lad,, d'-· un 7 3 -15 8 2
CI ¡\1)1{ \1)( ro. ¡:¡ minll'n> l(lll' ,k-hiér:nnos ""·rihir l ' II ¡,,, d .., CI ' ,\- --1 2x 2 =4
1)1{ .\J)I ·1()S lt t l.UIIL".Ullt.l' c.Ii, 1di\.·1u.l,, J~\ l{ll\." h.1h íJ 1110, , cr~•r;uit). l'lll rc 33'4
-1 Id ,l,,hk de la rni ✓/- -1[~H8= 18-t
En e,lc ~a,o .11 : -:1 da:,,;, Ulrtlo n>\.:11,,:nlc, pero .J!'\ , S = JX--l '-lue es nrn, ur
yu~ 1.3-t. cnh,nLc, en IU!!>I I de 8 lom:111h>S 7 para .-1 (TADI{ \ 1111 l) (7
t·, b ,1g.m'-·n1"· nlr;1 de la raí.1 ) 7 J.¡ 5 8
. .¡
Rt·s1 an10,;; '2lJ lk l .~~ yul·,bntlo 5. ha_1;111tl,, a <.:0111111u;u:1un el ~1gu1t·11l l·
gn1po lk d'-" ulr.1!',, voh 1endo,c a rcpclir el prol.'.'e,o 13'-l
J :! l)
5 5· X 21 .2 = 5-l
54 1 :>-t[IJ x[l = ~.¡¡
4.6 RAÍZ DE UNA l\lULTIPLICACJÚN INDICADA 17

,¡r---;- . •y- ,,¡-;-


'V a x n ce- 'v a x 'V h

4.7 RAÍZ DE UNA DIVISIÓN IND ICADA


~ =-va : -v'h
E_1emplo: V6Ll = y64 V8 4: 2 = 2

4.8 RAÍZ DE UNA POTENCIA


W' = ½Jll
1 MATEMATICA
1
SK:\ l\ lPR .\CTICA ®
l. Completar el siguiente cuadro: (Colocar un aspa si la división es inexacta)

r -10
-1 -2 +2 -4 +3 -7 - I5 -5

+100

-40

-55

+70

-80

-100

+150
-6j )

11. Escribir en los casilleros correspondientes los números enteros (si existen) que permitan
que la igualdad se cumpla:

(1) iiJ'=-8 (11) [~ l =+64 (21 l [-i- 11= -49


(2) ~ =E] (12) V+M= w (22) "V-4<J =EJ
2
(3) ~~ r= + 32 (B) □ 3 =-64 l23) [ : ] =+49

(4) 032 =1~2 1 (14) v-&4 =G] (24) V+49 = w


□ = -625
3 4

(5) E( = - 32 (15) 5J =+M (25)

(6)
~=□ 2
(16) V+M = □ (26) ~= □ 3
3
(7) r;J =+16 ( 17) {;] =+1000 (27) □ =-125

(8) "\1+16 = GJ
. 4
( 18) V+IOOO =GJ
3
(28)
-if-T2s = □ 5
(9) EJ =+ 81 (19) CJ =-1000 (29) □ =+243

l l O) ,v-81 =□ (20) ~ - 1 ººº =1-,".> I (30) 0243= □


. .. ------- ·-·-·-·--: .-:-.-:-.-.. ~:-.:-·-
1 ALFOI\SO ROJAS PUEMAPE 1

111. Hallar b raí✓. cuadrad~• ¡k cmla uno de los siguil'lllcs mí1lll'í<l'- ckctuando la comproha-
c ión c.:orrl·spondil·111l·:

t 1¡ 753 (111 -Dl5 <2 1) 72 51>


(2¡ 5-lX ( 12) 7 (129 (22) -l-l:!18
/3) 87(, (13) 5 629 (23) 25 >14
(-l) 915 {1--l) 6903 (24) .16 hl-l
(5) 717 (15) 7 818 (25) 28 !02
lhJ 9XX (16) 9 515 (261 54 726
17) 501 ( 17) 1 xu (27) :n 6-l:!
(X¡ 613 ( 18) 7415 (2X) 57 101
(l)) xnx ( 19) 5 703 (29) 62 713
< 10) 7h9 (20) 8 -l()-l (.10) 51 415

IV . Co111pktar c.:ada casil k ro con un número entero p.ua que la igualdad sea crnrn:ta.

(6) -.j( lh):IXI) =8:[~J


(_
1 ) ~( 6--1): () IHJO) =l - -l) : ( 10) l7) □ifIOO Nocxisteen Z

(X) # - =~•
(-l) -v1ITIT = ( - 2) : (5) (lJ) ~
o= XI

(5) -~ = □ ,10) -✓ O" ]6 = 12

\'. En l'.Íl'rc:iciosn,n opl·rac.:io11,:scomhinadasdondc no hay signos de c.:olcc.:cilín,cmpaamos


a operar poll'IKias y raín:s, para luego seguir con multiplicaciones y divisionl'S y
finalllll'llll' ._•kc1uar la-. sumas y restas. Si hay signos de c:okc:ción, operamos primero
segun ..-1 111is1110 cmkn l'll el interior de los mismos. Según csto.dccluar las siguil·ntcs
l>pcracilllh.:s c11mbi11ada!'\.

11) ( -JJ +i4) - (+_"\)( -2) ,,., ( -lJ) : ( +J) - ( -5)( -2) +1 -2)' vVJ6
(2) ( -7)( +2) + ( -XJ( -3) (7) ( -5f + ( -3)( 2, q 7)

131 ( -5 f - \ -3 )( +61 - ( - 11 1
(XI ( - 2) ( - 2) + 1 \ ) 2 + ~

(-l 1 ( - I )(-X¡-' + ( - 1 )t -h) -t- ( -."\ ¡2 ('ll ( -5)1 -hJ : I 2)+ ~


tSJ ~ + l -..
1 '
)"(+'.\)., - 1 (10) ( - 30): (+))-,. \ - :'Ílc ( -2)
ilfrJ@Y¡tj MATEMATICA
1

(11) (-3) 4 - (-2)(-7) + (-16) . (-8) (21 ¡ H-7H-21 + f-5 - 2 - (-3 ... (-2, 2 + VM n
(12) (-6)(+10) : (-3) + (-2) (22) { -h- ((-2)4 : (-2)(+3) -(-3))}
(13) (-4)2 • (+2)-(-7)2 + (-1)(-3) (23) (-8)(+2) - {(-6)(+2) 2 - (-51(-3)} --vai"
(14) ~ X (-3) 4 + (-5)(-8) - (-2) (24) { ~ (-5) t ((-2) 6 : (+4) - (-7)(-2)]}

(15) ~ + (+8) 2 (-3f • (·42) (25) (-10)2 : (-2) + {(-2)' · /+4)-(-7)(-2)}

(16) V10Q " (2) 3


- (-5)(-7)(-2) (26) (-6)(-2)(-3) + V64- ((-6)(-10)(+2) : (-4)1
(17) (-120) : (-24) , (-3) + (-3) (27) R : 1-2)2 + {(-8)(-3) : 1-21 + 1-1i=- 1-61(-8)}

(18) (-6)(-8) : (-3) + (3}2(-5) (28) (-10)2 : (-5) - ((-2)(-3) - (-6 + 8 - 5 - 7))

(19) ((-7)(-2) + (- 5f ) + ((-3)(-4)(-5) + (+1)) (29) (-4)(-6)(+2) : (-3) + (-7 - 6 + (-2) 2 - ~I

(20) {( V81+ (-6)( +2)-(-2f]+[(-8l:(+2)-(-2))} (30) (-6¡2 : (-2)2-{-6-3-1(-5)(-ll) + í - lJII

\ 'l. Si a= -2 ; b = -3 ; e = + I 6 : d = -X; e= +-l ; hallar el valor num~ric.:o de las ,iguil~ntcs


cxprl'SÍ1 mes:

(1) ab + e-d (7) ab + cd - {2e + $ + a - b}

(2) Vc -abcd + e (8) cde + [a + b + e - ¼"- (a - b - e¡)


(3) ab - cd + e' --v'c (9) J( ab -~ el
2
+ ~- ( abcd l

(4) 2a + 3b - 5c (1 O) -b - [ae - cd + a2 + b2 + e·)

(5) 7VC -r 8Ve + ad (11) a3 - ~- {-d - (a• + be - d2))

(6) 3d - 5a + Vd -1 (12)
3
Vec - (2bc- 5ae - e . d)
1 ALFOSSO ROJAS PUÉJ\IAPE 1

Aquí mis op.:rac:ion.::-. d.: SKA'\IPH.ÁCTICA ®


IV8i§Y¡tj 1 MATEI\IATICA
1
PROHLEl\1 .-\S RES( 1ELTOSSOHRE LAS PRII\IEIUSCI IATROOl'ERACIONES EI.E-
1\IE!\T.-\l.ES CO'.'. , f':\IEIH>S E:'\TEROS.

( 1) Entre Arturo y:'\ 1:mul'I ti,·nen S/. 126. Si la cantidad que


Licue Arturo,. ._ 17 veces la que ti,·n,· Manuel , ,·.ruámo
más lK'ne Arturo que l'vlanucl'!
a) S/.119 h> S/.112 c) S/.17 d) S/.7 e) S/. 12
O ¡IMPORTANTE'
Solul'itin:
,:; Si de dos números A y B
D donde A > B, se conocen:
Si IJ conliene q veces a d,enlonces d =q La SUMA (S)
Si ,\ tiene III unidades más que B,enlOnces El COCIENTE (q)
A - B=m.
Lo que se nos pide en el problema es la DI FE-
RENC/A enzreloquezienenam!Jospersonajes.
Ji:!:
r:·
Ent!~:;;~~~~: que:

S" q + R
A = - - --
Veamos el proccdimi,·1110: q+1
8 = S- A
Suma (S) de lo qu,· ti,·n,·n Anuro
y Manuel: S = 126
Anuro (Al tiem· 17 \'eccs lo que A
q = - = 17
ti,·ne Manud (1\1): 1\ 1
Entonces lo que ti,·ne Arturo se S · q + R 126 X J7 + ()
A=----
calcula así: C:: O q + 1 17 + 1
Operando: A= 1 llJ
Si restamos la suma total menos
lo que tie1w Arturo,oht,·nemos lo M = 126 - 119 = 7
qu,' tiene l\lanud: C:: O
Rl·Spucsta.- 00 ¡CUIDADO!

Arturo tiene 119 - 7 = S/.112 más que Manuel. Rpta. @ Si de dos números A
=== y B donde A > B, se
Hace 2 ai'ios. tu edad era mayor que la lk l\lanl/a { conocen:
por X ar'ios. Si actualm,·mc tu edad ~~s ,·1 tripk que ;:: La DIFERENCIA (D)
la de l\larilla. ¡_qu~ edad tendrás el próximo aiio'! . ( El COCIENTE (q)
El RESIDUO (R)
a) 11 h) lJ l') IO d¡ IJ e) 15
a:: Entonces, se cumple
Soluciún: \ que:

La diferencia de ccfacfes en lodo tiempo siempre es la A=D><q-R


misma. Si hace 2 wíos La diferencia de tu edad y La de q -1
Maritza era 8 mios, ac1ual1nen1e La dtcrencia (D)
siRue siendo 8 años. Ademár si tu edad es lriple que B=A-D
la de Maritza, enlonces el COCIENJE (q) de ambas
edades es 3, con RFSfO CERO .
1 ALFONSO ROJAS PUÉMAPE 1
El PROCEDII\IIENTO es el -;iguicntc:
Es1abkcemos la DIFERENCIA (D) de edades: 0=8
Ahora el COCIENTE lq) de ambas ctbdes: q=3, R=O
Dx lJ - R
• Calculamos la edad I\IA YOR así: Edad Mayor=---'----
e:: o 4- I
8 X 3 - ()
Rl'spul'sta.- = 12
3-1
Como la EDAD MAYOR l.'S tu edad. el pníximo aiio
tendrás 13 m'ios. Rpta. @

El produc10 tk las l.'dades de Juan y Víctor es


128. Si la edad de Juan es el doble que la de
Víctor, ¿cuál es la edad de Juan'! .
a) 12 h¡ 15 C) 16 d¡9 e) 18
Soludún: ~¡- O ¡IMPORTANTE!
Estamos anle un problema donde los datos son el W Si de dos números A y B
producto y el cocieme de dos números. "Si la edad de
Juan es el doble que la de Vírlor" vignifica que el
f donde A > B, se conoce:
cocicnle de ambaf edades es 2. ¡;;i
,.,.,
L~log:i~:z~~~~f)
El RESIDUO (R)
El producto P de ambas edad..:s es: P = 12X ~j!: Entonces, se cumple que:
El cociente q de ambas edades es: q=2 p A=D><q-R
Calculamos la edad MAYOR así:
C::: 00
A= ~

A=-✓ 128 x 2
A= V256 = 16
t . ~::~~
-. ·--~----·-·.-.••.•· ...

Rl'Sf>tll'Sla.-
Como Juan es el MAYOR, su edad es 16 años. Rpta.@

(-1) Si un librero vende a S/. IX l·:ub lihn, gana


S/.1 XO. pero si -;e clecitk \'Cnder L·ada libro a 0 0 ¡CUIDADO!
S/. 15 pierde S/.60. ¿Cuántos libros tiene para Dados dos numeros Ay
vender el librero'! . B (A> B) donde:
a)XO b) 2-1 C).'6 d)-12 (') 5-l El Producto: P
El Cociente: q
Solm:iún:
Entonces, se cumple
En problemas como este cm{'lcamos el MÉJDDO de las DJFE- que·
RENClAS. el cual nos permite hallar el número de obJclOs,
dividiendo D : D donde. A= ✓ Pxq
D7 es la DJFÉRENC/1\ TO/i'\Lentrc lo que se ganaría y luque se
f'Crdcría .
D es la DIFERCNCIA UNlfAR/1\ de los precio.\· de venta por B=~
u,iidad que originan la Raruincia o pérdida. -,,.....---,-,/
1 MATEMÁTICA
1
Proc~·dim iento:

Difaencia Total (D-.) º.- = 180 - (-60) = 240


Dil'-·rencia Un11~1ria (D.) 0=18-15=3
u
• Cantidad de libros pedida (C) C=D-.:D.
e = 240 : 3 = 80
Res¡HJesta.-

EI lihrcro tiene 80 libros para vender. Rpta.@


Una piscina inicialmente llena tod~, de apta.ha tenido
el desagüe abierto durante tres días.hasta que linal-
mcntc quedaron 8 litros. Si cada día ha estado (lcsa-
giiando la mitad más 4 litros de lo que quedaba del
día amerior. ¿cuál es el volumen total de la piscina'!.
a) 1201 hJ 1101 c) 1001 d) 1401 e) 801

Una forrna de re~olver esle problema es por el llamado


MÉTODO DEL Ci\NGREJO ( "De final a principio"), es
derir analizaremos en un cuadro lo que ocurre el lerrer
día, lue11oelsegundodíayfinalmcnle el primer día. Para
eslo,lengamos en cuenta lo siguiente:
SitomamoscornoeJemplouna varilla a laque recorta,rws
la MI IJ..D +2 cm, entonces quedará la M ffi\D -2 cm, as[:
O ¡CUIDADO!
MITAD- 2
MITAD
Bhtros es lo que queda
?cm al final del tercer día
que equivale ala MITAD
MITAD +2 de lo que queda del d1a
anterior, menos 4, es
• En nuestro problema (5). si cada día salen la MITAD +4 decir:
emonces quedarán la MITAD -4. Veamos el siguiente MITAD-4 = B
cuadro partiendo (k lo que queda al final del tercer día: Para que esta desigual-
dad se cumpla, dicha
QUEDAN: SALEN MITAD debe ser 12
DÍAS MITAD
MITAD - 4 MITAD+ 4 Como ese 3" día salen
la MITAD+ 4 entonces
3-· 8 = MITAD-4- ~ 12 - f.-- 16 saldrán 12 + 4 = 16
1
1

'
-
1

2' 24 = MITAD-4- 28 - ► 32
1 1

1"
'
56=MITAD-4-
- 60 -
- i
' 120 litros que habían al inicio del 1º día
1 ALFONSO ROJAS PUÉMAPE 1 114J@ff¡fj
Rt:"sput'sla.-

AI inicio del primer día la p1sc111a ,·,1aha 1u1~1lmc111c


llena rnn 120 li1ros de agua. Rpta. @

El dlll'lio dc 11111alk·1 ,111JLra1~1 a 1111 ohrao al


4ue k 11rrl·n· SI.X por l·;1da d1a q11,· 1raha_1,·)
SI. 3 por cada día que por causas de salud
se a11sl'llll' dd lallcr. 1 lll'),'.O de 2:'i dias,d
obrero recibe S/. 150. ¡,Cu:.í111os días no Lra- .
bajó'!.
a) 8 h) 12 e) 9 d) 10 e) 15

Soluciún:

Vamos a mostrar aquí el MÉTOIJO DE FALSA


SUPOSICIÓN. que romiste en .rn¡wru·r que anle dos
~itua< tones .~ólosecumple una de ellas, origiriando un
ERfWR JO/AL y un ERROR UNITAR!n . Errores
ruyo rorieltle nos permite obtener lo que buscamos.
E.~te mclodo se aplira por lo general cuando hay dos
i11róg11ilm y cuatro claJos.
n
-,,._:__¡

,;,r
.J , \

.
¡

Vamos a suponer lJlll' el obrero Lr.ihajó todos


los 25 días, c111011ces habría rccibido: 25 • S/.8 = S/.200

¡Pero sólo recihi1í S/. 150!,cstose debe a que


nucstrn suposición es falsa .
El error total comeudo es: S/.200 - S/.150 = S/50
Al considaar q1ll' 10dos los días r11cron tra-
bajados se coml'li1í 1111 error 1111itario de: S/. 8 - S/. 3 = SI. 5

El cociente dd l'rror total l'nlrl' el error


unitario nos da la CANTIDAD DE DÍAS N" Días que = S 1 · SO = lOdías
no trabajó S /. 5
NO TRABAJADOS :
Luego los días que trabajó serán: 25 - 1O= 15 días.
lkspul'sta.-

Los días que el obrero no trabajó fueron IO. Rpta.@

* OTRA FOl<l\1A: REGLA DEL llOl\11\0

Esta es una regla práctica del MÉTODO DE


TE
FALSA SUPOSICIÓN. Para su desarrollo se
cmpka un ROM RO que es una figura que tiene
la siguiente forma y elementos:
1 l\L-\TEMATICA
1
donde:
TE rcprc-scnla al número toUtl de elementos.
TR representa al total recaudado.
NM: representa al número mayor por unidad.
Nm: representa al número menor por unidad.
Las líneas punteadas expresan la fonna como debemos
de operar.
En el problema (6) esta REGLA DEL ROMBO se
aplica así:
8
N" de días que no trabajó 25 X 8 - 150
8- 3
25 = 10

Entonces ele un total de ::!5 días. (25 - 10) = 15 días trabajó.

(7) Se ha comprado 77 la1.as de leche de dos


capacidades distintas: unas tienen 8 onzas y
las olías 15 onzas. Si el contenido total es ele
861 onzas, ¡,cuántas la¡¡,s de 8 on1.as se
compraron?.
a)-lO b)38 c)44 d)41 e)42
Soluciún:
Observa que hay cuatrodatos:dos de ellos referidos
a onzas por UN/ DAD, otro referido al total de latas
y un últmw referido al TOTAL DE LITROS
RECAUDA DOS, entonces podemos aplicar la
R EGLADEL R OMBOquedespuescomprobaremos
con el METODO DE FALSA SUPOSICIÓN.
• Número total de elementos (TE) : TE= 77
• Total recaudado {TR) :TR=861
Número mayor por unidad (NM) : NM= 15
Número menor por unidad (Nm) : Nm=8
Por la Regla del Rombo:

Nº de latas de 8 on,as = 77 x 15 - 861 =42


15-8 TE=77
Rtsput>sta.-
Se compraron 42 latas de 8 onzas. Rpta.@
1 ALFONSO ROJAS PUÉMAPE 1 112\@Ui
* Ra1:rnk·mos la solución de este problema haciemlo una
FALSA SUPOSICIÓN :
Supongamos que tod;1s las 77 laUts son de 15 onza-;
(FALSA SUPOSICJONJ, entonces el contenido total
Sl'íÍa lk : 15 x 77 = 1155 onzas
Pero I 155 110 es el wrdad..:m conte111do tot.al sino que
hay un I RROR (o dill'rem.:ia) debido a que hay latas de
8 onzas, este ERROR TOTAL es: 1155 - 861 = 294 onzas
Como el error total se debe a que hemos con<;iderado
algunas latas de 8 como si fueran de 15 onzas, entonces
el ERROR UNITARIO será de: 15 - 8 = 7 l>llJ:as.
Luego el mímcro de Iat~is de 8 onzas ser.í: l
294 : 7 = 42 IatasJ 1-<pta.
OHSERV ACH)N.-

En el rombo: Si ckseamos hallar el número de latas de


15 onzas procedemos así:
15
N" delatasdc l5onzas= 77 x X - 861
8 - 15

77 k61

o también (si no usamos el rombo para este último


caso):
Nº de latas de 15 onzas = 77 - 42 = 35 latas

Mano y Felipe tienen _11111tos S/.300. Si Mario le diera a


Felipe S/.90. emonc~·s los dos t.endrían igual cantidad de ,{ O RECUERDA OVE-
dinero. ¡,Cuánto tiene cada uno de ellos'!.
l Siconocemoslasuma
al S/.240 y S/.60 h) S/.100 y S/.200 c) S/.80 y S/.220 } (S) y la diferencia (D)
d) S/.170 y S/.130 e) N.A. l de 2 números A y B
I (A>B), entonces se
Solucion:
:!!:! cumple qu;: + D
La '-llllla de lo que tienen ambos es: - M+F=300 \ A=--
. Si !\fario le da a Felipe S/.90 para
.,¡ tenn ahora cantidades iguales, en-
2
B= S - D
M - F= 180
~/rI 2
I ' ,._ ,. / "
1..0nces la diferencia inicial emre
{-¡,,:J· ambos era el doble de S/.90. es decir: ___/
,~.,.¡i
,,
~.:fü¡"
•• !
• Con la suma (S ) y diferencia (O) de
ambas cantid.llks ya podemos hallar
M = S + D = 300 + IXO ➔ 1\-I
,
- -
')
=- :!--l-(l

lo que tiene cada uno: ~ , O F= s; o = :mo; ixo ➔ IF = S/. 601


Rpta.@
'MATEMATICA
1 1
RAZONEMOS
1
Instrucciones:
A continuación te presento bloques de ejercicios para afianzar lo
leido hasta aqui. Marca la respuesta correcta y compárala con
la CLAVE DE RESPl 'ESTAS que aparece al final; sin embargo,
te recnmtendn que expliques en tu cuaderno las RAZONES de tu
resultado. No tnmes este tralmJo como una pesada carga, sino
como un DESAFIO a fu inteligencia.

IlLOQUE I ♦ (9) La diferencia dedos números enteros es 32 y su


correspondientcsuma cs-66. ¿Cuál csel número
♦ menor?
( 1) Entre dos personas tienen SI. 200. Si la cantidad
que tiene una de ellas es el triple de lo que tiene ♦ (10) En el problema anterior, ¿cuál es el número
la otra. ¿cuáles snn dichas cantidades? mayor disminuido en -7?

(2) Se Liene una regla de 30 cm. que luego se parte ♦ (11) Dentro de 6 años Luis será 8 años mayor que
en dos pedazos. Sí un pedazo es el doble del l'vloisés. Si actualmente la suma de sus edades
otro, ¡,cuánto mide el pedam menor? ♦ es de 34 años. ¿cuál será la edad de Moisés
♦ dentro de 6 años?
i3) Jés~ica y Rosa tienen juntas S/. 342, si lo que
tiene Rosa es 5 veces lo que tiene Jéssiea, ♦ (12) El señor Perez es 38 años mayor que el señor
¿cuánto tiene Rosa? Quispe. Si le restáramos 7 anos a la edad del
♦ señor Pérez, ésta sería el doble de la edad del
(4) Las edades de un padre y su hijo suman 85 anos. señorQuispe. ¿Cuál es la edad del señorQuispe?

Si la edad del hijo es la cuarta parte de la de su
padre. ¿cuál es la edad del hijo? ♦ (13) El señor y la señora Domínguez se casaron
cuando el primero aventajaba por 8 años a la
(5) Dos números enteros su man 3 2. Si dividimos el ♦ segunda. Si actualmente sus edades sumadas
mayor entre el menor. el cociente resulta 3 y el da'ii 72 años, ¿cuántos años tendrá el señor

residuo 4. ¿Cuál es el número mayor? Domínguez dentro de 10 años?

(6) Se parte en dos una varilla de 180 cm. Si ♦ ( 14) l'vlaritza es dos años mayor que Jéssica. Dentro
comparamos ambos pedazos resulta que la más de 6 años la suma de sus edades será 60 años.
grande contiene dos veces a la más pequena. ♦
¿Cuál es la edad actual de Mari tza?
con un sobrante de 30 cm. ¿Cuánto mide la
parte más pequeña? ♦ (15) 1lace 8 años Miguel era 6 a ños menor que
César. Si dentro de 5 años la suma de sus ed,1des

m Entre Juana y Pedro tienen en el Banco una será 40 anos. ¿cuál será la edad de César el
cuenta por S/. 920. Lo que le corresponde a próximo ano?

Juana es 4 veces lo que le corresponde a Pedro
con un adicional de S/. 20. ¿Cuánto le ♦ (16) El producto de dos números no positivos es 98
corresponde a Pedro? y su cociente es 2. ¿Cuál es la suma de estos
♦ números?
·Hace I O años el papá de Javier era 29 años ♦ (17) Un número contiene exactamente 4 veces a
mayor que él. Si le quitáramos 5 años a la edad otro. Si al multiplicardichosnúmerosseobtiene
del papá. entonces su edad sería el triple que la ♦
16. ¡_cuál es la diferencia negativa de ambo,
de Javier. ¿_Cuál es la edad del papá de Javier? ♦ número~?
1 ALFO~SO ROJAS PUÉ!\IAPE 1
(18) Dos números enteros de distinto signo dan ♦ (28) Si Manuel vende cada lapicero en SI. 12, gana-
como producto -43 2 y como cociente -3. ¿Cuál ría SI. 48 en todos los lapiceros que tiene. Si
es la diferencia positiva de ambos números"l ♦ cada lapicero le costó S/. 9, ¿cuántos lapiceros
vendió?
(19) Si al minuendo de una sustracción le ♦
aumentáramos en 10 unidades y al sustraendo ♦ (29) Un número se aumenta en 8, el resultado se
lo disminuimos en 8 unidades, ¿en cuánto varía triplica, se extrae la raíz cuadrada al resultado
la diferencia? ♦ obteniendo 6. ¿Cuál es el número?

(20) Hallar el valor de E = A+ B - C si: ♦ (30) Un sargento quiere formar a sus soldados en 5

A= + ✓"16[ 3( 4xl5-8x6 )+8x3 ]-24x4 • filas de 6 soldados cada una. pero observa que


le faltarían 4 soldados. Entonces los forma en 4
filas de 5 soldados. ¿Cuántos le sobran ahora?
2
B = +V-[ 3xl5:5+1+( 3 x2xl-31 )] ♦ (31) Un profesor tiene 2 salones, uno de 30 y el otro
• de 25 alumnos. Si en el primer salón reparte 2
c = 1-7-8--\18+11

(21) En Puno se registran las siguientes temperaturas ♦
caramelos a cada alumno, le sobran 20
caramelos; si en el segundo salón reparte 3
caramelos a cada uno. ¿cuántos caramelos le
durante la semana: -4°, 0°, 6°, -1 º. -3°, Oº. 5°. sobrarían?
¿Cuál fue la mayor variación de temperatura ♦
entre 2 días consecutivos? ♦ (32) Una señora tenía 26 años al nacer su hija;
cuando la hija cumplió 15 años. su abuelita
(22) LepreguntanaCocoporsuedadyésterespondc: ♦ tenía 68 años. ¿Cuántos años tenía la abuela.
cuando nació su hija?
"Si al doble de mi edad le suman 4, obtienen 40
años". ¿Cuál es la edad de Coco? • ♦ (33) Un submarino está a 400 m de profundidad. Un
helicóptero está justo sobre él y lanza una
(23) Willy recibe SI. 5 de propina de su tío, luego su
papá le duplica su dinero y por último gasta ♦ bomba que recorrió 600 m hasta acertar al
SI. 4. Si todavía le quedan SI. 1O, ¿cuánto tenía ♦ submarino. ¿A qué altura sobre el nivel del
inicialmente Willy? mar, volaba el helicóptero?

(24) Rosa tiene S/. 80y Lily tiene S/. 60. Diariamente
gana cada una S/. 20. Si después de cierto ♦
• (34) La suma de dos números es 406. su cociente es
2 y el resto 91 . ¿Cuáles son los números?
número de días, entre ambas tienen S/. 340,
¿cuántos días trabajaron juntas? ♦ (35) Dividir el número 358 en dos partes, tales que
♦ su diferencia sea 125. ¿Cuáles son los números?
(25) Un depósito tiene 360 litros de agua. Jorge y
Luis extraen agua con baldes de 8 y 5 litros ♦ (36) Han transcurrido 13 minutosdesdequecomenzó
respectivamente, cada vez que van al depósito. ♦ el examen y dentro de 25 minutos faltarán 12
¿Cuántos litros quedarán en el depósito después minutos para las 10:55 minutos en que termina
de 25 viajes? ♦ el examen. ¿A qué hora comenzó el examen'!

(26) Un auto viajó de una ciudad a otra en 4 horas, ♦ (37) Luisa le da S/. 8 a Fanny, Fanny le da luego
viajando a 60 km/h. ¿Cuánto demora otro ♦ SI. 6 a Luisa y por último Luisa le vuelve a dar
auto en hacer el mismo recorrido, si recorre ♦ SI. 5 a Fanny. Si al final Luisa y Fanny tienen
80km/h? SI. 25 y SI. 32 respectivamente, ¿cuánto tenía

(27) Gregorio le debe S/. 48 a Javier. Diariamente


Gregorio le devuelve S/. 6; sin darse cuenta, ♦
• cada una inicialmente?

(38) Betty puede resolver4 problemas en un minuto.


después de cierto número de días, Javier le ♦ mientras que Rosa es el doble de rápida en
debe a Gregorio SI. 12. ¿Cuántos días trans- resolver problemas. Si juntas resuelven una
currieron hasta entonces? ♦ tarea de 72 problemas. ¿cuánto demoran?
1 MATEMÁTICA
1
BLO(JUEU
• unidades al dividendo, la di visión ahora re~ulta


exacta y el nuevo cociente es una unidad mayor
(1) Dada una adición de varios números enteros que el inicial. ¿Cuál es el nuevo dividendo?
añadimos -5; -7 y -2 respectivamente a cada
• a) 3 148 b)3 648 c) 3 548
uno de tres sumandos. Si a otros dos sumandos
les aumentamos +10 y +4 respectivamente.
¿cómo queda alterada la suma inicial?

• (7)
d) 3 646 e) 3 446

Al sumar el minuendo. el sustraendo y la


a) Queda aumentada en 14
• diferencia de una sustracción se obtiene 3 260.
¿Cuáles el sustraendo si se sabe que es la mitad
b) Queda disminuida en 8
c) No se altera • del minuendo?
d) Queda disminuida en 14
e) Queda aumentada en 8 •

a} 815
d)817
b} 704
e) 811
e) 708

(2) Si la suma de los tres términos de una sustracción ♦ (8) Si en una multiplicación de tres números enteros
es 458, ¿cuánto es la suma de las cifras del
minuendo? • se duplica cada uno de ellos, ¿cómo queda
afectado el producto?
a) 13 b) 11 c) 17 •
d) 38 e) 229
• a} Queda multiplicado por 2
b) Queda multiplicado por 8
(3) Se tiene una multiplicación de tres factores. Si
• c} Queda dividido por 2
se duplica uno de ellos y se triplica otro, ¿en
• d) Queda dividido por 8
e} No se altera


cuánto varía el producto inicial?
a) Queda multiplicado por 12 • (9) Si en una mulliplicación de tres números enteros
b) Queda multiplicado por 6 sólo se duplica uno de ellos. ¿cómo queda
c) Queda dividido por 6
d) Queda dividido por 12 • alterado el producto inicial?

e) Faltan datos
• a) No se altera

(4) Cuando dividimos cierto número entre 50,


obtenemos como residuo 20. Si dividimos el


b) Queda multiplicado por 2
c) Queda dividido por 2
d) Queda aumentado en 2
mismo número entre 52, obtenemos el mismo
cociente pero 4 de residuo. Calcular el cociente
que se obtiene en ambos casos.

♦ (1 O)
e) Queda disminuido en 2

Si multiplicamos un número entero por 518,

a) 6 b) 10 c) 12
• dicho número estará ahora aumentado en
267 806. ¿Cuál es el número?
d) 16 e) 8
• a) 715 b) 361 c) 216
(5) Si dividimos dos números enteros obtenemos
por cociente 17 y por resto 31 . Si luego
aumentamos el dividendo en una unidad,

• (11)
d) 815 e) 518

Si un comerciante vende a SI. 11 cada cal-


entonces el cociente aumenta también en una ♦ culadora. gana SI. 75; pero si decide vender
unidad, pero esta vez ya no hay residuo. ¿Cuál cada calculadora a SI. 6. pierde SI. 50. ¿ Cuántas

es el dividendo inicial? calculadoras tiene para vender?

450 b) 525 515 • a) 17 b} 25 e) 26

•♦
a) c)
d) 575 e) 625 d}l9 e)28

(6) Al dividir dos números obtenemos por co- (12) El papá de Carlos decidió mejorar sus ingresos
ciente 63 y por resto 55. Si añadimos dos ♦ para lo cual se asocia con dos padres de familia
1 ALFONSO ROJAS PUÉMAPE 1
que también saben de carpintería. Cuando tienen ♦ ( 18) Se tiene un tanque lleno de agua al que abrimos
listas una cierta cantidad de mesas. discuten el el desagüe. Si en cada hora salen la mitad de lo
precio de venta. Si las venden a SI. 72 cada una, ♦ que quedó la hora anterior más dos litros,
ganan un total de SI. 2 210 y si las venden a quedando al final de la tercera hora sólo cuatro
SI. 47 cada una. pierden SI. 1 040. ¿Cuántas ♦ litros; determinar la cantidad de litros que había
mesa<; confeccionaron? ♦
antes de la primera hora.
a) 125 b) 38 c) 49
• a) 57 litros b) 48 litros c) 64 litros
d) 130 e) 69

(13) Un pequeño ganadero decide vender sus vacas; ♦


si las venden a SI. 2 900 cada una, tendría una
• d) 60 litros e)32 litros

(19) Una familia se provee de agua a través de un


caño conectado a un depósito grande. Si se
pérdida total de SI. 2 000. Si las vende a ♦
SI. 3 500 cada una. tendna entonces una ganan- ♦ dejara abierto el caño durante 4 horas, queda-
cia de SI. 2 800. ¿Cuántas son las vacas que rían al final sólo 2 litros. ¿Cuántos litros habían
piensa vender? ♦ inicialmente en el depósito. si se sabe que en
cada hora salen la mitad de lo que quedaba la
a) 8 b) 13 c) 17
• hora anterior más un litro?
d) 6 e) 11

(14) Para rifar una refrigeradora se hicieron cierto ♦


número de boletos. Si cada boleto se vende a
• a) 68 litros b) 62 litros
d) 84 litros
e) 75 litros
e) Faltan datos

SI. 8 se ganaría SI. 1 040, y si cada boleto se ♦ (20) Un reservorio de agua potable ha sufrido un
vende a SI. 3 se perdería SI. 21 O. ¿Cuántos ♦ desperfecto, de modo que cada día se está
boletos se hicieron? vaciando el agua hasta que el nivel de la
a)I 15 b) 275 c)250 • superficie disminuye la mitad más tres metros.
d) 300 e) 315

(15) Se organiza una función de teatro en nuestro ♦


• Si al final del tercer día no hay ni una gota de
agua. ¿cuál era la altura inicial de la superficie
de agua del reservorio?
colegio. Si el señor Femández paga SI. 6 por ♦
cada entrada, le sobrarían SI. 16 y si paga SI. 7 a)90m b)42m c) 186 m
por cada entrada, le sobrarían SI. 8. ¿Cuántas ♦ d) 18 m e) 6 m
entradas compró?
♦ BLOQUEIII
a) 11 b) 8 e) 13
d) 7 e) 6 • (1) Un contratista ofrece a un obrero SI. 6 por cada
(16) Si a la edad de tu abuelito la multiplicas por 8,
luego la divides por I O y el cociente lo ♦
• día de trabajo y SI. 2 por cada día en que a falta
de obra no trabaje. Si después de 17 días recibe
SI. 70, ¿cuántos días no trabajó?
multiplicas por 3 añadiendo enseguida 36,
obtendrías 180. ¿Cuál es la edad de tu abuelito? ♦ a) 9 b) 12 c) 11
a) 60 años b) 65 años
d) 72 años
c) 180 años
e) 84 años

• (2)
d) 8 e) 13

En un examen de matemática de 19 preguntas


(17) Multiplicamos un número por 4. producto ,ti
que luego restamos 12 dividiendo enseguida el ♦
• se asignan 9 puntos a las preguntas bien
contestadas y 4 puntos a aquéllas que no fueron
contestadas. Suponiendo que todas las que se
resultado entre 3, parn volver a multiplicar por
6 añadiendo luego 3 al resultado. dividiéndolo ♦ contestaron fueron correctas, ¿cuántas
finalmente entre 3 resultando 89. ¿Cuál es el ♦ preguntas no se contestaron si el puntaje final
número inicial? fue 11 1 puntos?
a) 48 b)40 c) 60 • a) 15 b) 11 c) 12
d) 58 e) 36
• d) 13 e) 9
.

MATEMÁTICA
1
(3) Se embotellaron 10 l onzas de alcohol en ♦ ( 1O) Para pagaruna deuda de SI. 305 empleo billetes'
botellitas de 7 y de 3 onzas. Si el total de de SI. 5 y SI. 20. ¿Cuántos billetes de los 28 que I
botellas de uno y otro tamaño es 23, ¿cuántas ♦ tengo son de SI. 5?
botellitas de 3 onzas tienen alcohol?
♦ a) 17 b) 13 e) 12
a) 15 b) 8 e) 7 e) 15
d) 11
d) 17 e) 13 ♦
♦ (11) En una función de teatro se obtuvo una
(4) Se desea envasar 279 litros de vino en toneles
recaudación de SI. 679. Si el total de boletos
de 15 y 9 litros respectivamente. ¿Cuántos ♦
toneles de 9 litros se necesitaron si el total de vendidos fue de 183, ¿cuántos de éstos
toneles empleados fue 27? ♦ pertenecían a adultos?
Datos:
a) 27 h) 20 e) 23 ♦
Valor <le una entrada para adulto: SI. 5
d) 21 e) 17 ♦ Valor de una entrada para estudiante: SI. 3
(5) En 41 envases, algunos de 17 litros y otros de ♦ a) 115 b) 65 e) 72'
5 litros. se desea vaciar un total de 385 litros de d) 47 e) 118
agua. ¿Cuántos envases de 17 litros se tuvieron ♦
que emplear? ♦ ( 12) Tengo 50 billetes, unos de SI. 1O y otros de
a) 26 b) 12 e) 19 SI. 50. Si uso todos los billetes que tengo para
♦ pagar una deuda de SI. 780, ¿cuántos billetes
d) 14 e) 15
♦ son de SI. 10?
(6) Maritza hizo dos llamadas telefónicas; una a
Quito y otra a Arequipa. ¿Cuántos minutos ♦ a) 35 b) 43 e) 26
duró su llamada a Quito, si pagó por las dos ♦ d) 41 e)29
llamadas SI. 244 y en las dos llamadas empleó
~~~~ ♦ (13) Se vendieron entre adultos y niños un total <le
Dato: 91 boletos para una función de cine. Si un
♦ boleto de adulto costó SI. 5 y un boleto <le niño
Comunicarse I min. con Quito cuesta SI. 13
Comunicarse I min. con Arequipa cuesta SI. 6 ♦ se vendió a SI. 3, ¿cuántos boletos de adulto se
vendieron si la recaudación total fue de SI. 311?
a) 12 min b) JO min e) 15 min ♦
d) 13 min e) 17 min a) 19 b) 72 e) 17
♦ d) 21 e) 23
(7) En un patio grande hay cerdos y patos. Si se ♦
cuentan 28 cabezas y 78 patas. ¿cuántos patos (14) Un barril contiene 69 Jitros de cierto líquido. Si
hay en el patio? ♦ éste debe ser envasado en 27 botellas, unas de
♦ 2 litros y otras de 3 litros, ¿cuántas botellas de
a) 15 b) 11 e) 17
2 litros se va a necesitar?
d) 19 e) 13 ♦
a) 8 b) 15 e) 13
(8) En una granja donde existen pavos y vacas se ♦ d) 14 e) 12
contaron 96 patas y 31 cabezas. ¿Cuántas vacas
hay en la granja? ♦
(15) Si vendo a SI. 7 cada pelota gano SI. 12, pero si
a) 14 b) 15 e) 16 ♦ las vendiera a SI. 5, perdería SI. 6. ¿Cuántas
d) 19 e) 17 pelotas tengo que vender?

a) 7 b) 6 e) 9
(9) Ciemo cinco litros de agua deben ser vaciados ♦
en depósitos de 11 y 4 litros. ¿Cuántos son de ♦ d) 11 e) 13
11 litros si en total se usaron 21 depósitos?
♦ ( 16) Cinco entradas para hombre y seis para mujer
a) 18 b) 15 e) 17 a un teatro nos cuesta SI. 27. Otro grupo de 8
d) 3 e) 6 ♦ hombres y 6 mujeres al mismo teatro paga
1 ALFONSO ROJAS PUEMAPE 1
SI. 36. ¿Cuál es el costo de cada entrada para ♦ (3) Hace algún tiempo cada habitante de un distrito
mujer? recibía 300 litros de agua por día. Actualmente
♦ el número de habitantes aumentó en 180
a) SI. 3 b) SI. 2 e) SI. 4
♦ teniendo que recibir cada habitante 6 litros
d) SI. 5 e) SI. 1
menos. ¿Cuántos habitantes tiene actualmente
♦ dicho distrito?
(I 7) En una cierta concentración de estudiantes
habían triciclos y bicicletas. Si se contaron 85 ♦ a) 7 000 b) 9 000 e) 8 000
timones y 185 llantas,¿ cuántos eran los triciclos ♦ d) 11 000 e) I 3 000
que había en dicha reunión?
♦ (4) Entre gallinas y conejos se cuenta en un corral
a) 11 b) 13 e) 15
d) 16 e) 70 ♦ 48 cabezas y 158 patas. ¿Cuántas gallinas y
conejos hay?

(18) Susana tiene SI. 315 en billetes de SI. 10 y de a)l6y32 b)l8y30 c)17y31
SI. 5. Si tiene un total de 46 billetes, ¿cuántos ♦ d) IO y 38 e) 22 y 26
eran de SI. 5? ♦

a) 21 b) 29 e) 23 ♦ (5) Se reunieron todos los alumnos yalumnasdc 1°


d) 27 e) 19 y 2° año de nuestro colegio, donde notamos lo
♦ siguiente: En ungrupode72alumnos, todos los
(19) El papá de Jorge desea invitar un helado a él y ♦ hombres tienen 13 años y todas las mujeres 11
a cada uno de sus amigos. Si cada helado le años. Si la suma de las edades de todos los de
cuesta SI. 2 le sobraría SI. 11; y si cada helado ♦ ese grupo es 860 años, ¿cuántas mujeres había
le cuesta SI. 3 le sobraría SI. 4. ¿Cuántos ♦ en dicho grupo?
helados está pensando comprar?
♦ a) 38 b) 36 e) 35
a) 7 b) 8 e) 9 d) 33 e) 34
d) 6 e) 10 ♦
♦ (6) Un tío quiere compartir entre sus sobrinos
(20) En el problema anterior, ¿cuánto dinero tenía el cierto número de caramelos. Si les da 8
papá de Jorge? ♦
caramelos a cada uno, le sobran 17 y si les da 12
a) SI. 21 b) SI. 18 e) SI. 25 ♦ caramelos a cada uno, le faltan 27 caramelos.
d) SI. 27 e) SI. 28 ¿Cuál es el número de sobrinos?

a) 7 b) 11 e) 12

d) 13 e) 15
BLOQUE IV

(]) Un obrero gana diariamente SI. 5 más que otro. ♦ (7 ) En el problema anterior, ¿cuántos caramelos
quiere repartir el tío?
Despuésdetrabajarcadaunoel mismo número ♦
de días, el primero recibe SI. 143 y el segundo a) 78 b) 100 e) 105
SI. 88. ¿Cuánto gana por día el obrero peor ♦ d) 98 e) 110
pagado? • ♦

a)Sl. 11 b)Sl. 13 c)Sl.5 ♦ (8) En un exameri de MATEMÁTICA se utilizan


d) SI. 12 e) SI. 8 57 números, unos de una cifra y los demás de
♦ dos. Si en total hay 66 dígitos, ¿cuántos nú-
(2) En el problema anterior, ¿cuántos días ♦ meros que aparecen en dicho examen son de
trabajaron juntos los obreros? dos cifras?

a) 7 b) 13 e) 10 a) 7 b)48 c)38
d) 11 e) 8 ♦ d) 9 e) 28
1 MATEMATICA
1
- (9) Unlitrodelechepurapesa 1030gramos.Cierta ♦ pierde un punto. Si la nota de un alumno es I O.
mañana se reciben 6 litros que pesan 6 120 ¿cuántas preguntas contestó bien?
gramos. ¿Cuántos litros de agua contiene esta ♦
leche?(EXAMENDEADMlSlÓN-UNMSM) ♦ a) 5 b) 15 c) 13
d) 12 e) 9
a) 3 t b) 4 t c) 6 t ♦
d) 5 t e) 2t ♦ (16) Carlos y David salieron simultáneamente de
dos ciudades opuestas uno al encuentro del
(10) Con tres desarmadores se obtiene un alicate, ♦ otro, el primero en motocicleta y el segundo en
con tres alicates un martillo.¿ Cuántos martillos ♦ bicicleta. Además el primero recorrió 4 km por
se obtendrán con 117 desarmadores? hora más que el segundo y se encontraron
(EXAMEN DE ADMISIÓN-UNIV. ♦ después de 7 horas de haber partido. Si el
CATÓLICA) segundo ha recorrido 35 km. ¿cuál es la distancia

entre las dos ciudades?
a)II b)I0 c)9 ♦
d)l3 e)l4 a) 75 km b) 69 km c) 72 km

d) 91 km e) 98 km
( 11) Juan compró un minicomponente que luego

vendió por SI. 457. obteniendo una ganancia
♦ (17) Diez personas efectúan un viaje de excursión a
igual al doble del precio de compra más SI. 37. Chosica. cuyos gastos convienen pagarlos en
¿Cuánto le costó el minicomponente?
♦ partes iguales. Al término del mismo, cuatro de
a) S/. 250 b) S/. 140 c) S/. 270 ellos no podían pagar, entonces cada uno de los

d) S/. 170 e) S/. 210
restantes tuvo que pagar SI. 8 adicionales.
♦ ¿Cuánto le costaba a cada uno inicialmente
(12) Un comerciante empleó S/. 1 910 en comprar ♦ dicha excursión?
50 camisas de SI. 40 y de SI. 35. ¿Cuántas
a)S/. 9 b)S/. 10 c)S/.12
camisas de SI. 40 compró? ♦
d) S/. 15 e) SI. 18

a) 32 b) 18 c) 16
d) 30 e) 28 ♦ (18) A las 7 a.m. sale un auto de A a 55 km/h y va al
encuentro de otro que sale de B a 70 km/h a la
( 13) Se desea rifar un automóvil haciéndose cierto ♦ misma hora. Sabiendo que se encuentran a las
número de billetes. Si se vende cada uno a ♦ 12 m, ¿cuál es la distancia entre A y B?
SI. 8sepierdeS/. 600, y si se vende a SI. JO cada ♦
uno se gana SI. 1 400. ¿Cuál es el precio del a) 625 km b) 615 km c) 610 km
automóvil? ♦ d) 720 km e) 750 km

a)S/.7600 b)S/.8600 c)S/.9400 ♦ (19) Entre 24 personas deciden pagar en partes


d) SI. 8 800 e) SI. 8 200 ♦ iguales una deuda; pero resulta que 8 de ellas
sólo pueden pagar la mitad de lo que les
( 14) En el problema anterior, ¿cuántos billetes se ♦ corresponde, obligando de esta manera a que
mandaron hacer?

a) 1 000 b) 800 c) 1 100




cada uno de los demás añadiese a su cuota
SI. 6. ¿A cuánto asciende la deuda total?
d) 1 200 e) 700 a)S/.516 b)S/. 418 c)S/.478

d) S/. 520 e) S/. 576
(15) Un examen bimestral de MATEMÁTICA de ♦
20 preguntas se plantea del siguiente modo: Por ♦ (20) Pablo tenía 30 años cuando Andrés nació.
cada respuesta bien contestada el alumno gana Ambas edades suman hoy 48 años más que la
un punto y por cada respuesta mal contestada ♦ edad de Esteban que tiene 30 año~. ¡_Qlé edad
·O· 1 ALFONSO ROJAS PUEMAPE 1
tiene Francisco que nació cuando Andrés tenía ♦ a) 12 años h) 11 años c) 13 años
13 años? ♦ d) 15 años e) 17 años
a) 14 b) 12 c) 11 ♦ (27) El comandante de un destacamento observa
d) 13 e) 15 que haciendo marchar a sus soldados de modo

que en cada fila entren 4. le resultan 132 filas
(21) Preguntando a un matemático por las horas ♦
más que si en cada una se tuvieran 6. ¿Cuántos
transcurridas responde: "Quedan 6 horas menos
soldados tiene dicho destacamento?
que las horas transcurridas". ¿Qué hora es? ♦
♦ a) 1 384 b) 1 484 c) 1 584
a) 1 pm b) 12 m c) 11 am
d) 1 684 e) 1 784
d) 3 pm e)4pm ♦

(22) Se ha pagado una deuda de SI. 265 con monedas ♦


de SI. 5 y SI. 2; el número de monedas de SI. 2 ♦
es mayor que el de SI. 5 en 17 monedas.¿ Cuán-
to suman las monedas de SI. 2 y de SI. 5'! ♦
(EXAMEN DE ADMISIÓN-UNIVERSIDAD ♦ CLAVE DE RESPUESTAS
NACIONAL DE INGENIERIA-71)
a)82 h)81 c)80 • RAZONEMO S

d) 83 e) 79

(23) El tío de Carlos fue el domingo al hipódromo. ♦


• BLOQUE 11
(1)c (6)b (11) b (16)a
En la primera carrera perdió SI. 40; en la ♦ (2) a (7) a (12) d (17)e
segunda ganó SI. 125; en la tercera ganó el ♦ (3) b (8) b (13) a (18) d
doble de lo que tenía hasta la segunda carrera. (4)e (9)b (14) e (19) b
En la cuarta carrera después de perder la mitad ♦ (5) d (10) e (15) b (20)b
de lo que tenía le quedan SI. 465. ¿ Con cuánto ♦
dinero empezó a jugar? BLOQUE 111
♦ (1) d (6) b (11) b (16) b
a) SI. 360 h) SI. 190 c) SI. 270
d) SI. 120 e) SI. 380 ♦ (2) e (7) e (12) b (17) e
(3) a (8) e (13) a (18)b
(24) La suma de las edades de Carlos y Fernando es ♦ (4)d (9)d (14) e (19)a
48 años. Al acercarse Carolina, Carlos le dice: ♦ (5) e (10) a (15) e (20) e
Cuando tú naciste yo tenía 4 años, pero cuando ♦
Fernando nació tenía 2 años. ¿Cuál es la edad BLOQUE IV
de Carolina? ♦
(1)e (8)d (15)b (22)d
a) 21 años b) 23 años e) 27 años ♦ (2) d (9) e (16) e (23)e
d) 25 años e) 24 años
• {3)b (10)d (17)c (24)a

l
(4) e (11) b (18) a (25) e
(25) En una lihrería por cada 3 lapiceros que se ♦ (5)a (12)a (19)e (26)b
vende se ohsequia 2. Si el stock disminuye en (6) b (13) b (20) e (27) e
180 lapiceros. ¿cuántos se obsequió? ♦
__
(7_)c--(-14_)_ª_ _(2_1_)d_ _ ____,)
a) 36 h) 45 c) 72 ♦
d) 48 e) 75 ♦

(26) Ludia tiene 28 años menos que su papá. ¿Cuál ♦


eslaedaddeLucilasisabemosquedentrodc 15 ♦
años la suma de sus edades será de 80 años?
MATEMATICA
1

LAS MATEMÁTICAS DE TODOS LOS DÍAS


( 1) La empresa TEXOM S.A. culminó sus operaciones en el año 1 992; el señor Ramírez
que es su dueño, desea hacer un balance a modo de reflexión acerca de lo que logró en
los últimos 5 años con la citada empresa, para lo cual analiza el siguiente cuadro-
rcsumen. en el cual falta llenar algunos espacios vacíos, considerando que la ganancia
o pérdida se calcula restando los ingresos menos los egresos.

AÑO INGRESOS (S/.) EGRESOS (S/.) GANANCIAS O


PÉRDIDAS (S/.)

1 988 73 517 68419 -h'.09'ó


1 989 195 614 113 417 .,\'?~¡q,
1 990 214 818 276 509 .( ) '_ o•
1 991 306 415 t 4-4~ _ "")Yi +58 407
1 992 318 716 'l ?. 1 ") (l Q -15 613

Según este cuadro contestar a las siguientes preguntas:

a) Al final de los 5 años, ¿,cuánto ganó? o ¿,cuámo perdió? ~


b) ¿,Cuánto fue el ingreso total logrado en los tres primeros años? ,q
c) ¿,A cuánto asciende el egreso total durante los 5 años?
d) ¿,En cuántos de los 5 años se lograron ganancias?
e) ¿,Cuánto se ganó o perdió en los últimos 3 años?

Solución:

Respuestas:
o
a) ~

b)

c)

d)

e)
1 ALFONSO ROJAS PUl:.\L\PE 1 ifrJ@Yál

(2) El dueño de una cadena de tiendas de venta de ropa. tiene la siguiente infonnación
acerca de las ganancias (cantidades positivas) o pérdidas (cantidades neg~1livas)
mensuales durante el año 1 992:

TIENDAS
MESE~ DEL
AÑO 1992 1 2 3 4 5 6

ENERO +573 -713 +618 +846 -:J24 -126


FEBRERO -815 +1 612 -116 +1 714 -1 804 +302
MARZO +617 -615 -67 -84 -10 +614
ABRIL +1 615 +819 -129 +647 -48 +517
MAYO -318 +768 -430 +259 -701 +313
JUNIO +816 -105 -437 +817 -820 +215
JULIO +715 +614 -508 -511 -613 +866
AGOSTO -607 +1 213 +101 +604 -781 +1 313
SETIEMBRE +1 810 -504 -406 -508 -659 +1 965+
OCTUBRE -468 ¼617 -507 -119 -808 +2 368
NOVIEMBRE +2 315 +908 +213 +205 -101 +3 515
DICIEMBRE +7 517 +6 313 +102 +763 -86 + 8769

a) ¡,Cuál o cuáles de las 6 tiendas deberían ser cerradas?


b) ¡,Cuál de las 6 tiendas tuvo la mayor gananua anual"! ¡_Cuál fue esta ganancia'!
e) ¡,Tuvo el dueño ganancia o pádida durante el mes de setiembre en todas sus
tiendas?
d) ¿Cuál de los 12 meses fue el más difícil'! ¡,Cuántopcrdió?o ¿cu,ínto fue lo poco
que ganó'!
Soluciún:

a)
_____
Respuestas:
__.

b) _____ _.

e)

d)
liW@Y;fl 1 MATEMÁTICA
1

RAZONAMIENTO MA TE1\1ÁTICO
'
t 1) Si ddinirnos la operaci11n * par.t n1ak¡u,er (6) Si : a D, h = 12a + h 3 + ahlª •
par 1k mí1n.·ros enteros del siguiente modo: ;,C'u:íl de las siguientes e,presiones es
x*y =3 x< 5y 111:tynr'!

Caln1lar: a)(5 D. 3) h)(I i:>, 2)


(-7) * (- 1) l") (X i:>, 4) d) (10 ,6 15)
e) Todas son iguales
a) 167 h\ 1-17 e) 152
d)5 e) 117 (7) Siemlo \ una opcraci1ln definida por:
xSy = xz - y3.
(2J Sean la, opcrac10ne< a y • definida~
nmm: a ,', h = 7a - 3ah + b 2 Calcular:
:1 • h = a -h 1( - 1) S (-2)1 S 1(+1) S (+2)1

Calcular el \'alor de: a) 22-1 b) 4-18 c)424


d) 22X c)420
1( -5) • (+3)1 .\ 1(+3) ó. ( -2)1
(X) Oatla la siguiente tahla de doble entr,Kla, ha-
a) 1 7111 l") 2 XX3 llar E ,i :
d) 2 X25 el 2 725
(3) Si,•,?_y=(,+ y)(xy) :
7 5 2
Calnllar el valor de: ª3 -1 -7 4
( - 1) r; ( - 2)
X X 3 -5
h) - 1 C) -6 3 7
l) -3
d) +t,
a) 1 b)3 e) 7 d) 4 e) 5
(-=i) l),•fim1111" la opcraci,,n 6 para niah¡uicr
par de números enteros de la siguiente manera: (9) Calcular cl \',1lor de P de izquierda a dere-
cha. si s,· sabe ,1ue la opcrnci1>n * est;Í
\ /._ B = _~,\ .\B deriimla por la sigmcnte tahla.
,c¡:un esto, calcular: 2 o 1
* -
-- 5
-
4 1 11 6 2
6 7 5 3 9
aJ --IX h) - 12 1.) +4X
() (, 4 5 4
di -IX c) -24
7 1 3 2 X
5) Si sc n1111pk quc: x *y= 2x + y 2
P=-1*2*1*0*5*1
llallar ten la si¡:uicntc iguakbd:
a) 6 e) 7 d) 12 e) 15
l( -2)*(+l)l+t=l(+3)*(-l)I
(10) Si :x #y= x +y
al 12 h) IIJ c)X X• y= X+ 2y
d)6 e) 15 Hallar: F = 1(3 # 2) # 71 * ((-3) * ( -2)1

a) -2 b) -1 e) 2 d) 3 e) -5
1 ALFONSO ROJAS PUÉMAPE 1 iiiMMPJi

Aquí mis operaciones de RAZONAMIENTO MATEMÁTICO.


ifr.iMjY¡tj MATEMATICA

CAPÍTULO 3

- -- ~

..

~
\
1
1

1 !1 PROPIEDADES
,, 1

1 DE LOS NUMEROS 1
1
!11
1
1 ;1
\

l'
~
=
- __),
1

OBJETIVOS:

• Reconocer si un número dado es divisible por otros, sin efectuar


división alguna.
• Estudio de los números primos y compuestos.
• E studio del MÁXIMO COM ÚN DNISOR Y DEL MfNIMO
COMÚN MÚLTIPLO, así como de sus aplicaciones prácticas.
1 ALFONSO ROJAS PUÉM.APE 1 iféM:41

ESTRUCTURA DEL CAPÍTULO:

í
/ t. DI\- ISIBILIDAD 4. MÁXIMO COMU N DI VISOR
1. 1 ¿Qué es DIVISIBILIDAD? (MCD)
1.2 ¡,Cuándo un número es divisible 4.1 Divisores comunes
por otro? 4.2 ¡,Qué es el MCD?
J .3 Múltiplo de un numero 4.3 Procedimiento para hallar el
1.4 Divisor de un número MCD:
SKANIPRÁCTICA 1 * Por "Golpe de vista"
* Por descomposición en fac-
2. CRITERIOS DE DIVISIBILIDAD tores primos
2.1 Divisibilidad por 2° * Método abreviado
2.2 Divisibilidad por 511 * Método de las di visiones suce-
2.3 Divisibilidad por 3 sivas (Algoritmo de Euclides)
2.4 Divisihilidad por 9 SKANIPRÁCTICA 4
2.5 Divisibilidad por 6
5. I\IÍNII\IO COI\IUN I\IÚLTIPLO
2.6 Divisibilidad por 7
(mcnl)
2.7 Divisibilidad por 11
5.1 Múltiplos comunes
SKANIPRÁCTICA 2
5.2 ¿Qué es el mcm?
3. NU:\IEROS PRII\IOS Y NÚMEROS 5.3 Procedimiento para hallar el
COMPUESTOS mcm:
* Por "Golpe de vista"
3. 1Número primo
* Por descomposición en fac-
3.2 Número compuesto tores primos
3.3 Números Primos entre sí (PESÍ)
* Por el método abreviado
3.4 Criba de Eratóstenes
* Por aplicación de la propie-
3.5 Cómo averiguar si un número dado dad que relaciona dos núme-
es primo. ros con su MCD y su mcm
3.6 Descomposición de un número en
una multiplicación indicada de sus SKANIPRÁCTICA 5
factores primos. RAZONEMOS
3.7 Divisores de un número LAS MATEMÁTICAS DE TODOS LOS DÍAS
SKANIPRÁCTICA 3 RAZONAMIENTO MATEMÁTICO
MATE1"IÁTICA

( 1. DIVISIBIUDAD)

Neces1tod1v1direstos
dos rollos de alam-
bre en parles ·;¡ua
les, lo más g•andes
posibles sin que
sobre alambre
,cuanto c!obo medir
cada parte?.

" ... DIVIDIR EN PARTES IGUALES, SIN QUE SOBRE


ALAMBRE ... ", expresiones como estas u otras parecidas,
son frecuentes en la vida diaria y están relacionadas con la
división exacta de dos cantidades que es el tema que nos
ocupa en adelante.

1.1 ;,Ql'É ES DIVISIHILIDAD?


Es una parte de la TEORÍA DE LOS NÚMEROS que analiza
las condiciones que debe tener un número para que sea DI-
VISIBLE por otro.

1.2 ;,CUANDO UN NÚMERO ES DIVISIHLE O ¡ IMPORTANTE !


POR OTRO? Si A es div1s1ble por B en-
tonces:
Un número A es divisible entre otro número B, cuando A AlL
contiene a H exactamente un número entero de veces. O K

Es decir: Si dividimos A entre B, el COCIENTE debe ser:


* EXACTO
1L yCooen1e
entero "'ª""
* NÚMERO ENTERO Residuo cero

Además, el RESIDUO debe ser cero. ~ 0


~o I ALFONSO ROJAS PUÉMAPE 1
Ejemplos:

(1) 114 es divisible entre 19


Porque al dividir 114: 19, el COCIENTE resulta ser 6
que es un número entero y el RESIDUO resulta ser O.

('.!) -27 es divisible entre 3


Porque al dividir-27: 3 el COCIENTE resulta scr-9quc
es un número entero y el RESIDUO resulta ser O.

{3) 87 NO es divisihk entre 2 O¡ CUIDADO!


Porque al dividir 87:2, el COCIENTE es exacto pero Si AesMULTIPLO DEBen-
NO ES Núrv1ERO ENTERO. tonces. A = KB
Ademas.
I.J i\l(LTIPI O DE t ·'.'\ :\L::\IEl{O A es d1v1s1ble por B
Acontiene exactamente a B
I
Por lo expuesto hasta aquí, 18 es divisible por 3 ¡,verdad? B es divisor de A
B está contenido exacta-
entonces 18 es l'vtÚLTIPLO de 3.
mente en A
Luego:
Un número A es MÚLTIPLO de otro n, si A
contiene a n un número exacto y entero de veces.

IA Dl\'ISOI{ DE l'N l\Ti\lERO


Si 18 es divisible por 3, entonces 3 es DIVISOR de 18.
Luego:
Un número B es DIVISOR de otro A .si H está
comen ido en A un número exacto y entero de veces.
Gráficamemc:

OHSERVACIONES: 00 ¡IMPORTANTE !

l. Todo número tiene infinitos múltiplos. Cualquier número tiene al


2. Todo número tiene una cantidad finita de divisores. menos dos divisores·
e oo La unidady el mismo número.
La excepción a esta regla es
3. Los DIVISORES también reciben el nombre de FAC-
el numero UNO.
TORES.
¿Porqué?
-'· El número UNO es DIVISOR o FACTOR de todos los
números.
5. El CERO es MÚLTIPLO de todos los números ME-
NOS de sí mismo.
1 MATEMATICA 1

l. Es los (si existen) de cada uno de los números dados:

B MÚLTIPLOS DE B

5
2

o
3
7
9
4

10

11. Escribir la mayor cantidad de divisores (llamados también factores) de los números
dados: (hallarlos por simple inspección o por sencillas divisiones).

A DIVISORES (0 FACTORES) DE A

8
o
20
36
90

111. Contestar las siguientes preguntas:

1. ¡,Por qué todo número tiene infinitos múltiplos?


2. ¡,Por qué el CERO es MÚLTIPLO de todos los mímeros menos de sí mismo?
3. ¿Cuántos divisores diferentes tiene la UNIDAD?
4. ¿Es CERO divisor de I O? Escribe las razones de tu respuesta.
1 ALFONSO ROJAS l'UÉMAPE ifa,@d:fi

Aquí mis respuestas de SKANIPRÁCTICA G)


1 MATEMATICA
1
e2. CRITERIOS DE DIV1SIB.IUDAD)

Para saber en forma inmediata si un número es divisible entre


otro, en algunos casos no es necesario efectuar la división co-
rrespondiente, porque bastará conocer algunas característi-
cas de tal situaci6n de divisibilidad; a estas características las
conoceremos como CRITERIOS DE DI VISIBILIDAD que O ¡CUIDADO!
son los siguienLes:
2 n es una POTENCIA de 2
Si n = 1 dicha potencia es 2.
2.1 DIVISIBILIDAD POR 2" Si n=2d1chapotenc1aes4. 1

Si n = 3 dicha potencia es B.
Un número es divisible por 2" si termina en n ceros
o si las II últimas cifras forman un númerodivi ... iblc
./
por 2". C::::' O

CASOS PARTICULARES.-

* Paran= 1: (entonces 2" será 2 1 ó 2)


Un número es divisible por:? si termina en un
cero o la última cifra es un número divisible
por 2. C::::' 00
Ejemplos:
16; 748; 600; 17004 son númerosdivisihlcspor2 porque
terminan EN CERO o EN CIFRA PAR.

* Paran= 2: (entonces 2" será 22 ó 4)


Un númcroesdivisiblcpor4 si tcnninaendos
ceros o las dos últimas cifras forman un
número divisible por 4.
Ejemplos:
1 300; 128: 316; 100 son números di\"isihlcs por 4,
porque sus do( últimas cifras son CEROS o forman un 00 ¡ATENCIÓN!
mímero que es múltiplo de 4 .
Es lo mismo decir·
* Paran = 3: (entonces 2" será 23 ó 8) "Número divisible por 2"
Un número es divisible por 8 si termina en que: "Número múltiplo
tres ceros o las tres últimas cifras forman un de 2'' y éstos se llaman
número divisible por 8. NÚMEROS PARES.
Ejemplos:
1000;7016;5000; 10088 sonnúmcrosdivisiblespor
8 porque terminan en tres ceros o las tres últimas cifras
forman un número múltiplo de 8.
1 ALFONSO ROJAS PUE\IAPE 1
., ., 01\'ISIUILIDAI> POR 5n
O RECUERDA QUE:
Un número es divisible por 5º, si termina en n ceros o si
las n últimas cifras forman un número divisible por 5". Es lo mismo decir:
"Numero divisible por 25"
que ''Numero mult1plo de
CASOS P..\RTICLILARES.-
25".
* Paran= 1: (entonces 5" será 5 1 ó 5)
Un número es divisible por 5 si tem1ina en
CERO o si la última cifra es 5.
Ejemplos:
50; 75 ; 10 065 ; 732 560 :-.on números divisibles por 5
porque terminan en O ó en 5.

* Paran = 2: (entonces 5" será 5 2 ó 25)


Un número es divisible por 25 si las 2 últimas
00 RECUERDA QUE:
cifras son ceros o si las 2 últimas cifras
forman un número divio;ibk por :!5. C:::: ' O s, 70 000 es divisible por 125
entonces 70 000 es mult1plo
Ejemplos: de 125 ó tambien: 125 es
divisor de 70 000
575; 800; 76 525; 786 5(Xl

* Paran= 3: (entonces 5" será 53 ó 125)


Un número es divisible pur 125 si las 3
últimas cil'ras son ceros o si las tres últimas
cifras form~m un número divisible por 125.
Ejemplos:
700(X); 5 250: 3 000: 152 375 son números divisibles por
125 ó múltiplos de 125 porque terminan en tres ceros o las
tres últimas cifras forman un número múltiplo de 125. @
w
C:::: 00

2.3 DIVISIBILIDAD POR 3

Ejemplos:
Un númcroesdi\'isible por 3, si la SUMA DE SUS
CIFRAS da un mímero múltiplo de 3. "
000 RECUERDA
SIEMPRE QUE;
1
(]) 178 407 S1 178 407 es divisible por
3 entonces 178 407 es
Sumando las cifras: MÚL T/PLO de 3 6 3 es
DIVISOR de 178 407
1 + 7 + 8 + 4 +O+ 7 = 27 que es múltiplo de 3.
Luego 178 407 es divisible por 3. e:::> 000
MATEMATICA
1
(2) 1 101 11 1
Sumando las cifras:
1 + 1 +O+ 1 + 1 + 1 + 1 = 6 que es múltiplo de 3.
Luego 1 101 111 es divisihlc por 3.

2.-t DIVISIBILIDAD POR 9


{:=::•:: O ¡CUIDADO'
Un número es divisible por 9, si la SUMA DE SUS ~-=-·
C'IFRAS da un número múltiplo de 9. \ Si Aes MÚLTIPLODEBen-
T tonces: A "' KB
[ Además:
Ejemplos: t, A es divisible por B
~::' AcontieneexactamenteaB
(1) 57 231
~' B es divisor de A
Sumando cifras: B está contenido exacta-
{. mente en A
5 + 7 + 2 + 3 + 1 = 18 que es múltiplo de 9. C O

Luego 57 231 es divisible por 9.


(2) 707 454
Sumando cifras:
7 +O+ 7 + 4 + 5 + 4 = 27 4ue es múltiplo de 9.
Luego 707 454 es divisible por 9.

2.5 DI VISIBILIDAD POR 6

Un número es divisible por 6,si lo es también por 2


y por 3 <;imultáncamente.
00 ES IMPORTANTE.
Ejemplos: ... recordar que un número es
(1) 1 068 { divisible por 2 s, termina en
CERO o en cifra PAR.
* ¿Es divisible por 2?
Sí, porque termina en cifra par ~ '> 00

* ¿Es divisible por 3?


Sí, porque la suma de cifras es 1 + O + 6 + 8 = 15
que es múltiplo de 3.
Luego 1 068 es divisible por 6.
(2) 53 670

* ¿Es divisible por 2?


Sí, porque el número dado termina en CERO.
1 ALFONSO ROJAS PUEMAPE 1
* ¿Es divisible por Y.'
Sí, porque la suma de todas sus cifras, da un O ¡MUY IMPORTANTE!
número que e,; múltiplo de 3; es decir: é' Si un número es divisible por 6, lo
5+3+6+7+0=21 .r es también por cada uno de los
,,- factores o divisores de 6.
Luego, 53 670 es di\ isiblc por 6. ~ O
En General:
S, un numero A es divisible por
2.6 DIVISIBILIDAD POR 7 otro B, entonces A tambien es
d1vis1ble entre cada uno de los
Un número será divisible por 7 si cumple con la factores o divisores de B.
siguiente regla:
Multiplicamos cada una de las cifras del número
dado de derecha a izquierda por los siguientes
factores:
I; 3; 2;-1; -3; -2; I; 3;2 ;-l;-J;-2; ... etc
• Sumamos los mínll·ros cilleros obtenidos. Si el
rl•,tJltado final es CERO o múltiplo de 7 el núnwro
dado ,;erá emonces divisible por 7.

Ejt>mplo: ¿Es 626 934 di\'isiblc por 7?


Veamos:
6 2 6 9 3 4
j, j, j, j, j, j, X
¡ATENCIÓN!
2 -3 -1 2 3
Si un número termina en
-12 -6 -6 + 18 +9 +4 CERO, es d1v1s1ble por 10.
Sumando los enteros obtenido,: -12 - 6 - 6 + 18 + 9 + 4 = 7 Si un número termina en
Luego 626 934 es di\'isiblc por 7. dos CEROS, es divisible por
100.
2.7 DIVISIBILIDAD POR 11
S, un número termina en
Un mímero será divisible por 11 si la suma de sus cifras tres CEROS, es divisible por
ck orden impar (cmpcnmdo por la dcn:cha) menos la 1 000.
suma de las cifras de orden par, resulta ser CERO o
m1íltiplo ck 11.

Ejemplo: ¡_Es 9 873 226 divisible por 11?


S1111ll'lllOs primero las cifras de orden impar
a partir de la cifra ck las unidades: 6+2+7+9=24 . ..... (1)
Sumemos luego las cifras de orden par a
2+3+8=13 ...... (2)
partir de la cifra de las decenas:
24 - 13 = 11; luego 9 873 226 l.!S
Restemos ahora ( 1) - (2):
divisihle por 11.
1 1\-lATEMATICA
1
SKANIPR1\CTICA CD
l. Marca con un aspa si consideras que el número A de la columna izquierda es divisible
por alguno de los números de la fila horizontal superior:

3366 X X X X X

72110 ' y
4496 Y, X
392 X
2 585

6180 X
2528 y
5080 )<

2235 ._,
48265 ,

43 767

8046 V
775

69575

5 712 \.-"'

3588 )'.

18046

49347

104 265 ('


3401734

626 813

455 792 '(

~=--•·_.' :;:: ·,::; -.-; --~


~ .-:- ·:=:=:· ~:;::::-:--::-:=: :-:•·.- :=:-: ·-.:-:-:-·-.-:::;:,:.
1 ALFONSO ROJAS PUÉMAPE 1

Il. Completa el siguiente cuadro escribiendo deh.~jo de 2~3; 4; 5;etc.,los menores números
que reempl.11.ados por separado en el casillero del número A de la columna izquierda,
la conviertan a su turno en múltiplo de 2; 3; 4; 5; etc. Si no es posible entonces, marcar
con un aspa en el casillero correspondiente.

NÚMERO A
1 l l lJ6 l liJ¿J
2 3 4
- -
1
-

741 □ o o 2 o o 3 6 o
279 □ 2 o 1- e,.
-
I
:> 3 ()
742 □
52450 ?-
'2
<2,
J
2
n
e,_
?
J
....
--
..___
.,...
t:; o
f')
-
302 ,1 1 I -¡ ~
<
~
457 □ ~ 2 1 s ,/ ~ ) ú
7300 ll ~ q -::J.. 9 d
5236□ -2 '2. u < '2 o 2
714 380 l'L ~ é. o .J e LJ (()

5 612 305 xt r j. - ..;._,


8615 90 o z '2 li 0 '2
3 210 386 o A x -1
52111 D ~ t''], ~ ({)

8415 602 '3 t ',( 'Y


,.
72 315 48□ 'v C- ( J
67 819 605 },_ 0 X
52 318 76□ o ó
95 311 x1D t)
Lf
J\,L\TEMÁTICA

111. Si el número 415 350 es divisible 1x1r 3, entonces la división 415 350: 3 es exacta,
luegodecimosque4 l 5 350liene tercia; si el mismo número es divisible por 5,entonces
tiene quinta y así sucesivamente. Según esto llenar el siguiente cuadro, marcando con
un SI o con un NO según tenga o no TERCIA, CUARTA, QUINTA, etc. el número
dado en la izquierda .

MITAD TERCIA CUARTA QUINTA SEXTA NOVENA

58950 SI SI NO SI SI
SI
418 f: p 1 r
2 544

72950

646 516 -

318 714

52 618

715 625

71 416

808 708

111111

I \'. Contesta las siguientes preguntas:


( 1) b1.:rihe 5 números que sean divisibles por 15. ¿Tus cinco números son tamhién
divisibles por .~'! ¿Lo son también por 5'!
(2) Escribe 8 números que sean múltiplos de 10. ¿Tus ocho números son también
divisibles por 2? ¿Lo son también por 5?
(3) Escribe 10 números que sean múltiplos de 14. ¡,Tus diez números son también
divisihk-. por 2'! ¡_Lo -.on también por T!
(4) Escrihe 12 números que sean múltiplos de 12. ¡_Tus doce números son también
múluplos de 2·! ¡_Lo son tambi.:n lk 6"!
(5) Cambia por otra la cifra de las centenas del número 73 543 pam que el número
sea múltiplo de 3.
(6) ¿Cuánto deberíamos agregar como mínimo al número 71 315 para que -.ca
múltiplo de 7'!
º"
4
J AlPONSOROJASPUÉMAPE , ___

Aquí mis operaciones de SKANIPRÁCTICA 2


iif.'M!MI 1 MATEMATICA
1

e3. NÚMEROS PRIMOS Y NÚMEROS COMPUESTOS)

Claroquesí,sison
SKAN I TO,
12 podemos colo-
¿es más lacil
car 1 fila de 12 ó 2
encajonar 12
filas de 6 ó 3 lilas
botellas que
de 4, si son 10
10? _..,.::::::.;::::,._ _ _..:::;;......;:........i podemosdisponer
·~,_.,..-.....;,._ __,_____..._---1 sólo 1 fila de 1O ó
2 lilas de 5

¿Y si fueran
13 botellas
las que tuvie-
ramos que
encajonar?

Tengo menos posihil idades en guardar decenas que docenas


de objetos. Peor aún si son 11 objetos, pues éstos podemos
disponerlos sólo en I fila de 13, ya que si ensayamos
cualquier otra manera de colocarlos en una caja rectangular
por ejempllJ, siempre sobrará o rallará algún objeto.

La razón decstors muy scncilli el número n sólo purdc ser


descompuesto comü LL 1; dicho de otro modo, n no admite
más divisores que la unidad y el mismo. Estamos tratando
emonces con un NÚl\lERO PRIMO. O 1ATENCIÓN!

3.1 r-;úl\1ERO PRII\1O Oy 1


No son números PRIMOS
Llamado también PRIMO ABSOLUTO. es aquel mínk'm
ENTERO y POSITIVO que se deja dividir sin residuo por 2
números entl~ros: LA UNIDAD y EL MISMO. ~ O Es
decir: Los números PRIMOS tienen sólo 2 divisores.
•O· 1 ALFONSO ROJAS PUÉMAPE 1
Algunos de estos números primos son:
2 ; 3 : S ; 7 ; 11 ; 13 ; 17 ; 19 ; 23 ; 29 ; cte.
O ;CUIDADO!
3.2 1':Úl\1EROS COMPUESTOS
Si dos números son PRI-
Se les llama así,a los números ENTEROS y POSITIVOS que MOS ENTRE SI (PESiJ no
tienen más de dos divisores. es necesario que cada uno
Ejl'm1>los: de ellos sea NÚMERO
PRIMO ABSOLUTO.
4 ; 6 ; 9 ; 10 ; 12 ; 14 ; 16 ; cte.

3.3 NÚMEROS PRll\IOS ENTRE SÍ. (PESÍ)


Llamados también PRIMOS RELATIVOS, son aquellos
que sólo tienen 1 DIVISOR COMÚN: La UNIDAD C::::> O
Ejemplo: C:::: 00
oo¡ IMPORTANTE!
1 NÚMERO DIVISORES bes DIVISOR (o factor) de a
si b divide exactamente a a.
14 1 : 2: 7; 14 Asl.
25 1; S; 25 2 es divisor (o factor) de 14
18 1;2;3;6;9:18

Según la Tabla: 14 y 25 son PESÍ(únicodivisorcomún: la unidad)


25 y 18 son PESí (únieodivisorcomún: la unidad)
14 y 18 NO SON PESÍ
J.-1 CRIBA DE ERATOSTENES
Es un modo de formar la tabla de los números primos que no
son mayores que un número dado N.
Para esto escribamos los números del 1 al N.
RECUERDA:
1 2 3 4 s 6 7 8 9 10 FACTOR es lo mismo que
11 12 13 14 15 16 17 18 19 20 DIVISOR de un número.
21 22 23 24 25 26 27 28 29 30
31 32 33 34 35 36 37 38 39 40
41 42 43 44 45 46 47 48 49 so
51 52 53 54 55 56 57 58 59 60
61 62 63 64 65 66 67 68 69 70
71 72 73 74 75 76 77 78 79 80
81 82 83 84 85 86 87 88 89 90
91 91 93 94 95 96 97 98 99 100
...............................................
............................................. N
1 J\lATEMATICA
t
• En esta tabla suprimimos el l PORQUE NO ES PRI-
MO. t:;>o
• Tomamos el primer primo que es 2, (porque es divisible
O ¡CUIDADO!
por sí mismo y la unidad).
• Suprimimos de la tabla todos los múltiplos de 2, excepto Un NÚMERO PRIMO tie-
el mismo 2. ne dos divisores
Un NÚMERO COMPUES-
• De los que no hemos suprimido, el primer número es 3
TO tiene más de dos d1-
que es primo.
v1sores.
• Suprimimos de la tabla todos los números que sean ~I 1 es un caso especial
múltiplos de 3, excepto el mismo 3. porque NO ES PRIMO, pe-
• El primer número no suprimido es 5 que también es ro tampoco es COMPUES-
primo. e;> 00 TO.
• Suprimimos ahora todos los múltiplos de 5 excepto el
mismo 5 y así sucesivamente.
• TODOS LOS NÚMEROS NO SUPRIMIDOS serán
primos.

Es decir, en una tabla del I al 100 tendremos los siguientes


números primos:

2 3 5 7
lJ 13 17 19
23 29
31 37
41 43
53
47
59 (:\- !'
61 67
71 73 79
83 89
97

3.5 ¿CÓMO AVERIGUAR SI UN NUMERO DADO


,,,, RECUERDA QUE:
ES PRIMO?
Un NÚMERO es PRIMO
sólo si es DIVISIBLE entre
Veamos un ejemplo: 149
sí misno y entre la UNI-
PRIMERO: Extraemos la raíz cuadrada del número dado, DAD.
tomando sólo la parte entera. Así:
5 es divisible entre 5.
Es decir: -{149 = 12, 0000 5 es divisible entre 1.
pero sólo tomamos 12. Luego, 5 es pnmo.

SEGUNDO: Dividimos el número dado entre todos los


números PRIMOS menores o iguales a 12 (en
este caso, no tomanos 12 porque no es primo).
1 ALFONSO ROJAS PUE\IAPE 1
Así: 149 Ll_ 149 Ll_ 149 Ll_ 149 L1._ 149 l.!!_
Q)74 @49 @29 @21 @ 13 ,, ¡ATENCIÓN!

TERCERO: Si todas las divisiones efectuadas son inexac- Como FACTOR es lo


tas, el número dado es PRIMO. En este caso mismo que DIVISOR, un
149 es NÚMERO PRIMO. FACTOR PRIMO, es un
divisor del numero dado
3.6 DESCOMPOSICIÓN DE UN NÚMERO EN UNA quealavezesNÚMERO
MULTIPLICACIÓN INDICADA DESL'S FACTO- PRIMO.

RES PRIMOS (TEOREMA DE GAUSS)


Cualquier número compuesto, puede ser expresado como la
multiplicación indicada de sus factores primos, elevados a
exponentes enteros y positivos.
Veamos el procedimiento a seguir, en el siguiente ejemplo:
Descomponer 540 en sus factores
primos. t;> O
Iº. Se escribe el número a descomponer. colocando una 540
raya vertical a su derecha:
540 2
2º. A la derecha de esta raya vertical se escribe el número
270 2
primo menor que divide exactamente a 540: Facrnrcs o
135 3
divisores primos
3º. El cociente de dividir 540: 2 se escribe debajo de 540. 45 3 de 540
4º. Volvemos a proceder como en el 2° paso y así suce- 15 3
<;ivamente hasta que el último cociente sea 1. 5 5
Luego 540 se escribe como la multiplicación indicada de
estos factores primos, es decir: 540 = 2 x 2 x 3 x 3 x 3 x 5
o mejor aún: 540 = 21 x 33 x 5 t;> 00

3.7 DIVISORES DE UN N(MFRO o,, ¡IMPORTANTE/


Si un numero compuesto N, se descompone en sus factores A esta descomposición
primos del siguiente modo:

t
de un numero compuesto
entero y positivo. también
N = A'" .B"-C';dondeA.B yCson númerosprimos,setiene se/e/lama DESCOMPO-
que el número de DIVISORES (primos y no primos) de N SICIÓN CANÓNICA yes
está dado por la siguiente fórmula: única para cada número.
1 Nº de divisores = (m + J)(n + J)(r + 1) 1 ~U;'
Ejemplo: Si 540 = 22 x 33 x 5 ;
El número de divisores (primos y no primos) de 540 es igual a (2~\+ 1;1 + !) = 24
E~ponentcs de los factore, primos
en la descomposición canónica.
MATEMATICA

SKANIPRÁCTICA @
l. MarcarconunaspasegúnelnúmerodadoseaPRIMOABSOLUTOoCOMPUESTO.

NÚMERO PRIMO COMPUESTO NÚMERO PRIMO COMPUESTO


7 X 341
24 X 311
111 321
173 409
187 413
119 477
213 419
217 509

11. Escribir todos los divisores de los números dados y formar las parejas de dichos
números cuyos elementos sean números primos entre sí.

NÚMERO DIVISORES NÚMERO DIVISORES

12 45
15 48
28 50
33 54
42 55

Pares de números pri-


mos entre sí: 15 y 28:

111. Descomponer canónicamente los siguientes números y establecer la cantidad de


divisores de dichos números.
(1) 120 (8) 520 (15) 1 200 (22) 3600
(2) 240 (9) 400 (16) 1580 (23) 3800
(3) 90 (10) 480 (17) 1 620 (24) 4500
(4) 180 (11) 560 (18) 1840 (25) 5200
(5) 240 (12) 724 (19) 2000 (26) 6000
(6) 300 (13) 846 (20) 2380 (27) 6800
(7) 360 (14) 900 (21) 2600 (28) 7200
1 ALFOl\'SO ROJAS PUÉM-\PE 1

Aquí mis operaciones de SKANIPRÁCTICA G)


1 MATE!\lÁTICA
1
( 4. MÁXIMO COMÚN DIV1SOR (MCD))

4.1 DI\ ISORES COI\IUNES ,, RECUERDA QUE:


Un número p > 1 es divisor común de los enteros m y n
FACTOR oDIVISOR deun
si p divide a m y p divide a n. número expresan lom,smo.
Veamos un ejemplo: Si 2 es DIVISOR de 8,
entonces lo divide exac-
Busquemos todos los divisores de 12 y 30 por separado, tamente, es decir:
esto lo escribiremos en una tabla: Qo
8:2 = 4

NÚMERO DIVISORES
30 Q);@;G); 5 ·®
, ·, 10 , 15 , 30

12 Q);@;G); 4 ·®, ·, 12

¿Cuáles de estos divisores, lo son simultáneamente de 30 y 12?

Los que veo en el cuadro son


1: 2: 3y 6. De todos ellos 6 es
el MAYOR DIVISOR .

Esto significa que si 30 y 12 fueran las longitudes en


centímetros de dos cintas de alambre, entonces:
* Podemos cortar ambas cintas en pedazos de 2 cm cada
uno. ¡Y NO habrá sobrante en ninguna cinta porque 2 es
di visor común de 30 y 12 !
* Podemos cortar también ambas cintas en pedazos de ,,o ¡IMPORTANTE!
3 cm cada uno y tampoco habrá sobrante en ninguna cinta
porque 3 también es divisor común de 30 y 12. Los divisores comunes
* La medida más grande de cada pedazo de alambre en las menores son también
dos cintas deberá ser 6 cm para que no haya sobrante, ya divisores del MCD.
que 6 es divisor común de 30 y 12.

..t.2. ;.Ql É ES EL l\lÁ '\.1:\10 COMÚ:-..1 Dl\'ISOR O l\lCD"?


EIMCDdedosomásnúmerosnaturales,eselMAYOR
DIVISOR común de los números dados.
En el ejemplo anterior, el MCD de 30 y 12 es 6.
Esto último lo escribimos con mayor comodidad así:
MCD (30 y 12) = 6 Qoo
1 ALFONSO ROJAS PUÉMAPE 1

4.3. PROCEDIMIENTO PARA HALLAR EL MCD.


Podemos hallar el MCD de varias formas.

A.- POR "GOLPE DE VISTA".


Aplicamos aquí algunos principios sencillos.
Ejemplos:
(1) ¿Cuál es el MCD de 6 y 3?
Observando 6 y 3 nos damos cuenta que el ,, ¡CUIDADO!
MAYOR número que divide a ambos a la vez es En general se cumple que:
3. c;>o Si un número es divisible
¿Notaste ya que 6 es divisible entre 3? entre otro, EL MCD DE
Si esto ocurre, entonces el MCD es el menor, es AMBOS, ES EL MENOR DE
decir: 3 ELLOS.
(2) ¿Cuál es el MCD de 28 y 7?
Permíteme contar con tu participación.
Completa con un SI o con un NO según estimes
conveniente:
~
* 28 es divisible entre 7
* 28eselMCDde28y7 ~
* 7 es el MCD de 28 y 7 ~
(3) ¿Cuál es el MCD de 24 y I 8? -s;( ¡Alerta! )
24 no es divisible entre 18, entonces podemos
aplicar la siguiente propiedad:
"Si A no es divisible entre B, el MCD de ellos
es igual al MCD del número menor B y r, donde
r es el residuo de dividir A entre B".
En nuestro ejemplo, ¿cuál es el RESIDUO de
dividir 24: 18? Tal RESIDUO es 6, entonces si
aplicamos la propiedad, tendremos:
MCD(24 y 18) = MCD (18 y 6)
Por lo visto en el ejemplo ( 1): MCD (18 y 6) = 6
Luego: MCD (24 y 18) = 6
( ¡IMPORTANTE!
En general,SI DOS OMÁS
1
Ayúdame con el siguiente ejemplo: NUMEROSSONPRIMOS
ENTRE SI, SU MCD ES
(4) ¿Cuál es el MCD de 36 y 24? LA UNIDAD.
Completa con un SI o con un NO:
* 36 es divisible entre 24 D
* El residuo de dividir 36 : 24 es 12 D
* El MCD (36 y 24) es igual que el MCD (24 y 12) D
* El MCD (24 y 12) = 12 0
* Luego el MCD (36 y 24) = D
1 MATEMATICA
1
(5) ¿Cuál es el MCD de 7 y 5'!
El único divisor común que tienen amhos números O IMPORTANTE
es 1, luego su MAYOR divisor común es 1. En general: Si dos o mas
MCD (7 y 5) = 1 e:> O números son primos entre sí.
su MCD es la unidad: así:
Recuerda además que si dos MCD(72 y 25) == 1
números tienen como único divisor
común a la unidad, tale$ números
son PRIMOS ENTRE SI.

(6) ¿Cuál es el MCD de 12 y 25?


Completa con un SI o con un NO:

12 es número primo D 12 y 25 son primos entre sí D


25 es número primo D El MCD de 12 y 25 es 1 D
B.- POR DESCOl\IPOSICIÓN EN FACTORES PRII\IOS.
(a} Descomponemos los números dado" en "us laclorcs
primos.
(b) Extraemos los factores SÓLO CO'.\ 11 1 NES con su 0 0 RECUERDA QUE:
MENOR exponente.
(c) El producto de estos factores CO\llJNES es el A la descomposición de un
número en sus factores pri-
MCD buscado.
mos también se le conoce
Ejemplos: como DESCOMPOSICIÓN
CANÓNICA.
( I ) Hallar el MCD de 30 y 2--l.
Solución:
1() 2
• Descomponemos 30 en sus l:'i 3
factores primos: 5 5
1
Luego: 30 = 2 x 3 x 5 ... ( 1)
2-t 2
• Descomponemos 24 en sus 12 2
factores primos: [.,) () 2
Luego: 24 = 2J x 3 ........... (2) 3 -'
1
• Los factores COMUNES con su MENOR expo-
nente en ( 1) y (2) son 2 y 3.
PROPIEDAD
• El MCD estará dado entonces por 2 x 3 ó (l.

MCD(30; 24) = 6 Si multiplicamos o divi-


dimos varios números
(2) Hallar el MCD de 120; 350 y 2--l0. por una misma cantidad,
Solución: su MCD también queda
• Descomponemos los tres mult1pflcado o dividido
1·20 = 2'x3x5
por esa misma cantidad.
números. cada uno en sus 350 = 2 X 5! X 7
factores primos:~ 00 ~-l() = 2~ X 5 X 3
1 ALFONSO ROJAS PUEMAPE 1
• El MCD está dado entonces
porel PRODUCTO DELOS
FACTORES COMUNES MCD( 120; 350; 240) = 2 x 5 =[!Q}


con su MENOR exponente:
C.- POR EL MÉTODO ABREVIADO. ( O ¡CUIDADO!
Aquí buscamos en forma directa y simultánea SÓLO Este método es aplicado
los factores comunes. Ejemplos: .:; al cálculo del MCD sólo
(1) Hallar el MCD de 120; 350 y 240. I de dos números. Si son
tres o más, se calcula el
I 20 - 350 - 240 2 Luego: )t MCD de los dos prime-
ros, Juego se calcula el
60 ¡75 120 5 MCD(l20; 350; 240) = MCD del resultado ante-
12 35 24 2 X 5 =[IQ]
rior con el tercer número
y así sucesivamente.
Detenemos el proceso cuando ya no hay factores comunes
o cuando los elementos de la última fila ( 12; 35 y 24) son
PRIMOS ENTRE SÍ.
(2) Hallar el MCD de 30; 84 y 66
Completa y comprueba:

30 - 84 - 66D El MCDesO xD ¡MUY IMPORTANTE/

D D D D Es decir: Si se divide a varios

D D D números entre su MCD,


MCD(30; 84; 66) =6 los cocientes obtenidos
son números PRIMOS
D.- POR EL MÉTODO DE LAS DIVISIONES SUCESIVAS. ENTRE SÍ.
Ejm:
Llamado también ALGORITMO DE EUCLIDES. ~ O 32 y 18 su MCD es 2
Veamos el procedimiento en un ejemplo: 32:2 = 16
Hallar el MCD de 30 y 24. 18: 2 = 9
16y9son
Solución: PRIMOS ENTRE SÍ.
Por divisiones sucesivas:
l O
Se colocan en fonna horizontal
• 1 Cocientes
los dos números dados: 1 30 24
2°. Se divide 30 : 24 escribiendo el
1 Residuos
cociente y residuo donde indica el
cuadro: 1 Cocientes 1
3°. El residuo obtenido en la primera 30 24
1
división pasa a ser divisor,
efectuando una nueva división de 1 Residuos 6v
24: 6, procediendo como en el 2° Cocientes 1 4
1
paso. El último residuo que NO
ES CERO es el MCD buscado. 1 30 24 6
En este caso MCD(30 y 24) = 6 1 Residuos 6 o
1 MATEMÁTICA
1
SKA'.':IPRACTICA C±)
l. Considerando 4uc un DIVISOR di\'idc exactamente a un número dado completar el
siguiente cuadro hasta hallar el MCD de los números señalados aplicando sólo el
concepto de MCD.
DIVISORES COMUNES a:
NÚMERO DIVISORES 36 y27 40 y 18 38 y 30 72 y40 45y 30 42 y 32

72

38

45

36

40

32

27

18

30

42

MCD ◊
11. Calcular el I\ICD de los siguientes números por "golpe de vista":

NÚMEROS 5y3 6y3 12y 4 7y 8 3y4 18 y 3 18 y 6 24y 5 16y 12

MCD
NÚMEROS 20y 12 9 y 11 12 y 25 13y 14 32y 12 30y 18 45 y 20 13 y 2 16y 14

MCD

111. Hallar el MCD de los siguicmes números aplicando DESCOMPOSICIÓN CANÓNI-


CA y comprueba tu rcspucslll hallando el mismo MCD por el Método Abreviado:
(1) 60y90 (5) 35; 70 y 80 (9) 25; 40; 15 y 80
(2) 32; 40 y 50 (6) 45: 85 y 100 (10) 16; 30; 64 y 72
(3) 54'. 80 y 64 (7) 12:60y72 (11) 180: 300; 240 y 360
(4) 18;64 y 72 (8) 18, 60 y 54 (12) 720; 400; 520; 800 y 640

·.·.•·. •.··-· .. .,•,., ••••••••• •-:-•.•,,•,·.·,·••••••••, ·••••••,••"'•"•"•v•.••.T.-:-.•.• ·-.• • •••• -.Y1.•-.•·-:-.-,•.v·•.v·v•.v.•,•· • .-. ··-·-· ......-
1 ALFONSO ROJAS PUÉMAPE
1 112'M/UI
Aquí mis operaciones de SKANIPR ·\CTICA0

.·==~•-.:=::: ·:;;;:::r.-=:::::.::: ::i:-:i=?: :::~:::-:=:=-~--~:·•·:· ·¿:::-:;/;:~H--'=::::;::::::~:-:-:::::::::::.:~:-~-.:=:=:=:~·:-:::::::«::::=::..:.:. · :~~=:-:::::. ·:-:=:~:=:=:=~:=:~::=;:-::\'::::-=:~:::::~


;.·~¿y:,,.:-;. ,ci:~.-:-:.-..-:..::*'="".-:---· - -:-:-:-:-·--...,.::w,--v:,·.--:,;.:-:=:&:-·i•~=-:--..:-: • :-. ·. -,:-:-:-:-:-·-·-:-:-·--..:v,;-;-- i :-:-:--:-- :-:~-- -._ · ·-'.'11:
1 MATEMÁTICA 1

e5. MÍNIMO COMÚN MÚLTIPW (MCM} )


S. l :\IL LTIPLOS COMUNES.
Si m es múltiplo de p y también lo es de q entonces m es ( RECUERDA QUE:
múltiplo común de p y q.
Veamos un ejemplo: 18 es MÚL T/PLO de 2 por-
B usq ucmos algunos m úlli plos de cada uno de los números 12 que 18 contiene exacta-
y 18, los cuales escribiremos en la siguiente Tabla: mente 9 veces al número 2
18=9x2
NÚMERO MÚLTIPLOS As, también decimos que
2 es DIVISOR de 18
12 12: 24: 36; 48; 60: 72; 84: 96; 108; 120; ...
18 18;36;54;72;90;108;126;144; . . . o o
¿Cuáles de estos múltiplos lo son a la vez de 12 y 18?
o ¿cuáles son los múltiplos comunes a 12 y 18?

>Q
-"
Según el cuadro, puedo notar que los mult1plo1
comunes de 12 y 18 10n 36· 72; 108: bueno,
;V~\ ·no habria cuando acabar de nombrarlos. pero el
·¡:.:1' MENOR de estos mul!,plos comunes es 36.
::¡

Usemos los múltiplos comunes de 12 y 18 del siguiente


modo:
Supongamos que un padre y su hijo Licncn ocupaciones taks
que el primero sólo está en casa cada 18 días y el hijo cada O ¡ IMPORTANTE I
12. Si el día 1º de enero están ambos reunidos en familia, se
Los MUL TIPLOS de un
volverán a encontrar dentro de 36 días o dentro de 72 días o numero conforman un
dentro de 108 días así sucesivamente. con1unto INFINITO
¿Por qué sería la fecha más próxima de reencuentro dentro de Los DIVISORES (o fac-
tores) de un numero,
36 días a partir del 1° de enero?
conforman un conjunto
Porque en ese transcurso, el padre habrá regresado a casa 2 FINITO
veces exactamente y el hijo 3 veces exaclllmcnte. Es decir, el
MENOR número que contiene exactamente a 12 y 18 es 36.

5.2 ;.QUE ES EL MÍNIMO COMUN ML:LTIPLO O mcm?


El mcm de dos o más números naturales es el menor
MúLTIPLO COMúN de los números dados.
En el ejemplo anterior el mcm de 12 y 18 es 36.
Esto último lo escribimos con mayor comodidad así:
mcm(l2 y 18) = 36
1 AlFO~SO ROJAS PUÉ\1APE 1
5.3 PROCEDl'.\lIENTO PARA HALLAR EL mcm.

Hallamos el mcm de varias formas así:

A. Por "GOLPE DE VISTA".


Hay situaciones en las que sin mayor méLodo, podemos
hallar el mcm de dos números.
Ejemplos: O ¡CUIDADO!
(1) ¿Cuál es el mcm de 5 y 6? S1 dos numeras A y B
Veamos los primeros múlLiplos son PRIMOS ENTRE
s,; entonces su mcm.
5; 10; 15: 20; 25; 30; 35; 40; ... es el PRODUCTO de
los numeras A y B
Ahora los múlLiplos de 6: Luego.
6; 12; 18; '.!4; 30; 36; 42: 48; ... mcm (A y B) .. A ~ B
Para A y B PESí
De ellos el primer múlLiplo común que Lamhién es el
MENOR MÚLTIPLO COMÚN o el ml"m es JO. ~ O
Pero30es 5x6,esdccir: mcm(5 y6) = 5 x6 yestoocurr~
cuando los números dados son PRIMOS ENTRE SI.
ÓO
(2) ¿Cuál es el ml"m de 12 y 25?
EsLos números Licncn como único di\'isor común a la
UNIDAD, enLonces son PRIMOS ENTRE SÍ (PESÍ),
luego podemos escribir dircctamcnle que:
mcm(12 y 25) = 12 x 25
o mcm(l2 y 25) = 300
(3) ¿Cuál es el mcm de 18 y 3?
Observa que aquí 18 es múltiplo de 3 y es múlLiplo
de sí mismo, cnLonces el mcm de ambos será 18, es
decir: mcm(18 y 3) = 18 O 00
00 ¡ATENCIÓN!
(4) ¿Cuál es el mcm de 3 y 16'!
Analiza y n.:spondc: S1 A contiene exacta-

mcm(3 y 16) = D mente a B.


o A es mú/1,plo de B.
¿Por qué? o A es di~1s1ble por B.
entonces:
(5) ¿Cuál es el mcm de 7 y 28?
mcm {AyB),.,A
Analiza y responde:
mcm(7 y 28) = D
¿Por qué?
1418@41 1 MATEMATICA
1

H. POR DESCOMPOSICIÓN EN FACTORES PRIMOS.

Lo veremos en el siguiente ejemplo:

(1) Hallar el mcm de 120; 36 y 30


o RECUERDA OVE:
Solucicín:
S1 B divide exactamente
Escribimos cada núme- 120 2 36 2 30 2 } a A. entonces B es FAC-
60 2 18 2 15 3 TOR o DIVISOR de A.
ro como la multiplica-
ción indicada de sus ◊ 30 2
15 3
9 3
3 3
5 5
1 As1:
3 d1vrde a 12
factores primos: O O 5 5 1
1 Luego:
Es decir: ◊ 120 = 23 X 3 X 5 3 es factor de 12;
36 = 22 X 3 2 y s1 3 es número primo
30 = 2 X 3 X 5 entonces 3 es FACTOR
PRIMOde 12.
• Extraemos los factores
COMUNES y NO
mcm = 23 x 32 x 5
COMUNES cada uno
con su MAYOR expo- ◊ ó mcm = j 360 1
nente. El producto de
éstos es el mcm bus-
cado:
¿ Me permites contar contigo para el siguiente ejemplo?

(2) Hallar el mcm de 450; 168 y 240.


PROPIEDAD.
Solud{m: Si se mult,pl1can o divi-
. Descomponiendo cada 450
D o D o
o 0 168 240
D o
o den var1os numeras por
una misma cantidad. su
número:
D o D o D o mcm tambien queda
¡◊ CJ D CJ D CJ D mult1phcado o dividido
fa decir: 4so =I
D o D o D o por esa misma cantidad.
168. =I 1 D D D D
D ,COMPRUÉBAL 0 1
240=1 1

• Extraemos los factores


COMUNES y NO """(; Conipru~ba lo 1 )
COMUNES, cada uno
con -;u MAYOR expo-
◊ mcm = ~I ==~
o mcm = 1 25 200
nente. Su producto es
el mcm buscado:
•O· ~ ALFONSO ROJAS PUEMAPE 1--,s,--~-
C. POR EL MÉTODO ABREVIADO.
Al empicar este método, buscamos en un solo esquema
todos los factores comunes y no comunes sin excep- O RECUERDA OUE:
ción:
El mcm es el MENOR
En el ejemplo (1) den. número que cont1eneexac-
)j lamente a var,os números
(1) 1 Hallar el mcm de 120; 36 y 30 1 O 0 dados.
Ik, ...,.-,-- e· .. . •; •, · •~❖-\ "-. ».

Solución:
120 - 36 - 30 2
. Escribimos en un solo es- 60 18 15 2
quema los tres números 30 9 15 2
<lados y vamos averiguan- 15 9 15 3
<lo si al menos uno de ellos 5 3 5 3
cada vez es divisible por 5 5 5
2; 3 ; 5 ; etc. Así:
. El proceso se detiene
mcm = 2 3 x 32 x 5
cuando los números infe-
riores son todos iguales a ó 1 mcm = 360
!a unidad, luego: 1

Aplica tú ahora este método al ejemplo (2) den ¡será motivador!

(2) 1Hallar el mcm de 450; 168 y 240


00 PROPIEDAD:
Solución: El producro de dos nú-
meros A y 8 es igual al
450 - 168 - 240
Luego: .-----. producto de su MCD por
sumcm.
mcm=:='======. Es decir:
o mcm = l 2s 200 1 t
A "- B ■ MCD x mcm

D. POR APLICACIÓN DE LA PROPIEDAD QUE RELA-


CIONA DOS NÚMEROS CON SU MCD Y SU mcm : 0 00

Ejemplo: Hallar el mcm de 12 y 18.


Soluciém:
• Hallamos el MCD de 12 y 18 MCD(l2y 18)=6
• Por la propiedad referida: 12 x 18 = 6 x mcm
• Descomponiendo 12 como: 6 x 2 x 18 = 6 x mcm
• Agrupando e identificando: 6x(2xl8)=6x(mcm) ➔ mcm= 18x2=~
1 -C: 1 T
1 l\-1ATEMATICA
1
r:: SKANIPRÁCTICA @
l. Considcrand,, que un MÚLTIPLO contiene exactamente a un número dado, comph.:tar
el siguiente cuadro hasta hallar d mcm de los números señalados, aplicando sólo el
concepto de mcm. (Escribir sólo los 10 primeros múltiplos de cada número).

MÚLTIPLOS COMUNES A:
Nú-
mero MÚLTIPLOS 6y8 15 y 16 16 y8 18 y32 15 y 20 24 y 16

6 6; 12; 18; 24; 30; 36; 42: 48; 54: 24

8 8; 16; 24: 32; 40; 48 : 56; 64 : 72: 48

12

15

18

16

20

24

32

36

mcm ◊
11. Calcular el mcm de los siguientes números por "golpe de vista":

NÚMEROS 5y3 6y2 12 y 4 7y8 3y4 18 y 3 18 y 6 3y9 6y7

mcm

-: fü NÚMEROS 10 y 5 17y 3 6y8 2 y 11 4 y 10 6y3 9 y 10 2y3 8 y 12

mcm

111. Hallar el mcm de los siguientes números aplicando Descomposición en FACTORES


PRIMOS y comprueba tu respuesta hallando el mcm por el MÉTODO ABREVIADO:
(1) 60 y90 (5) 35; 70 y 80 (9) 25; 40, 15 y 80
(2) 32; 40y 50 (6) 45: 85 y 100 (10) 16: 30; 64 y 72
(3) 54; 80y64 (7) 12; 60 y 72 (11) 180; 300; 240 y 360
(4) 18; 64y72 (8) 18; 60y 54 (12) 720; 400; 520; 800 y 640
.- .... :-· ·=-- ·· :: · .
1 AlFo,so ROJAS PUÉ\IAPE 1

Aquí mis operaciones de SKANIPRÁCTICA0

,/

• I :;--_-::.¡-

'I,,

. -. -: :-. • -:-.-:-:-: :-.-: : .-:-:-:-:-: :-:-:-:-:-:-: .-: :-:-:-:-:-:-·-,¿-:-:-:,:,:-:-:-:-=·=-~--: ::;-.-:-:-:-:-:-.-:-:-:-:,:•:•·-~=~-:-:~-::.::-:-:-:-:-:-:-:-~:;-·-:-:;:-:-:::-=-~=~•~::;-,¡;·:;:.;-::;::-:-i.·.-:-:-:-:-:-:-:.• :-."l-.-:-:-:-:-:-:-:-:-:-:,.:-:•::/·-=-=-:-:-:-:-:-:-:-:-:-:-;:-:-:-:-:-:-:-. : :-:-:-: i
·-:-:-:.:---.-»:-: :=:~::-~:::::~=--:-P..;::;:=-.:*=~=~$J"::~»:~..1:!:-1:f.~:-=!'-::-~~:-~-!.~-=~~-~:!::~:::=~=-~=-~4"~~~:!:~:=:=:=:~:=:=:=:=:~;;~-::.;:-~::!-~:=-~:~=~=:t-:::=:::=:~..~=:-x~=:-:!:=:~
1 MATEMATICA
1
RAZONEMOS

Instrucciones:
A continuación te presento bloques de ejercicios para
afianzar lo leído hasta aquí. Marca la respuesta correcta
y compárala con la CLAVE DE RESPUESTAS que
aparece al final; sin embargo, te recomiendo que expliques
en tu cuaderno las RAZONES de tu resultado.
No tomes este trabajo como una pesada carga, sino como
un DESAFÍO a tu inteligencia.

llLO(.)CF. I ♦ (8) Al contar las hojas de mi cuaderno, de 2 en 2,


de 3 en 3 ó de 5 en 5, no me sobra ni me fal-
(1) Una familia tiene 2 hijos y otra familia tiene 3. ♦ ta ninguna hoja. ¿Cuántas hojas tendrá mi
¿Cuál será el menor número de caramelos que cuaderno, si es mayor de 80 pero menor de
deberá tener una bolsa, para que pueda ser ♦ 100?
repartida en cualquiera de estas familias?

(9) Con las personas que hay en una reunión, se
(2) Las mesas de un restaurante son para 3; 4 ó 6 ♦ pueden formar grupos de 3; 5 ó 7 sin que sobre
personas. ¿Cuántos vasos de refresco, como o falte alguna persona.¿ Cuántas personas como
mínimo, debe contener una jarra, para que ♦ mínimo hay en la reunión, si se sabe que no hay
pueda ser servida en cualquier mesa? más de 150?

(3) En un salón hay 30 alumnos y en otro salón 36.
♦ (10) Julio tiene 18 años y tiene dos hermanas
Si se forman grupos del mismo número de menores, cuyo producto de edades es 120.
alumnos en ambos salones, ¿cuál será el máxi- ♦ ¿Cuáles serán estas edades, si ninguna es menor
mo número de alumnos que podrá tener este de to años?
grupo? ♦

(11) Le preguntan a una profesora por su edad y ella
(4) Dos depósitos tienen 36 litros y 40 litros responde que su edad es el producto de los 2
respectivamente. ¿Cuál será el mayor número ♦ mayores números primos de una cifra. ¿Cuál es
de litros que debe tener otro depósito, para que la edad de la profesora?
los 2 primeros lo contengan un número exacto ♦
de veces? (12) Del caño de agua fría cae una gota cada 8 min
♦ y del caño de agua caliente cae 1 gota cada
(5) Un profesor promete a un alumno un punto por ♦ 12 min . Si de ambos caños cae una gota
cada divisor de 36 que el alumno encuentre. simultáneamente, entonces después de cuánto
¿Cuántos puntos como máximo podrá obtener ♦ tiempo caerá otra gota de ambos caños, a la vez.
el alumno? ♦ (13) Una abuelita, tiene 3 nietos; uno la visita cada
12 días, otro la visita cada 15 días y el otro cada
(6) Un profesor de Matemática asigna a Coco y ♦
Willy los números 60 y 72 respectivamente y 20 días. Si casualmente se encuentran los 3 en
un mismo día, ¿cuánto tiempo después se vol-
les pide que encuentren los divisores de sus ♦
verán a encontrar los 3 juntos?
respectivos números. ¿Cuántos de estos ♦
divisores son comunes? (14) El producto de 2 números enteros positivos es
♦ 12. ¿Cuáles son los números, si la suma es la
(7) Un profesor de Matemática le encarga a Alberto menor posible?
hallar los números primos que hay entre el I y ♦
el 50; a Carlos le encarga hallar los números ♦ (15) Hallar la suma de los múltiplos de 6, com-
primos entre el 51 y el 100. ¿Quién encuentra prendidos entre el menor número primo de dos
más números primos? ♦ cifras y el primo inmediaco inferior a 70.
·t~ ALFONSO ROJAS PUÉMAPE 1u,: éái
1

(16) ¿Cuántos números primos están comprendidos ♦ (30) El producto de 3 números enteros es 36; si la
entre el menor múltiplo de 6, de 2 cifra~ y el suma es 1O, ¿cuáles son estos números?
mayor múltiplo de 17, de 2 cifras? ♦
(31) En elcolegio se reúnen 3 grupos; el de guitarra,
♦ cada 6 días, el de teatro, cada 9 días y el de
(17) Un comerciante compró 40 caramelos y 30
chocolates . Desea hacer paquetes que tengan el ♦ Matemática, cada 15 días. Si cierto día
mismo número de dulces de cada tipo. Si hizo coincidieron los 3 grupos, entonces después de
el máximodepaquetes,¿cuántos dulces habían ♦ ¿cuántos días volverán a coincidir nuevamente?
en cada uno de estos paquetes? ♦ (32) En cierto planeta "x" se aparece el corneta ..A"
(18) El número 48 se descompone en 2 factores. ♦ cada 180 años, el corneta "B" cada 150 años y
¿Cuántos de estos factores son impares? el corneta "C" cada 120 años. Si cierto año
. ♦ coincidieron los 3 cornetas, entonces después
(19) Un libro tiene 142 hojas. En las páginas cuya ♦ de ¿cuántos años, corno mínimo, volverán a
numeración es múltiplo de 25, hay una lámina coincidir nuevamente?
a color, ¿cuántas láminas a color hay en el libro? ♦
(33) A lo largo de una carretera se observan avisos
♦ comerciales. El aviso de "TOME COLA" está
(20) ¿Cuántos números enteros hay desde el 61
hasta el número primo superior más próximo? ♦ cada 6 km; el aviso de "BANCO DEL PAÍS"
está cada 8 km y el aviso de AEROLÍNEAS
(21) ¿Cuánros números primos de dos cifras, que ♦ "PUCALLPA" está cada 10 km. Si en cierto
terminen en 7, hay? ♦
punto están los 3 avisos, entonces; ¿después de
cuántos kilómetros volverán a estar los 3 avisos
(22) La suma de dos números primos consecutivos ♦ juntos?
es 90. ¿Cuáles son estos números?

(34) En el problema anterior, ¿cuántas veces apa-
(23) La suma de dos números primos consecutivos ♦ recieron los avisos de "TOME COLA", hasta
es 112. ¿Cuáles son estos números? que volvieron a estar juntos los 3 a, ,ws?

(24) Un gallo canta cada 8 min y otro cada 12 min. ♦ (35) La suma de 8 números pares con 5 números
Si ambos cantan a las 6:00 a.m., ¿a qué hora impares, ¿será par o impar?
volverán a cantar juntos otra vez? ♦
(36) L1 suma de 7 números pares con 12 impares,

(25) La suma de 2 números es el mayor número ¿será par o impar?
primo de 2 cifras, si uno de ellos es el menor ♦
número impar de 2 cifras diferentes, entonces (37) En un salón están matriculados 32 alumnos; la
¿cuál es el otro? ♦ quinta parle de los que faltaron, era por
♦ enfermedad y la mitad de los que faltaron fue
(26) ¿Cuál es el menor número impar no primo de por estar de viaje. Si hay más de 20 asistentes,
2 cifras? ♦ ¿cuántos faltaron?

(27) En un avión viajan 200 personas. Si el avión se (38) Las edades de 3 niños menores de IO años son
estrella, entonces los sobrevivientes se pueden ♦ números primos, donde la edad del mayores la
agrupar de 5 en 5; o de 6 en 6 ó de 8 en 8. suma de las edades de los otros 2. ¿Cuáles son
¿Cuántos fueron los muertos? ♦
estas edades?

(28) En un costal hay 18 kg de harina y en otro hay (39) Se tienen 2 depósitos de agua, de 80 y 90 litros
24 kg. ¿Cuál será el m<1yornúmero de paquetes ♦ ¿Cuál será la mayor c,1pacidad que debe tener
que se podrá formar, tal que tenga un número otro depósito. tal que esté contenido un número

entero de kilogramos de ambos costales? exacto de veces en cada uno de ellos?

(29) ¿Cuántos múltiplos de 8 y l O comunes, hay (40) De qué número es siempre múltiplo, )asuma de
entre 50 y 100? ♦
3 números enteros consecutivos?
1 MATEMATICA
1
HLOQU ·:11

(1) Llegó una donación de 180 tarros de leche, 300 paquetes


de fideo y 450 bolsas de avena. Deseamos empaquetarlos
de tal modo que en cada paquete haya el mismo número de
artículos. ¿Cuántos paquetes como máximo podremos
hacer?

a) 25 b) 41 c) 30 d) 45 e) 20

Solució11:
Para que cada paquete tenga el mismo número de
tarros de leche, este número debe ser DIVISOR de
180; lo mismo ocurre ron los fideos y la avena.
Como debe ser una misma cantidad de paquetes
donde esten los tres artículos, el DIVISOR arriba
referido debe ser COMUN. Si este numero de pa- O ¡ATENCION!
quetes debe ser el mayor posible, entonces tenemos
que hallar el MÁXIMO COMÚN DIVISOR. Hemos empleado aqu, el
MÉTODO ABREVIADO
• Hallamos el MCD de 180; 300 y 450. 180 para el cálculo del MCD,
- 300 -450 2
en e/cualsólonosmteresa
Qo 90 -150 -225 3
buscar los factores o
30 - 50 - 75 5 divisores COMUNES a
6 - 10 - 15 los tres números, de mo-
Luego: MCD (180; 300 y 450) =2X3X5 do que los numeras de la
ultima fila sean primos
= 30 entre sí.
Respuesta.-
Podremos hacer 30 paquetes como máximo. Rpta. ©
(2) Tres líneas de microbuses salen de un mismo paradero
inicial. De la primera línea salen microbuses cada 2 horas;
de la segunda salen microbuses cada hora, y de la tercera
cada hora con 12 minutos. Si a las 6 a.m. salen las tres
juntas, ¿a qué hora volverán a salir al mismo tiempo?

a) 11 a.m. b) 12 m. c) 1 p.m. d) 2 p.m. e) IOa.m.

Solución:
Si transformamos las horas a minutos, los microbuses salen cada
120 minutos de la primera línea, cada 60 minutos de la segunda
linea y cada 72 minutos de la tercera. Volverán a salir juntos
después de varios de esos intervalos. Es decir el tiempo dentro del
cual volverán a salir juntos debe ser MÚLTIPLO COMUN a esas
tres cantidades, para lo cual bastará hallar el mcm de 120; 60 y 72.
•$• 1 ALFO~SO ROJAS PUÉl\1.-\PE 1
• Considerando que I hora tiene
60 minutos, tendremos: De la l O línea salen cada 120 minutos
De la 2° línea salen cada 60 minutos
De la 3° línea salen cada 72 minutos
. El número que buscamos debe 120 -60 - 72 2
contener exactamente a 120; 60 60 -30 -36 2
y 72, entonces procedemos a 30 -15 -18 2 O CUIDADO
calcular su mcm: 15 - 15 - 9 3
Hemos empleado aquí, el
c:;>o 5- 5- 3 3 MÉTODO ABREVIADO para el
5- 5 - 5 cálculo del mcm en el cual
1- 1- tuvimos que extraer /os factores
mcm (120; 60 y 72) = 23 x 3 2 x 5 o divisores COMUNES y NO
COMUNES, de modo que los
= 360 números de la ultima fila son
• 360minutos equivalen a 6 horas, TODOS rguales a la UNIDAD.
si salieron a las 6 a.m. entonces
volverán a salir juntas a las
12 m. Hasta esta hora: La 1º 1ínea sacó 360 : 120 = 3 carros (aparte del primero)
La 2° línea sacó 360 : 60 6 carros (aparte del primero)
La 3° línea sacó 360 : 72 = 5 carros (aparte del primero)
Respuesta.-
Vol verán a salir al mismo tiempo a las 12 m. Rpta.@

(3) ¿Cuál es el mcm de 2 y 3? ♦ a) 18 min. b) 27 min. c) 20 min.


d) 25 min. e) 32 min.
a) 2 b) 3 c) 5 d) 6 e) 8 ♦
♦ (7) Hallar el mayor divisor común de 72 y 90.
(4) Se tiene un cilindro con agua, el mismo que
puede ser llenado por dos caños. ¿Por lo menos a) 18 b)9 c) 27 d) 54 e) 36
de cuántos litros es el cilindro,siel primer caño ♦
vierte 2 litros por minuto y el segundo vierte 3 ♦ (8) ¿Cuál es el divisor común más grande de
litros por minuto y ambos llenan el cilindro por 32x40 y 60x 16?
separado en un número exacto de minutos? ♦ a) 350 b) 160 c) 320 d) 80 e) 360
a) 7 litros b) 5 litros c) 12 litros ♦ (9) Tenemos 90 galletas, 54 chocolates y 150
d) 18 litros e) 6 litros
♦ bombones; necesitamos empaquetarlos en
bolsas que contengan la misma cantidad de
(5) ¿Cu{il es la mínima capacidad en litros de una ♦
cada artículo. ¿Cuál es la máxima cantidad de
piscina, si se sabe que un caño la llenaría a 20
bolsas que se necesita?
litros por minuto; un segundo caño la llenaría a ♦
54 litros por minuto y un tercer caño la llenaría ♦ a) 12 b) 3 c) 6 d) 8 e) 9
a 15 li Iros por minuto; conociendo además que
el llenado por separado de cada caño es un nú- ♦ (10) Enelproblemaamerior,¿cuáleselnúmeroque
mero exacto de minutos? representa a la suma de galletas y bombones en
♦ cada bolsa?
a) 540 litros b) 270 litros c) 300 litros ♦
d) 320 litros e) 560 litros a) 49 b) 39 c) 35 d) 42 e) 40

(6) En el problema anterior, ¿cuánto demoraría el ♦ ( 11) En una librería se tiene en STOCK 300 lapice-
primer caño en llenar la piscina? ♦ ros, 180 reglas y 240 borradores. Si el librero
1 MATEMATICA
1
<le.sea venderlos empaquetados al mismo precio ♦ número posible de trozos del mismo tamaño.
c,1da bolsa, ¿cu.íl es el mayor número de bolsas ¿Cmíl es la longitud de cada trozo'!
que estarían listas para ven<lerse, sabiendo que ♦
a) 8 m b) 10 m c) S m
cada bolsa <lebe contener lapiceros, reglas, ♦
d) 12 m e) 7 m
borradores y que no debe sobrar ni un solo ar-
~~ ~ ra ~ ~~ ♦ (20) Calcular el menornúmero de cuadrados iguales
a) 52 b) 63 c) 65 d) 60 e) 80
♦ en que se puede dividir un terreno rectangular
♦ que tiene como dimensiones : 81 O m y 684 m.
(12) En el problema anterior, ¿cufü es el número que
a)l620 b)1710 c)1825
representa a la suma de lapiceros y borradores ♦
d)l750 e)l8
en cada bolsa? ♦

a) 9 b)l2 c)6 d) IS e) 1O ♦
♦ HLOQUE III
( 13) ¿Cuál es el producto del MCD y mcm <le los
números 21; 39; 7 y 3? ♦ (1) Calcular el menor número posible que dividido
entre 72; 120 y 80 nos dé siempre residuo cero.
a) 263 b) 283 c) 266 ♦
d) 186 e) 273 a) 1 400 b) 850 c) 360
♦ d) 1440 e) 720
(14) El producto de dos números es 215 930. Si su ♦
MCD es 302, ¿cuál es su mcm? (2) ¿Cuál es el número menor que dividido entre

b) 715 30; 84 y 64 resulte siempre en una división
a) 730 c) 810
♦ exacta? ·
d) 515 e) Faltan datos
♦ a)S700 b)6700 c)S720
( 1S) El mcm de dos números es 68. Si el pro<lucto de d) 6720 e) 7720
los mismos es 1 836, ¿cuál es su MCD? ♦
a) 22 b)32 c) 27 d) 35 e) 42 ♦ (3) ¿Cuál es el menor número que dividido entre
30; 84 y 64 resulte siempre 7 por residuo?

(16) La suma de dos números es 6 veces su MCD y
a) 6 527 b) 6 727 c) 7 727
el pro<lucto de dichos números es 8 veces su ♦
d) 6 713 e) 6 217
mcm. ¿Cuáles son estos números?

a) 32 y 4 b) 40 y 6 c) 6 y 34 (4) Calcular el menor número de 3 cifras que al

<l) 52 y 10 e) 40 y 8 dividirlo por 4; 6 y I Odé siempre residuo cero.

(17) Dos números son primos entre sí. Si su produc- a) 180 b) 210 c) 120
to es 3 264, ¿cuál es su mcm? ♦ d) 140 e) 160
♦ (S) ¿Cuáles el menor número mayor que 200, que
a) 3 264 b) 3 615 c) 3 178
d) 2 615 e) 1 632 ♦ al dividirlo por 12; I Oy 8 da siempre residuo 6?

( 18) Si multiplicamos el MCD por el mcm de dos ♦ a) 246 b) 146 c) 216


números, obtenemos 288. Sabiendo que uno de ♦ d) 256 e) 286
ellos es el MCD de 810 y 144, calcularla suma
de dichos números. ♦ (6) ¿Cufü es el menornúmero divisible por 7 que al
ser dividido por 2; 3; 4 ó S da siempre 1 de
a) 34 b) 17 c) 68 d) 64 e) 38 ♦
residuo?

( 19) Se tiene tres alambresde35 ; 40y 125 metros de a) 201 b) 261 c) 311
longitud, los cuales se dividen en el menor ♦ d) 281 e) 301
1 ALFONSO ROJAS PUEMAPE l
(7) El número de alumnos de primer grado de ♦ menor que 400, calcular el número exacto de
secundaria de un colegio es tal que si los páginas.
agrupamos de 30 en 30 sobran 9; si los agru- ♦
pamos de 18 en 18 sobran 9; y si los agrupa- ♦
a) 315 b) 317 c) 327
mos de 24 en 24 también sobran 9. ¿Cuántos d) 319 e) 313
alumnos de primer grado tiene el colegio si nos ♦
han dicho que no son más de 400? ♦ (l 3) En una urbanización, el número de habitantes
está comprendido entre 4 000 y 4 500. Si los
a)369 b)315 c)319 ♦ agrupamos de 16 en 16 sobran 6; si se agrupan
d)360 e)300 de 12en 12sobran 2 ysi se agrupan de 18en 18
♦ sobran 8. ¿Cuántos habitantes tiene dicha
(8) Los alumnos de 1°A más los de 1°B son tantos ♦ urbanización?
que si los agrupamosde4en4sobrarían 3;si los
agrupamos de 6 en 6 siempre sobrarían 3; y si ♦ a)4320 b)4310 c) 4 340
los agrupamos de 8 en 8 también sobrarían 3. Si d) 4 215 e) 4 500

el número de alumnos de 1°A y 1°B está com-
prendido entre 70 y 80, ¿cuál es este número? ♦ (14) El profesor de Matemática de un salón de clase
desea en acuerdo con sus alumnos conformar
a) 70 b) 72 c) 75 d) 74 e) 78 ♦ grupos de trabajo y observa lo siguiente: Si los
♦ agrupa de 4 en 4 sobra I alumno; si en cambio
(9) Si agrupamos a todos los alumnos de un colegio los agrupa de 8 en 8 sobran 5 y si los agrupa de
de6en6sobran5; si los agrupamos de 16en 16 ♦
10 en !O sobrarían 7. ¿Cuántos alumnos hay en
sobran 5; y si los agrupamos de 20 en 20
también sobran 5. ¿Cuántos alumnos tiene dicho ♦ clase?
colegio si nos han dicho que son m,is de 900 ♦ a)37 b)40 c)43 d)46 e)39
pero menos de 1 000?
♦ (15) El número de libros de mi biblioteca no es
a) 915 b) 965 c) 960 menor que 400 ni mayor que 500. Si los agru-

d) 970 e)980 para de 4 en 4 me sobrarían 2, si los agrupara
♦ de 8 en 8 sobrarían 6, si los agrupara de 18 en
(!O) Un comerciante de golosinas agrupa sus choco-
lates por docenas y le sobran 8, si los agrupara ♦ 18 sobrarían I 6 y si los agrupo de 16 en 16 me
por decenas también le sobrarían 8; sabiendo ♦
sobran 14. ¿Cuántos libros tengo en mi
que cada chocolate los vende en S/. 0.50, biblioteca?
¿ cuántos chocolates tenía si al venderlos ♦ a)432 b)430 c)436
recaudó por ellos entre S/. 90 y SI. 100? ♦ d)440 e)448
a) 68 b) 128 c) 158
d) 188 e) 148
♦ (16) Un empleado del colegio intenta improvisar un
♦ auditorio en el salón más grande , para lo cual
(11) Es necesario llenar cuatro cilindros de una ♦ trata de disponer las sillas en filas completas. Si
capacidad de 50; 75; 100 y 80 galones las arregla por docenas le sobran 8, si las arregla
respectivamente. ¿Cuál es la mayor capacidad ♦ por decenas le sobran 6 y si las arregla en filas
del balde que podremos usar para llenarlos con de 8 cada una Je sobran 4. ¿Con cuántas sillas
cantidades exactas de baldes? ♦ cuenta el empleado?

a) 7 Gis. b) 6 Gis. c) 8 Gis. a) 114 b) 116 c) 120
d) 5Gls. e) 9 Gis. ♦ d) 124 e) 132

(12) Si las páginas de un libro se cuentan de 3 en 3 ♦ (17) Una revista científica tiene entre 40 y 50 pági-
sobra 1, si se cuentan de 7 en 7 sobran 5 y si se ♦ nas. Al contar éstas. notamos que si lo hacemos
cuentan de 9 en 9 sobran 7. Sabiendo que el por parejas nos sobra una página; si las con-
número total de páginas es mayor que 300 pero ♦ tamos de 3 en 3 sobra un par, y si las contamos
1 MATEMATICA
1
de 4 en 4 nos faltaría sólo 1 para que no nos ♦ (4) ¿Cuántos números enteros menores que 880
sobre nada. ¿Cuántas páginas tiene la revista? son divisibles simultáneamente por 6: 15; 8
♦ y 10?
a) 49 b) 48 c) 45 d) 47 e) 46
♦ a) 1 b) 8 c) 6 d) 7 e) 5
(18) Calcular el menor número de cuadrados iguales ♦
en que podemos dividir una pizarra que tiene ♦ (5) ¿Cuántos números enteros mayores que 500 y
por dimensiones 360 cm y 210 cm respec- menores que 900 son divisibles a la vez por 9:
tivamente. ♦ 12; 15 y 18?
a) 74 b) 30 c) 84 d) 90 e) 60 ♦ a) 1 b) 3 c) 2 d) 4 e) 5

(19) ¿En cuántos cuadrados iguales como mínimo ♦ (6) Un comerciante tiene tres barriles de vino de
es posible dividir un terreno rectangular que ♦ 420; 580 y 1 800 litros respectivamente, pro-
mide 420 m de largo y 300 m de ancho? poniéndose vender este vino en recipientes
♦ pequeños e iguales de la mayor capacidad y que
a)35 b)60 c)l2 d)65 e)38
♦ estén contenidos exactamente en los tres
(20) Se tiene un terreno de I 240 m de largo por ♦ barriles. ¿Cuántos recipientes debe usar el
860 m de ancho que se desea vender en partes · comerciante?
cuadradas de la mayor área posible cada una. Si ♦ a) 122 b) 140 c) 84
ya se vendieron 955 partes, ¿cuántas faltarían ♦ d) 66 e) 20
vender?
♦ (7) En un taller de carpintería el total de los salarios
a)l611 b)l717 c)l711
♦ es SI. 525 y en otro S/. 81 O, recibiendo cada
d) 2 666 e) 955
trabajador el mismo salario; ¿cuántos traba-
♦ jadores hay en cada taller si el salario es el
BJOQLE I\ ♦ mayor posible?

♦ a)27y35 b)5ly37 c)54y35


(!) Se ha dividido 3 barras de acero de longitudes d)51y54 e)35y39
540: 480 y 360 mm en trozos de igual longi- ♦
tud, siendo ésta la mayor posible. ¿Cuántos ♦ (8) Calcular lasuperficiedel menor terreno rectan-
trozos se han obtenido? gular que puede ser dividido en lotes rec-
♦ tangulares de 6 m por 20 m ó 1O m por 16 m
a)IS b)21 c)23 d)25 e)18
♦ ó 12 m por 32 m.
(2) ¿Cuál es la menor cantidad de locetas cuadradas ♦ a)1720m 2 b)3540m 2 c)2613m 2
que se necesita, sin partir ninguna, para cubrir d)1920m 2 e)1810m 2
un piso de 744 cm por 528 cm? ♦
♦ (9) ¿Cuál es el número más pequeño posible que
a) 24 b) 682 c) 662
dividido por 8; 6; 1O y 5 da un residuo común
d) 702 e) 632 ♦
que sea al mayor posible?
(3) Lms, César y Wilfredo visitan a Maritza en su ♦ a) 124 b) 120 c) 116
casa cada 3: 6 y 8 días respectivamente. Si los ♦ d) 128 e) 248
tres juntos la visitaron el 2 de enero, ¿cuál será
latech,1máspróximaenquevolveránacoincidir ♦ (10) Hallar el menor número posible que dividido
en la visita los tres·! ♦ por 4; 15 y 18 dé un residuo común que sea
también el menor posible.
a) 24 de enero b) 22 de enero ♦
c) 1º de febrero d) 8 de febrero a) 161 b) 301 c) 207
e) 26 de enero ♦ d) 307 e) 181
·t··, . 1 ALFONSO ROJAS PUÉ.\1APE 1
( 11 ) ¿Cuál es la menor capacidad de un depósito de ♦
agua que se puede llenar en un número exacto
de minutos por cualquiera de 3 caños que vier- ♦
ten 16; 15 y 32 litros por minuto respectiva- ♦
mente?

CL\VE DE RESPUESTAS
a) 520L b) 460L c) 500L

d) 480L e) 560 L RAZONEMOS

(12) ¿Cuántos números pares menores a 1 500 son BLOOUEII

divisibles simultáneamente por4; 5; 6 y 8?
(1) e (6) b (11) d (16)e

a) 15 b) 18 c} 12 d) 14 e) 10 (2)b (7) a (12} a (17) a
♦ (3)d (8)c (13} e (18) a
(13) ¿ Cuántas formaciones tipo rectángulo se puede (4)e (9)c (14) b (19)c

lograr con 120 alumnos? (5) a (lO)e (15) e (20} b

a) 8 b) 12 c) 15 d) 7 e) 30
BLOOUElll

(14) Dos ciclistas recorren una pista cerrada. El (1) e (6) e (11) d (16) b

primero tarda 15 minutos en dar la vuelta y el (2)d (7)a (12) e (17)d
segundo 18. Si ambos parten del mismo punto, ♦ (3) b (8) e (13)b (18) e
¿al cabo de cuánto tiempo volverán a encon- (4)c (9) b (14) a (19) a

trarse? (5)a (10)d (15) b (20)c

a) 1 hora b) hora y media
BLOQUE IV
c) 2 horas d) 35 minutos ♦
e) 45 minutos ♦ (1) e (5) e (9) a (13)d
(2)b (6) b (10) e (14) b
(15) Se han plantado árboles igualmente separados ♦ (3)e (7) e (11) d (15) b
en el contorno de un campo triangular cuyos (4)d (8) d (12) e

lados miden 114; 180 y 240 metros. Sabiendo
que hay un árbol en cada vértice y que la dis- ♦
tancia entre dos árboles consecutivos está com-

prendida entre 4 y 8 metros, ¿cuál es el número
de árboles plantados? ♦

a) 79 b) 89 c)56 d) 86 e) 91 ♦
1 MATEMATICA
1
LAS l\IATEl\'IÁTICAS DE TODOS LOS DÍAS
( I) El Dr. Moreno tiene 3 pacientes que asisten a su consultorio de manera regular. El Sr.
Alva visita al Dr. cada 2 días; el Sr. Benavides cada 3 días y el Sr. Cruz cada 4 días. Si
cierto día coincidieron los 3 pacientes, entonces:
a) Después de cuántos días volverán a coincidir los 3.
b) Después de cuántos días coinciden sólo el Sr. Alva y el Sr. Benavides.
c) Después de cuántos días coinciden sólo el Sr. Cruz y el Sr. Benavides.

Solución:

Respuestas

a)I
1
b) I
1
c)I
1
(2) Se encarga a una imprenta, la elaboración de facturas, que estarán numeradas desde
00IO0 hasta 09999. Se quiere saber:
a) ¿Cuántas facturas habrán en total?
b) ¿Cuántos tipos de imprenta se usarán?

Solución:

Respuestas

a)I
1
b)I
1
1 UFONSO ROJAS PUEMAPE 1 ifrJW!Fi
(3) Un antiguo constructor, disponía de bloques de piedra de 60 x 80 x 120 cm.
a) ¿Cuál es la medida de la menor arista que tendrá un cubo construido con estos
bloques?
b) ¿Cuántos de estos bloques se emplearán en la construcción de dicho cubo?

Solución:

Respuestas

(4) Empleando el procedimiento de la "CRIBA DE ERATOSTENES", hallar los números


primos del 101 al 200. ¿Cuántos son estos números?

101 102 103 104 105 106 107 108 109 )lO
111 112 113 114 115 116 117 118 119 120
121 122 123 124 125 126 127 128 129 130
131 132 133 134 135 136 137 138 139 140
141 142 143 144 145 146 147 148 149 150
151 152 153 154 155 156 157 158 159 160
161 162 163 164 165 166 167 168 169 170
171 172 173 174 175 176 177 178 179 180
181 182 183 184 185 186 187 188 189 190
191 192 193 194 195 196 197 198 199 200

Sulucüí11:

Respuesta
iff.i@u¡lj 1 MATEMÁTICA
1
RAZONAMIENTO MATEMÁTICO

(1) ¿Cuántos triángulos hay en la siguiente fi - (6) Calcular el valor de la incógnita:


gura'!

a) 7 435 h) 8 663 e) 9 149


a) 11 h) 18 e) 12
d)6317 e)9839
d)l6 e) 15
(7) Cuál es el número que sigue en la siguiente
(2) Calcular el número de triángulos que hay en sucesión:
la siguiente figura: 2: 3; 8; 63; . ...
a) 396 h) 3 968 e) 1 875
d)2615 e)3619

(8) Calcular el valor de P en la siguiente figura:


a)45 b)42 c)35
d)9 e) 27

(3) Si en la figura del problema anterior, tra7.a-


mos una línea recta hori1ontal que corle a las
líneas que parlen del punto superior: ;.Cuán-
a) 53 h) 63 c ) 47
tos triángulos contaríamos ahora?
d) 62 e) 51
a) 45 h)50 c)46
(9) Hallar x en d siguiente gráfico:
d)90 e) 51
20 16
(4) ¿Cuántas figurasde4 Iados aparccencnclsi-
guienle gráfico? 8614~21
78

1111111111
566x
a) 62 h) 73 c) 67
a) 45 h) 9 e) 18 d)M e) 58
d)27 e)42
(10) ¿Cuántus triángulos aparecen en la siguien-
(5) Escribe el número que falla: te figura?

15

29

a) 32
18
33
,...
47
•)

h)42
23
26
37

c) 36
a) 22
A
d)15
h)24
e) 18
e) 12
d) 38 e)45
1 ALFONSO ROJAS PUÉMAPE 1

Aquí mis operaciones de RAZONAMIENTO MATEl\1ÁTICO.


1 MATEMATICA
1

CAPÍTULO 4

r ·- -- -- -~
1
-
r 1
~
1

1
,, 11 1
1
1 1

rr NUMEROS
1
' 1
1
1
1
1 11 :1
'¡ 1
!
1
1
1 i RACIONALES 1
1
1
1
i

11 1 !
1
~- , _
~

-~ ___¿

OBJETIVOS:

• Establecer si un número pertenece o no al conjunto de los números


RACIONALES.
• Manejar las técnicas operativas de números racionales en la
resolución de problema..<; prácticos.
1 ALFONSO ROJAS PUÉMAPE 1
ESTRUCTURA DEL CAPÍTULO:

l. FRACCIONES 2.5 Operaciones combinadas de Adición


y Sustracción en ([)_.
1.1 ¿Qué es una FRACCIÓN?
SKANIPRÁCTICA 4
1.2 Comparación de una FRACCIÓN
con la unidad. 3. MULTIPLICACIÓN Y POTENCIA-
CIÓN EN ([)_
SKANIPRÁCTICA 1
3.1 Propiedades de la multiplicación de
1.3 La FRACCIÓN como elemento de
números racionales.
un PRODUCTO CARTESIANO.
SKANIPRÁCTICA 5
1.4 Signos en una FRACCIÓN.
3.2 Potencia de una fracción.
1.5 Fracciones EQUIVALENTES. 3.3 Signos de una Potencia de base ra-
1.6 Simplificación de Fracciones. cional.
3.4 Multiplicación de Potencias de
SKANIPRÁCTICA 2
BASES RACIONALES IGUALES.
1.7 CLASE DE EQUIVALENCIA. 3.5 Potencia de potencia de BASE RA-
1.X NÚMERO RACIONAL. CIONAL.
3.6 Potencia de una multiplicación.
1.9 Relación entre IN, '11.. y (!)
SKANIPRÁCTICA 6
1.1 O RepresenLaci6n de (!) en la recta
numérica 4. DIVISIÓN Y RADICACION EN ([)_
1.1J Tran,1 , mnación de fracciones a de- 4.1 ¿ Cómo dividir fracciones?
nominador común. 4.2 Otra forma de presentar una división
1. 12 Comparación de números raciona- de fracciones.
les. 4.3 División de potencias de igual base
racional.
SKANIPRÁCTICA 3
SKANIPRÁCTICA 7
LAS MATEMÁTICAS DE TODOS LOS DIAS
4.4 Radicación en ([)_
RAZONEMOS 1 4.5 Signos de radicación eT! ©
4.6 Exponente fraccicnario.
4.7 Raíz de una !'"nultiplicación.
2.1 Adición de FRACCIONES DE 4.8 Raíz d<! Raíz.
IGUAL DENOMINADOR. 4.9 Potencia de una Raíz.
4.1O Operaciones combinadas
2.2 Adición de FRACCIONES DE DIS -
TINTO DENOMINADOR. SKANIPRÁCTICA 8
2.3 Propiedades de la Adición DE RAZONEMOS2
NÚMEROS RACIONALES.
LAS MATEMÁTICAS DE TODOS LOS DÍAS
2.4 Sustracción de NÚMEROS RA-
RAZONAMIENTO MATEMÁTICO
CIONALES.
1 MATEMÁTICA
1
e 1. FRACCIONES )

Skanito.si deseo repartir


20 panes entre Juan y
Fernando: ¿Cuánto les
toca a cada uno?.-·.:.._ __,.

S1 luego llega Carla y


deseo repartir por igual
los 20 panes: ¿Cuantos
les t a, u_n-=-9_?_ _

Cuando csLudiamos el CONJUNTO DE LOS NÚME-


ROS NATURALES (IN ), vimos que era necesario exlcndcr
dicho conjunto a otro más amplio que nos permitiera cfcc:Luar O IWCOIW,\R QUE:
la RESTA o SUSTRACCIÓN para Lodos los casos, apare-
ciendo cnLonccs el CONJUNTO DE LOS NÚMEROS EN- IN = { O; 1; 2; 3; . . .}
TEROS ( ~ )_ C O 72. = { .. . -3;-2;-l; O; l; 2; 3;
..}
Pero aún ahora, se nos prcscnla olra clificulLad, al Lralar
luego: IN e 72_
de efectuar ciertas divisiones de números enteros como en
nueslro ejemplo. Es decir : No Lodas las divisiones se pue- o
den efectuar en ~-
.:.:-
"IN e.\lá contenido en 72.
t\),:\"'·::""
:::.""
.-:.""
:·::'"'°'
:.·-:'""
> "'
.;.;..=_.•.
-"":·:·--;-:··""---·."·-·:"'
" ·-·-•,-
.. ..,.
____..,.
~ ,,,
.._.,,
.. ,,:::_.__. ,

Veamos algunos ejemplos:

(1) 27:3=9
porque 3 x 9 = 27
o porque al resolver la ecuación 3.x = 27
resultax = 9

(2) (+32): (-4) = -8


porque (-4) x (-8) = +32
o porque al resolver la ecuación (-4)x = (+32)
resulta : x = -8
J ALFO!';SO ROJAS PUEMAPE 1
(3) ¿Cuál es el resultado de dividir:

(+20): (+3)?

Como verás, NO EXISTE en el CONJUNTO DE NÚME-


ROS ENTEROS 71. un número que multiplicado por ( + 3 ), dé
como resultado (+20),luego en lL, no es posible resolver la
ecuación (+3)x = (+20)

Ante esta situación, surge la necesidad de ampliar el con-


junto de números enteros 7L , a otro que en adelante llamaremos
EL CONJUNTO DE NÚMEROS RACIONALES, que lo reco-
noceremos por la letra (D. y que empica símbolos o numerales
llamados FRACCIONES.

1.1 ¿QUÉ ES UNA FRACCIÚN?

Una fracción es una división indicada de dos


i~· ---- -
-- -. ..,·.··.. O lll:COIU>,\NIJO:
números enteros.
Si dos o mas frac ciones
En tal división, el divisor es DIFERENTE DE tienen el mi.vno denomi-
CERO. nador, las llamamos fra c•
ciune.1· I/OA10GÉNEAS,
Es decir: .!!.. , donde b ~ O si 110 es así las fracciones
b s011 IIEJEROGÉNEAS.

Además a y b son.los TÉRMINOS de la fracción y


reciben el nombre de NUMERADOR y DENOMINA-
DOR respectivamente e::> O

Ejemplo:


Numerador
---,
Denominador

Términos de la fracción 1#!!), ¡IMPORTANTE!

~1~
El co11j1111rn de los NUME-
ROS RACIONALES ( (D )
El d,,,,,,,.,,,,,,,, 6, ,ep,m"'6 la canúOOd de par,e,
en que dividimos a la UNIDAD . ~~im
.;;,
aparect•porque no rmlas las 1
cl11•i.llO/lf'!, S0/1 pos 11J/t,!, {' /1
l'I co11¡1111ro d e l01 NÚ-
MEROS ENTEROS ( Z ) j
El numerador 5, represenJa la canJidad de
partes que se ha tomado de la UNIDAD .
1 MATFMATICA 1
Aplicación :
¡IMPORTANTE!
Si dividimos en 6 partes iguales la pizarra del
salón, y pintamos sólo 5 partes, entonces toda 112 se lee ·un medio"
la parte pintada de la pizarra, la representa- 113 se lee ·un tercio·
mos por: 314 se lee "Tres cuartos•
2/5 se lee 'Dos quintos
516 se lee ·cinco sextos'
1110 se lee •un dedmo•
31100 se lee "Tres centésimos
7/1000 se lee ·s,ete m1les1mos
7/15 se lee "Siete qwnceavc-s•

1.2 COMPARACION DE UNA FRACCH)N CON LA UNIDAD


• Si el NUMERADOR es MENOR que el DENOMINA-
DOR, la fracción es MENOR que la UNIDAD.
Ejemplo:
De una 1or1a. tomemos las ¾ partes: e:>
• SI el NUMERADOR es IGUAL que el DENOMINADOR,
la fracción es igual a la UNIDAD
Ejemplo:
De una torta, tomemos las t partes: Q
Es decir: la UNIDAD tiene "4 cuartos" ó "5 quintos" ó
"6 sextos". etc.

,-.
• Si el NUMERADOR es MAYOR que el DENOMINADOR.
la frncción es MAYOR que la UNIDAD
Ejemplo:
.. ..
¾

·.· . . ·.
De una torta no poderrws servirnos las partes, en- .
. .

tonces debemos emplear dos TORTAS. así: Q .

* Transfonnación a Mixtos.-

Le llamamos MIXTOS, a una fonna de representar a las fracciones mayores que la


unidad.

Así: 7 ~ es un MIXTO, donde:

La PARTE ENTERA es 7
La PARTE FRACCIONARIA es !
1 ALFONSO ROJAS PllEMAPE 1

Este MIXTO puede ser desdoblado también así:


7_!_ = 7 + _!_
9 9
Entonces también es cierto que:

¿Cómo transformaremos una fracción MAYOR QUE LA


UNIDAD A MIXTO? ¡ATENCIÓN!
Veámoslo en un ejemplo:
Un MIXTO rcpresulla
a una fracción mayor
Transformar !!. a MIXTO
que la unidad.
3
PASO l. Dividimos NUMERADOR entre el DENOMINADOR .- --❖• ••.·-·-·---·-·········-·.-• •-. ---• --- -- - --.-.--.

Cociente: 3
Residuo : 2
PASO 2. El MIXTO buscado lo escribimos así:

• PARTE ENTERA · Está dada por el COCIENTE


obtenidn en la división.
En nuestro ejemplo: La parle
entera es 3

• PARTE FRACCIONARIA
- El numerador de la fracción está dado por el residuo
e:> O obtenido en la división.
- El denominador de la fracción es el mismo de la
fracción inicial.
ORI::CUHIWA Q UE:
En nuestro ejemplo: !!. = 31.
3 3 En una división como
¿Cómo transformaremos un MIXTO a una .fracción 8 L!:..
bxc e
MAYOR QUE LA UNIDAD? d
Para efectuar esta transformación, multiplicamos a es el Di,-idendo
el denominador de la parte fraccionaria por la b es el Divisor
parte entera y a este producto le sumamos el nume- e es el Cociente
des el Residuo
rador obteniendo el NUMERADOR de la fracción
buscada. El denominador es el mismo:

En el ejemplo anterior Y=3x.3+2 11


x\___3 3 3
IW8 éifi
1
MATEMÁTICA

SKANIPRÁCTICA

l. Completar en los recuadros los números que faltan para que se cumpla la igualdad:

(a} 3 x □ = 15 (b) 2x □ =6 (c) 5x D = 30 (d) 7x O =21


(e) 2x D=
10 (0 4 x O= 12 (g) 15>< 0 =45 (h) 13x O =26
11. Resolver las siguientes ecuaciones y escribir el valor de x de cada una en el recuadro
correspondiente:

ECUACIÓN X ECUACidN X ECUACIÓN X ECUACIÓN X


3x = 15 4x =12 7x = 14 5x =25
2x= 10 5x=30 6x=24 3x=33
2x=6 15x =45 3x = 18 4x=48

III. ¿Qué puedes afirmar de las siguientes ecuaciones?

ECUACIÓN X ECUACIÓN X ECUACIÓN X ECUACIÓN X

3x=5 4x =3 3x =7 6x =7
2x =7 2x =9 5x = 1 3x=32

IV. En las siguientes figuras colorea la parte correspondiente a la fracción referida:

<•>¾ EB (b) 2.
8
(c) l
8

V. Escribe como MIXTOS las siguientes ffacciones:

FRACCIÓN MIXTO FRACCIÓN MIXTO FRACCIÓN MIXTO

8 13 15
'3" T T
11 17 137
7 T -3-

Escribe como fracciones los siguientes MIXTOS:

217
1 MATEMÁTICA
1
1.3 LA FRACCIÓN COl\10 ELEl\lENTO DE
O lll:·cmW M l QUI~:
u~ PRODl:CTO CARTESIANO. c;>o
Si 11= {1;2)
La fracci6n .!!. también se representa por {a; b¡ B = {m; n}
h
para be#- O LI PRODUCID Ct\RfES/t\NO
Como a puede ser cualquier número emero: a E 7l. A x ll es el cnnju11ln de pares or-
Como ll puede ser cualquier número enLero denados ( a: b ).
EXCEPTO CERO: b E 7l. •
don.de a E¡\
El PRODUCTO CARTESIANO 7l. " 7l. • es el y b E ll
conjunto de Lodos los pares ordenados (a; b) que dan /1 fl :
lugar a todas las fracciones posibles .!!. ó ¡J/b_ llxB ={(/; m ) ,(l; n),(2, m ),(2;n.)}

~ --~> oo b
~ Una fracción cualquiera f pcncnece
.' ·';1'J:. al produclo cartesiano 7l. x 7l. •
•-,..J_
~:::~ :1
,,
'••f • t
'1: :1
1.-t SH;NOS EN U'.\JA FRACCH)N
Una fracción es POSITIVA si ambos L~rminos O O I ,\11'0/ff,\l\'TI~
Lienen el mismo signo
Así: ?Z. rq,rt•senw al conjwzlO de
+a - a nwncros t·rtlcro,\ .
6
+b -b
?Z. + r,11rc~cnw al co111un.to de
Una fracción es NEGATIVA si ambos térmi- n.um,,ro.f en.teros positivos.
nos Licnen DIFERENTE SIGNO
?Z. - rcprc.w:rtla al co11ju1110 de
Así:
+a , - a 11um,:ros entero.\· negall-
-- o -- vus .
-b +b
Vamos a convenir, que el signo positivo o ne- 72. •rcprcscma al ro111wzlo de
gaLivo ele una fracci6n esté determinado por el signo n.iímcrns ,·,ueros E.'fCEP-
'/0 EL CERO
positivo o negatirn del NU:\IERADOR, ya que con-
sideraremos al denominador ele signo positivo, ele
modo que:

Frac:cion pm itiv,1: ~
Fracuon ncgati\'a: ~# ooo ¡ Cl 'll).11>0!

El .wnbolo ,,--..,, s1g11i/ica

1.5 FRACCIO:\ES EQCI \' A LENTES j_r~ ''EQL'/l ALF. A". En la prá, ri-
i·a .,e le suele rcan¡,la:zar por el
Dos fracciones !!.. y .E. son EQUIVALENTES ~Ímholo =
b el
s1 se rnmplc que ad = be j C:::> Oí)í)
·O I ALFONSO ROJAS PUÉMAPE 1
Ejemplos:
3 9
( 1)
5 Y 15 son equivalentes
f( 0 ¡ATENCIÓN!
3 9
ó -rv-
5 15 lli: Una fracciC:111 cicnc una canei-
dad i/imi1ada de fracciones
3 9 EQUIVA LENTES oblcnidas
ó
5
= 15
porque3 x 15=9 x5 : por AMl'L/FICACJÓN
45 =45
~(¿,::· -- . º/ •••• • •••••• •• ••••••••••

(2) l "".!Q.
7 35

6 2
- =10
- porque 2 x35 = 7 xlO
7 35 70 = 70

Podemos obtener FRACCIONES EQUIVALENTES de


dos maneras:

111 POR Al\1PLIFICACIC)N


Para esto MULTIPLICAMOS numerador y denomi-
nador por un mismo número distinto de cero.

Ejemplo: 3
-rv--
3X 3 3 - -9
5 5X 3 5 15

3 3x (-2) 3 -6
◊ o
-rv --
5 5x(-2) 5 - 10
00 /U~COIW,\R QUB:
211 POR SIMPLIFICACIÓN
Para esto, DIVIDIMOS numerador y denominador A/ir.mar que dos nú,ncrns
por un mismo número distinto de cero. tienen como unit:o divi-
sor común a la unidad
l.!i.rv rn 2 18 9 si11nijica que cales núme-
Ejemplo: ó -=- ro.~ son f'IUMOS l~N-
90 90:2 90 45
TR B sl Ejemplo: 3 y5 .
18 18: 3 18 6
-""--
90 90:3 ó 90 30
/
Si los ténninos de una fracción tienen como único DIVI-
SOR COMÚN a la unidad. dicha FRACCI()N ES
IRR EDUCTIHL E o IRR EDUCIULE O OC.~
E1emplo: 3
5
1 MATEMATICA
1
1.3 LA FRACCIÓN COl\1O ELEMENTO DE
O lll~CO/Wt\ll QUI~:
UN PRODUCTO CARTESIANO. Qo
Si A= { l; 2)
La fracción* también se representa por ta; bJ n= {m;n}
para b c1-- O EL PRODUC/"O CARILS/1\NO
Como a puede ser cualquier número entero: a E &'.'. ;\ x ll es el conjwito de pllres or-
Como b puede ser cualquier número eme ro clc,uulo., ( a: b) .
EXCEPTO CERO: h E &'.'. •
dorule a E ,\
El PRODUCTO CARTESIANO &'.'. " &'.'.. es el y b E 11
conjunlo de Lodos los pares ordenados (a; b) que dan Así:
A,B = {( I; m),( 1; n),(2; m),(2;n)}
~a:;~O{ias las fracciones posibles ~ ó a/b.

f; Una fracción cualquiera t pcncne.:c


.,.,,,_
! ·':ZT-
,.... ,..
al producto cartesiano &'.'. " &'.'. •

,.,
~~¡ljj
IA Sl(iNOS EN UNA FRACCH>N
Una fracción es POSITIVA si ambos L~rminos O O / ,\f/'0/lTA N TE
tienen el mismo signo
Así: '11. r cp r l ·,·e11/a al umjwuo de
+ a ó - a 11111ncros enteros.
+b -h Z!. + r,-presenta al conjunlO de
Una fracción es NEGATIVA si ambos ténni- nwn,·ros enteros po\ilivos.
nos Licnen DIFERENTE SIGNO
Z!. · rqJrt'scnla al cm 1ju1110 de
Así:
+a , - a 11wn.,~ros l'lúcro.,· ne,r:ati-
-- o -- vos.
- b +b
Vamos a convenir. que el signo positivo o ne- Z!. •rqJf"l'Sl'tIUl al
con1w110 de
gativo de una fracción esté dcLcrminado por el signo numerns c1IIE'rr1s E.\"CF.P -
TO F.LCERO
positivo o negativo del NU:\IERADOR. ya lJUe con-
sideraremos al denominador de signo posillvo. de
modo que:

~
+a
Fracci6n po,;itiva:

::@jf~
+ h
ooo ¡CUJJ..\l>O!
Fracción ncgati, a:
+b r;J .w n/Jolo r--/ siw zifica
'T.QUH /\LE A" . F.n la prác-li-
1.5 FRACCIOI\ES EQL'IVALENTES <a .,e le suele recm{'lazar por el
Dos fracciones .!!. y .E. son EQUIVALENTES ~Ímho lu =
h d
si se cumple que ad = be I C::::'> 000
1 MATEMATICA
1
1.6 SIMPLIFICACIÓN DE FRACCIONES O RECORDAR QUE:

Simplificar una fracción significa transformarla en


Para hallar el MCD de
otra EQUJV ALENTE y a la vez IRREDUCTIBLE.
dos o más ntímeros, los
Para simplificar una fracción podemos proceder de descomponemos en sus
dos maneras: factores primos. entonces
el MCD estará dado por
l' Dividimos los términos de la fracción entre su MÁXI- el producto de los facto -
MO COMÚN DIVISOR (MCD) 0 O res COMUNES co11 su
Ejemplo: MENOR e.J..ponente.
Simplificar la fracción 24
180
Solucióll:
24 = 3 )( 23
PASO 1: Descomponemos los términos de la frac-
180 = 32 )( 2 2 )( 5
ción en susfaccores primos: __x
PASO 2.- Establecemos el MCD de los términos de MCD=3x22 = 12
la .fracción C'> O : __x
PASO 3.- Hallamos la fracción equivalente e irre- 24 _ 24: 12
ductible de La fracción dada dividiendo 180 - 180: 12
ambos términos entre el MCD. __x 24 2
RESPUESTA:
180 =rr
La fracción equivalente e irreductible de
OO ¡ATENCIÓN!
24 2
180 es 15 · En la práctica, emplea-
mos esta forma de sim-
Veamos una segundaforma de simplificar una fracción . plificar unafracci6n ex-
trayendo MIFAD - MI-
2' Dividimos sucesivamente los términos de la fracción TAD, T ERCIA-TERCIA,
entre divisores comunes hasta logmr una fracción irre- etc.
ductible. 12
.J<r
Ejemplo: Jz' 12
Así: j,$(J =L)
Simplificar La fracción 72
150 ?r
Solución:
72 72: 2 36
PASO 1: Dividimos ambos términos entre 2:
150 = 150: 2 =75
PASO 2: Dividimos ambos términos entre 3:
36 36: 3 12
-=--=-
75 75: 3 25
RESPUESTA.-
72 12
La fracción equivalente e irreductible de 150 es 25 O 00
1 ALFONSO ROJAS PUÉMAPE 1

Examinemos OTRO EJEMPLO:

Simplificar la fracción 36
300

Solución:

lºFORMA:
MCD (36 y 300) = 12
• //aliamos el MCD de 36 y .,;,,
300: ~
Rpta. O ¡CUIDADO!
• Dividimos el numerador y
denominador de la fracción ~ 36/12 Í3l ..á+b+c b+c
entre el MCD hallado: __/ 300 / 12 · Lfil Ám
=-
m
-
¡Incorrecto'
OBSERVACIÓN:
.A'(b+c) b+c
Si multiplicamos los términos de la fracción
. &'.m = m,
simplificada por el MCD de los términos de la ¡Correcrol
fracción inicial, obtenemos esta fracción inicial.
Esto último es correcto,
2ºFORMA: porque "a" está como
factor y no como su-
• Ambos términos tienen MI- 36 18
TAD. entonces los dividimos ;,r --=--
300 150
mando en el numerador
y denominador de la
entre 2: ~
fracción.
• Los nuevos términos siguen
teniendo MITAD . por lo que ;,r 36 = 18 = -9
los volvemo.rn dividir entre 2: ./ 300 150 15
• Los términos ahora obtenidos
tienen dividor común 3 (¡Tie-
nen tercia!) ¡entonces los divi- ;,r
dimos entre 3!: ~

• Como ambos términos, ya no


tienen divisores comunes, te-
nemos entonces la fracción ;,r
simplificada, es decir: ~

3ºFORMA:

• Descomponemos los térmi- 36 2>< 2" 3>< 3


nos de la fracción dada en ;,r 300
= 3><2><2><5><5
sus factores primos: ~

• Luego suprimimos factores


comunes al numerador y de-
nominador, obteniendo la ;,r
fracción simplificada: ~
e:> o
1 MATEMATICA
1
:=····· 1
.•,• ..
SKANIPRÁCTICA 0 l
l. Dados los siguientes conjuntos: A= [ l; 2; -3} ; B= {7; 5; -6}
C= [l; -4; 2; -9} D= {2:-1: 3; 5}
Escribir el producto cartesiano con el menor número de elementos

: 1

11. Si cada par ordenado (a;b) de un PRODUCTO CARTESIANO da lugar a una


fracción a señalar el número de fracciones que se obtiene en cada producto
~::
-¡;-·
.:-:
cartesiano señalado con los conjuntos dados en G), estableciendo además la
i cantidad de fracciones negativas y positivas, así como las que son mayores o
¿::
¡;: menores que l.
t
:::
PRODUCTO CARTESI ANO Nº FRACCIONES ( +) ( -) >I <I

Ax B
A xC
Ax D
B xC
BxD
CxD
¡: DxC

III. Escribir en cada recuadro SI, si las dos fracciones respectivas son equivalentes y
NO,si ocurre lo contrario.
•=·
Av lf2 1
14/49 6/8 21/24 28(35 12/1 6 35/21 21/28
'
;·: 3/4
.. 2{7
4/5
.==
:.: 2/4
;:: 7/8
f 5(3
{
t IV . Completa lo que falta, para que las igualdades sean ciertas
.• a) 3 0 b) 2
=
10 e) -22 =□ d) 5
7 =21 5 =º
•.!
:-
D 16 15 75
;: v. Escribir en el casillero correspondiente la fracción simplificada respectiva:
a/h 4/40 18/300 24/180 75/100 32/90 64/360 36/1800 38/1440
Fracción
IrrcducLible
~. -===~~==:-:. •,.;;~::;: -. ~:--~ ..-. , .-.--:-~•;::•:=.:-... ·'- ,:'.- -..--... .-... :~:-=••... :-:·:·:•<•:••:-•••:•=•J•:•?:·•·:·:···-.:•:·:•:•:·•·•·•·:•:::•:·:·:·•·•·•·•·:❖•-:-,-:-x•:•:•:·:·:-:·:ü:·.-:-•:•·•·••n·•·:·:•:•:·--:--::-:·=·•-:-•·:::·:·:·:·:·:·=-
] ALFONSO ROJAS PUEMAPE

Aquí mis operaciones de SKANIPRÁCTICA @


MATEMATICA

1.7 CLASI:. DE EQUIVALENCIA


Tomemos una fracción como 3/5 y establezcamos todas las fraccio-
nes equivalentes agrupándolas en un conjunto:

Observemos lo siguiente:
(1) A tal conjunto se le llama CLASE DE EQUIVALENCIA.
(2) Si de todas las fracciones del referido conjunto tomamos la irreducti-
ble de denominador positivo ésta será la FRACCH)N CANÓNICA
o REPRESENTANTE CANC>NICA de la CLASE DE EQUIV A-
LENCIA.
Ya podemos esL'lblecer ahora lo que significa NÚMERO RACIONAL

1.8 NÚMERO RACIONAL


ORECUERD,1 QUE:
Le llamamos NÚMERO RACIONAL a cada CLASE DE "72 x 71. • Si¡;nijica :
EQUIVALENCIA que se encuentra en el conjunto Z x Z * "Producto cartesiano for -

Ejemplos:
ºº ,,,.,
Í,,
mado por pares ordena-
dos ( a; b )""dorule :
a E 71_
(]) Un número racional cuyo REPRESENTAN1E CANÓNI- r bE 71. •
CO es 3/5 podrá ser escrito así: Además (a;b) repre:.enta
¡,,-
¡- a la fracción: alb.
{ j ;1~ : ~ ; ~t ;.....;J: } n ;é O

1 Resolver la ecuación 5x = 3 1
(2)
Solución:
¿Qué número x multiplicado por 5 da como resultado 3? 00 l ,UPOR TANTE
La solución de eslll ecuación no es un número entero,
A L CONJUNTO DE NU
pero sí unafamilia de fracciones equivalentes o CLASE MEROS RACIONA LES "
DE EQUIVALENCIA o NÚMERO RACIONAL que es: clases de equi1•a/e11cia e11
l._ . .!2._. 2._. !.1_ . . 3n } n "#-O Z X Z * se le reconoce por
{ 5 · JO · 15 · 20 · · · · · · 5 n la letra (D, de modo que
Para no referirnos a todo el NÚMERO RACIONAL sólo
emplearemos su REPRESENTANTE CANÓNICO que es
3/5. O 00
Comprobación.
Aseguramos que 3/5 es solución de la ecuación dada )
porque multiplicada por 5 da como resultado 3 O s( 53 =3
◄Q:· 1 ALFONSO ROJ\S PUÉMAPE 1

En la pracuca, es común referirnos a cualquier


fracción como l como a un NúMERO RACIONAL
5

1.9 RELACHl;\; EI\TRE [N ?_y~

¿Puede un número cnwro ser también NÚMERO RACIONAL'!


Veamos:
El número entero -7. podemos expresarlo también como
-7/+l que es una fracción Cll}OS Lérminos son números ente-
ros. Dicha fracción expresada con más propiedad como un
número racional puede ser escriL:t así:
- 7 _ - 14. - 21. - 2.8. }
{ + I' + 2 ' + 3 ' + 4 , .. . .

Luego: Los NÚMEROS ENTEROS 71.. pueden ser expresados


como fracciones y fonn:m parte del conjunLo de los
NúMEROS RACIONALES (!) .
O l<ECl '/: IW,\ QL'E:
EsLo es:
71. e (!) IN :C o 1111u110 de mémc r o f
naturales.

Además: Si IN e 71. y 71. e (D Z!.. C onjunlo de ntímcrof


c111eros.
EnLonces:
(!) :Conjunto de n úmeros
l1Nc7l.. e(!) 1 c:::>o racionalc.f .

GráfieamenLe: ...
. - . ~-- --~- _._: .....-. '."•: . . .. - . ·-. ;-._. _._.-·-·
RACIONALES ((D)

Naturales (IN) Enteros Fracciones

º· 1; 2: 3, .... (~ ) 1.
2· T
- I

5
-l;-2;-3 ; .. 1; 3

1.10 REPRESENTACIÚN DE ([) E N LA RECTA I\U i\ IÉRICA


Ya sabemos que los números emcros 71.. se represen-
Lan en la rccLa numérica así:

• • . -3 -2 -1 o +1 +2 +3 •• -
1 MATEMÁTICA
t
Si asignamos numerales que representen a los puntos medios de cada
unidad, tendremos la REPRESENTACIÓN GRÁFICA EN LA RECTA
NUJ\1ÉRICA de las fracciones de denominador 2.
Así:
-- -
-
.. . -3 -2 -1 o +l +2 +3

~· 1
1
lo . l ' l l
.
T
-5
T
'
-4
2 '
-3
2 2
-2 - 1
T 2
l
-22 3
2
4
2
5
2
6
2
' .
123456 7''
4 44 4444
Podemos observar en la gráfica lo siguiente:
• No podemos terminar de subdividir la distancia entre dos números
enteros consecutivos.

Esto significa que es imposible señalar el SIG UIENTE ni el


ANTERIOR de un número racional cualesquiera porque
ENTRE DOS NÚMEROS RACIONALES SIEMPRE
SERÁ POSIBLE HALLAR AL MENOS OTRO.

Un mismo punto en la RECTA NUMÉRICA puede ser representado


por varias fracciones que son EQUIVALENTES entre sí, por lo que
reafirmamos que el CONJUNTO DE DICHAS FRACCIONES
(CLASE DE EQUIVALENCIA) representa a un mismo NÚMERO
RACIONAL.

1.11 TRANSFORl\lACIÜN DE FRA C CIO NES A


DENOi\llNADOR C OI\I ÚN.
Dos o más fracciones de denominador diferente podemos transfor-
marlas a sus fracciones cquivalcmcs de igual denominador o DENOMI-
NADOR COMÚN por cualquiera de las dos formas siguientes:
1º FORMA.- EMPLEANDO EL mcm DE LOS DENOMINADO-
RES
Ejemplos:
(/) Tran.~formar a denominador co-
mún las siguien1es.fracciones:
3 2
T; s
AlFOl'iSO ROJAS PUEM.\PE

Solució11

PASO 1.- Hallamos el mcm de los denomi- O O mcm (7; 5) = 35


nadores: --"

PASO 2.- Dividimos el mcm entre cada


3
7
◊ (35: 7)x 3=15
denominador y lo multiplicamos
por el numerador de cada frac-
ción, obteniendo el nuevo nume- 1 f= ~~ 1
rador respectivo. El denomina-
dor de cada nueva fracción es el
2
5
◊ (35: 5)>< 2= 14

~ = ~~
mcm hallado: __,,
1 1

RESPUESTA:
Las fracciones dadas, transformadas a denominador f O RHCUER/J,\ QUE:
común son: lj:! Para hallar el mcm de dos
15 14
o mll'.t números, los des-
35 y 35 l
!!!!! :;;n¡~:;:::::,s :;:vu:[:t:~
(2) Transformar a denominador común mcm está dado por el pro-
las siguientes fracciones: ducto de los factores
COMUNES y NO COMU-
1 5 - 3
2 ;3;4 NES con su MAYOR EX-
PONENTE
Solución:
PASO l. Hallamos el mcmde los denomina-
__,:,, mcm(2;3;4)=12
dores:
PAS02. Dividimos el mcrn enzre cada de-
nominador y lo multiplicamos por
1 ◊(12:2)><1=6
el numerador de cada fracción.
obzeniendo el nuevo numerador
respectivo. El denominador de
cada nueva fracción es el mcm ha-
5
3
◊ (12: 3) x 5 = 20
1 f=~~ 1
llado: --"

-} ◊ (12: 4) x ( - 3) = - 9 - 3 - 9
--
4
--
12

RESPUESTA.-
Las fracciones dadas transformadas a denominador común son:
J MATEMATICA
1
2o FORMA. EMPLEANDO LA REGLA DE PRODUCTOS
CRUZADOS

Ejemplos: Ji O ¡ATENCIÓN!
f Podemos simplificar las
(1) Transformar a denominador común
las siguientes fracciones: li ::/º;::rv';:'~ni::,::~
los denominadores sigan
7
3
5
2
rf siendo iguales.
f:'
\;4 ::;:~ ;:::·:::;:::.:,:.:,.:"r;:·~ =::•·-=-•-.:.;;.;•::~~~~~;.;.:;;;::~
Solución:

□□
PASO 1.- El denominador común en am-
bas fracciones está ck,do por el
producto de los denominadores rxs: 7 x5
dados. ~

PASO 2.- Los numeradores respectivos se J><L


7 5
obtienen mulliplir.cmdo en forma
cruzack, el numerador ck,do por 3 x5 2 x7
el denominador de la otra frac- 7 X5 ' 7 X5
ción: ~

(2) Transformar a denominador común


las siguientes fracciones:
1 5 - 3
2 ; 3 ;4

Solución
PASO l. El denominador común en cada
una de las tres fracciones,está
dado por el PRODUCID de los 2 x 3 x 4 • 2 x 3 x 4
denominadores dados: ~

PASO2. Los numeradores se obtienen


multiplicando el numerador de
la fracción respectiva por Los 1 X 3 X 4. 5 X 2 X 4. - 3x 2 X 3
demás denominadores. así: ~ 2x3x4'2x3x4' 2x3x4

.!1..40 . - 18
24 ' 24 ' 24
PASO 3. Si podemos simplificar las tres
fracciones lo hacemos, pero
siempre y cuundo los denominu- Ji_.20.-9
12 ' 12 ' 12 Rpta.
dores sigan siendo iguales: ~
1 .-UFOMO ROJAS PUÉMAPE 1
l 1
CO\IPARACIÚN DE NÚI\IEROS RACIONALES.-
• Si comparamos dos fracciones de SIGNO DISTINTO. es
MAYOR la fracción POSITIVA.
Así: 0 RECUER/M QUE:
2 - 1
(1) es mayor que a < b
7 5
"a es menor que b"
, 2 - 1 porque el lado de a cue11ta
O ->-
7 5 con un SOW PUNTO;"bes
(2) - / es menor que f mayor que a" • porque el
lado de b cuenta con DOS
PUNFOS.
-8 <2.


ó 3 5
ºº
Si comparamos dos fracciones del MISMO SIGNO se nos
presentan dos casos:
I'. Si los DENOMINADORES SON IGUALES compara-
mos sólo los numeradores, el NUMERADOR MAYOR
indicará la FRACCIÓN MAYOR.
Ejemplo:
¿Cuál de las siguientes fracciones es mayor?
- 2 - 10
5;-5-

RESPUESTA.
Comparamos sólo los numeradores : como -2 es mayor
que -10 entonces -2/5 es la fracción MAYOR O 00

2°. Si los DENOMINADORES SON DIFERENTES, Trans-


formamos las fracciones a DENOMINADOR COMÚN
y luego procedemos como en el caso anterior.

Ejemplo: 00/U::CVEIW,\ QUE:

¿Cuál de las siguientes fracciones es mayor? En la RECTA NUMÉRICA


los números "crecen" de
7 . 3 izquierda a derecha.
9'5 10 •-.1 -1 O +I +1 ••• +10
Por la REGLA DE PRODUCTOS CRUZADOS:
-JO< -2
e===::::>
ó -2 > - 10
35 = - ]_><l.- =27
9 5
Al transformarlas a DENOMINADOR COMÚN, el ma-
yor numerador es 35, luego: l 719 > 3/5 1
1 MATEMATICA 1
SKANIPRÁCTICA ®
l. La expresión t de 50 nuevos soles significa que hay que operar l x 50 y dividirlo enLre
2; es decir: -]x50
2- = 2.SO = 2'i. Iª
e En general : b de e= -b- , segun eso completar: axe¡ ,
(a) }de 100 = D (d) ¾de 120 D = (g) tde D =80

(b) ¼de 200 = D (e) fde 180 = D (h) j-<le D = 160

(c) f de 80= D (0 fdc5 500 =□ (i) fde D =200

11. Complclar con un SI o con un NO según corresponda en el siguienLe cuadro:


5 1/2 -1 -7 o -7/2 -8/4 10/5 1/7 -6

IN SI

"1J.. SI

(D SI

III. Señalar el númao ANTERIOR y SIGUIENTE al número indicado en los respectivos


conjuntos numéricos Si no es posible determinarlo o no lo tiene, marcar el casillero
con un a:-.pa.

3 -7 1/2 to - 10/3 15/3

En IN

En "1J..
En (D

IV. Transfonnar a fracciones con denominador común a los siguientes conjuntos de fraccio-
nes:

(a)
1 1
s; 7 (d) 1. . _!_ . -5
2' 3' 7

V. Completar con > ó < según corresponda:

/ 2 1,2 -1/2 .\/2 -m -5{3 7(3 1/4


-5

5/4 <
-2{3

7/2

.:-..•.-.-!-:-:-:-.-.--:-:-.- ·-:-:-.-: .-.-:-:-: ..


1 AlFOl\'SO ROJAS PUE1\l-\PE 1

Aquí mis operaciones de SKANIPRÁCTICA 0

!i~.
~'J.
?.. ...,¡
lcq:,
,,
'1i.H
MATI:MATICA

1 RAZONEMOS - 1

Instruccion es:
A conti nuacion te ¡>r nnto bloques de ej, rci.N'JS para afianzar
loleulohasta aqu• \fo rea l.~ respuestacorreclaycomparala
crm la CIA VE nE RF.C:,p[Jf,;.C:Ti\S r <,paree al final, "m
embarf;!o, te vm ""' qu tu cuadrrno las
RAZOVESd tur. 't J .Not< trabajocomouna
pesada rar{!a, sino omo w1 DI'. AFÍO a tu mtelfrrenr,a.

In OQUF 1 PASO 6.- Calculamos la



diferencia de los términos:
(1) Al simplificar una fracción, obtuvimos In. ♦ 35 -5 = 30
Sabiendo que la suma de los términos de la
fracción es 40, calcular la diferencia de los ♦ Rpta.@
mismos. ♦
(2) ¿Qué fracción de 16 es 2?
a) 25 b) 35 c)40 d) 30 e) 28 ♦
a) 1/2 b) 1/4 c) 1/6 d) 3/4 e) 1/8
Solución: ♦
Solución:
Simplificar unafracción significa ♦
transformarla en irreductible. Tenemos que identificar el
♦ "todo" y "la parte", recor-
Antes de simplificarla: mklnk
♦ dando que con respecto a la
Después de simplificarla: mln
unidad, los términos de una
donde k es MCD de (mk) y (nk) ♦ fracción expresan:
♦ "la parte"
PASO l. -Ubicamos la.fracción
simplificada: 1/1 ♦ "el todo"
PASO 2.- Establecemos la lxk ♦ PASO 1.- ldentificamos "el todo": 16
fracción alltes de su simpli- 7xk ♦
ficación: PASO2.-ldentificamos "/aparte": 2

PASO3.- Pordato: lasumade PASO 3. -Establecemos lafracción: 2116
lostérmilwses40: 7k+k = 40 ♦
2 l
♦ PASO 4.- Simplificamos lafracción: - =-
PASO 4. - Buscamos un valor 16 8
de k que permita que la igual- ♦
dad sea cierta: k = 5 Respuesta. -

PASO 5. - Establecemos la 2 es 1/8 de 16
lx5 5 ♦
fracciónalltes de simplificarla: Rpta.(v
7x5 35 ♦
•t"' ALFO!°'ISO ROJAS PUÉMAPE

(3) Si simplificamos una fracción, obtenemos 1/3. ♦ 1 11. -2 111. +3 2


l. - IV. -
Si la suma de sus términos es 28, calcular su
diferencia. ♦
7 7 -5 -3
♦ (18) Señalar la fracción mayor que 2/5.
(4) Al transformar una fracción en irreductible 1.1/4 11.4n 111.ln IV.3/11 V.7/19
queda convertida en 2/5. Si la diferencia de sus ♦
términos es 12, encontrar la suma de ellos. (19) ¿Cuál de las siguientes fracciones es menor que
♦ 3n?
(5) El MCD de los términos de una fracción es 7. ♦ 1.5/8 11.6/11 111.2/3 IV. 2/15 V. 3/5
Si dicha fracción es equivalente a 8/9, encontrar
su numerador. ♦ (20) ¿Cuál de las siguientes fracciones es mayor que
1/6?
♦ l.5n 11. 3/19 lll. 2/43 IV. 2/3 V. I y IV
(6) Una fracción es equivalente a 3/5. Encontrar el
denominador si se sabe que el MCD de los
♦ (21) ¿Qué fracción del círculo representa a la región
términos es 15.
♦ sombreada en la siguiente figura?
(7) Al simplificar una fracción a/b obtenemos 7/9. ♦
Si el MCD de a y b es 6, hallar a + b

(8) Si deseamos simplificar la fracción 32/48, ♦
dividimos ambos términos entre el MCDde los
mismos. ¿Cuál es dicho MCD? ♦

(9) Si simplificamos la fracción dada en el problema ♦


(8) empleando el MCD. ¿cuál es la fracción ♦
simplificada?
♦ (22)
( IO) ¿Cuál de las siguientes fracciones es mayor? ♦
1. 5n 11. 3n 111.-wn 1v. 11n v. 1n

1 1 1 1 1 1 1 1
(1 1) ¿Cuál de las siguientes fracciones es mayor? En la figura mostrada: ¿Qué fracción del
1.-1/9 11. -7/9 111. -15/9 IV. -2/9 V. -5/9 ♦ rectángulo mayor, representa la región som-
♦ breada?
(12) ¿Cuál de las siguientes fracciones es mayor?
l. 12/3 11. 12/5 111. 12n IV. 12/11 V. 12/13 ♦ (23) En la siguiente figura, ABCD es un cuadrado.
¿Qué fracción de ABCD representa la región
(13) ¿Cuál de las siguientes fracciones es menor? ♦
sombreada?
l. 1/3 11 . 2/5 111. 3/4 IV. 2n V. 1/5 ♦
A 2 2 B
(14) ¿Cuál de las siguientes fracciones es mayor? ♦
l. -2/3 II. 2/3 III. -7/11 IV. -In V . -3/8
♦ 2
(15) Señalar la fracción menor: ♦
l. 5/3 11. 2n III. 1/4 IV. 2/11 V. -11/2

2
(16) ¿Cuántas fracciones mayores que cero mos- ♦
tramos a continuación?

2
l. -
-2
11. -
1
111. - - IV. -
2 D c

7 7 9 -5
♦ (24) En el problema anterior: ¿Qué fracción del
(17) ¿Cuántas de las siguientes fracciones son me- cuadrado mayor representa la parte no som-
nores que cero? ♦ breada?
1 MATEMÁTICA
1
(25) En la figura siguiente ABCD es un cuadrado y ♦ (35) ¿Cuáles el númeroquedeberíamosescribiren
O el centro. ¿Qué fracción del cuadrado re- el casillero mostrado para que la igualdad sea
presenta la región sombreada? ♦ ciena?


(36) ¿Cuál es el número que deberíamos escribir en
♦ lugar de x, para que la siguiente igualdad sea
ciena?

3
♦ -de x= 36
7

(26) En el problema anterior: ¿Qué fracción delcua- ♦ (37) ¿Cuál es el número que deberíamos escribir en
drado quedó sin pintar? ♦ el casillero para que la siguiente igualdad sea
ciena?
(27) En la figura mostrada: ¿Qué fracción del círculo ♦ D-
representa a la región sombreada? ♦ de 143 =26
11

♦ (38) Calcular:
1 1 4
♦ -de-de-del05
2 3 5


(39) ¿ Cuál es el número que deberíamos escribir en
el casillero para que la siguiente igualdad sea
♦ ciena?
(28) En la siguiente figura: ♦ 1 1 2
-de-de - de630=12
♦ 7 3 0

(40) ¿Cuál de las siguientes expresiones representa
♦ a la región sombreada de la figura mostrada?
¿ Qué fracción del rectángulo mayor representa
la región sombreada? ♦ A .--------,---------, B

(29) En el problema anterior: ¿Qué fracción del
rectángulo mayor representa a la región que ♦
quedó sin sombrear? ♦

(30) Calcular 1/2 de 7 000 ♦


(31) Calcular 2n de 2 604 D c
♦ 3 1 3 l
(32) Hallar 1/2 de 1/4 de 560 (1) -de-de ABCD (11) - de - de ABCD
♦ 5 2 4 2
(33) ¿Qué fracción de 102 es 68? ♦ l 1 3 l
(111) - de - de ABCD (IV) - de - de ABCD
(34) ¿Qué fracción de 105 es 45? ♦ 2 4 8 4
f AlFO\'SO ROJAS PUEMAPE 1
BLOQ UE 11
♦ (2) Una computadora pesa 8 kg más un tercio de
(1 ) Calcular el número cuyos dos tercios es 34. su peso total ¿Cuánto pesa la computadora?

a) 26 b)62 c)51 d)56 e) 63 ♦ a) 8Kg b) 12 Kg c) 10 Kg d) 14 Kg e) 6Kg

Solución: ♦ Solución

¿Cuántos tercios tiene la unidad? ♦


Lo que se pide averiguar es el TO -
Tres ¿verdad? ♦ TAL (peso de la computadora)
Porque 1 = 3{3 empleando el dato que consiste en

¡Cuidado! Esta misma unidad se el peso de una PARTE. para lo cual
puede expresar así: ♦ empleamos el mismo razonamiento
del problema anterior.
2 3 4 5 ♦
]= 2=3=4=5= ... ete.

l°FORMA.
J>,tSO 1.- Si dos tercios es:34 ♦
Luego un tercio es la mitad de 34: 34 / 2 = 17 PASO l.- El peso
♦ total: 8 kg + 1/3 de
PASO 2: El total pedido es igual a tres ter- su peso total
cios ó tres veces UN TERCIO : 3 x 17 = 51 ♦
PASO 2.- Como el


8Kg. equivale
RESPUESTA: ♦ peso total se puede a dos de los
El número cuyos dos tercios es 34, será 51 . ♦ subdividir en tres tres tercios
tercios: (2{3)
Rp. c ♦
PASO 3.- Entonces


NOTA. 8
I tercio equivale a
2 =4Kg.
Este problema también puede ser planteado ♦ la mitad de 8 kg
así: PASO 4.- El peso

-J-de □ =34 ♦
total está dado por
tres veces un tercio,
3x 4= 12Kg

es decir:
LTotal ♦
Para el cálculo del número a escribirse en el ♦ RESPUE'iTA
casillero, multiplicamos 34 por 3 y dividimos
el producto entre 2: ♦ La computadora pesa 12 kg .

♦ Rpta. b
□ _34 x3
- 2

□ =5/ ♦
J•ASO J.- Leyendo

En General, si se tiene: el problema, se ◊ 2 □

nota que 2/3 del 3 de =8
peso total equivale
c¡;-de □ =c ♦ a 8 kg. es decir:

el número del casillero se calcula así: ♦ l'ASO2



□ =c:b ♦
Luego;
1 MATEMÁTICA
1
♦ ( 10) Una botella de gaseosa de litro y cu:irto de
/',\SO 3: Para hallar
la ranzidad a escribir- ♦ capacidad. está con líquido hasta sus 315
se en el número del ¡,Cuántos lilros de gaseosa tenemos"!
rasillero. que e.f el ♦
e) 2l
peso total buscado ♦
7
multiplicarnos 8 x 3 y d) 2_ t e) ] l
dividimos el producto ♦ 5 4
e,11re 2 (ver problema ♦ Solución
( 1)):

-operando. □ = 12Kg ♦
Lo que vamos a hacer r11 .._,te
pro/Jl,·ma se par,:ce al ral1.11lo de la

mllad tic I O que e., 5; lo cual wmlú, "
RESPUESTA:
se expre.\11 <L~í: / 12 de /0.
La computadora pesa 12 kg ♦
La pafohra "de" si¡;11(firn c¡11t'
♦ tc,u·mos que 1111tltiplirar:
(3) ¡,Cuál es el número cuyos 5n es 85'?
♦ I • /{) =/ "/0 =5
a) 117 b) 129 e) 119 d) 139 e) 149
2

(4) ¿De qué número es 78 sus 3/4?

a) 99 b) 93 c) 102 d) 104 e) 106 P,\SO 1.- Rcprcsc111.11nos
♦ "litro y cuarto" :
(5) ¿Los 2/5 de qué número es 30? ♦
l',\SO 2.- Tran.iformamos
a) 85 b) 75 c) 65 d) 55 e) 70 ♦ MIXTO afracrián:

(6) Los 4n de la propina de Luis equivalen a 52 ♦ /',\SO 3.- La parte de la 1OS j3l e
I 5 ,.IITO
botella con líquido es: 4
nuevos soles. ¿Cuánto es la propina de Luis? ♦
a)S/.103 b)S/-g3 c) 51-97 d)SJ.91 e) 51102 /',\SO 4.- Expresamos lo
♦ l.5 x~4 litro
a11terior como una multi-
♦ plicación:
(7) Los 219 del costo de un artefacto es SI. 34.
¿Cuál es el costo del artefacto? ♦
/',\SO 5 .- La multiplica-
3 X 5 .
a) SI. 153 b) SI. 117 c) SI. 162 ♦ ción de Jraccwnes se rea- x 11tro
5 4
d) SI. 148 e) SI. 178 liza TÉRMINO a TÉRMI-
♦ NO, es decir:
(8) Un alumno del colegio pesa 16kg más los 3n ♦
3X 5
de su peso total ¿Cu.ínto pesa dicho alumno'!

/',\SO 6.- Simplificamos:
5x 4 =--¡3 Jº
1tros

al 22 kg b) 24 kg e) 19 kg RESPUESTA:

d) 21 kg e) 28 kg
♦ Tenemos en la botella 3/4 de litro de
(9) Una caja de herrannentao; en un taller pesa ♦ gaseosa
55kg m.1~ los 6/11 de su peso total. ¡,Cuánto
pesa la caja de hLTiamientas? ♦ (11) Disminuir 300 en sus 7/12
a) 119 kg b) 127 kg c) 121 kg

a) 125 b)75 c) 25 d)l50 e) 175
d) 126 kg e) 133 kg. ♦
t ALFONSO ROJAS PUÉMAPE 1,
Soluri,i11: ♦
(15) Una piscina puede llenarse totalmente con
Toda cantidad tiene dos medios. tres ♦ 300 litro~. Si actualmente está llena hasta sus
ll'rno.,·, r·uatro cuartos, . .doce 13/25 (que lcem06: "Trece veinticincoavos")
♦ ¿Cuántos litros de agua debemos aumentar
don ·,n-os. ,·ir
Si u11a n1111iclad tic11e don: do,.cavos ♦ antes que la piscina empiece a rebalsar?
( 12/12) y Le re.,w11w.,· ,; clisminuimo., 7

do, t avos nos quedan 5 doceavo., . b) 112/ c) 144/
a) 104/

d) 124/ e) 100/
/>ASO l. - Identificamos
300 ♦
ta canzidad total :
♦ (16) Disminuir 180 en sus 11/15
/>ASO 2.- Identificamos 7
La cantidad a disminuir : 12 de 300 ♦ a) 36 b) 24 c) 96 d)48 e)44
l',\SO 3.- lde111ijicamos 5 ♦
la cantidad que queda: 12 de 300 (17) Un alumno tiene 13 años de c<.lad: si se dismi-
♦ nuye la edad en sus 2/13 ¿Qué edad dice te-
l',\SO 4.- Expresamos ner?
"de" como una multipli-
_2_ X 30() ♦
12
ració11: ♦ a) 10 b) 11 e) 9 d) 8 e) 12
l',\SO 5.- Opera11do _'í X 3(1() = 125 ♦
12 (18) Aumentar 1 19 en sus 5n
RESPUESTA : ♦
a) 204 b) 200 c) 202 d) 206 e) 208
l\l dismi11uir JOU e11 su.,· .2.. resulta 125 ♦
12
♦ ( 19) La hermana de César tiene 15 años, pero gusta
(12) Una botella de dos litros está llena de agua aumentarse la edad en sus 2/5 frente a sus
h:L~ta sus 2(3 ¿Cuántos litros de agua hay en la ♦ amigos ¡,Qué edad dice tener'!
botella'! ♦
a) 17 b)18 c)19 el) 20 e) 21
b) lt e) 2-t ♦
3 3 ♦ (20) Un jugador en su primer juego, pierde la mitad
d) i / e) 2 l de su dinero, en el segundo juego pierde 1/4 de
3
♦ lo que le quedaba y en el tercer juego pierde
(13) Un depósito de cuatro litros de capacidad está

1n del nuevo resto. ¿Qué fracción del dinero
lleno de gasolina hasta sus 3/5.¿Cuántos litros inicial le ha quedado?
de gasolina hay en el depósito? ♦
a) 11/28 b) 9/28 c) 13/28

a) 7_ l b) 11 t c) 12 /
5 5 5 ♦ el) 17/28 e) 15/28
d) 13 t e) 17 /
♦ (21) En nuestro colegio 4 de cada 7 alumnos postu-
5 5
lan a la universidad, de los cuales solo ingresa
(14) En el problema anterior. ¿Cuántos litros debe- ♦ la cuarta parte ¿Qué fracción de los ~ilumnos
ríamos agregar para que se llene el depósito? del colegio ingresan a la universidad'!

a) 1/4 b) 1/5 c) 1/6
b) 'li t c) 2. l ♦
5 5 d) 1/7 e) 1/8
d) Jl / e) 12 l ♦
5 5 (22) En un cierto país, hubieron elecciones con dos

candidatos A y B, donde 3 de cada 5 habitan-
1 MATEMÁTICA
1
tes prefirieron no votar. Si de las personas que ♦ P✓lSO 2.- ldcntijica-
votaron 5/6 lo hicieron por el candidato A mos "el total" de la 36 + 18 =54 litros

¿Qué fracción del total de habitantes represen- mezcla:
ta a los que votaron por A? ♦ PASO 3.- Estable-

a) 1/4 b) 1/2 c) 1/5 d) 1/3 e) 2/3 ♦ cemos la porción o


fracción de cada
♦ parte en la mezcla: leche : ~ Ltrs. de Leche
(23) Supongamos que en el distrito de Ventanilla, 54 Ltrs. de MeLcla
4 de cada 5 estudiantes de 1º grado de secun- ♦
daria aprueban el curso de MATEMATICA ♦
este año. Si de éstos, las 2/3 parles obtienen de simplificando: 1Leche Í I de mezcla
nota 15 ó mas ¿ Qué fracción del total repre- ♦
senta a estos alumnos?
♦ 1X Llrs. de Leche
/\gua: - ,
♦ 5.., Ltrs. de Mezcla
a) 7/15 b) 8/15 c) 11/15
d) 1/15 e) 13/15 ♦

(24) En el problema anterior ¿Qué fracción del


♦ simplificando: 11\gua TI de Mezcla

total representa a los alumnos aprobados 4ue ♦


obtienen menos de 15 de nota'!
♦ /',\SO./.- /..o ,:,u:crrado en recuadros signifi-
a) 1/15 b) 7/15 c) 8/15 ♦ ca que la IERCFJM PAR"/ E DE LA ME/.CLI\
es agua y los VOS TERCER,\S P,\RJES DF..
d) 11/15 e) 4/15 ♦ LA ME/.CLJ\ es leche.
♦ l',1S0 5.- Si extrae-
(25) Un depósito contienc36 litros de leche y 18 de
1
agua. Se extrae 15 litros de la mezcla ¿Cuán- ♦ mos 15 litros de Agua (15 ) = 5 /
mezcla, "la lercera
3
tos litros de leche salen?
♦ parte es agua y las ,.,
a) 7 b) 10 c) 6 d)9 c)8 ♦
dos terceras partes
es leche" :
Leche :
3(15) =10 l
SulucMn: ♦
♦ RESPl)ESTA:
Como r.• una mezcla. dl·lerminare-
♦ En 15 litros de mezcla, salen JO litros de
mos ron t·xaclitud lav rarlt:s y el
leche
lodo. para lue:;o esla/Jlcccr [Cl.\ ♦
/ra, ,·wnc.v que rcf>rt'.,c11tc111 a aula Rpta. b
1:0lllf'OIIOlte ele la mezcla. ♦
(26) Un tonel contiene 40 litros di! vino y 1O litros
H,V CU,11.QUl/:"ll C,\ NTIIJ,\/J ♦ de agua. Si extraemos en otro dep<ísito 35
/ )/;' Ml~ZC/.,\ Ql 'J.; SI -: 1-:xnu1-:,
lilros de la mc:tcla ¡,Cu;íntos litros de vino
/.,\ llU' /ll-:SJ:ºf\'F,\ CJ()N nuc- ♦
salen'!
C/01\ ',\/U ,\ /JE /,✓ \ S l',\/lT/~S ES

/.,\ MIS.\1,\.
♦ a) 28/ b) 26/ e) 32/
d) 30/ e) 25/
PASO 1.- lde11tificamos "las ♦
partes" de la mezcla: 36 ltrs. de leche ♦ (27) Un depósito contiene 10 litros de lNCAKO-
18 ltrs. de agua LA, 18 litros de COCACOLA y 42 litros de

1 ALFONSO ROJAS PUÉMAPE 1
FANTA . Si extraemos 14 litros de la me:,.cla. ♦ /',\!)(} J.· Empezamos a razonar a parlir del
¡_Cuallll>S litros de INCAKOLA salen'!
últi.,no dato

a) 3l b) 5l c) 4/ d)2l e) 6/ "Luego se ve,ule 1/3 ,Je/

resto" enlmu:c.,· quedan
1:.n un deposito se mezdó 30 litros de agua y ♦ 2(3 del resto que n¡uivalcn
50 litro!> de leche, luego se extrae 16 litrns de a 24 ·1011clad<Lf:
la me:r.cla y se le reeempla1:a por la misma ♦
cantidad de agua. Si de la nueva mezcla se ♦ :? del rel·to = 24
vuelve a extraer 18 litros ¿Cu:íntos litros de 3
leche salen'! ♦
6 :? x resto = 24
a) 6/ b)9l c)JOl d)l2l e)lll ♦ 3
24 3
/?esto= x = 36 Ton.
♦ i
(29) En un tonel hay 60 litros de vmo A y 40 litros
de vino R. Si cada litro de vino A cuesta S/. l(J ♦
J'i\SO 2.- El RESfO hallac/,,"c11 C'i P1\SO I es
y cada litro ele vino R cuesta S/.5 ¿Cuánto ♦ Lo que queda de la {'rimera venia . S, se vendió
cuesta 45 litros de la me1:cla? 1/4 de La C/t.,VJJ[),\l) l,\ 'IC/1\L e111"11<'cs que-

daron 3/4 de la CAVflD,\D /1\º/Cl,\L que
a) S/ 320 b¡ S/. 500 c)S/.25()
♦ equivalen a Las 36 UJNEú\D/t.S
cl)S/ 3<,{) e) Sf.421)
♦ 3/4 de La C,\NTJDMJ 1;\'!Cli\L = 3ñ
♦ = 48
(.\0) Tenemos una botella de gaseosa de un litro y C/t.N'JJD/t.D J,VJCJAL = 3ó ,..-4
se hehe la mitad que se recmpla/11 ,x,r ,1gua; 3

otra ve/. se bebe la mitad de la gaseosa con RESPUES'f/t. :
agua y vuelve a llenarse la botella con agua; se y
/nicfrúmcnic luúJiu .J8 toneladas de harina
repite lo mismo por tercera vez ¿Qué cantidad

ele g.L~eosa queda en la botella?
(32) Cada ve:, que emro a un salcm de PRIMER
♦ GRADO dejo la mit:tcl más 8 hojas que tengo;
a) 1/8 b) 1/6 c) 1/4 d) 1/2 e) Nacl:1
♦ si entro sucesivamente a 3 secciones y al final
(31) En un molino se tiene una cierta cantidad de me quedo con 61 hoja.,; ¡,Cu.il es la cantidad de
♦ hojas que tenía al entrar al primer salcín?
toneladas de harina de las que se vende 1/4.
Luego !>e vende 1(3 del n:~to quedando por ♦ a) 800 b) 500 c) 6U0 d) 400 e) -150
vender 24 toneladas. ¿Cuantas tonclad,L~ de
♦ SUGERENCIA. consider;IT lo siguiente
harina hab1a inicialmente'!
♦ Si gasto 1/2 nuis 1, me quedan 1/2 mcnos 1
h) -18 c) 24 el) 34 e) 26
♦ Si gas10 1/2 m:ís 5, me qm·cl.m 1/2 menos 5
5,,/ul'iá11:
Si ¡;:L-;to ]{3 m:is 7, mequ..-dan 2(3 mcnos 7

En prohlcm.is donde ~e pide ha-
ll ir una C,\NTID..\D INICl,\L ~ (33) Un ;ilumno entra a dos lihrería,- en fon na suce-
e, rcu1111,·ml.1hle cmpc ✓ ;tr ar.,,,,,_ siva con una cierta cantid;,d ele dinero. En la
n.,r por d 11lt11m1 el .no. Adcm,i,.
t primera gasta 1/3 de lo que tenía m:ís S/.1 n y
,1plKarc1uo~ lo ,1g111c111e: ♦ en la segunda gasta 1/10 de lo ~1ue tenía m:ís
S/.1 O. Si regresa a su casa con S/.53. ¡_Cu:íl es
♦ la cantidad que tenía al inicjo'!
♦ a)st- 100 b)s, 811 c) s r. 70

♦ d) Sf. l ~O e)Sf. 140


1 MATE l\1ATICA

(34) Una señora va al mercado con una cierta ♦ ¿cuántos litros de gasolina de 90 octanos se ha
cantidad de dinero para hacer tres compras consumido?
distintas en tres lugares diferentes. Cada vez ♦
que entra a un lugar gasta la mitad de lo que ♦ a) 2 L b) 2.!. L
tiene más SI. 20. Si al final se queda con SI. 65, ♦ 2
¿cuánto dinero tenía al inicio?
d) 1 _!_ L e) 3 L

a) SI. 600 b) SI. 800 c) SI. 700 3
d) SI. 720 e) SI. 840 ♦
♦ (5) Arnaldo gasta su dinero de la siguiente manera:
(35) Un depósito está lleno hasta una cierta al tura en 114 en un libro, 113 del resto en pasajes y
metros. Si abrimos el desagüe y encada hora el ♦ todavía le quedan SI. 24. ¿Cuánto tenía Amaldo
nivel del agua baja la mitad más I metro, siendo ♦ inicialmente?
que al final de la 3r, hora ya no hay agua; ¿cuál a) SI. 26 b) SI. 34 c) SI. 24
es la altura del nivel inicial de agua en el de- ♦ d'[SI. 48 e) SI. 36
pósito? ♦

♦ (6) En una construcción se han mezclado 500 kg de


a) l2m b)l8m c) 16 m
arena y 300 kgde cemento. Se utilizan l 60 kg
d) 20m e) 14m
♦ de la mezcla en el llenado de los techos. Se
agrega a lo que queda 160 kg de cemento.
♦ Si se vuelve a extraer 180 kg de la nueva mez-
BLOQUEIII
♦ cla para las paredes, ¿cuántos kilos de arena se
(1 ) En una reunión se observa que 17 caballeros ♦ usaron?
fueron con terno azul, 20 con ten;o marrón y 13
a)60kg .l b)I00kg c)l20kg
con terno negro. ¿Qué fracc!ón del total fue con ♦
,!IJ90kg e)II0kg
temo marrón? ♦

a) 315 b) 2/5 c) 13150 ♦ (7) El tanque de gasolina de una moto tiene una
d) 17150 e) 40150 capacidad de 8 L. Si se encuentra Ileno hasta sus
♦ 314, ¿cuántos litros faltan para llenarse?
(2) Enunabolsahay25caramelos; 12sondefresa, ♦
1
8 son de limón y el resto de menta. ¿Qué ♦ b) 1..!.. L e) 2-L
2 3
fracción del total son de menta?
1
♦ d) 3L e) 3- L
a)2/5 ./ b)315 c)3/I0 2
JT) 115 e) 7110 ♦
♦ (8) ¿Qué fracción del rectángulo mayor, representa
(3) En una tienda venden una camisa en SI. 30y un
la región sombreada?
pantalón en SI. 48. En otra tienda venden la ♦
camisa en SI. 32 y el pantalón en SI. 40. Si en ♦
la primera hacen un descuento de 1/6 y en la
segunda hacen un descuento de 118. ¿ Cuánto se ♦
pagó en la tienda que conviene más?

a) SI. 62 ~SI. 63 c) SI. 60 ♦
d) SI. 65 e) SI. 66
♦ a) _!_ b) ~
(4) Un chofer, acostumbra llenar su tanque de
gasolina con 1{i ía de 84 octanos y~ Ju.
de 90 ♦ 4
d)-
5 8 3
e)-
octanos. Si y~ ha consumido 5 litros de mezcla, ♦ 16 8
1 ALFONSO ROJAS PUÉMAPE 1
(9) Si los 3n de un terreno. pertenecen a un ♦ ,ÍSI. 96 b) SI. 72 c) SI. 48
hermano, y está valorizada esta parte en 24 mil d) SI. 192 e) SI. 94
dólares; ¿en cuánto estará valorizada la parte ♦
que pertenece al otro hermano? ♦ (15) ¿Qué fracción del círculo representa la región
sombreada en la siguiente figura?
a)$ 36 000 b) $ 28 000 c) $ 30 000 ♦
cM$ 32 000 e)$ 35 000

(10) En una reunión, los 3/8 son varones, de las ♦


mujeres, 1/5 son casadas. ¿Qué fracción del ♦
total son solteras?
1 1 ♦
a) - b) -

4 3
1 1
3 5 ♦ a) - b) -
d) - e) - 3 4
4 8 ♦
d) ~ e) 1
♦ 4
(11) Un industrial gastó 2/5 de su dinero en comprar
materia prima y los SI. 120 000 restantes en ♦ 0 6) En una reunión, los solteros son 1/3 del número
hacer mantenimiento. ¿Cuánto tenía el indus- ♦
dec~ados. ¿Qué fracción.del.total son casados?
trial? . ·1 2 1

a) - b) - c) -
a) S/.180 000 b) Sl.21 O000 c) S/.195 000 ♦ 3 3 2
d> s1.220 ooo ,rs,.
200 ooo 1
♦ e)-
(12) Un niño cada vez que toma de una botella llena ♦ 4
con gaseosa, bebe la tercera parte. Si ha hchido
3 veces seguidas, luego de las cuales qunlan ♦ CLAVF DE RESPUESTAS
240 cm3; ¿cuántos cm 3 habían inicialmente·.' ♦
RAZONEMOS 1
a) 850 cm 3 b) 820 cm 3 e) 900 cm 3 ♦ BLOQUE 11
3
d) 810cm e) 920cm 3
♦ (1)c (11)a (21) d (31) b
(13) Se entrevistan a los alumnos de un salón y se ♦ (2) b (12) a (22) d (32) e
obtienen los siguientes resultados: 12 visitaron ♦ (3) e (13) e (23) b (33) d
el Museo de la Nación; 24 visitaron el Museo (4) d (14) b (24) e (34) b
de Arte; 18 visitaron el Museo de Oro. ¿Qué ♦ (5)b (15)c (25) b (35) e
fracción del total no visitó el Museo de Arte? ♦ (6) d (16) d (26) a
(7)a (17)b (27) d
3 7 1
♦ (8) e (18) a (28) b
a) - b)- c) -
(9) e (19) e
24 (29) d
8 3 ♦ (10) b (20) b (30) a
4
d) - ♦ BLOQUE 111
9
♦ (1)b (5)d (9) d (13) e
(14) Cada vez que apuesta un jugador pierde la ♦ (2) d (6) d (10) e (14) a
mitad de su dinero. Si después de apostar cuatro ♦ (3) b (7) a (11) e (15) e
veces seguidas, el jugador se queda con SI. 6, (4) e (8) e (12) d (16) d
¿cuánto tenía inicialmente? ♦
1 MATEMATICA
1
!] LAS MATEl\1ÁTICAS DE TODOS LOS DÍAS

l. Skanito cava toda una zanja en 7 días, si se trata de un joven muy trabajador y hemos
notado que su esfuerzo diario es parejo ¡,Qué fracción de zanja cavó en 2 días'!

Rpta. '•---__.
2. El papá de Carlos recibe un sueldo extra como gratificación en Julio y un sueldo extra
como aguinaldo en diciembre ¿Qué fracción de sus ingresos anuales representa lo que
recibe en diciembre'!

Rpta. •1_____.
En un distrito de la capital se estableció luego de una encuesta que el número de
habitantes entre 10 y 100 años de edad es de 480 000 . En el gráfico siguiente
mostramos la distribución respectiva. Según los datos mostrados ¿Cuántos habitantes
entre 16 y 35 años de edad hay en dicho distrito?

L:::::::::I Habitantes entre I O y 15 años


lllilIIllIIIII Habitantes entre 16 y 21 años
[=:J Habitantes entre 22 y 35 años
~ Habitantes entre 36 y 100 años

1___,. .
Rpta . •

4. Los alumnos del Icr año de un colegio son en total 300. De ellos las 2/5 partes obtienen
en un cxámcn bimestral notas de 12 a 14; la tercera parte obtiene notas de 15 a 16; los
2/15 obtienen de 17 a 18 de noLas, teniendo el resto de alumnos, menos ó igual a 11 de
nota. ¿Cuantos alumnos tienen notas de 15 a 18?.

1____ .
Rpta. •
I, 1 ALF01'SO ROJAS PUÉM ~

e 2. ADICIÓN Y SUSTRACCIÓN EN ([¡ )

...a;;;::._

ueso.o 1 1Bp12arra
- -

Podemos notar al distribuir el pintado de la pinma


que 1/10 de Luchín má<, los 7/10 de SKANITO equiva-
len a los 8/1 O de pizarra pintada.
O ¡CUIDADO!

Simplificando: ~ equivale a 1 Toda una przarra se puec,e d1v1c


lO 5 en
Es decir: dos mea,os ➔ 212
o tres tercios ➔ 3 '3
Si la parte pintada es 4/5 y ack-más toda la pi1.arra o cuatro cuartos ➔ 4/4
se puede dividir en 5/5 O O entonces la parte que falta o cinco qwntos ---,. 5 5
pintar es 1/5.
etc
2.1 ADICIO N D E FRACCJO;\ES DE IG UAi
DEI\Oi\11NADOR.

Al efectuar la SUMA o ADICIÓN de dos o


más fracciones de igual denominador. se su-
man los numeradores y se escribe el mismo
denominador.
IP8ik!ifi 1 MATEMATICA
1
Ejemplos:


(/) -1 + - +-
7 =1- 7 1
-+-=-
7 8
10 10 10 10 10 10

(2) Efectuar:
¡CUIDADO!
Para sumar MIXTOS con denomi-
s2+2l... nadores iguales:
11 11 Sumamos las PARTES ENTERAS
Sumamos las PARTES FRACC/0
NARIAS
Es decir: ¡CUIJJ 1/JO!
+ Si dos Jracuoncs.
~


a c
5 i_ 2 _l__ - 7 1-±._1_
~ 11 - 11 h y d
so11equivalu1U'.f .\e
cumple que:
ADICIÓN DE FRACCIONES DE DIFERE N- axd=hxc
TE DENOI\IINADOI~ ·-, ~......,=-------
·-·- _._.-.-.-~-- .· -.-.-·-.- -·-· . ..

Para efectuar la SUMA o ADICIÓN de fracciones de dife-


rente denominador, buscamos transfmmar las fracciones a otras
equivalentes. de tal forma que todas tengan ahora el mismo
denominador, procediendo luego como en 2.1.
Para esto podemos considerar los sigmcntcs casos:

1.- DENOMINADORES MÚLTIPLOS DE


OTROS. O llECl:Dl/J,\ QUE:

Veamos la siguiente suma: Una forma de hallar u11a frac


non EQU /\ ALF.N /E a, ,,11,
F'f' 5 3
.ectuar: 7 +¡;r ·.1 ef por ;\AIPL/FIC,\CJON, es
clccir.
14 es múltiplo de 7
'j
• Podemos transformar SÓLO la primera
fracción a otra equivalente~'> O de modo
que ambos denominadores sean iguales.
5 X :2 + ..l._ Ó ..!Q_+ _l_
7 X 2 14 14 14

• Por fin, aplicamos ADICIÓN DE FRAC- .,..,. --._.__ __


CIONES DE IGUAL DENOMINADOR

.!Q + .l..=
14 14
lnl
~
#-t:• 1 ALFONSO ROJAS PUÉMAPE 1
2. MÉTODO DEL MÍNIMO COMÚN MÚLTIPLO (mcm)

Veamos el siguiente ejemplo:

I~fcctuar · 1-+l.+_]_
' ·3 8 30

Para sumar fracciones por este método, procedemos así:

• Hallamos el mcm de los denomina-


3= 33 }
dores y lo escribimos como DENO- 8=2 mcm(3;8;30)=
MINADOR del resultado: 0 O
30 = 2 X 3 X 5 z3 )( 3 )( 5 = 120
Entonces l3 + l+ - 7-= .!'=====-
8 30 120

• Para calcular el numerador del resultado, efectua-


mos la suma de los números logrados así:
O RHCUE/l/J,\ Q UE:
- Dividimos el mcm en- Una forma de calcular el
tre cada denominador. (120: 3) x 2 = 80 monde dos o más números
(120 : 8) X 3 = 45 crmsiste en D~SCOMPO-
- Y el resultado lo mul- NER cada número e,i .rns
tiplicamos por el res- (120 : 30) X 7 = 28
r ,\CTORr:.'i PRl,\IOS.drm-
pectivo numerador. (. de:
f:
111c m = Producto de facto -
Luego: 2 3 7 80+45 +28
3 + 8 +-3-0 = ....:.;_;__12--=--0-- res COMUNliS y
N O COM UNE S
con sus Al,\ }' O-
Efectuando la suma
2 3 7 RES expo11c111es.
en el numerador: -+-+-=
3 8 30

3. REGLA DE PRODUCTOS CRUZ_t\DOS

Esta es una regla práctica. recomendable para


sumar dos o tres fracciones de términos pequeños
Para dos fracciones.-
,,.
-~~. . g:_:, ad+ he
b"{l bxd
•'~',
Ejemplos:
22
(1) Efectuar: - 35
1 MATEMATICA 1
Es posible efectuar "mentalmente" la suma de
productos cruzados que se escribe en el nume-
rador, de modo que podamos dar el resultado ~
en forma más directa : __/
1 1
(2) Efectuar:
2 +3 0 ¡ C UID,W O!
Decir "qué parte de la
Por la REGLA DE PRODUCTOS CRU'ZADOS pitia" es lo mismo que
dcrir "qué fracción de
1 1 5
2 + 3=6 la pitia"

Para tres fracciones.-

Ejemplos:
1+_!+.!= ]x3x4+Jx2x4+Jx3x2
(1) Efectuar:
2 3 4 2x3x4

operando:
00 llE COIW.\.Yl>O:

1 1 1 26 f'ROf'I Efh\V IJ/SIR!Dl '-


-+-+-=- TI\ "¡\ ele los numero.~ c111c -
2 3 4 24
ros:
Simplificando: 1 1 1 13
-+-+-=- a(b+r) = a/J + ar
2 3 4 12 a la inversa
ab + ac = a(b+c}
(2) Maritza recibe las dos terceras partes de una dmule a recibe el nom/Jre de
piña. Carlos recibe la quinta parte y Guise/la lf) Ft\CfORCDMÚN
de la misma.¿Qué parte de la piiia reciben los
tres juntos? C:- O
Solución:

1 Al aplicar la re;;la de productos cruzado.\


1 podcmm, SI/IIPLIFICAR la f ran:ián rc.,ul-
lllnte. a¡,lic11ndo en el nwnl'mdor el pro1 eso
im-cr.\o de la PROPIEDAD DISTRIBU rt-
\ A C::· 00
, fa decir. si lw:~o ele aplicar la RCGLA DE
PRODUCTOS CRUZADOS no.\ queda .
ah+ ac e.\1(1 e., i,;ual a a(b+c)
ad ad
h +r
Súnplifirnndo:
d
.,..
1 ALFONSO ROJAS PUÉl\.fAPE 1 IWMié&•
PASO 1.- Veamos la fracción de piña que recibe cada uno :
Maritza 2/3 de piña
Carlos 1/5 de piiia
Guise/la 1/9 de piña
PASO 2 .- Los tres junlús recibirán:

PASO 3.- Por PRODUCTOS CRU-


ZADOS:
PASO 4.- Por la inversa de la propie-
dad DISTRIBUTIVA:
2 1 1 44
PASO 5.- Simplificando : -+-+-=-
3 5 9 45
RESPUESTA.-
Los tres juntos reciben 44/45 de la piña

'.!.3. PROPIED ,\DES DE LA ADICIC)N DE J\ÚI\IEROS RACIONALES

• PROPIEDAD DE CLAUSURA: "Si dos o más números son RACIONALES la


suma de ellos también es RACIONAL"
Ejemplo :
Si
1
3
E(D y
1
5
E(D
Entonces: (~+~)E@ ó -E <D
8
15

• PROPIEDAD CONMUTATIVA: "El orden de los sumandos RACIONALES no


altera la suma"
!!..+.!::..=.!::..+~
b d d b
Ejemplo: '.! 1 1 2 B
-+-=-+-=-
3 5 5 3 15

• PROPIEDAD ASOCIATIVA: "La manera como se agrupan los sumandos RA-


CIONALES no altera la suma"

(~+.E.)+~=.!!.+(.!::..+~)
bd fb clf
Ejemplo:
( .!...+
2
.!...)
3
+ .!...=.!...
5 2
+(.!...+
3
.!...)
5
5 1 1 8
6+s=2+IT
31 31
30 = 30
I~___M_AT_EM_ÁT_I_CA_ _ _ I
• ELEI\IENTO NEUTRO: Es el número CERO
"Si sumamos un número RACIONAL con el CERO, a O a
el resultado es el mismo número RACIONAL"
-+-=-
b c b

• ELEI\IENTO OPL'ESTO:
"Es el número que sumado con cualquier número RACIONAL da como resultado
CERO"
Si f es un número RACIONAL, - ba es su OPUESTO, tal que f + -ba =O
2A SUSTRACCIÚN DE NÚI\IEROS RACIONA LES
Efectuar la SUSTRACCIÓN de dos números racio-
nales equivale a efectuar la ADICIÓN ele uno ele ~= O RH'UERIJ 1 Ql F:

(Pi
ellos con el OPUESTO del otro. Ó O ,:. · En e 11alq11ic r lW.trt
sedisti11g11c11 los sigm nt,
Ejemplo: elemellfos:

Efectuar f- ; ~j{t 8 5
T
3
, _'l ,, T T
• Esta SUSTRACCIÓN tam-
bién puede .~er escrita así:
3+
..
s ....
.. r6'-.. O0
••
"-111111L11do
1
1
+
Di , '
• Ahora aplicamos la REGLA 2 -2 10+(-6)
DE PRODUCTOS CRUZ4- 3+-5-=--15--
DOS.
• Suprimiendo paréntesis: 1+ -2= I0-6 Qoo
3 5 15
2 2 4
---=- 0 0 ¡CUIDADO !
3 5 15
l- 1acci611¡,m11i1·a
OBSERVACIONES:
3 +~
(1) En la práctica, podemos efectuar la RESTA o t--=-=-
h +h -h
SUSTRACCIÓN, sin convertirla en ADICIÓN. ~ l- 1acc10n 11eRati1•a
En el ejemplo anterior: .>.:: ,.- --
el -e:\
-=-
+J
{'V'¿._ b +h -h
! ~ 1 ~~~
1_1=10-6=..i_
3 5 15 15 1Wll
,,
Si M' tiene

-
a
Sil 0/Jlll'.\lu t
(2) Para SUPRil\1IR signos de colección como pa- h
a -,d
réntesis, corchetes o llaves, procedemos como lo - n
h b
hicimos en NÚMEROS ENTEROS (Z) .
Así:
• Si el paréntesis está precedido de signo POSI-
TIVO. suprimimos el paréntesis y los signos
interiores NO CAl\1nlAN.
J ALFONSO ROJAS PUÉMAPE 1

Ejemplo ·
Suprimir signos de colección en: (1._ .!...) +[.!. - .!... +1]
3 2 2 5 7
Delante de Los signos de colección, sólo hay sig-
nos POSl77VOS: C::- O
5 l 1 1 2
---+---+-
3 2 2 5 7
• Si el paréntesis está precedido de signo NE-
GATIVO suprimimos el paréntesis y los sig-
!' 0 RECUERIJA QUE:

nos interiores CAMBIAN.


f Si un número no lleva sig-
} no positivo se supone PO-
Ejemplo: ··= SITJVO
tr 5 = +5
Suprimir signos de colección en : (¡ 2/3 = +2/3

(;-1)-[¾- !- ;] ~
!t.,,. ..... --~-~--~- ~-- --·-········ ~
Delante del corchete tenemos SIGNO NEGA17-
VO, entonces:

l_.!..._ 5 +.!...+1.
7 3 9 4 7
2.5 OPERACIONES COMBINADAS DE ADICIÓN Y
SUSTRACCIÓN EN (D •
Veámoslo en ejemplos:

2 4 7
(1) Efectuar: - +---
3 5 9
t,{ IMPORTANTE
Solución: ~::
? A l sumar dos números
• Por La REGLA DE PRODUCTOS CRULADOS: ( R/\ CJONALES OPUES-
¡i:. TOS el resultado es nulo 6
1 + ~ _ _]_ = 90 + 108 - 105 = 93 r:: CERO .
3 5 9 3X 5X 9 135
• Simplificando
t f +(-bª) =0
l+i._.2.= 131 1
i.1
),:-
::--· ..!.. _~ =0
3 5 9 45 b b
OTRA FORMA.- ·t¼ '};:·.c::::·::. :'.::::·;:. :_:;::::-;:::::;;:::.i:;: : ••:··· ;;<·,:;·,::~:.··::Y
• Por el mcm de denominadores
mcm (3;5;9) = 45
• Dividimos el mcm entre el denormnador de cada l. + i__ .2.= 30 + 36 - 35
fracción y el resultado se multiplica por el re~pecti- ~ 3 5 9 45
vo numerador: _____,/
MATEMÁTICA
1
Efectuamos las sumas y restas del numerador : 2 4 7
-+---= llli
3 5 9 ~
(2) Efectuar:

Solució11:

• Suprimiendo puréntesis:
• Anulamos las fracciones
opuestas:
• Por la REGLA DE PRO- E= 2x7+1x5
DUCTOS CRUZADOS: 5X 7

Ayúdame a resolver un ejercicio con MIXTOS

(3) Efectuar: P= ( 3 ~ +5 ~ -1 )-[ 1¡ -7 ! -1] /


f --
0 ¡CUIDA DO!
:i~Para sumar o res-
Solució11 tar MIXTOS, ope-
==º ramos por SEPA -
Suprimiendo paréntesis: j:; RADO las PARTES
ENTERAS y las
• Si convertimos 3.!.. a f PARTES FRAC-
4
fracción ,tcndre mos: ,:- CIONARIAS y lue-
f
Anulamos los opuestos :

• Efectuamos la suma de
5+7=12 ~
MIXTOS indicada: O O 1 1 5 - - P=t22.
- + - = - ---- 6
2 3 6

(-') Efectuar:

Solució11
• Eliminamos primero el cor-
chete interior .-
F=[~ + ~ -¾]-[!- ~ + ~]
• Eliminamos todos los cor- F = _!_ + -2. - l__ -1+ _!_ - -2.
chetes: 3 6 8 4 2 6
• Anulamos los opuestos: F= _!__l_l_
3 8 4
+1-
2
• Operamos empleando el
mcm:
ALFONSO ROJAS PUÉMAPE

;[ SKANIPRÁCTICA @)
l. Efectuar las siguientes operaciones:

a) t+f= D d) 1j+2}= D g) 5~-3¾= D

11. Empleando la REGLA DE PRODUCTOS CRUZADOS efectuar las siguientes ADI-


CIONES: ~

(+ 1/2 2{7 3/4 1/11 -1/4 -3/7

1/3

1/5
2/9

5/6

Haciendo uso del MÍNIMO COMúN MúLTIPLO (mcm) efectuar y completar:

a) 1~ + ~ +i = □ g+ □ = □ e) i- ~+i na+□ =[]


b)i+1~+-½ □ g+□ =□ t) ..1_ +.1 _..! = D+D-□ =□
10 4 2 LJ
c) ti +i +-¼ □ et □ =□
1 2 2
d) 7+--"j~ 1 D+D+D+O □ h) 7 + 13 _ 1 _D+D- □ - □
+6 □ = T81í TITT nu- - CJ -
IV. Empleando la REGLA DE PRODUCTOS CRUZADOS efectuar las siguientes SUS-
lRACCIONES.

r- 1/2 1/4 5/6 7/10 5/13


5/2

1 2/3

3/5

3/11
lil~1~l~¡~I 1 MATEl\lATICA
1 ·t
V. Completar con los signos " > •• ó " < •• según corresponda:

a) (1 + D□ (-¾+ D () (1+ 1--¾) □ (1-¾)


b) ½4 □ 524 g) ( i + ~ +-j ) □ (-~ + ~)

c)
(5i +1)□ 1 1 h) (1i +2 ~) □ (3i)
d) H+ ;-4 )□ (1--½) i) (11-&%) □ (21+1-½)
e) ( 3¾ + 2 D□ (
2-f) j) (3i +2 !) □ (3-½-2¾)
VI. Efectuar las siguientes operaciones combinadas de ADICIÓN y SUSTRACCCIÓN. cscri-
bienclo el resultado en el casillero correspondiente.
1 2 3 1 1 1 1
□ 32 + 23- 15- 28

1. 5 +1 - 8 15.

2 1 1 1 2 1 1
□ □
2. IT +5 - 2 16. 47 + 33 + 22 - 17
3 2 1 3 3 1

7


3. 12+4+3 17. 5-+2--1--2-
4 8 4 2
1 1 1
□ □
4. 1 18.
2+3+4 +s 3+-(½+½)
7 13 1 1 )-(-!-4 +l)
□ □
5.
9 +Tir +12 - 8 19 . ( l+2
5 3 7

11 3 1 3
□ □
6. (-!-+2-l)-(-!-+
12+4 -2 - 8 20. 6 7 5 2 l) 5

□ □
7. 13 11 3 1 2 1. 1 2) - ( 152 +23)
( 351 +26-3
Tif +15- 4 - 1f
1 5 51)(1
+ 25- 31)
□ □
7 7 1 (3
8. 12+9-2-3 22 · 75 +7-

1 5 1 3
9. Tif+ 12 - 4 - 8
□ 23. 1
(½+ +½)-( ~ +½-½)-(¼-½)

_ l7 ) - ( l6 + _141 )

+ 1~

10. 32 -71 -41 -61 24 . ( i.
7 3 ) - ( 3l
9

3 2 5 2
□ □
1 l. 25. (1++)-(2½-½)+(~-1+)
4 +9 - 16 +3
-, 4 1 3 1
L...5+7+5 - 2
5 1 1 1
□ 26. (2l+~)-(i.+_!2_)-(-l
7 9 3 7 18 _l)
3

□ □
13. -¡-17-12 -4 27. [1 1 1][11
32+25-13 - 5 - 61]

□ □
14. 3 5 2 1
4+6 - 3 - 2 28. [½+(~-2)]-[(f-¾)-2]
1 ALFONSO ROJAS PUÉMAPE 1 i)zj@mfi

VII. Escribir la propiedad correspondiente a cada ADICIÓN mostrada:

ADICIÓN PROPIEDAD ADICIÓN PROPIEDAD

3.!_ + .!.= .!_ + 3.!_


2 2
7 8 8 7 rr+-11= 0
l l -O l l
0
2 + =2 o +s =s
3 o 3 12 o 12
IT + rr= IT rr+rr=rr
({ +-D+ i =i +(~ +-D -2 o
-r +TI =-y-
-2

~ 3 2) =2x""22+2x"5
2:-22+""3 3 2 - 3 +3
-::::, + -::::, = o
5 -5 -2 l l 2
7 +7 = o 9+3=3-9
o l l 5 19
,+9=9 7 + 2= /
- 5 5
1+1= 0 { +(2+~)=({+2)+{
5 5 -2 -2
-::::, +,=o -+-=O
+3 - 3

VIII. Contesta las siguientes preguntas:

! l. ¿Es 3/-7 el opuesto de -3n? ¿Por· qué?


l 2. ¿En qué casos un MIXTO equivale a una fracción menor que la unidad?
1 3. ¿Es posible efectuar la suma de un MIXTO y una fracción? ¿Cuál es el procedimiento
a seguir?

.l 4.
¿Tiene sentido lógico, restar una fracción mayor que la unidad de una fracción menor
que la unidad? ¿Por qué? Proponer un ejemplo
5. ¿Por qué crees que al CERO se le llama ELEMENTO NEUlRO de la ADICIÓN?
6. ¿Es -5/-4 el opuesto de +5/+4? ¿Por qué?
Un entero sumado con un racional menor que la unidad, da como resultado otro racional.
¿Si o no? y ¿Por qué? Escribe 3 ejemplos
8. Un entero sumado con cualquier racional, da como resultado otro racional.¿Si o no? y
¿Por qué? Escribe 3 ejemplos
9. Un número natural sumado con un entero, da como resultado un racional.¿Si o no? y
¿Por qué? Escribe 3 ejemplos
10. Un natural sumado con un racional da como resultado otro racional.¿Si o no? y ¿Por
qué? Escribe 3 ejemplos.

..•..·.·s·=·····: •.··•.•
1 MATEMÁTICA
1
Aquí mis opcrncioncs de SKANIPRÁCTICA @
,~:· 1 AlFOl\SO ROJAS Pl'ÉMAPE I IWJMMi
( 3. MULTIPUCACIÓN Y POTENCIACIÓN EN (fJ)
Luchm. los v1dnt, que 1-
1'lcamo. representan los

, ~:;;::~ rn"""' ... •I t ¡No Skarnlo1, mas bien los v1dnos pues-
tos cubren los 3/8 de toda la ventana.

, / ,,r ,r:
-
r. /-
/ f' J,
/
~

Tal parece que en la primera situación SKANITO y


Luchín están en desacuerdo, pero si analizamos con cuida-
do, veremos que ambas afirmacmnes son equivalentes.

Es decir: 0 RECUEIW,1 QUE :


Luchín asegura que los vidrios colocados representan [ n la expre1·ion
los 3/8 de toda la ventana, lo cual es cierto porque toda !!..de e
la ventana está dividida en 8 partes de las cuales se han b
cubierto 3. La palabra "de" e1· como
·'·I¡ u,w "orden" que indica
Además: que len.cmos que multipli-
car a.1í:
SKANITO asegura que los vidrios colocados repre-
a a
sentan los 3/4 de la mitad de toda la ventana, lo cual -¡; de e=-¡;>< e
también es cierto, porque se nota que la mitad de la """"".,,...,,,,,.....,....,- ✓
ventana está dividida en 4 partes, de las cuales se ha
cubierto 3 de ellas.

Esto también puede ser representado


Así:
l del ó lxL=l
4 2 4 2 8
1 MATEMATICA
1
¿Cómo hemos efect uado la MULTIPLICACIÓN de
FRACCIONES?
El numerador fi nal es el resultado de multiplicar los numeradores; el
denominador final es el resultado de multiplicar los denominadores
Es decir:

Veamos algunos ejemplos:


(1) Efectuar:
5 7 1 5><7><1 ◊
[j] l O ¡IMPO/ff,\ NTE!
6><2><3=6><2><3 6
!i Si reemplazáramos los nú-

t ;;?:;/'~'·":~:•m
(2) Efectuar
4 20 1 4 >< 20 >< l
><->< - = - - - -
5 3 8 5><3><8
'"ci "7"°'( - d
Descomponiendo en
factores primos: - -

Simplificando:

OTRA F ORMA:
Casos como este, pueden ser simplificados de
modo mús directo en la misma MULTIPLICA -
CIÓN INDICADA, de fracción a fracción. ¡ CUIDADO!
Así: ¿A qué es igual
J J J
- de - de - de 300 ?
3 2 5
ú, palabra "de" es una
(3) Efectuar: orden que irzdica que
18 5 tenemos que multiplicar.
><-
30 15 / / I
- X - X - X 300 = JO
3 l I 2 5

~5 "l3 = ~
Simplifiquemos primero )( -
l
cada fracción : 3

_;?( l
Simplificando de fracción -5 )( ;5"
a .fracción. e::> O
1 ALFONSO ROJAS PUEMAPE 1
3.1 PROPIEDADES DE LA l\1LLTIPLICACIÓN DE I\Úl\1EROS
RACIONALES

• PROPIEDAD DE CLAUSURA
"Si multiplicamos dos números racionales, el resultado que obten-
dremos es otro número racional"
Es decir:
Si !!.. E <D. y .E. E <D. entonces: .!!. x ~ e <D.
b d b d
Ejemplo:
Si y le©. entonces: 1x 5 =2-e<D.
8 7 8 28

• PROPIEDAD CONMUTATIVA
"El orden de los factores, no altera el producto".
Es decir:
.!!.x ~= ~>< .!!.
b d d b

Ejemplo: 2 2

~><;=~><~ ◊ 6
-=-
35
6
35
5 5

• PROPIEDAD A.SOCIA TIV A


"La manera como agrupamos los factores, no allcra el producto"
Es decir:

Ejemplo:


lO 4 2
><- = - > < -
20
21 3 3 21

• ELEMENTO NEUTRO.-
Multiplicando cualquier número racional por 1, se obtiene el
mismo número racional.
Luego 1 ó 1/1 es el ELEMENTO NEUTRO de la multiplicación
de racionales.
A~ AxL=A
b 1 b
MATEMÁTICA

• INVERSO DE UNA FRACCIÓN


Si un número racional es diferente de CERO o NO NULO.existe
otro número racional que multiplicado por el primero.da como
resultado 1.
Es decir si: .!!.. es NO NULO
h
entonces : .!2. es su inverso
a
porque : -ª- x
h
º-=
a
1
Ejemplos:
(]) El inverso de 7/8 es sn
(2) El inverso de 8 es l/8
(3) El inverso de -1 es l/- 7
(4) El inverso de 0/3 NO ESTÁ DEFINIDO

• PROPIEDAD DISTRIBUTIVA DE l\1ULTIPLICACIÚN RES-


PECTO A LA ADICIÚN
Al multiplicar un número racional por una ADI(::IÓN indicada de
otros, obtendremos como resultado la ADICION de PRODUC-
TOS P~RCIALES del primer racional por cada elemento de la
ADICION.
Así:

Ejemplo:

Comprobemos:

◊ 11
15
= 11
15

OHSERVACH>N.-
Para transformar una suma en una multiplicación de números racionales podemos
empicar la PROPIEDAD DISTRIBUTIVA en ''sentido contrario", es decir:

Ejemplo:
donde 113 recibe el nombre de
FACTOR COMÚN.
* 1
. _.;:
ALFONSO ROJAS PUÉMAPE

SKA '.\I IPR1\CTICA


1
®
#1@@á:fj

l. Completa el siguiente cuadro simplificando el resultado de la operación indicada.


(X 2/5 -1/3 2[7 -3/4 -1/2 5/-4 2(3
5(3
2/9
4[7

-12/18
11/-15

II. Completar los casilleros con los números que permil.an que la igualdad sea correcta.

]) l.xlxl = D 5) 2_ x.:l X □ =j_ 9) ..1... x.1 xll. = D


3 5 1 □ 8 4 0 6 1s 5 6 D
2) .:.1 x-2 x.:Z. = D 6) Ax± x D =R JO) _± x 36 x..:.11. x..:l. = D
4 2 3 D 5 9 0 3 90 15 s 12 D
3) lix..:f!.x.:Z.= D 7) D x.± xl _ __:1__ ] 1) H. x..:ll.. xl. x.:.1 x--2... = D
30 9 4 D 0 15 3 - 135 16 9 6 1 10 D
4) l.x D x2=..1_ 8) .:.1 X □ x.:l =-1 12) .1 xi x-± x.:l xl = D
5 0 8 60 2 0 4 24 14 15 30 2 3 D
III. Al lado de cada mull.iplicación escribe la propiedad corrcspondicnl.e.

MULTIPLICACIÓN PROPIEDAD MULTIPLICACIÓN PROPIEDAD

(½ x¡)xt =½ x(¡ xt) ½(++%)=½X+ +½ X%


l. x.:1 =.:1 xl. .1(..1..._J_)
=]_ x..1... _ _l xl
7 5 5 7 5119 511 59
..:ixl=..:i ~2.+l.)=3 x-2+3 x_l_
8 l 8 4 5 4 5
ixl.L = J
11 -2
¼(
13
1 +1-) =..1.__X 1 +_L xl_
2 13 13 2

IV. Contesta las siguientes prcgunt.as:


1) ¿Cuál es la característica de dos números 4) Escribe 5 multiplicaciones de 4 fraccione~
racionales cuyo producto es 1? cada una cuyos resultados sean iguales a 1
2) ¿Por qué una fracción nula no tiene inver- 5) Describe las características de dos fraccio-
so? nes cuyo producto sea -1
3) Multiplicamos 27 fracciones negativas: 6) Si se tiene una multiplicación de 73 fraccio-
¿Qué signo + ó - tiene el resultado? nes siendo una de ellas nula : ¿Cuál es su
producto?
- .--- - ----:-.-------""'
•• :_,..,
~---:--
·; ,:=:-•--:---
-- __ ~.,.......
MATEMATICA

Aquí mis operaciones de SKANIPRÁCTICA G)


ALFONSO ROJAS PUEMAPE IJ©MUI

3.2 POTENCIA DE UNA FRACCH)N.


Es el RESULTADO de multiplicar "n" veces una misma
fracción.
Así:
xb xb x. x1!..b = Potencia n-ésima
.!. 1!.. 1!..
b

n veces ~

~
= p 1

donde: n es exponente NATURAL e:;> O 0 ¡ l.lfl'OR T,\ ,\ '1R!


-ª- es la base RACIONAL o fracción Cn una l'O/ENC!t1 DE
b
P es la potencia o resultado de la operación BASE R1\CIONJ\L no nula.
POTENCIACIÓN. si n = O el resuluulo es I .
u
Ejemplo: (-¡;-) = l a.<O

3 significa que la base racional 2/5


(¾) debe ser multiplicada por si misma
tres veces.

2 2
Es decir:
( 25)3 =sxsxs
2 ¡Tres veces la base racional 2/5!

2 X2X 2 8
5 X 5 X 5= 7
= 125
Puedes observar que el exponente 3, que inicial-
mente afectaba a la fracción 2/5 termina afectando
ahora al numerador 2 y al denominador 3.
Entonces, podemos afirmar de modo general que:
n
a
n
b

3.3 SIGNOS DE UNA POTENCIA DE BASE RACIONAL


Observa los siguielll,'- cicmrl, ,-,.

(1) + 2)~ + 2 X +2 X +2 = (+ 2) X ( + 2) X ( + 2) = 1 + 81
( 5 - 5 5 5 5X 5 X 5 125
1 MAl'EMAl'JCA
1
(2)

De los ejemplos (1) y (2) podemos concluir que:

Una potencia de base POSITIVA y exponente PAR o


IMPAR,siempreespositiva O O

( - 2) x (- 2) x l - 2) x (- 2)
_,., 0 RECUERDA QUE :
=r·
Potencia es el RESULIADO
=::- de la operación POTENC"IA-
CJÓN. A.\í.:
2
(4)
(!) = ¼
la potencia es I 14

Si observas con cuidado los ejemplos (3) y (4) notarás que:

Una potencia de base NEGATIVA puede ser:


POSITIVA, si el exponente es PAR
NEGATIVA, si el exponente es IMPAR

3.4 MULTIPLICACIOI\ DE POTENCIAS DE BASES RACION ALES


IGUALES.
m n m +n
Ci) X(~) =(i) ¡Escribimos la misma
base y sumamos los
¿Lo comprobamos? .... veamos un ejemplo: exponentes'
Efectuar:

5 veces

Entonces: ¿Qué ocurrió con los


exponentes iniciales?
,-e~ 1 ALFONSO ROJAS PUEMAPE 1
J.5 POTENCIA DE POTENCIA VE HASE RACIONAL

[(tfT =(tf · ~ ~ 1Escnb1mos la misma base


y mult1phcamos los expo
nantes'

Comprobémoslo en el siguiente ejemplo: \~,


Y ~)
v

[m'I=(-D'-m'•m' Tres veces


ñ ¡,Qué ocurrió con los
cxponcmcs iniciales?

¡6 veces la base 2/31

3.6 POTENCIA DE UN/\ l\ll"I TIPI ICACION


1EI exponente n lo afocta-
mos a cada uno de los fac-
tores de la mult1plicac1ón m-
d1cada1

Tres veces

Por la propiedad asociativa de la multiplicación:


1: x-±: x-±: x 3 x2. x2 O IWCO/W \U QUI;:
5 5 5 7 7 7

Tres veces Tres veces


"'º
Si la RASE R1\CJONliL e.,
NULA. ClllOIU"C.C
11
(f) =1

o
(2) (t) =l
o
( -/) = 1

(3)

ºº
1 MATI" MA TIC,\
'

(4)

Solución:

fí"" ' llr/(1\ prc'CII{(' l{lll


Sl.

E11tonr, ., ta111h1, n .,e , wnplc ,¡11c·

1a
h
¡,,. r, ..!el )
1:.., cÍ<'t'lf, de, 110 11ft1111,, p,•dc1111•, afirmar ,¡tu·
Para mulup/icar poi, 11, w. de e \¡11111cnt, ,
1s.:11alc•1 11111//1¡>/1, e111w1 fa~ ho.,c.~ v e nhw ,
el /lli.1/rl I l'\f'rll/l /1l1

• De las 6 potencias que


multiplicamos conforme P,
4 de ellas tienen exponente
27 y 2 de ellas_tienen expo-
nente 42, entonces pode-
mos agrupar los exponen- -"'
tes: __/
• Efectuando las multiplica-
ciones al interior de los _1'
P= [-43 54 37 75] [-95 59]
x
x
x
x
x
x
27

x x
x
42

corchetes: __/
27 42
• Simplificando: P=(-1) x(-1)
O 00
P=(-l}x(+I)

P= - 1 Rpta.

(.',Q ¡/\ll'OI{ /'\ \ 'TI:"!


O ¡ CUDWO !
{lmu1 prt w·m, Ir•
1/, mm alterado el úlde11. ¡ 1,,rq11c•
,., UJmrv fJE LOS F-\( TORU,
fl'O \LTER4 El P!WfJl c_·,o .L ~ h 1 \1 \I
(P, ,de, 111111 ,11 ,r < d
-$:· 1 ALFO'\SO ROJAS PUÉM \PE
1 11f.,@41

1
SKA!\IPR ,\CTIC \ ® I
l. Completa los casilleros vacíos:

(1) (+/J' =8
(2) (-j=§
(3)
(+/f =8
(4)
(-j=§
(5) (-j=§
(6)
(-.'J'=8
(7)
Cf=§
(8) (~)7 (;/ = (~)º
3
0 11 11 11 11
(9)
[(-/f]1 =[-/] (20) (-/) x(-/) x(~:) x(-t) = □
5 □ 17 13 17 17 13

(10) (( -43) 2) =(-/ J 7


(21) (- )
5
x(~2 ) x(-65) x(-/) x(_\) = □

(I 1) [(i-)2 (L)3J2 =D
3
x
3 □
(22) [(-¼/X(-~/ X(-Dºr =(!~ (-~)º
1 !\L\TEMATICA
1
11. Escribe en los casilleros en blanco las potencias indicadas:

ré-it 1
Al Cuadrado
1
Al Cubo A la Cuarta

2 4
-2
3
- 1
2
3
2
-3 - 27
2 -8-
-2
-'i-

III. Contesta a las siguicnLes preguntas:

1.- Según el cuadro anLerior:¿Qué signo tienen las poLencias, cuyos exponenLes de
sus respectivas bases ncgmivas son pares? ¿Por qué?
2.- Según el cuadro amcrior:¿Qué signo tienen las poLencias, cuyos exponentes de sus
respccLivas bases negativas son impares? ¿Por qué?
3.- Según el cuadro amerior:¿Que signo tienen las potencias cuyos exponentes de sus
respectivas bases positivas son pares? ¿Por qué?
4.- Según el cuadro anterior:¿Que signo tienen las potencias cuyos exponentes de sus
respectivas bases positivas son impares? ¿Por qué?
5
3
5.- La expresión (½) ¿Es POTENCIA DE POTENCIA? ¿Por qué?
s
6.- La expresión
3
[( ½/] ¿Es POTENCIA DE POTENCIA? ¡,Por qué?
2
1
7.- (½) = 256 ¿Es corrccLO'! ¿Por qué?
3 2

8.- [(½) =[(½/]2


] ¿EscorrccLO? ¿Porqué?
3
2
9.- (-
2
1) = ~41 ¿Es correcto? ¿Por qué?
3
1
10.- [( - /] = ~1 ¡,Es corrccLo'! ¿Por qué?
2
•ti • AU=ONSO ROJAS PUÉMAPE 1
e4. DIVISIÓN Y JMDICACIÓN EN IZJ)

Cuántos kilos de
rroz hay en el pa-

El paquete
grandecon-
hene 6°/ 7
Kg.dearroz
Skanito ¿Qué debo

Jl hacercon estos numeres


para saber cuántas
bolsas necesito?

Para que SKANITO sepa cuántas veces 4n kg está


contenido en el paquete grande de ~ kg, deberá hacer una
- 6 7
DIVISION de ~ entre 4n kg. o: o 1n·cu1~RJJ,, QUE:
7
~:.i Como en tuda Dl\ '/SIÓN la
7 7 d
pruncrtl Jraci wrz al b t.,
IJl\'IDEN/JO y la \·e¡:umla
Transformemos el MIXTO a FRACCIÓN: frarnmi cid u. el DI\ /SOR
48. ~
7 . 7
-U ;,C0\10 DIVIDIR FRACCIONES?
Dividir una fracción a/b entre otra NO NULA
c/d equivale a multiplicar la primera fracción a/b
por la INVERSA de la segunda c/d.

Es decir: ~:~=~X~ = axd QO


b d b e bxc
1 MATEMÁTICA

Para terminar de resolver el problema de SKANITO L\!ne-


mos que efccLuar la división: 0 ¡ATEl\~CUfN!
48 . ~ o lo que es lo mismo: Una expresión como
.,
7 .7
¡,
48 x L= 48 x 7 = 4~ = ful e
7 4 7 x4 4 LJ d
J."e le conoce 1ambicn
como FRACCIÓN DE
¡Ya tenemos respuesLa!: fÉRMINOS RAC/0
NALES
Los ~ kg del paqueLe grande debemos dislribuirlos en 12
7
bolsas de 4n kg cada uno.

4.2. OTRA FORMA DE PRESENTAR Ul\A DIVISIÓN DE FRACCIOI\ES


Escribir .!!..: .f. es equivalente a escribir:
a b d
h donde a y d se llaman TÉRMINOS EXTREMOS...._ O O
c b y c se llaman TÉRMINOS MEDIOS /
d
Para obtener el mismo resultado, operamos así:
a
-=
b Producto de términos extremos
c Producto de términos medios
d

(/)

ó también:
00 ¡CUIIHJJO!
~
h e
¿-' =-¡;
5 e
(2) Efectunr: 9 m
5
O 00
iT ~=~
r r
Empleando : Producto de extremos "R.
Producto de medios

=~=1111
5

95 9 X 5 9
11

Puedes notar que hemos suprimido 5, pero . ...


1 1
'º ALFONSO ROJAS PUÉMAPE

.... 5 esel 1ÉRMINO EXTREMO en unafraccióny5 esel


TÉRMINO MEDIO en la otrufracción, entonces:
1#1@Mi

Si uno de los términos medios en una fracción es


IGUAL al término extremo en la otra (o uno es múlti-
plo del otro) se simplifican (o se dividen entre su
MCD).
En nuestro ejemplo:
5 1
9 9 ¡CUIDADO!
-=- , como I I es EXTREMO va al numerador,
5 1
¡Si!
11 11 luego: 5/9 =.!.!_ r
5/11 9 t L±..Q=b ¡No!
(3) Efectunr: t )Y
Los elemcnlos comunes en
t: ambos TÉRMINOS se su-
primen siempre y cuando
(4) Efectunr: es1én como FACTORES y
no como SUMANDOS.

Solución:

En fracciones cuyos términos son otras


fracciones, efectuamos primero por sepa-
rado las operaciones en cada uno de los
términos mayores y después dividimos los
resultados.

• Efectunmos la suma del numerador ma-


yor:
• Efectunmos la suma del denominador
mayor:
• Escribimos la fracción mayor con los
resultados anteriores:

.i.3. DIVISIÓN DE POTENCIAS DE IGUAL BASE RACIONAL

1Escrib1mos la misma base, y


restamos los exponentes!
G)
1 MATEMATICA
1
Lo comprobamos en un ejemplo ¿de acuerdo?
Efectuar:

Simplificando:
lo que equivale a escribir:

(2 /3)
( 2 / 3)
5
3
= ( 2 / 3) 5 -3 que finalmente es igual a (213) 2 ó @]
4
9

Si en la misma expresión (D amerior se nos presenta


Q
r- - -J'
una situación donde m = n, tendremos:

Jt Ejemplos:
®

Exponeme ~--- _- rn <sn>º = 1 (3) (-7/2)º= I


Nulo vi/ (2) ( 7527 )º = I (4) (0/5)º = NO ESTÁ DEFINIDO
13

¿Qué ocurriría si en la misma expresión(D, n es cualquier número


natural y m = O? ..... En ese caso tendríamos:

-n
(a/ b) = -- -,,-n Exponente
(a/ b) @) Negativo

Ej<'mplos:

ADEMÁS: Si hacemos algunas transformaciol'les en III, tendremos:

(a/b)·" = _1 = J.LL= b" = (!!.)" ó 1 (,;i_)·" = (!!.)" 1


(a/b)" a"/b" a" a h a j
"EL EXPONENTE NEGATIVO INVIERTE A LA BASE"
Ejemplo:
1 ALFONSO ROJAS PUEMAPE 1
SKANIPRÁCTICA 0
l. Completar el siguiente cuadro efectuando todas las divisiones señaladas:

í: -3/2 -7/3 2/5 -1/2 3/4 -3/2

1/2
-2/3
5/4
-3/40
8/25

11. Dado el siguiente cuadro. llenar los espacios en blanco y hallar la fracción de términos
racionales simplificada:
Se Fracción Se Fracción
DIVISIÓN Suprime QUEDA DIVISIÓN QUEDA
l3implificada Suprime Sim_2lificada
2/7 7 2/6 1/3 12 / 11
6/7 12 /13
8 /9 - 8 /7
9 8/11 8/11
11 / 9 8/5
3/5 7 /17
9/5 9 /17
-18 / 13 - 2 / 23
24 /13 4 / 23
7 /15 2/7
7/5 2TTT
- 6/ 5 - 11 / 2
7YT5 2T7T
111. Escribir la expresión más simple, equivalente a:

½+½ □
(5) -,-= □
4
1 1 1
(10)
5+6+12
1

3

(3)
~o
11

-.~5

(4) 1: =º
-8 □
5
:... --. ·.-:-:_. -- . . ..
1 MATFI\L\TICA
·t·
IV. ¿Cuál es el número que falta en el casillero par_a_q_u_e l~i-g-ua- ld_a_cl_s-ea- c-ie-r-ta; -- 7
73
2)□
25
(1)(5/2)70 (5 / (6) (1/2) (1/2)5
(11) (76 / 113)º= □
(5 / 2) (l /2)□

(2)
(7 / 9)125

(7/9)
117
(7 /9)□ (7) (5/8)□
(S / 8)
13
(5/ 8/
(12) (- s!)
o
= D
3)□
(2/3)76 23
(3)
(2 / 3)19
(2 / (8) (3 / 4)
16
<0>7 (13) ( 26¼f= 1
(3 / 4)
29
(1})
/7)□
o
(4) (9 (9) (7 / 9p=( 7 ¡9/ (14) (0) = No
Definido
(9/7)17
38
(1 / 9t 5 5
(53 / 5) 17
(5) -(53, 5 P (10)(3/11)
(3 / 11)
6 (15) (1 / 4) 2 2 =□
15
11)□
(10{) (3/

V. Dar el número que debemos colocar en lugar de m en cada caso para que la igualdad sea
cierta:
-3 m
( l) ( 5 / 13)
m_
-
1
'[3'7"i'3j" (9) ( 2 / 3) = ( 3 / 2)

-m k

c10> (1 !) =C~)
m 8
(2) (3/ 17)m =---4 (6) ( 2 / 9) =(9 / 2)
( 3 / 17)
3 -m
(3) (7/9)-7 = 1 m (11) (5 / 7) =(7 / 5)
(1 / 9)
o
(4) (1 / 19)-
3
=-m__
3
(8) mº = NO DEFINIDO (12) (}½) =m
(l / 19)

Nº DE PREGUNTA I' 4 5 6 8 9
m
I ' I ' 1 1 1 I ' 1 1 l'°l''l''I
VI. Efectuar:
1 1 1 1 1
(1) 1 / 7 (4) 4 X'"!+ "J (7) "7+4 46
-8- I / 24 1/28+2+'37"

I 2 10 3 19
1[ 1 1]
(2) 5+ j (5) 4 '"!+J" (8) 3'" x 1 + "7 XT 1
"TIT'" 1 / 24 6 J 35 ms
1 1
1
(3) -z-)
-4-
1 2
(6) 5 X J
1

1 / 30
+ 2
1
(9) -
7+2
-x
1
(1
- + - 2+ 3 )
1 1
N+I 3 2
1 AlF01'S0 ROJAS PUÉMAPL 1
-IA RADICACl()'.'J F~ (D
Hemos estudiado, que dada la expresión: ( : Y=P
La operación que permite el cálculo dr P dados a/b y n O ¡,lTENCI Ó\"!
recibe el nombre de POTENCIACIÓN. O O
Dado: 4
Pero ahora: (½) =P
La operación que permite el cálculo de la base a/b por medio de la opcració11
dados P y n,se llama RADICACIÓN. l'01ENC/ACIÓN podemos
ralc u/ar el rcsttlt<ulo o PO-
Es decir: TENClt\ P, que en e~lc
caso e.f: 16 / 81

donde:
P : Se llama ahora RADICANDO y es un número RACIONAL
n : Se llama ÍNDICE y es un número NATIJRAL I ¡
n? 2

ªb : se llama RAÍZ
V: le llamamos OPERADOR RADICAL
Ejemplo: _ /8
Calcular iJ -&
Solución:

Al lwllor RAÍZ DE ÍNDICE 3 e; 00


es/amos trmandn d<' calc:ular un númC'ro
ra,.wnal que clnado al C'xponc:nle 3 vuelva
a rc¡,rodurir ,.¡ radicando 8/125.

Tenemos que:
3

/¾s= 5
2
porque ( 52) 8
= 125 ~, cio 1u-:cu1::1w,t Ql"H:
La RMZ DE ÍJ\'D!Cl 2 n
ci/Je el ttumbrc ele R,\ÍI
Puedes observar que al aplicar el operador
CU,\DRADA .
RADICAL a una fracción. hemos afectado di-
La R,\ÍI, DE ÍNDICE 1 rc-
cho operador a cada término de la fracción, es ribc el nombre de R1\ÍI,
decir: ClÍIJIC,1
1 MATEMÁTICA
1
4.5 SIGNOS DE RADICACION EN (D

Ejemplo: ~=±_]_
(1) . ~ +e
b =d
unpar
◊ Porque: 3
10

(~) =~
10 1000

.n-c Ejemplo:
#= -/

unpar ___
(2)
b - d
Porque: s
(-/) = 2ii
Ejemplo:

+e 2
+2 ( + 2) + 4
(3)
T f+4 =-3-
✓T porque -3- =9

Ejemplo: ~
✓ zs
(4) no tenemos raíz, porque no existe un
número RACIONAL que elevado al ex-
ponente 2, reproduzca el radicando
- 16/25

-1.6 EXPOl\E~TE FRACCIONARIO

.JT 1 ,10
. vz =(~)
-t7 R.\ÍZ DE UNA l\1t:LTIPLICACH):\

~={j_><{i
• ✓ 1~9 ✓1 ✓9

. 'V4
/1 )( -V7
3/32, =✓ 7 )( 32 = 2
4 7
.\1 FO:\SO ROJA' PUEM.\PE

.u,; 1{ \17 DI R \1/

~.9 POTE'.\CI.\ DF 1·, \ R.\ÍZ


M7 ~ 16

1..;,r;;¡¡;m ~../ta/ h) m • '1lrrJ =:f-f • {llm-J


¿Por qué?
=81

~ to OPERACIO'\ES ( 'O\IBli\ .\Dr\S


En expresiones donde intervienen dos o más de las operaciones ya cstudiadas,se siguen
los siguientes criterios para su simplificación:
(1º) Eliminamos todos los signos de colección efectuando las operaciones interiores.
(2º) Procedemos a operar lo que queda, respetando el siguiente orden:
• Se operan las RAÍCES y POTENCIAS. O O
• Luego las MULTlPLICACIONES y DIVISIONES.
• Finalmente las SUMAS y RESTAS.
Ejemplos: O ¡Cl /1) \/JO!

_ _,. ✓. á. . _!_;_x_.C-;_+_1_+_4_..J.) x---7_4


Efectuar: 1
(1) 'i1 I> 'J , un
E radual ªl'''r "
3+121 13 frac, , 1111. \ t·n , 1p1.
Yllf ionc,, r,m/,uw
das, l'runcro L/'-'

Solució11: D, ,arr,,1/,mu•.\ pr1111,·ro la l'Xprt'.\Í11n ba¡o d rnd,- lullmos é,w, como


<al. utu• ali, a¡,arn ('11 .\/UtlllJ \' unu ,r111/up/1ra-
, 1011. , mp, ::arcwUJ.\ ¡1or Ül\ .Hl/lW.\ de· la.\ trC.\ frar- ,
.\l r,tuvu•r,m 01n·-
r<1claf en u,i pu
' wn, \ p,ir,111¡,/1, ·ur l,u ~o- ,, /
y(a/h)
" ..1
T,
l YClllc \1\

- - - --------------'---
Sumando por productos cruzados las tres fracciones del paréntesis:
1 1
2
1 12 + 6 + 8 26 13
-+-+-=---- -=-
3 4 24 24 12
............(!)
• Efcctunmos la suma del denominador: 3 + _1 =E. ············®
12 12
• Reemplazamos ([) y {¿) en E:
E y(13 / 12) x(l3 / 12) x _ 74 X--
_ 74
37 / 12 13 13
• Eliminando el radical: E _ 13 / 12 x_-74
-
- 37 / 12 13
• Simplificando: E _& _ 74 - 74
E=-- 1 E=- 2 1 Rpta.
= 37 X~ 37
ifü,@UI 1 MATEMATICA
1

(2) Efectuar:

E=
.JJJr QR ECUEIWA QUE:

.Jrn=(f;);.
Solución:
No nos preocupemos todavía por la poten- 1
cia de exponente 72. Vamos a efectuar pri- Potencia de Exponente
mero la RAIZ DE RAIZ, para lo cual escri- Fraccionario
bimos un solo radical cuyo indice está dado
por el produrto de los Índires iniciales.

• Aplicando RAÍZ DE RAÍZ:


E~,JN
E~,,,,g E=~li-)
~
1
72
• Escribiendo la expresión obte- ( ) 36 2
nida como potencia de expo-
nente fraccionario: c:>o ~
E= 1
5
= (15 ) = 1 4 1
25 Rpta.

(3) Efectuar:
2

[ (1 / 2-1 / 5) ] X (-=-29)
1/25+1/4 9

Solución:
• Por RAÍZ DE RAÍZ: (1 / 2 - 1 / s>2 ] X (- 29)
1/25+1/4 9
• Operando dentro del cor-
9 /100 X - 29
chete:
29 / 100 9
• Simplificando:
F=J{f x -, =~

1 F = - 1 1 Rpta.
1 .-UFOI\SO ROJAS PllÉ\fAPE 1
SKANIPRACTICA ~
l. Encontrar las fracciones que elevadas al cuadrado reproducen las respectivas fraccio-
nes dadas:
2
(□Y (4)( □f = 64 (□) = 100I
25 49
(1) = 16 (7)

(□Y (□Y
2

(8) (□) = 1IT


1 4 16
(2) = =
9 (5) 81

(□)2 = 25 (6)(□J2 = 49
2

(□)
36 HX)
(3) (9) = 1~
Las fracciones que acabas de escribir en los casilleros ¿Qué vienen a ser de cada una de
las fracciones dadas?:

11. Comple1.ar con fracciones según corresponda:

(IO)H= □
(2) .fli= □ (5) □
.J¼ = (8) !lf-=D (11)/iJ=D
(3)-f-ff;-=□ (6) H= □ (9) .¡¡f-=0 {12)~= □
111. Efcc1.uar:

cl)C!)
¡/2
(8)
9
[16 X 36 49]
1/ 2

(14) JJJo /3)


24

1/2 -1/2 -3/2


(9) [...!.
4
X _!
9
X _!_ ]
16 (]5) (1) X (1)
(3) ( 1~;) )
2/3
(10) [2~ 8\] X
- 1 /2

(16) J~
3/2
(4) (~~) (11)
49
-x-
16
9
64 (17) j J✓(2 /9)84

(12), m 0 8) G)
-3 /2

,----- 1
x ( 1dxi -½) x
1/3

(13)JJJM
1/2
(16/ 7) (25 1)
(19) (81 / 7) X 36X 9
1 MATEMATICA 1

::I IV. Contesta las siguientes preguntas:


(1) ¿Por qué no podemos determinar la raíz de índice par de racionales negativos en el
conjunto Q?
(2) ¿Por qué las raíces de índice impar de racionales negativos son también de signo
l negativo?
¡ (3) Demostrar que ¿Podemos extraer o introducir el
exponente m sin importar el valor del índice n?
•. ·1
r;;:y . )m /n
(4) Demostrar que: Vlb) =(i;-
:1
l V. Efectuar y comprobar:
l
6/5+1/6 ~
;i (l) 7 /3 -3 / 10 " ✓ lfiJ . Rp: [!) {?) Rp: eJ
•. ::1 -1/2
1/2+1/3 1/8+1/8
(2) [.!.x
2
1-x
9
'!:...x2.J
7 5 (8) 2 + I / 6 + 1 / 12 + 1 / 2
1 Rp: (!] 5 / 13 + 11 / 14 Rp: [!)
1+ 2

Rp: ~ (9) Rp: (!]

2 ]- 17 / 3
(4) (1/3x3/7) +1/2 (10)
[
11 / 98 + 20 / 49 Rp: [!) Rp: [!)
(~-s-1)
_1_ X 1 / 2 + 1/ 3 3/7 +7 /8+ 1 /9]
(5) + 1/ 8 1 / 4 + 1 / 6 Rp: [!) (11) [ 5 /7 - 1 / 8 Rp: [!)

6
( ) w' ti 1
s 1
1
Rp: [!]
(12)
3
4 + 2 3+4
--+--x
-1_
4
1

1.+1.
7

6
1

2
[93]-I
-
56
Rp: [!)
4$ 1 ALFONSO ROJAS PUEMAPE 1

1 RAZONEMOS - 2

lnstrr, cciorres:
A continuacion te presentv bloques de e1ercicios p ara a fianzar lo
leido hasta aqu(. Marca la respuesta correcta y comp arala con
laCLAVEDE RESPUESTASqueaparecealfrnal; sin embargo,
terecomiendoc¡ueexpliquesentucuader nolas RAZONESde tu
resultado. No tomes este trabajo como u na pesada carga, sino
como un DESAFI O a tu inteligmcia.

m OQl FI ♦ ó
35 34
- =_!_ de día
35 35 35
(l ) Rubén emplea 2/5 del día para trabajar; la sép- ♦
RESPUESTA
tima parte del día en comer y los 3/7 del día, los ♦
emplea para donnir. ¿Qué fracción del día los Rubén dedica 1135 de día a otras cosas.
dedica a otras cosas? ♦ Rpta. ~
a) 8/35 b) 34/35 c) 1/35 ♦
(2) La Dirección del Colegio ha efectuado compras
d) 7/34 e) 3/35 ♦ dedos tipos de tizas en iguales cantidades. Los
Solución: ♦ profesores usan en clase 5/6 de un tipo y los
3/4 del otro tipo. ¿Qué fracción de la cantidad
C'vmotraba1a2! 5drl día, come 1 /7 del ♦ total quedó sin usar?
mismo d,a y duerme 3 / 7 del mismo

dia . Sumamos 2/5 + 1 /7 + 3/7 para a) 7/24 b) 5/12 c) 1/12
snberque fmcciondel din trabaja, come ♦ d) 5/24 e) 1/24
ydu<>rme.

Solució11:
PASOl: ♦
Escribimos los datos: 2/5 del día trabaja, líl S i D represen ta la cantidad de
del día come, 3í7 del día duerme ♦ tizas de un tipo, entonces 2 ( O J

representa el TOTAL de tizas de
PAS02:
ambos tipos; luego hav que buscar
♦ la fracción a/ b de 2 ( 0 ) de tizas
Entre trabajar, comer
♦ que queden sin usar.
y dormir emplea:
PASO 1:

Los profesores usan 5/6 l
♦ de un tipo, entonces que- -de □
6
dan:

PAS03: PAS02:

Por la REGLA DE PRO- 34 Los profesores usan 3/4 l
de día ♦ del otro tipo, entonces -de □
DUCTOS CRUZADOS: 35 4
quedan:

PASOS 4: PA.S03:
El resto de dí~ para dedi- ♦ Tizas de ambos tipos que
1-34
carlo a otras cosas es: 35 ♦ quedan sin usar:
1 MATEMÁTICA
1
PAS04: ♦ (10) Un libro deMATEMÁTICA tiene 300 páginas
Por PRODUCTOS CRU- .!.Q de 0 y 7 capítulos, de los cuales 3 de ellos represen-
ZADOS: 24 ♦ tan la tercera parte del libro; otros dos repre-
♦ sentan las 2/5 partes del libro.¿ Cuántas páginas
PASOS:
conforman los dos capítulos restantes?
El total de tizas que se ♦
compró es 2 O luego: .!.Qde 0
24 ♦ (11) Se tiene un tonel de vino vacío; inicialmente
ó 5x2 D vaciamos en él 3/7 litros de agua, luego vacia-
♦ mos 7 ½ litros más de agua y finalmente 1/6
24
♦ de litro más de agua. ¿Con cuántos litros más
ó 2-x2 (D) llenaríamos el tonel, si éste tiene una capacidad
24 ♦ total de 18 litros?
ó 2...de2 { □) ♦
24 (12) Para equipar al colegio con una sala de cómputo,
♦ se realizan varias actividades. Una rifa dejó
RESPUESTA
para la 1/7 parte del dinero necesario; la pro-
Lafracci6n del TOTAL que quedó sin usares ♦
yección de una película permitió obtener las
5/24. ♦
5/6 partes del dinero requerido. ¿Qué parte del
Rpta.G) ♦ dinero necesario falta para equipar la sala de
cómputo?
(3) De (1/2 + 1/3) restar 1/4 ♦
♦ (13) Dos velas del mismo tamaño se encienden y
(4) Restar (2/3 - 1/9) de ( 1/2 + 1/5) apagan adistintahora. Si unadeellasseconsume
♦ en sus 5/7 y la otra en sus 3/5, ¿qué fracción de
(5) De una piña, Victoria se come los 5/7. Si el
una vela inicial quedará por consumir?
resto se lo come Blanca, ¿qué fracción de piña ♦
se come Blanca? ♦ (14) Se compran dos latas de leche para el desayuno.
(6) Femando dedica 1/8 del día a jugar en la ♦ Si de la primera se consume la cuarta parte y de
computadora, 1/16 del día lo dedica a comer, y la segunda se consume la mitad; ¿qué parte del
1/4 del día lo dedica a donnir. Si el resto del día ♦ total de la leche comprada queda sin consumir?
lo dedica a cumplir con los trabajos del cole- ♦ (! )
5 Liz y Carolina compran iguales cantidades de
gio, ¿qué fracción del día dedica a esta última
carne para su consumo diario. Liz emplea en el
labor? ♦
almuerzo las 4/5 partes de su carne y Carolina
♦ emplea también en el almuerzo los 6/7 de la
(7) En la Biblioteca del Colegio, las 2/3 partes de
los libros son de MATEMÁTICA, la quinta ♦ cantidad de carne que compró. ¿Qué parte del
parte son de LENGUAJE, el resto de libros ♦ total de carne comprada les quedará a las dos
corresponden a los demás cursos.¿Qué fracción juntas para la cena?
del total de libros pertenece a los demás cursos? ♦
(16) Jorge, Gerar~oy Angélica compran una misma
(8) Un automovilista demora en ir de Lima a ♦ cantidad de hojas cada uno; de las cuales Jorge
Chimbote, 4 ¼ de hora, quedándose a desean- ♦ emplea la mitad de sus hojas, Gerardo emplea
sar 2 ½ de hora en dicha ciudad. Si parte con ♦ 3/4 partes de las que compró y Angélica gasta
dirección a Trujillo y se demora 3 ½ de h en la cuarta parte de las suyas. ¿Qué parte del total
llegar allí, ¿cuántas horas empleó para ir de ♦ de hojas compradas queda sin usar?
Lima a Trujillo?
♦ (17) Octaviotiene20 Y, años,Alonsotiene2 ¼ años
(9) En el problema anterior: ¿A qué hora llegaría a ♦ más que Octavio. Si Héctor tiene 1 ½ años me-
Trujillo el automovilista si sale de Lima a las nos que Alonso, ¿cuál es la suma de las tres
8a.m? ♦
edades?
1 Al.FOIi.SO ROJAS PUÉMAPE 1
(18) Juan recibe como pago semanal SI. 120 ½; si ♦ tiempo demoran juntos en resolver dicha sepa-
una semana recibe como reintegro SI. 79 ½. rata?
¿qué parte de S/. 700 recibe esa semana? ♦
♦ (28) Mi mamá demora 30 minutos en cocinar, pero
( 19) De una cierta cantidad de dinero que tenía me ♦ sólo estuvo I O minutos en la cocina y mi
robaron la séptima parte; si de lo que me que- herrnana terminó de cocinar lo que faltaba.
daba presté la mitad, ¿qué parte del dinero que ♦ ¿Qué parte de la comida cocinó mi hermana?
tenía antes del robo me quedará? ♦
(29) De una bolsa con caramelos, un niño cogió
(20) Se tiene un terreno para sembrío que se recibe ♦ 2/5 del contenido menos 3 caramelos. Si dejó
en herencia, de los que se regalan 2/5; si los ♦ 15 caramelos, ¿cuántos caramelos había ini-
cialmente?
2/3 de lo que queda se siembra de hortalizas y ♦
el resto de frutas, ¿qué parte de la herencia se
(30) ¿Cuánto le falta a4/9 para ser igual a los 2/3 de
sembró de frutas? ♦
5/2?

(21) Un padre de familia gasta 1/3 de su dinero en ♦ (31 ) María comió l /3 de una tona y Lucía 2/5. ¿ Qué
alimentación y l/4 de su dinero en alquiler. parte queda?
¿Qué fracción de su dinero le queda? ♦
♦ (32) María comió 1/3 de una torta y Lucía 2/5 del
(22) Un viajero recorre 2/5 del camino total en auto; resto. ¿Qué parte queda?
luego recorre 1/3 del camino total a caballo. Si ♦
el resto lo hace a pie, ¿qué fracción del camino ♦ (33) Un alumno demora cierto tiempo en resolver
viajó a pie? un problema. Los 2/5 de ese tiempo en leer el
♦ problema; l/3 en operaciones y el resto en
(23) Un vendedor tiene cierta cantidad de libros por ♦ comprobar el resultado. ¡,Qué fracción demora
vender. Primero vende l/3 de los libros que ♦ en la revisión?
tiene y luego 2 libros más. Si aún le quedan 8
(34) Un caño demora 6 horas en llenar un depósito
libros, ¿cuántos libros tenía inicialmente? ♦
y el desagüe demora 8 horas en vaciarlo. ¿En
♦ qué tiempo se llena haciendo funcionar el caño
(24) De un depósito con agua, se extrae 3/5 de su ♦
y desagüe a la vez?
comenido y luego 6 litros más. Si aún quedan
14 litros, ¡_cuántos litros habían inicialmente? ♦ (35) De una fuente con ceviche, Luis se comió l /3 ;
♦ luego Manuel se comió 1/3 del resto. ¿Qué
(25) Andrés puede hacer una obra en 4 días, mientras fracción del total se comió Manuel?
que Beatriz demora 6 días en hacer una obra ♦
similar.¿Qué tiempo demoran juntos en hacer ♦ (36) Maritza demora 3 ½ minutos en comer ¼ de
tal obra? pizza.¡_Cuántodemoraencomer2 ¾ de pizza?

(26) Un grifo llena un depósito en 6 horas, mientras ♦ (37) Son las 3 pm; ¿qué parte de lo transcurrido del
que otro grifo demora 12 horas en llenar ese ♦ día falta transcurrir?
depósito.¿ Qué tiempo demoran juntos en llenar
el depósito? ♦ (38) ¿Cuánto le falta a 3/4 de 2/5 para ser igual a lo
♦ que le falta a l/2 para ser igual a 3/4?
(27) Un alumno demora 3 ½ horas en resolver una
separata de problemas; otro alumno demora ♦ (39) Rosaura le debe a porfi rio $ 700. Si sólo le pago
4 y, horas en resolver dicha separata. ¿Qué ♦ $ 400, ¡,qué parte de lo que le pagó, no le pagó?
1 MATEMÁTICA
1
IlLOQUEII ♦ parte
Pracción ==--
todo
1) Fracción = : 5
(1) Se tienen tres costales A, B y C de dikrcnte ♦ 6
tamaño en los que se depositan 30 kg de harina ♦
36
en cada uno; si la capacidad de C es igual a la Fracción =
suma de los costales A y B y además el costal ♦ 89
A quedó lleno hasta sus 4/5 y el costal B quedó ♦
lleno hasta sus 9/11, ¿qué fracción del costal C ♦ RESPUESTA
está lleno con harina?
Los 30 kg de harina que se depositaron en C,

a) 23/89 b) 18/89 c) 36/89 representan los 36/89 del total de kg que allí
d) 1/89 C) 3/89 ♦ pueden depositarse.

Solución: Rpta.(v

Los 30 ilg que se depositan en C, ♦ (2) Si dos libros de lenguaje del mismo autor
representan una parte o fraccúm cuestan SI. 30, ¿cuánto cuestan 4 de dichos
del total ele hgr¡ue C puede aceptar. ♦ libros?
30 ♦
Fracción busl'Qda = a) S/.40 b) S/.60 e) S/.70
Total de C
♦ d) S/.80 e) S/.160
1 Por dato, el total de C es la suma

de A + B, entonces nos centrare- ♦


Solución:
j mo.~ en í'/ ctílculn de las capa• ♦
1 cidaclc.~ totales de Ay B.
Si dos libros cuestan SI. 30, entonces uno

costará SI. 15; si I cuesta SI. 15, entonces 4
PASO 1: 30 kg equivalen a Los 4/5 de la ♦ costarán 4 X 15 = SI. 60.
capacidad total ele A. entonces:

OTRA FORMA:
3 5 75 ♦
i de A== 30 Í> A== ox =
Si dos libros cuestan SI. 30, entonces4 costarán
5 4 2
♦ el doble, es decir: SI. 60.
PASO 2: 30 kg equivalen a lus 9/11 de la ♦ Podemos observar que si la CANTIDAD DE
capacidad total de B, entonces: + LIBROS A UMENTA, entonCl's la CANTIDAD
DE SOLES también AUMENTA; a esto le

~deB =30
11
l ♦
Llamamos MAGNITUDES DIRECTAMENTE
PROPORCIONALES (MDP).

PASO 3: Lo capacidad total de Ces igual a la ♦ EL cálculo anterior también podemos


suma de A y B, Luego: ♦ representarlo así:

♦ Si 2 libros -----►► SI.JO


C=A+B (
-----►► CJ
♦ Entonces 4 libros

La cantidad buscada (recuadro) será:

4x30
PASO 4: Lo pregunta será entonces: ¿Qué ♦ CJ 2
= S/.60
fracción de 44516 es 30 kg que se depositaron
también en C? ♦ Rpta.@
I 1 1 U FO'ISO ROJ "-5 Pl'É\IAP[ 1
En General, para MDP: (4) Un caño A puede Henar un dep<Ssito vacío en 2

horas y otro caño R puede llenar el mismo
_ .. ---1►'. b
Si a .....
.......... - ♦ depósito vacío en 5 horas. Si el depósito está
Entonces c·----- --- k'=r:1 vacío y se abren ambos caños a la vez: ¿En
♦ qué tiempo se llenará todo el depósito?.
_c><b ♦
Luego:
D - a a) 1
7h
3

(3) Si tres cuadernos tipos A cuestan S/.7 y otros

tres cuadernos tipo B cuestan S/10 ¿Cuál será Solución
el costo total de 5 cuadernos tipo A y 7 ♦
cuadernos del tipo B? "¿Qué parle del depósito es llenado en
♦ J hora por cada uno de Los caños?" .
a) S/.50 b) S/.25 c) S/.55 Es una preg1mla que nos vamos a hacer

d) S/.35 e) S/.45 en este tipo de problemas. Si abrimos
♦ los caños a la vez, podremos ver la
Solución
parle del depó.rito que pueden lle11ar

f'rimcro hallarcmo.f el cmto efe to~ 5 j1mtus en una hora .
ruilClcmm tipo/\ y dc.\puc.~ el co.110 de ♦
lo.\ 7 cuadernos tipo n para luego PASO 1.-Averiguar la parte del depósito que
,wnar ambos co~ro~ . Obferva que ♦
puede llenar el caiio A en J hora:
CU \DFRNO'i) SOLES \Un AIDf' ♦ - El caño A lo llena solo en 2 horas.
PASO 1.- Costode5 cuadernos:
♦ Luego:
TipoA TIPOA SI. ♦ - En 1 hora llena 1/2 de depósito.

3 ----e> 7 ♦ PASO 2.-Averiguar La parte del depósito que


puede Llenar el caño n en 1 hora:
5 -----e> □ ♦
Luego: - El caño n lo Llena soló en 5 horas

C] =~=~ Luego:
3 3 ♦
- En I hora llena 1/5 de depósito.
PASO 2.- Costo de 7 cuadernos ♦
PASO 3.-Averiguar que parle del depósito es
TipoB TIPOB SI.
♦ llenado en I hora si se abren ambos ca,ios a la
3 -=--=C> JO vez:

7 6
-====-=-={>
♦ (Í + ~ ) de depósito
Luego: 6 + C]

PASO J .- Costo total


7 >< 10 +ZQ_
3 3 ♦

y 7 de depósito
♦ Tu
♦ PASO 4.- Si en I hora, ambos caños llenan
7/10 de depósito:¿En cuánla.r horas limarán
♦ 1 depósilo?
RESPUESTA:
El costo 101al pedido será de Sl.35 ♦ Depósilo Tiempo

Rp . © ♦ 1/ JO ~ 1h

♦ ==C> fjfil
f....___MA
_ TE_MA
_ _TI_C_A_ _~li

Luego: ♦ (10) Si 7 reglas cuestan S/.1: ¿Cuánto cuestan 28


reglas?
Etl=1;1 fü'm =
Eill
1
7° horas ♦
a) S/.8 b) S/.7 c) S/.12 d) S/.4 e) S/.6
1o ♦
(11) Si dos computadoras cuestan SI. 2 500. ¿cuán-

§ffi = 1-73 horas to cuestan I Ocomputadoras?

RESJ'UESTA.- a)S/. 10500 b)S/.12500 c)S/. 7500
Todo el depósito se llenará con los dos caños ♦ d)S/. 15000 e)S/.13500
3 ♦
abiertos en 1 horas.
7 (12) Si tres calculadoras tipo A cuestan S/.40 y
Rpta. (v ♦ nueve calculadoras tipo B cuestan S/.120:
(5) Se distribuyen 300 litros de leche entre tres ♦ ¿Cuánto costará comprar 7 calculadoras tipo
depósitos en partes iguales. El primero se lle- A, más 14 calculadoras tipo B?
na hasta sus 3/5 y el segundo hasta los 3/4. ♦
¿Qué fracción del tercer depósito se llenará si a) S/.270 b) S/.300 c) S/.280

su capacidad es la suma de las capacidades de d) S/.180 e) S/.100
los dos primeros?. ♦
(13) TresautosmarcaTOYmecuescanS/. 85000 y
a) l/6 b) 1/2 c) 2/3 d) 1/3 e) 3/4 ♦ tres autos marca LAD me cuestan SI. 47 000.
(6) Se tiene dos tanques de distintas capacidades. ♦ ¿Cu.íl es la cantidad de dinero <1ue debemos
En el primero se depositan 200 litros de agua, tener para comprar 5 autos marca TOY y 17
cubriendo los 3n
del tanque; eñ el segundo se ♦ autos marca LAD?
depositan 700 litros cubriendo los 3/4 del tan- ♦ a)S/. 412000 b)S/. 308000 c)S/.450000
que. Calcular la suma de las capacidades tota-
les de ambos tanques. ♦ d) SI. 408 000 e) S/. 418 000

a) 1200 Ir b) 1400 lt c) 1500 lt ♦ (14) Un caño puede llenar un depósito en l O minu-


d) 1700 lt e) 1800 lt ♦ tos y otro en 20 minutos.¿En cuánto tiempo
pueden llenar el depósito las dos llaves juntas?
(7) Una pisl:ina está llena hasta sus 2/7. Si le ♦
afiadimos 1 080 litros de agua, el nivel de agua 2 . b) 2 mm
. ) 2 mm
.
♦ a ) 6-fmm 5
3 e 1
3
sube hasta los 4/5 de su capacidad total .¡, Cuál
es su capacidad total? ♦
d) sf min e) 4t min

a)2030i b)2000i c)2l00i ♦ (15) De los dos caños que fluyen a un tanque, uno
d) 3 l00i e)4 IOOL solo lo puede llenar en 6 hora~ y el otro solo,
♦ lo puede llenar en 8 horas. Si abrimos los dos
(8) Un depósito de agua está lleno hasta sus 3/11; ♦ caños a la vez estando el tanque vacío:¿En qué
si le añadimos 1O litros, el nivel del agua sube tiempo se llenará dicho tanque?
hasta la tercera parte del depósito. En este caso: ♦
¿Cuántos litros después de los 10 debemos ♦ b) 3; h c) 3lh
7
añadir para que el depósito se llene totalmente? 1 5
♦ 4 h e) 3 h
ajJOOi ~1mt tjlWi
7 7
d) l50t e) 11ot ♦ (16) Un caño puede llenar un depósito en 3 horas y
♦ otro lo puede hacer solo en 4 horas. Si el
(9) Si 3 lapiceros cuestan S/.2:¿Cu,ínto cuestan 9 depósito está vacío y abrimos los dos caños a
lapiceros? ♦ la vez:¿En cuánto tiempo se llenará los 3/4 de
a) S/.8 b) S/.9 c) S/.15 d) S/.7 e) S/.11 ♦ depósito?
1 ALFONSO ROJAS PUÉMAPE 1
a) 64 b) 56 c) 52 d) 58 e) 68
a)1
5
h b) ¡l_ h ♦
7 7
d) tt h


Solución

Vamos a represenlar por Pala canti-


(17) De los tres caños que fluyen a un estanque, dad de pasajeros que salen de Mira -
uno de ellos lo puede llenar solo en 36 horas, ♦ flo res. Como P tiene 1[7 y 1[7 se que-
otro en 30 horas y el otro en 20 horas. Abrien- ♦ da en Lima, entonces de Uma a Co-
do los tres caños a la vez :¿En cuanto tiempo mas siguen viaje 617 de P.
se llenarán las 2/3 partes del estanque? ♦

a)9h b) 8h c) 5h . ♦ PA SO 1.- Si en Lima se baja la séptima parte


y suben 15,entonces hacia Comas irán:
d) 6h e) 6,5h ♦

!i. de P + 15
(18) Un caño llena un estanque en 4 horas, y el 7
desagüe lo vacía en 6 horas. ¿En cuánto tiem- ♦ PASO 2.- Si a Comas llegan 63 pasajeros, 15
po se llenará el estanque, si la llave del desa- de ellos subieron en Lima, entonces de Mira-
güe empezará a funcionar una hora después de ♦ flores a Comas habrán:
abierto el caño? ♦
63-15 = 48 pasajeros
a) IOh b) 12h c) 9h ♦
d) 7h e) 6h l'ASO 3.- Estos 4Fi pasajeros corresponden a
♦ los 6[7 de P que salieron de Miraflores:
(19) Un estanque tiene 2 llaves y un desagüe. La

primera Jo puede llenar en 12 horas y la segun- !i_ de P = 48
7
da en 4 horas. Si estando lleno. se abre el ♦
desagüe y se vacía en 6 horas¡r,En cuánto tiem- Luego: P =48 x7
♦ 6
po se llenará el estanque si estando vacío se
abren los tres conductos a la vez?. ♦ Operando: P = 56 pasajeros

a) 8h b)7h c) 6h ♦ RESPUESTA.-
d)5h e) 4h ♦ De Miraflores salieron 56 pa.mjeros
(20) Estando el desagüe de una piscina cerrado,un ♦ Rpta. (f)
caño demora 6 horas en llenarla; y estando
abierto el desagüe, el caño se demora 9 horas ♦
♦ (2) 16 personas tienen que pagar una deuda de
en llenarla. Si llenamos la piscina y cerramos S/.7 200 en partes iguales.como algunas no
el caño:¿En cuántas horas se vaciará comple- p ueden hacerlo, cada persona de las restantes
tamente? ♦
· debe pagar S/. 270 más de lo que Je correspon-
c) 20h
♦ cle para cancelar la deuda. ¿Cu.íntas personas
a) 18h b)12h
no pagaron?
d)15h e) 16h ♦
a) 8 b) 12 c) 16 d) 6 e) 4

BLOQL'Elli
♦ Solución
(1) Un ómnibus se dirige de Miraílorcs a Lima
♦ ¿Cuánto pagaría cada una si todaf cum-
con una determinada cantidad de pasajeros. plieran? Entonces dividimos la deuda
En Lima se queda la séptima parte pero suben ♦ enlre el total de personas. Como algu-
15 pasajeros porque ahora se dirigen a Co- nas NO P1\ GAN, se rarga la deuda per-

mas, a donde llegan 63 pasajeros. ¿Cuántos .wnal de los demás en Sl.270.
pasajeros salieron de Miraflores? ♦
1 MATEMÁTICA
1
PASO 1.- lnicialmenie cada persona debe ♦ (7) Una moción fue aceptada por una votación de
5 a 3. ¿Qué parte del voto total está en contra
pagar ♦
SI. 7 200 = SI. 450 del movimiento?
16 ♦
a) 1/8 b) 3/8 b) 7/8 c) 5/8 e) 1/7
PASO 2.- /\lgunos NO PAGAN. Pero los que

pagan cancelarán:
♦ (8) Deseamos extraer agua de un tonel de 720
450 + 270 = S/. 720 litros de modo que queden en él, los 7/8 del
♦ contenido.¿Cuántos litros deben salir?
PASO 3.- El número de personas que pagan
será: ♦ a)10l b)90l c)l00ld)ll0le)ll2l
SI. 7 200 = IO
♦ (9) Ernesto me debe los 4/5 de S/.160; si me paga
Sl . 720
♦ los 5/8 de S/,160:¿Cuánto me debe'!
RESPUESTA:
El número de personas que no pagaron será: ♦ a) S/.30 b) S/.28 c) S/.36
d) S/.12 e) S/.20
16 - 10 = 6 Rpta. 0 ♦
♦ (1O) Me deben los 3/7 de SI. 252. Si se me paga
(3) Un tren sale del paradero A. La novena parte
1/9 de SI. 252: ¿Cuánto me deben?
de los pasajeros bajaron en el paradero B. Los ♦
restantes y 20 más que subieron en esta esta- a) S/.80 b) S/.100 c) S/.120

ción continuaron hasta el paradero C. ¿Cuán- d) S/.140 e) S/.125
tos viajeros partieron de A,si a C llegaron ♦
420?
♦ (11) ¿Cuál es el número por el que hay que dividir
a)400
d) 520
b) 250
e)450
c )480
♦ 18 para obtener 3t ?

♦ a) 5.!.. b) 5.l c) 5l
(4) Una deuda de S/.3 600 debe ser cancelada por 5 9 1 5
12 personas en partes ig uales, pero como al- ♦ d) 5.!.. e) 53
9
gunas son insolventes, cada persona restante ♦
debe pagar 1/3 más de lo que le corresponde (12) ¿suál es el número que multiplicado por
1
para cancelar la dcuda.¿Cuántas personas no ♦ 5
4
da como resultado 20 ?
2
pagaron? ♦
a) 5 b) 6 c) 4 d) 3 e) 2

a) l~i b) 5~6 c) 6--
21
1

d) 3_!2_ e) 1.!..
21 2
(5) Una deuda de S/. 3 000 debe ser pagada en ♦
partes iguales por 15 personas. Como algunas
de ellas no podrán pagar, los que sí pueden ♦
(13) Maritzacompra 7t litros de pintura de pared.

hacerlo deberán pagar la mitad más de lo que ·+ Si gastó S/. 48½:¿Cuánto pagó por cada litro
les corresponde. ¿Cuántas personas no pagan? de pintura?
♦ 27 27 1
a) 6 b) 8 c) 4 d)3 e) 5 a) S/. 6 b) S/.5 c)SI. 2
♦ 44 1 22 1 22
d) S/.4-:¡ e)Sl.6
(6) Quince personas deben pagar una deuda de 7
S/.1 500 en partes iguales. Como algunas de ♦
(14) En una calle de 100 metros de longitud, desea-
ellas son insolventes, cada una de las restantes ♦
tienen que poner S/50 más para cancelar la mos colocar postes de principio a fin. cada
deuda.¿Cuántas persona~ no pagan? ♦ 6¼ metros. ¿Cuántos postes necesitaremos?
a) 6 b)7 c) 5 d) 3 e)4 ♦ a) 12 b) 14 c) 16 d) 17 e) 19
(15) Necesitamos distribuir 800 litros de vino en ♦ DLOQUEIV
toneles de 16f • ¿Cuántos toneles debernos
♦ ()) ¿Cuánto le falta a la siguiente expresión-para
tener listos?
ser igual a la unidad?

a)45 b) 42 c) 62 d) 24 e) 48 I
♦ 1
(16) Elizabeth gastó la tercera parle de su sueldo en 1+ ---=-.-
comprar ropa, la cuarta parte en comprar zapa- ♦ 2+ --r
tos y la quinta parte en comprar libros. Si le l + 2

sobraron S/.117:¿Cuál es su sueldo? c) 1
a) 1/11 b) 8/11
♦ d) 10/ll e) 3/11
a) S/.170 b) S/.480 c) S/. 540
d) S/.620 e) S/.580 ♦
Solución:
(17) Un vendedor de enciclopedias recibe corno ♦
Operamos de "abajo hacia arriba",
comisión, 3/16 del total de las ventas de libros ♦ aplicando según el caso : REGLA DE
dcGEOGRAF1A y los 5/18 del total de las ventas PRODUCTOS CRUZADOS
de libros de MATEMÁTICA. Si luego de una ♦
a c ad+bc
Jornada se vendió SL400 en libros de Geografía ♦ b +d = bd
y S/. 576 en libros de Matemática: ¿Cuánto e INVERSA DE UNA FRACCIÓN:
recibió de comisión el vendedor? ♦
1 b
♦ a=a
a) S/.235 b) S/.135 c) S/.255
ti"
d) S/.270 e) S/.315 ♦
(18) En un salón de I grado de 30 alumnos, las 2/3
O
♦ PASO l.- Empezamos sumando fracciones
partes tienen buzos dcportivos.¿Qué fracción desde abajo:

de los que tienen buzos, no tienen buzos?

a) 1/4 b) 1/3 c) 1/5 d) l/2 e) 1/8
♦ PA SO 2.- Sobre este resultado está la UNI-
(19) Si dejamos caer una pelota desde cierta altura:
DAD, lo que permite emplear la INVERSA de
¿Cuál es esta altura sabiendo que después del ♦
cuarto rebote se clcva 32cm y que en que cada una fracción :
rebote se eleva 2/3 de la altura anterior? ♦

a) 81 cm b) 162 cm c) 324 cm ♦ 2
d) 62 cm e) 72 cm ♦ PASO 3.-Al costado de este resuliado, se nos
pide sumar 2:
(20) Un hambriento caminante, encuentra a dos ♦
pastores que se disponen a repartir con él los 2 8
♦ 2+ 3=3
panes que iban a comer. Uno de los pastores
tenía 5 panes y el otro 3. Todos comieron por ♦ PASO 4.- Sobre este resultado está la UNI-
igual. Al partir, el caminante les dejó 8 mone-
DAD, entonces:
das para que se repartan. ¿Cuántas monedas ♦
les tocó a cada uno?
1 3
♦ -¡¡-=1r
a) 5 al primero y 3 al segundo 3

b) 4 a cada uno PA SO 5.-Al costado de este resultado se nos
c) 7 al primero y 1 al segundo ♦ pide SUMAR 1:
d) 6 al primero y 2 al segundo ♦
e) No se puede repartir exactamente

1 MATEMATICA
1
PASO 6.- Sobre este resultado está la UNI- a)A< B b)A > B c) A= B

DAD, entonces: d)2A=B e) No se puede afirmar nada.

(8) Dar la cuarta parte de E si se sabe que:
♦ 1 1
E=l- l +5- l
RESPUESTA.- ♦ 3- --3- 3- - - 1
Para ser iguala la UNIDAD ( que también po- ♦ 2-2 1-2
demos escribir como 11/11, a 8/11 le falta
3/11 ♦ a) -1 b) 1 c) 2 d) 4 e) O
Rpta.0 ♦ (9) Calcular la décima parte de F si se sabe que:
1 4 2 a) 1 b) 50
(2) Simplificar: 1 + --1- ♦
2+ - F= TTfo+ T7s c) 1/50 d) 5
3 ♦ -1- - -1- e) 1/5
1/3 1/2
a) 117 b) 1..!_ ♦
7 (1 O) Al dividir un número entre su inversa se obtie-
d) 5I ♦ ne 81. Hallar dicho número.
7
a) 1/9 b)9 c) 3 d) 1/3 e) 1/81
(3) Dar la fracción irreductible y equivalente a: ♦
(11) Si los 3/4 de un número es 45: ¿A cuánto
1 a) 1/3 b) 5/3 ♦ equivale el doble menos la mitad del mismo
2+ 1
1+-- c) 7/3 d) 8/3 número?

1 - .!. e) 11/3 a) 60 b) 30 c) 100 d) 90 e) 120
2
♦ (12) La edad de Elizabcth es los 4n de la edad de
(4) Señalar una fracción equivalente a: Víctor y las 2/3 partes de la edad de Walter. Si

1 al sumar las tres edades nos resulta 102 años:
1+---1-
3--- ♦ ¿Cuál es la edad de Elizabeth?
2 _ .!. ♦ a) 26 años b) 22 años c) 18 años
2 d) 24 años e) 36 años
20 ♦
30 3 (13) Una pelota pierde las dos quintas partes de su
a) 2Í b) l4 c) 7 ♦ altura en cada rebote que da. Si se le deja caer
7 desde un metro de altura:¿Qué altura alcanza-
d) Tif e) ay b
♦ rá después del tercer rebote?.
(5) Reducir: 1 3
♦ a)20 cm b)21 cm
1 2 5
a) 12/5 b) 1/5
2+ 1
l+-- c) 13/5 d) 7/5 ♦ d) 6
1
cm e) 23
1
cm
1 + .!. e) 6/5
2 2
2 ♦ (14) Un albañil y su ayudante pueden hacer una
obra en 12 días, después de haber trabajado
(6) ¿A qué es igual?: ♦ juntos durante seis días se retira el ayudante y
el albañil termina lo que falta de la obra en 10
1 a) -1 b) O ♦ días. ¿En cuántos días puede hacer toda la
1+ 1
1---1 c) 1/2 d) -1/2 ♦ obra el ayudante trabajando solo?
1-2 e) 1/4

a) 23 b) 32 c) 35 el) 40 e) 30
(15) Liliana, gasta en alimentos la mitad de lo que
(7) Señalar la alternativa correcta si: ♦ gana y los 2/3 de lo que resta lo gasta en otras
1 1 necesidades. Al cabo de dos años, ahorró
A=3+ l B =3 + ♦ S/. 3 000. ¿Cuánto gana por mes?
1
1+--- 2---
2 _ .!_ 1 1 ♦ a) S/.650 b) S/.720 c) S/.750
-+- d) S/.850 e) S/.700
2 2 3

1 ALFO~SO ROJAS PUÉMAPE 1

(16) Un granjero reparte sus gallinas entre sus cua- ♦


tro hijos. El primero recibe la mitad de las
gallinas, el segundo la cuarta parte, el tercero ♦ CIAVE DE RESPUESTAS
la quinta parte y el cuano, las 7 restantes. RAZONEMOS - 2
¿Cuántas gallinas repartió el granjero'! ♦
a) 120 b) 160 c) 150 d) 140 e) 180 ♦ BLOQUEII
(17) Un niño compra limones a 3 por 2 nuevos ♦ (1)c (6)b (11) b (16) d
soles y los vende a 4 por 3 nuevos soles. (2) b (7) e (12) e (17) d
¿Cuántos limones debe vender, para ganar ♦
(3) d (8) e (13) d (18) a
diez soles?

(4)a (9)b (14) a (19) e
a) 160 b) 120 c) 80 d) 140 e) 150 (5) d (10) d (15) e (20) a
(18) Al tesorero de una sección de 1°grado le falta ♦
1/9 del dinero que se le confió.¿Qué parte de ♦ BLOQUEIII
lo que le queda restituirá lo perdido?
(1) b (6) e (11) e (16) e
a) 1/8 b) 1/3 c) 1/6 d) 1n e) 1/9 ♦ (2) d (7) b (12) d (17) a
(19) Dos tercios de los profesores de nuestro cole- ♦ (3) e (8) b (13) a (18) d
gio son mujeres . Doce de los profesores va- (4)d (9)b (14) d (19) b
rones son solteros mientras que los 3/5 de los ♦ (5)e (10)a (15) e (20) e
mismos son casados.¿Cuál es el número de ♦
docentes? BLOQUE IV
a) 70 b) 120 c) 60 d) 56 e) 90 ♦
(1)e (6)b (11) d (16) d
(20) ¿Cuál es el número cuya mitad, más su duplo, ♦ (2) a (7) a (12) d (17) b
más su tercera parte y más su triple, da el ♦
(3) e (8) b (13) b (18) a
número 1 435?
(4)e (9)d (14) e (19) e
a) 123 b) 326 c) 286 d) 246 e) 320 ♦ (5) e (10) b (15) e (20) d

1 MATEI\IÁTICA
1
LAS l\lATEl\lÁTICAS DE TODOS LOS DÍAS

( 1) En un Restaurant, atienden una madre y su hija. La madre demora 15 minutos en


preparar un pedido, mientras que su hija demora 20 minutos. Llega un cliente y las dos
comienzan a preparar el pedido, pero la madre se retira a los 5 minutos para atender a
otro cliente. ¿ Cuánto demora la hija en terminar de preparar lo que falta?

Solución:

Respuesta

1
(2) En la Compañía Constructora "GREMACO S.A." trabajan 6 obreros especializados en
poner parquet y que demoran 18 horas en instalar el parquet a un edificio de 3 pisos.
En la Compañía "SUCASA S.A." trabajan 8 obreros, con la misma especialidad y
demoran 9 días en instalar el parquet a unedificiode 3 pisos. Si se contratan 3 obreros
de "GREMACO S.A." y 4 obreros de "SUCASA S.A.", ¿qué tiempo demoran los 7
obreros en hacer una instalación?

Solución:

Respuesta

1
•t I ALFONSO ROJAS PUE."tlAPE 1

(3) En una de las plantas industriales de TUERCA S.A. se observó que una gran rueda
dentada ha sido puesta fuera de uso. Esta y otra más tienen juntas 180 dientes. La que
se malogró fue reemplazada por otra cuyo número de dientes representa los 3/5 del
número de dientes de la rueda malograda.¿ Cuántos dientes tenía cada una de las ruedas
iniciales si el nuevo total de dientes ha sido disminuido en 48?

Solución:

Respuesta

j
(4) En un balde de 3 litros de capacidad, se ponen dos litros de leche y uno de agua. Luego
se elimina 1/3 de la mezcla y se llena el balde de agua; después se elimina la 1/4 parte -
de la nueva mezcla y se vuelve a llenar de agua el balde; por último se elimina la mitad
de esta última mezcla y se llena de agua el balde nuevamente. ¿Qué cantidad de leche
contiene un litro de la última mezcla?

Solución:

Respuesta

j
Cif.\Ma:fi 1 MATEI',,lÁTICA
1

1 RAZONAMIENTO MATEMÁTICO
1W --

(1) En el auditorio del colegio hay 75 (7) El reloj grande del colegio marca las
alumnos de 111 grndo entre hombres y 11 am, entonces un travieso alumno
mujeres; la séptima parte de ellas. usan forzando las agujas del reloj las hace
anteojos y la onceava parte de ellos girar a cada una media vuelta; en ese
llevan calcetines blancos. ¿Cuántas inslante:¿Qué hora es?
mujeres usan anteojos?
a) 3pm b) 11 am c) 1pm
a) 5 b)7 c) 6 d) 9 e) 11 d) 9 pm e) No se sabe

(2) ¿Qué número debe ir en lugar de x en: (8) ¿Cuántos postes se necesitan colocar
en un terreno triangular, si se nos pide
1;2;4;x; 16
que debe haber uno en cada esquina y
a) 8 b) 12 c) 10 d) 5 e) 6 5 en cada lado?

(3) Si 6 lapiceros cuestan SL30:¿Cuántos a) 15 b) 18 c) 12


lapiceros podré comprar con S/.m? d) 9 e) 10

a) 5m b) 10m c) m/5 (9) ¿Cuántos surcos tiene un disco de 45


d) 6m e) m/6 RPM que disca 2 minutos 35 segun-
dos?
(4) ¿Qué número sigue en la sucesión?:
a) 352 b) 163 c) 82
40; 39; 37; 34; 30;.... d) 51 e) 1
a) 26 b) 25 c) 28 d) 27 e) 21
(10) Cinco trailers van por una carretera
(5) Habiendo perdido un jugador la mitad con 6 cajones grnndes. teniendo cada
de su dinero , volvió al juego y perdió cajón 8 cameros. ¿Cuántos carneros
la mitad de lo que le quedaba, repitió vienen?
lo mismo por tercera y cuarta vez a)48 b)240 c)24
hasta que no le quedaba más que S/.6 d) 120 e) Ninguno
rCuánto dinero tenía al comenzar el
juego? ( 11) Completar el número que falta:

a) 92 b) 108 e) 48
d) 96 e) 120

(6) Si una caja tiene 6 caras:¿Cuántas ca-


ras tienen un bloque formado por dos
cajas pegadas por una de sus caras?
a) 8 b) 12 c) 10 a) 185 b) 215 c) 305
d) 6 e) 11 d) 255 e) 285
1 ALFONSO ROJAS PUÉMAPE 1 11©@Ui
e Aquí mis operaciones de RAZONAMIENTO MATEMÁTICO

..-.-:.
t=
1 MATEMATICA
1

. --.
-:-- , .....,_ _ -----:" . - - , .

CAPÍTULO 5

a
REPRESENTACION
,
--; \
r
.,
DECIMAL DE LOS
1
NUMEROS RACIONALES l
1 1
1
\;'

, :--.... - ~~-·- -
1/

OBJETIVOS:

• Reconocer otra forma de representar a los números racionales.


• Manejo de técnicas operativas con números decimales.

' ' - - - - - - - - - -- - - -- - - - - - - - - - - . - - - - - -
1 ALF01'SO ROJAS Pl'.E\fAPE 1 ilWi:i41
ESTRUCTURA DEL CAPÍTULO:

l. ' l \11 ·. l<l > 111-TI \I \ 1

1.1 Fracci6n ordinaria y fracción decimal


1.2 Número decimal
1.3 Tabla de valor de posición 1.k las cifras de un número decimal.
1.4 Propiedades de los números decimales
1.5 Comparación de números decimales
1.6 Clasiíicación de números decimales
- Decimal Exacto
- Decimal Inexacto
* Periódico Puro
* Periódico Mixto

SKANIPRÁCTICA 1
,

2.1 Generatriz de un número decimal exacto


2.2 Generatriz de un número decimal periódico puro
2.3 Gcneratri1 de un número decimal periódico mixto

SKANIPRÁCTICA 2

"\ OPl·J< \CIO,ES ('Oi\ 'l \IEl<OS l>ECl:\1 .\1.ES

3.1 Adición y sustracción de números decimales


3.2 l\lultiplicación y potenciación de números decimales

3.3 División de números decimales


3.4 Radicación de números <kcimales

SKANIPRÁCTICA 3

RAZONEMOS
LAS MATEMÁTICAS DE TODOS LOS DÍAS
RAZONAMIENTO MATEMÁTICO
1 J\,L\TEMArICA

e
1. NÚMERO DECIMAL)

SKANITO, tene-
mos que repartir O¡ ~.,
ª~~~\
SI. 17 entre 4
personas: yo se 1·

/ que m1 par111 es
s: 1714 pero Jor-
ge dice que es
$/, 4,25
¿Crees que te
engañaron?

Ni lo engañaron, ni le dieron de más, ya que ambas cantida-


des son iguales como veremos más adelante. ¿Hay alguna
otr.a forma de representar a las fracciones?
Si, esta otra forma emplea los NúMEROS DECIMALES
que será el tema que ahora nos ocupará.
( O RECUERDA QUE:

1.1 FR ACCI< lN OIU>INARIA Y FRA(Tl<>N En cualquier FRACCl6N.


DEC il\1.\L el denominador debe ser
DIFERENTE DE CERO.
Si una fracción tiene su denominador diferente a una
potencia de 10 se le llama FRACCIÓN ORDINARIA.
c::>o
Si una fracción tiene como denominador a una potencia de
10 recibe el nombre de FRACCIÓN DECIMAL.
1 AlFO:\SO ROJAS Pl'É.\l.\PE 1
Ejemplos:
O ¡CUIDADO!
(I) ; · ~ ; ~ son fracciones ordinarias
~-:NUMERO
()
2 To1 ; 7 ; 3 son fraccmnes
. d . 1es
ec1ma
100 1 000
~CIFRAS
1.2 ,C\IERO DFCI\IAL
Es la expresión lineal de una fracción ordinaria o decimal
que se obtiene al dividir el numerador emre el denominador.

Ejemplos:
RECUERDA QUE:
(1) I= O. 2 que resulta de dividir 1: 5 • 0,02 se lee: "Dos centes,mos"
• 3.152 se lee. "Tres enteros ciento cm-
(2) l = O, 666. .. que resull.a de dividir 2: 3 cuenta y dos m1lesimos ·•
3
• 15,0007 se lee: "Q1,,nce enteros. siete
(3) 2.. = O, 4666... que resulta de dividir 7 : 15 d1ezm1les1mos"
15 _)
I.J TABLA DE VALOR DE POSICIC)N DE
1 AS CIFR.\S DE l ''.\ ;\Ú I\IERO DECI \I.\L
Recordemos que un NÚMERO DECIMAL consta de las siguienles

-
parles: PARTE ENTERA, COMA DECil\1AL y PARTE DECIMAL.
Ejemplo: 72. 345
...___..
L_ PARTE DECIMAL
COMA DECIMAL
PARTE ENTERA

Aquí, el numeral 5 Licne un valor relalivo en el número dado; e::> O


esle valor depende de la posición como cifra en dicho número; esta
posición es la de los milésimos.
¿Porqué no revisamos una Tabla con los principales valores de posición
en un número decimal? ¿Me ayudarías? ...
. . . veamos:
PARTE ENTERA PARTE DECIMAL

I
5º 4° 3° 2º 1º 1º 2º 3º 4º 5º +- ÓRDENES
Ejemplo: 3 2 8 5 6 3 5 2 4 6

DECENA DE MILLAR
UNIDAD DE MILLAR =...__Jt
__j~~
CENTENAS
DECENAS
UNIDADES

COMA DECIMAL
IV8!éifi 1 MATEMÁTICA t
IA PROPIEDADES DE LOS NÚI\IEROS DECII\IALES
1°. Un número decimal no ve alterado su valor si se le añaden o
suprimen CEROS A SU DERECHA.
Ejemplos:
(1) 5,7 = 5,70 (2) 5,7 = 5,70000
(3) 786,520000 = 786,52 (4) 2,5000 = 2.50

2°. Si a un número decimal le corremos la coma decimal a la derecha


uno o más lugares. el número decimal queda mulLiplicado por la
unidad seguida de tantos ceros como lugares se corrió la coma
decimal. f - - - -0- ¡C-UIDADO!
Ejemplo: 9. 076 x 100 • 907. 6
56 , 13 X 10 = 561 3
9, O 7 6, corriendo dos lugares la coma,tendremos:
L..J 7, 0156 X 1000 • 7 015 6
9 0 7,6 = 9,076 X 1()() 0 0 • 0, 2 X IO QOO "'2 OQO

como la coma fue corrida dos lugares hacia la derecha, el número


inicial quedó multiplicado por la unidad seguida de dos ceros.

3°. Si a un número decimal, le corremos la coma decimal a la


izquierda uno o más lugares, el número decimal queda dividido
por la unidad seguida de tantos ceros como lugares se corrió la 00 ¡ ATENCION I
coma decimal.
8156,32 -::: 815632
Ejemplo: 1000 '
8 156,32 , corriendo tres lugares la coma.tendremos: • 573.485 = 5,73485
L.J 100
8 156 32 6 783,3 :;: 678 33
8 15632 = • 0 OO 10 '
' 1 000

como la coma fue corrida tres lugares hacia la izquierda el


número inicial quedó dividido por la unidad seguida de tres
ceros.

1.5 CO:\IP.\R ,\CI<>N DE 1'Úl\1EROS DECII\L\LE~


• Si dos números decimales son de signo diferente será menor el
de signo negativo sin mayor discusión por su ubicación en la
recta numérica.
Ejemplo:
Entre los números-18,032 y+ 1,4 357 es menor el primero por ser
negativo.
1 ,UFOi'liSO ROJAS PUÉMAPE 1
• Si dos números decimales son de igual signo, se procede
del siguiente modo: se iguala el número de decimales con
ceros para luego eliminar la coma decimal y comparar O RECUERDA QUE:
como si fuesen números enteros.
Un número decimal no a/te·
Ejemplos: ra su valor si se le añaden
CEROS a su derecha
(1) Comparar 5,75 con 5,076
f¡m ·
* Como el primer número tiene sólo dos decimales,
le agregamos un CERO a la derecha e::> O para 6, 73 • 6,730 a 6, 7300
que ambos números dados tengan tres decimales
cada uno: 5,750 5,076
* Ahora eliminamos la coma decimal en ambos nú-
meros.
5 750 5 076
* Como 5 750 es mayor que 5 076, entonces:
5,75 > 5,076

(2) Comparar -8, 156 con -8, 15


* Igualamos la cantidad de decimales en ambos nú-
meros añadiendo un cero a la derecha del segundo
número: -8,156 -8,150
* Eliminamos la coma decimal en ambos números: oo ¡IMPORTANTE'
-8 156 -8 150
* Como -8 150 es mayor que -8 156, entonces: En la r9dB numi§flca
e;, o o -8,15 > -8,156
◄ ►
(3) Comparar: 72,315 con 72,31500 0
8156 -8150 O

* Por propiedad de números decimales, podemos


suprimir dos ceros de la derecha del segundo número
··crecen de izquierda a derecha"
dado.
Entonces • 8 156 < ·8 150
* Entonces ambos números quedarán así: 6 · 8 150 > -8 156
. 72,315 = 72,315
* Esto es: 72,315 = 72,31500

U , l'L\~IF IC \( ' l(>N DE NÚ MEROS DECIMA LES


El siguiente cuadro, grafica una clasificación de números

<
decimales que luego pasaremos a estudiar con mayor detalle:

NUMERO
DECIMAL
NÚMERO DECIMAL EXACTO

NUMERO DECIMAL INEXACTO


< DECIMAL PERIODICO PURO

DECIMAL PERIODICO MI XTO


1 MATEMATICA
1
* NÚMERO DECIMAL EXACTO.-
O ¡CUIDADO!
Le llamamos así a aquel que tiene un número limitado de
cifras. Al transformar una fracc,on a
Ejemplos: su ri?presentacion decimal se
0,25 3,75 1,8 DIVIDE el numerador entre el
d,:mom,nador. pero la fraccion
debe haber sido convertida
Una fracción da lugar a un NÚMERO DECIMAL EXAC- en IRREDUCTIBLE.
TO, si en el denominador aparecen sólo factores que son
potencias de 2 ó de 5 ó de ambos. C::> O ·-··-·-:-:-:-:-;-:-:-:-.-:;.-:-j;·•:-:-:-·-

Ejemplos:
(1) .!.4 = O• 25 porque 1 : 4 = 0,25
( o o RECUERDA QUE:
::
además el denominador 4 es una potencia de 2.
Para este tipo d0 divisiones
1 añadimos un CE RO al d1v1d0n·
4 = 22 do y un ·CERO-COMA" en el

(2) La fracción '>7'í :¿Equivale a un decimal exacto? ¿Por


¡~ coc,ente:

qué? -·
90~
Dividimos 7: 25 y obtenemos 0,28 corno cociente.
80 0,225
Luego
~
225 = O' 28, portJue en el denominador apa- 100
80
rece una potencia de 5, es decir: .2.2 = o, 28
5 200
. 9 E . 1
. , -:¿ l . 1exacto.¿
'> P or
(3) Lalracc1on qu1vaeaun{ec1ma 200
qué? 40 . .
000
Dividamos 9: 40 y obtenemos 0,225 como cocien-
te C:: OO

Luego ..2.. = O, 225, porque en el denominador apa-


40 9
recen como factores, potencias de 2 y 5, es decir: -3- - = O, 225
2 X 5
Observando estos tres ejemplos:¿Pmlemos saber cuán-
tas cifras decimales tendrá el número decimal resultan-
te antes de efectuar la división?

S1; bastara con saber cual es el MAYOR EX-


PONENTE de 2 ó 5 en el denominador de la
fracción Jrreduct1ble.
As1 1/8 tiene 3 decimales en su numero decimal
correspondiente, porque en el denominador 8 ó 2',
el MAYOR exponente de 2 es 3. iCOmpruebalo!
1 ALFONSO ROJAS PUÉMAPE 1
* NÚI\IERO DECIMAL INEXACTO.-
Le llamamos así, a aquel que tiene un número ilimitado de cifras
decimaks. Estos números decimales pueden ser a su vez de dos tipos:

(a) DECII\IAL PERH)DICO PURO


Es aquel en cuya parle decimal aparece una o un grupo de cifras
llamado PERÍODO que se repite indefinidamente a partir de la coma
decimal.
Ejemplos:
(1) 0,333 ... e::· período: 3 ; representación: OJ
(2) 0,757575 .. C:: período: 75; representación: 0,75
¿Ccímo podemos saber si una fracción puede ser reprC'scntada por un
DECil\lAL PERIÓDICO PURO? --=-------- - -
O RECUERDA QUE:
V - vamo- despaoo
Una fracCtón es IRREDUCTIBLE
S1mpl·'1camos la tracc1ó'l has .1 que sea orre- cuando sus términos ya no tienen
duct ,,e L; O
D,. ::or1por 110s el~ , nominador en sus tacto- fac•,res comunes
' DI TICS E1emplo.
L · ro dec1maI correspondiente será PE
RIOOICO PURO, si los factues pnmos del
denominador son d1s11nros a 2 y 5 !_!_ FRACCIÓN REDUCTIBLE
3x5
7
Por eIemµlo 1'3 3, 1 33
°Y FRACCIÓN IRREDUCTIBLE )
(b) DECII\IAL PERl()DICO MIXTO
Es aqud cuyo período empieta luq~o de una cifra o grupo de
cifras después de l.1 coma decimal: a esLa cifrn o grupo de
cifras le llamamos PARTE NO PERIÓDICA.
Ejem¡,los: ¡CUIDADO!

( 1) 0,5222 . . . C:
PARTE NO PERIÓDICA: 5; PERÍODO: 2 '\fi 1, 4142135
Rcprcscnt:ici<ín: 052 V3 1, 7320508
Q

(2) 0,73-l15415415. C: PARTE NO PERIÚDICA: 73 V5 ., 2. 23to679


PERÍODO: 415 V7 = 2, 6457513
""
Rcprcscntaci,ín: 0,73415 n 14159 65

¿Cómo podemos saber si una fracción puede ser representada e - 2 , 7162B183


Son números con la parre
por un DECIMAL PERIÓDICO MIXTO'! docimaf 1/imitacla pero SIN
PERIODO, éstos no son mí-
~,;,s, ~
forma similar al caso de periódico puro. es meros racionales, más bien
-ne la fracc,on y descomponemos el deno- reciben el nombre de NÚ
nador sus · ,ctores pnmos . E· on . el numero decI- MEROS IRRACIONALES
:.1I co,, __ ,1ond1" ·te será PERIÓDICO MIXTO e )S facto -
~ P n1c.~ dL denom,nadorson2o5oan !}()~ ADEMÁS DE
OTll~.3 FACTORE S PRIMOS OISTIN I OS DE 2 y 5

o w - - - - = O 1166.6
2.
l·- - - - - -
2 <5><3
-- - -- -------'
#fr.\{@:i¡k■ 1 MATl'MATICA
1 ~$,

SKA ~ I PRACTICA CD
l. Colocar \' si es ,·cnbdcro o F ),¡ es falso en cada casillero:

(1) 7,5 = 7,50 D (13) 763,512 = 7 635, 12 D


(2) 6,36 = 6,360 D (14) 0010,31 =0l0,31 D
(3) 05,5 = 5,5 D (15) 785,7U = 785,713000 D
(4) 7,62 = 70,62 D (16) 0,5 = 0,5(K)()00 D
(5) 3,250 = 3.2 D ( 17) 08,25 = 8,250 D
(6) 7,2 =:: 7,20 D (18) 0.000072 = 72,()(X)O D
(1) 10,003 = 1,003 D (19) 56,75 = 560,750 D
(8) 3.-l200 = 3,42 D 110) 42. 132 = 42, 1320 D
(9) 5,027 = 5,27 D (21) 42,132 = 42,132(K> D
(10) 7,08 = 07,08 D (22) 42,132 = 42,132(K)0 D
( 1 1J 007 ,3 = 07 J D (23 l 42.132 = 042.1320000 D
(12> 1152 = 011.s20 D (2..t > 42. 132 = oo.n. 13200000 D

11. Colocar > lÍ < según corre~ponda:

(1) - 62,508 D + 87,52 (11) - 15,08 D - 1,08


(2) -13,113 D + 113,13 ( 12) - 17,17 D 17,17
(3) - 6,2 D + 8,2 (13) - 61,01 D - 17,69
(4) 1,5 D o.o ( l ..t) + 17,86 D + 17,086
(5) - 6,13 D + 1,1 ( 15) +235,973 n + 235,00973
(6) + 7,12
D + 12.05 (16) -16,2..t6 - 15,241
'v J
(7) - 0.6 18 D 3,018 (17) - 1,0685 - 1,068
i
(XJ - 612.12 D o.o ( 18) 6.oo:n D - 6.()()32 .t
(9) + 51.136 D + 71.23 ( 19) - 5.031
D - 5,032
( 1O) + 42,057 D + 89,15 (20) -875.3 D o.o
(
1 AlFO;\'SO ROJAS PUÜ1APE 1

111. Marcar con un aspa según estime conveniente:

NÚMERO RACIONAL
NÚMERO NÚMERO
DECIMAL DECIMAL INEXACTO IRRACIONAL
EXACTO PERIÓDPURO PERIÓD MIXTO

0,72.S X
5,233:tt ..
07,52
58,58765
6J218756 . . ...
3,14159 .. .. .
7,6-12424:2 .. . ..
0,55555 .. . . .
-l 7X,05
7,6185743 .... .
635563556 . .. ..
8,6478478478 .. ...
65,723-l-l4 . .. .
6 l 8,S65-l656 ...
1.4142135 . . . .

1V. Comes La las siguil'ntes preguntas:

¿Cuántos milésimos hay en un décimo'!


¿Cuál de los siguientes números es mayor y por qué?: 67,7272 . .. ; 67 ,722'2 .. .
¿Por qué no se altera un número decimal si se añadl'n ceros a la izquierda de su parte
entera? . ~ -t
(-l) ¿Por qué ne .se altera un número decimal si se añaden ceros a la derecha de su parte
decimal?
(5) Teniendo una fracción, ¿c6mo se logra encontrar su número decimal equi \'a leme'!
(6J Si V2= l,-ll4213S .. . ¡,cuüles elnúmcroirracional: V2 ó IAl-l 2 135 ... ó
ambos'! ¿Por qué?
(7¡ ¿Tiene 3,14159 . ... un equi\'alente en fracción'! ¡,Por qué'!
MATI!\L\TICA

\'. Completa con V o F según sea verdadero o falso:

(1) 7,25 = 72,5: 10 D (11) 32,015 X 10 = 320,15 D


(2) 653,2 = 6,532 X 100 ·□ (12) 12,15: 10 = 1,215 D
(3) 68,58 X 10 = 6,858 D (13) 6,70038 = 67,0038: 10 D
(4) 68,58 X 10 = 685,8 D (14) 1,685 X 100 = 168,5 D
(5) 715,2: 100 = 7,152 D (15) 62,274 : 100 = 6 227,4 D
(6) 4,2075 = 7,4}5 X 10 D (16) 5,013 : 10 = 0,5013 D
(7) 74,15 = 7,415 X 10 D (17) 6,282 : 100 = 0,06 282 D
(8) 6,015 X 1000 = 6 015 D (18) 52,46 = 5,246 X 10 D
(9) 13,2 x 100 = l 320 D (19) 75,03 = 750,3 : 10 D
(10) 1,4215 X 1() = 0,14215 D (20) 75,68 = 7,568 D
VI. Completa el siguiente cuadro según sea el tipo de número decimal al que la fracción
dada dé lugar:
NÚMERO NÚMERO NÚMERO
FRACCIÓN DECIMAL AL FRACCIÓN DECIMAL AL FRACCIÓN DECIMAL AL
QUEDA LUGAR QUE DA LUGAR QUE DA LUGAR
1 EXACTO
5 PERIÓDICO 23
8 6 MIXTO 880
1 3 17
10 14 231
1
30
13
-
40 - 5
221
5 5 1
21 7 54
7 2 1
13 5 375
7 1
-112 PERIÓDICO
PURO 3 19
3
-358 -
4
5
25
7 17 2
16 27 9
22 8 1
55 15 2
9 19 7
26 32 30
1 ALFO!'/SO ROJAS Pt;EMAPE 1

Aquí mis operaciones de SKANIPRÁCTICA Q)


1 MATEMATICA
1
e 2. FRACCIÓN GENERATRIZ) O ¡MUCHO CUIDADOI
Antes de escrib,r el numerador
de la fracc,on generatriz se
Todo número decimal Liene su equivalente en forma de SUPRIMEloscerosdeladerecha
fracción. de la PARTE DECIMAL si
La fracción que genera un número decimal se llama FRAC- hubieran
CIÓN GENERATRIZ.
As,. 0,500 = 0,5
1.1 (;1-:NEIUTRIZ DE l lN N Úi\1EH.O DECII\IAL EXAC-
TO.
• Cuando el número decimal tiene la PARTE ENTERA O O PRIMOS ENTRE Si
NULA:
111 Se escribe como numerador toda la parte decimal (sin la
Son los numeros que sólo
coma) i:::> O llenen como divisor común a
2° Se escribe como denominador, la unidad seguida de
la UNIDAD
tantos ceros como cifras tenga la parte decimal.
Al s,mphf,car una fracción
3ll Se simplifica la fracción si es que todavía NO ES
para mayor segundad des-
irreductible (es decir, si todavía sus tém1inos no son
componemos/os terminos en
PRIMOS ENTRE SÍ) sus factores primos y supri-
Ejemplos: mimos los que son comu-
(1) Hallar la fracdón generatriz de 0,75.
Soluciá11:
• En el num(·radur l.'.s1:ribimos: - - - - - - - - - - - - - 75
• En el denominador escribimos 1 seguido de DOS CEROS (por-
que la parte decimal tiene dos cifras): - - - - - - - - - ~ ](X)

75
• Luego, la fracción será:
~ 100
• Como 75 y 100 no son PRIMOS ENTRE SÍ
75
O O O podemos simplificar la fracción: l(X) =
Respuesta.-
La frucción generatriz de 0,75 es 3/4.
• Cuando el número decimal tiene la parte entera NO NULA
lo desdoblamos para luego efectuar una suma linal, así:
(2) Hallar la fracción generatriz de 3,25.
Soluciá11:
• Desdoblamos el número así: 3.25 = 3 + 0,25
• Escribimos la fracción generatriz de la parte decimal: 3 25=3+1?...
' 100
I
3, 25 = 3 +
4
• Finalmente volvemos a sumar pero ahora como una suma de
fracciones: 3• 25 = 11
4
Respuesta.-
La fracción generatriz de 3,~5 es 13/4
·ti 1 .-\1 ro,so ROJAS P( 'EM\PE 1
2.2 GFNEIU n i1 z IJ E l 'N Nt'II\IERO DFCI!\IAL PERIODICO PURO.
Veamos el cálculo de esta fracción generatriz en un ejemplo:
Hallar la fracción generatriz de 0,545454 ... O O
So/ució11:
• En el numerador de la fracción escribimos el período, es decir: 54
• En el denominador de la fracción escribimos TANTOS NUEVES
COMO CIFRAS TENGA EL PERÍODO. En este caso el período 54
tiene dos cifras, entonces en el denominador escribimos: 99
• Luego, la fracción será: ➔ O 54

=2!
99 r o CUIDADO!

• Simplificando: ➔ 054=9x6=6 / Para abreviar la escritura, se puede


' 9xll 11 ):!: escribir sólo un período asf:
Respuesta.- 3
l:i o tamié! · · .. o.'j
La fracción generatriz de 0,5454 ... es 6/11
,,== · 0,333 . = 0,3
Obser\'acilin.-
Si un número decimal periódico puro tiene parte entera distinta de
CERO, procedemos a desdoblar como en el ejemplo (2) de la p~ígina
anterior.
Si el número decimal fuera 2,5454 ... 6 2,54
desdoblando tendríamos: 2,54 = 2 + 0,54

=2+~=1281
11 11

2.3 GEl\rtn rn.lL DE l , l'l \IE RO D EC II\IAL PEIU()DICO 00 ¡ATENCI0N!


:\11 \'. ro.
Podemos abreviar
Observa con cuidado: también un periódico
Calculemos la fracción generatriz de 0,15909090... O OO MIXTO así:

Solució11: ;.::=· 0, 159090... = o, 1590


• En el numerador de la fracción generatriz escribimos la parte NO (j PARTENOPERIÓDK:A 15
': PARTE PERIÓDICA 90
PERIÓDIGA seguida de la PARTE PERIÓDICA menos la PARTE
NO PERIODICA: ➔ 1 590 - 15
• En el denominador escribimos tantos NUEVES como cifras tenga el
pcríodp seguido de tantos ceros como cifras tenga la parte NO
PERIODICA, es decir: ➔ 9 9CX) ,.....,. I 590 _ 1S l 575
• Entonces la fraccicín generatriz será: O, 1590 = () = --
9 90 9900
• OOS CEROS en el denominador ----♦ t
• T>OS l\:lJEVES en el denominador - - - - ~ -

'"' 7x~x~ 7
Descomponiendo los términos y simplificando: O, 1590 = 2 2
= 44
44x ~X~
Respuesta.-
La fracción generatriz de 0,159090... es 7/44.
1 MATEMATICA
1
l SKA:\IPR ,\CTICA 01
Hallar la fracción generatriz ele:

(1) 0J2 (8) 0,175 (15) 18,62 (22)10,105


(:!) 0,17 (9) 1,2 (16) 15,85 (23) 12,08
(3) 0,35 (10) 1,35 (17) 16,25 (24) 25,25
(4) 0.125 (11) 2.75 (18) 5,02 (25) 1,75
(5) 0,012 (12) 3,05 (19) 11,1 (26) 30,5
(6) 0,25 (13) 3,125 (20) 6, l 2 (27) 68,85
(7) 0,8 (14) 7,075 (21) 8,18 (28) 72,75

(29) 0,1 (36) 3,2 (43) 6,6 (50) 3,Í5

(30) 0,7 (37) 10,3 (44) 17,36 (51) 9,16


...---. ...---.
(3 l) O, Í3 (38) 15,63 (-t5) 2,111 (52) 3,018

(32) 0,63 (39) S,09 (46) 5,2 ,.. (53) 5,54


,.......
CH) o;n (40) 2,0I (47) 6,15 (54) 6,75
,...._
(34_¡ ll,X1 (41) 6,Í8 (48) 31,18 (55) 111
..... ,.----..
( 35) O,, 11 (42) 2,315 (49) 6,Íl (56) 17,Í2

(57) o.:!'J (64) 0,745 (71) 2.69 (78) 4,165

(58) 0,56 (65) 1,23 (72) 6,56 (79) 8,15Í8


,...._
(59) 0.232 (66) 5.7?; (73) 8,6~4 (80) 3,162Í4

(60) 0,17 (67) 7,623 (74) 2,6315 (81) 2,0532

(61) 0,12) (68) 8,5í1 (75) 6,732 Í8 (82) 7,123

(62) 0,436 (69) 12,6Í2 (76) 2,11 112 (83) 2,156

(63) 0.1245 (70¡ l .8Í5 (77) 9,136 8 (84) 8,1235


1 AlFOl\SO ROJAS PUEMAPE 1

Aquí mis operaciones de SKANIPRÁCTICA ®

I
MATEMATICA
1
,: 3. OPERACIONES CON NÚMEROS DECIMALES)

., .1 ADICIO',1 , Sl'STR-\CCl<>N DE N(\IEROS DECI-


\ I \L ES
* Si se trata de decimales exactos. buscarnos que tengan la misma O ¡IMPORTANTE!
cantidad de cifras en la parte decimal completando con ceros.
Al sumar o restar números
S1 se tratara de sumar o restar 7 ,65 con 13,7563
decrmales, todas las colum-
Entonces igualamos la cantidad de cifras de la parte nas verilea/es de crfras co-
dccmrnl, es decir: 7 .6500; 13,7563 ffesponden a un mismo or-
Al sumar o restar escribimos un número hajo el otro den respectívo.
cuidando que la COMA DECIMAL esté ALINEADA.
para Iuego proceder a operar como si se tratara de nümeros
naturales.
En el resultado. volvemos a escribir la COMA DECI-
MAL en la misma línea vertical que las demás. RECUERDA QUE:

l~iemp/us: En la ADICIÓN-
A+ B s C
(1) Efectuar: 7.3 + 15.18 + 2,0156 AyB ➔ Sumandos
C ➔ Suma
Solución:
• Compktando con ceros a la derecha de la parte En la SUSTRACCIÓN
decimal: 7,3000 + 15,18CX) + 2,0156 M- S =D
• Escrihiendo uno ba_ío el otro: 7 , 3 O O O+ M ➔ Minuendo
N ➔ sustraendo
15,1800 t.:; o
D ➔ diferencia
2,0156
• Electuando como si sumáramos 2 4, 4 9 5 6
nümeros naturales: __/
L la coma conserva el
lugar de la.,; demás.
(2) Efectuar: 31,22 - 8,076
So/uciú11:

• Completando con ceros para igualar


la cantidad de decimales: 31,220 - 8,076

• Escribiendo uno bajo el otro: 31,220-


8,076
• Efectuando como si estuviésemos 23,144
restando nümeros naturales: __/
L la coma conserva el
lugar de las demás.
•-$ 1 ALfO",¡S0 ROJ<\S PUÉMAPE 1
* Si se Lralara de decimales inexacLos, operamos con sus fracciones generatrices:
Ejemplos:
(1) EfecLuar: 0,3 + 25 + 1,6'
Solución:
• Vamos a reemplazar los decimales periódicos
puros por sus fracciones generatrices: _/

• Sumando las fracciones de igual denomina~ 3+5+6


9 + 3

.!.! + l.
9 1
• Sumamos ahora la fracción resultante más el enlcru: 41
9
• Si dividimos____________
41 : 9 encontramos el resultado 4ue ;, 9
41 == 4 , 555 •••
buscamos:
Respuesta.- o .3 + 2 . s+ , • e; =1 4. s l

(2) Efectuar: 2 1 , 6 2 • 6 ,36


Suludá11:
• Reemplacemos los decimales periódicos mixtos
por sus fracciones gcneraLrices: - - - - - - - -
l21 + 62 - 6
90
J-l6 + 36 - 3 ]
90

• Suprimimos paréntesis: 21 + 56
90
_ 6 _ 33
90
~ 00 4 1 .. 1
• Agrupando y operando las parles enteras y las ~ 15 + 56 - 33
fracciones: - - - - - - - - - - - - - - 90
15 23 1 373
= -+-=--
1 90 90
• Finahnenle dividimos los Lérminos de la fracción
resullanle para obLener el decimal respecLivo: 1 373 = 15, 2555 . ..
90
Res¡rnesta.- 2) .6 2 _ 6.36 =l 1s.2s\
O RECUERDA QUE: ¡_, ,, RECUERDA QUE:

Al sumar frac~ o.'les poae· JS t>mole.1r la Al suor1mir parentes1s s1 éste esta precedido por el
REGLA DE PRODUCTOS CRUZADOS signo(+} los signos mreriores NO CAMBIAN
S, elpart>ntes1s estáprec0dido del signo{·) los signos
.::.
o
-e -- -
bO
- ,nrerrores CAMBIAN.
1 MATEMATICA
1
* Para efectuar OPERACIONES COMBINADAS de ADICIÓN y SUSTRACCIÓN procedemos
así:
Efectuar:
7,8 - {6,5 + 3,2 + [5,1 - (7,8 + 2,2 - 1,3)]}

Solución:
• Suprimimos el paréntesis más interno: 7,8 - {6,5 + 3,2 + [5,1 - 7,8 - 2,2 + 1,3]) O O
• Suprimimos el corchete: 7,8 - (6,5 + 3,2 + 5,1 - 7,8 - 2,2 + 1,3)
• Suprimimos la llave: 7,8 - 6,5 - 3,2 - 5,1 + 7,8 + 2,2 - 1,3
• Sumamos los positivos O O O
y negativos por separado: ( 7, 8 + 7, 8 + 2, 2) - ( 6, 5 + 3, 2 + 5, 1 + 1, 3)

• Efectuando la SUSTRACCIÓN final:


Respuesta.-
El resultado de operar la expresión dada
17, 8

--------1,7
16, 1

O 1ATENCIÓN!
Poctemos proceder tambien OPERANDO ya el 1ntenor del
es 1,7. ,: PARÉNTESIS, luegoOPERAN00elinteriordelcorcheteypor
/ ·~ ~/ inter~ de la llave

J.:! \ll 'I T IPI.ICACIOI\ \ POTEJ',;CIACH>N DE NÚMEROS DECIMALES.


* Para multiplicar DECIMALES EXACTOS operamos como
si se tratara de números enteros. La cantidad de cifras en la
parle decimal del resultado es la SUMA de la cantidad de
O O ¡IMPORTANTE!
cifras de los factores.

Ejemplo: Al operar con números decima~s-·


se emplean las mismas leyes de

l
Efectuar:
signos que en el CONJUNTO DE
(2,53) X (-3,4)
LOS NÚMEROS ENTEROS ( ;Z)
. .

Solución: En el primer factor hay dos decimales y en el segundo hay un


d('cimal, lo-; que SUMADOS dan tres dec11nalcs l¡ue hay que
:.epar.1r a parur de la derecha en el resultado.

• Multiplicando los signos y los números sin las COMAS


DECIMALES: (+253)(-34) = -8602

• En el resultado separamos TRES decimales (2 +l) a partir


de la derecha: (2,53) (-3,4) =l -8,602 I
Respuesta.
•t"' 1 ALFONSO ROJ\S PliÉMAPE 1
* Para multiplicar DECIMALES INEXACTOS. es preferible
hacerlo operando con sus fracciones generatrices.
Ejemplo:
Efectuar:
(- 2. 4) (+O. 3)

Solución:
• Hall~mos las - 2. 4~ = - [2 + 94] = - 922 Q O
fracciones _J ~
3 1
+ O. 3 = +
generatrices:
9 =+3 O DOS FORMAS. ..

. • • de hallar la generatnz cuando


• Multiplicamos - + O, -3)
(- 2,4)( =(- 22 )( + -
9
1) la parte entera NO ES CERO:
estas
.
_J 22
3
2, ¡ = 2 + º· :r = 2 + ! = 11
generatnces: = - 27 9 ()
o también:
= 1- 0,8141
Respuesta.- - -
( - 2 • 4 )( + O , 3 ) =-O , 8~
14 .

* Para multiplicar POTENCIAS DE BASE DECIMAL opera-


mos como si se tratara ele potencias ele números enteros,
considerando que el resul Lado tiene una cantidad de cifras ele
la parte decimal igual al procluctodemultiplicarelexponente
por la cantidad de cifras de la parte decimal de la base.

Por ejemplo:
o o RECUERDA QUE·
Si efectuamos: (-2,53)3

Tenemos: exponente 3 En POTENCIACIÓN de base entera:


cantidad de cifras (•af' a + B" paran par
ele la parte decimal 2 (-ar ., - él' para n impar.

Emonces: cantidad de cifras ele


la parte decimal en el
resultado: 3 >< 2 , es decir 6.

Luego, operamos sin la coma decimal:

(-253)3= (-253)(-253)(-253) = -16 194 277 e::> O O


A partir ele la derecha separamos 6 cifras que conformarán
la parte decimal.

Finalmente: (-2,53)3 = -16,194 277


1 MATOlATICA
1
* En OPERACIONES COMBINADAS de números decima-
les exactos podemos emplear también POTENCIAS DE O RECUERDA QUE:
BASE 10.
1
Observa lo siguiente: a- n
= -::-,;
a
0 l=-1 =-1 =10-l 00 o también:
, 10 101
1 -n
I 1 - 2 -:ñ = a
o, 01
= 100 = - 2 = 10 a
10 ••••.,... .... •••• ··---❖ ..............-••-.-.

1 I - 3
O~l = 1000 =-3 = 10
10
O O ¡IMPORTANTE!
....__--13 decimales _ ____.
Potencia de Potencia:
-n
O , O{l.. Cll = 10 (a"'_r. a'"'"
En general:
n decimales
Potencia de una Mult,plicación:
Ejemplos: (ax b)" ,. ti' xtt
3 Combmando ambas:
(1) Efectuar: E = (O. 0004)
2
(O, 02)

Solució11:
• 0,0004 se puede escribir así : 4 x 0,0001 • POR DIVISIÓN DE BASES
o también : 4 x l()-4 IGUALES:

• Del mismo modo en 0,02 2 x 0,01 E=24 x 10-s


o también : 2 x 10-2
-4 3
(4 X 10 ) o también:

1
• Reemplazando en E: E= · -2 2
(2 X JO )
-8
• Aplicando POTENCIA DE E= 16x 10
UNA MULTIPLICACIÓN E= O, 000~01~
Y POTENCIA DE POTENCIA: e;. 0 0
8 decimales
3 - 12 2 3 - 12
E = 4 x IO (2 ) x 10
2 -4 2 -4 Respuesta.-
2xIO 2xl0
6 - 12
E= 2 X 10 La expresión E es igual a 16 cienmi-
2 -4 llonésimos.
2 X JO
~-$:· 1 ALFONSO ROJAS PUÉMAPE 1
.U Dl\'ISIO!\ DF Nl i\lEROS DECli\lALES.
1

* Para esto multiplicamos el DIVIDENDO y DIVISOR por la


unidad seguida de tantos ceros como sea posible transformar los
números decimales en enteros.
Ejemplos:
¡ATENCIÓN!
(1) Efectuar 13,5 : 7
Para dw1d11 numeros de•
Multiplicamos ambos cima/es peflodicos se
términos por 10 -+ 135: 70 ¡división de enteros! opera con sus FRAC-
(2) Efectuar 68: 1,42 CIONES GENERA TRI-
Multiplicamos ambos CES.
términos por 100 -+ 6 800: 142 ¡división de enteros! , --- ----~--

Para efectuar una división de enteros procedemos como en el siguiente ejemplo:


16 l1_ 16 L2._ 16 Ll_
15 3 15 3, 15 3.2
""T 71'f" -ro
10
m
Entonces el ejemplo (1) se efectúa del siguiente modo:

13,5 : 7 6 135 : 70 , es decir: 135 l.2Q_ _,, O ¡CUIDADO!


--2Q_ 1,92
650 -1 Sólo tienen raíz cuadrada los numeros
630 decimales positivos No as, los números
200 decimales negativos.
140 \.. _
60
Respuesta: 13,5: 7 = 1,92

.' 4 IUDICACION DE NÚi\lFROS DECli\lALES.


Sólo nos referiremos a la raíz cuadmda e::> O
Veamos un ejemplo: Hallar la raíz cuadrada de 123, 723

2 3, 7 2 3 o 11,12 Hemos procedido como en radicación de


1 números enteros. pero debemos observar
,,,. #j X 2 = 2
o 2 3
., ., ,,,. 2 []x [D= 21
lo siguiente:
2 1 ., ,,,. • Al separar el número dado en grupos de
2 7 2 11 X 2 = 22 dos cifras Jo hacemos a partir de la coma
2 2 1
__5_1_
22 [D X [D= 221 decimal (como lo indican las flechitas).
3 o • La parte decimal debe tener completo
111 X 2 222 cada grupo de dos cifrac;, si faltan com-
4 4 4 4
6 8 6
222 g] X g] = 4444 pletamos con ceros a la derecha.
• Cuando bajamos el primer grupo de
cifras decimales, se coloca la COMA en
la raíz.
1 MATE!\lA TICA
1
Sh. \ '\ 1PR \CTIC A G)
l. Efectuar las siguientes ADICIONES y SUSTRACCIONES de números decimales:
(1) 5.7 - 3,2 (11) 5.6 + 3,92-1,12 (21) 7,13 -5,Ó8 + 6,2
(2) 10.2 + 5,3 (12) 8,72 + 6,373- 1,8 (22) 3,52 +3,67- 1,1
(3) 12.25 + 27,7 (13) 6,13- (5,08 + 0,12) (23) 6.2 + (3. 1 - 12.8¡ + 5.6 l
(4) 18,09- 3,01 (14) 3,2 + 5,3 (24) {3,9 + (5,6 - 3.08) + 6, 1)
(5) 16.03 - 3,2 (15) 7,15 + 3.08 - 2,153 (25) 5,2 - (6,9 + (17,3 - 12,9)]
(6) 8,3 - 5,27 (16) 3,16 + 6,15 - 2,2 (26) -6,9 - {5,7 + (8,9 - 6,3))
(7) 16,35 - 9,658 (17) 2,18 - 1,13 + 5,2 (27) 8,4 + {6,2 + (5,7 + 2, 1 - 3,2))
(8) 176,3 - 98,56 (18) 7.8 - 4,2 (28) 3,7 - (5.1 - (6,3 + 3,6)]
(9) 39,256 - 17,17 (19) 7,6 + 5.7 - 0,6 (29) 2,3 - (5,4 + 6,8 - 32,03)
(10) 45,111 + 12.09 (20) 5,32 + 6,68 (30) 6,9 + (5.2 - [3, 1 - (6,3 - 8,2l])

11. Efectuar las siguientes MULTIPLICACIONES y POTENCIAS DE NÚMEROS DE-


CIMALES.
(1) (-3,5)(+2,7) (5) (-3,1)2 X (1 ,7)2 (9) (6,3)(2.5)
(2) (-5. 13)(-6.05) (6) (1,2)(-2.5)(7,2) (10) (-2.7)(0,6)(-2.2)
(3) (-6,2)2 X (1,1) (7) (-2.8)(-5,9)(1,1) (11) (2,34)(5.3)
(4) {+4.3)2 X (1,2)2 (8) (-6,8)2(3,7) (12) (3,6)(0,2)(5,34)

111. Efectuar las siguiemes operaciones de DIVISIÓN y RADICACIÓN:


(1) 7,2 : 3.1 (4) 6.32 : 5.3 (7) 27,36 : 2,42 (10) ~ (13) -v' 1632 , 576
(2) 5.7 ·0.2 (5) 8,56 · 5,8 (8) 48,5 : 3,63 (11) ~ (14) "\/'3865 ,865
(3) 6.5 : 1,7 (6) 9,16 : 2,12 (9)~ (12) "\/'138 ,415 (15) "\/'6715 , 17

IV. Efectuar las siguientes operaciones combinadas:


(1) 7,2 : 2, 1 + (2,3) 2 -6.3 X 5,1 (4) 3, 7 - [8,6 +(5,2 - 3,4)] - (2,3)2
(2) 8.2 - {6, 1 - (2.5 + 5,2) - (1. 1)2) (5) (5,4)(-2,3) + 1(3,1)2 - (6,2 : 3, 1 + 5,2))
(3) 7,4 + {2.5 + (7.2 - 8,3)1' (6) y'i:, + (5,3) 2
- [6,8 : 3,2 - (7, 1 + 6,2)](3,6)

,·. Empicando POTENCIAS DE DIEZ efectuar las siguicmes operaciones combinadas.


4 5 5 8 2 2 4
(O. 3¡ x (O. 002 ) (0.81) (O, 01) (O. 2) (O . 03) x (O, 00005 )
(1) 2 3 (3) (5)
3 2
(0.9) x (O 0016) (O , 006) (O, 15) (O, 01)

3 7 2 7 3 e
(O. 00004 ) , (O, 001 ) (O. 04 ) (O. 0000001 ) (0,05) x (O, 008 )
(2) (4) (6)
2 3 2 2 2
(O, 0000002 ) (O. 2) (O. 01) (O, 5 ) x (O. 2)
1 ALFONSO ROJAS PUÉMAPE 1 IWJ@UI
Aquí mis operaciones de SKANIPRÁCTICA Q)
1 MATE\1ATICA
1

1 RAZONEl.YIOS
lnstruccinnes:
A continuacion te presento bloques de ejercicios para afianzar
lo leido hastaaquz. Marca la respuesta correcta y compárala
con la CLAVE DE RESPUESTAS que aparece al final; sin
embargo, te recomiendo que expliques en tu cuaderno las
RAZONES de tu resultado. No tomes este trabajo como una
pesada carga, sino como un DESAFÍO a tu inteligencia.

BLOQUE I ♦
(7) Un terreno de 650,34 m2 se reparte entre 3 hijos,
en partes iguales. Si cada hijo a su vez tiene
(1) Un padredefamiliadebecomprar 5 cuadernos ♦ 2 hijos a los cuales deja su parte, ¿cuántos m 2 le
para sus hijos. Si tiene SI. 20 y además compra corresponde a cada uno de estos últimos?
5 lapiceros J. SI. l ,50 cada uno, ¿cuánto costará ♦
cada cuaderno que compre? ♦ (8) Cuatro amigos aportan por igual para comprar
un auto de $ 4 630. Si luego lo venden en
(2) Una frutera vcnde46 manzanas a SI. 1,80 cada ♦ $ 6 550, ¿cuál fue la ganancia de cada uno?
una. Si se le malogran 9 manzanas y vende l,1s
restantes a SI. 2,50 cada una, ¿cuál será su ♦
(9) En un zoológico corresponde dar a cada león
ganancia? ♦ 25,6 kg de carne industrial y a cada cocodrilo
20,2kg. Si hay 4 leones y 6cocodrilos, ¿cuántos
(3) Un carpintero gastó SI. 145,20 en comprar ♦ kg de carne corresponde a cada uno de los
madera. Construyó 3 sillas y 2 mesas. Cada 3 tigres que hay, si la Administración dispone
silla la vendió en SI. 32,50 y cada mesa en ♦ de 280 kg de carne diarios?
SI. 55,70. ¿Cuál fue su ganancia? ♦
(10) Jorge pesa 73,5 kg y Rosa pesa 52 kg. Dia-
(4) Abel tiene SI. 4,50 más que Beto; Beto SI. 1,25 ♦ riamente Jorge y Rosa consumen entre ambos
menos que Carlos. Si Carlos tiene SI. 24, 15; 5 kg en alimentos en partes iguales. Jorge eli-

¿cuánto tienen entre Abel y Beto? mina 2,5 kg y Rosa 2 kg. ¿Después de cuántos
♦ días Rosa igualará el peso de Jorge?
(5) Un alambre de 24 m se di vide en 5 partes
iguales. A las 3 primeras partes se les suelda en ♦ (11) Una frutera tiene 12 manzanas que vende a
un extremo un pedazo de alambre de 0,75 m y ♦ SI. 1,80 cada una; 15 peras a SI. 1,30 cada una
a las otras 2 partes se les suelda un pedazo de y 20 naranjas a SI. 0,80 cada una. Cada
0,25 m. Si se unen todas las partes, ¿cu{into ♦ comprador lleva un número igual de frutas de
medirá todo? cada tipo. Si ya vendió todas las manzanas,

¿cuánto obtendrá del resto de las frutas que le
(6) Un depósito tiene 16,5 l. de capacidad; otro ♦ quedan?
depósito tiene 22, 7 l. de capacidad y otro depó-
sito tiene 10,21.decapacidad. Un depósito ma- ♦ (12) El Sr. García debía SI. 63.20; SI. 180,50 y
yor que los 3, llena los 2 primeros y sólo la SI. 234,40. Adem..í.s le debían a él SI. 264,50 y

mitad del tercero. ¿Qué capacidad tiene el SI. 300,50. ¿Cuál es entonces el estado finan-
dcpúsito mayor? ♦ ciero del Sr. García?
,t> 1 ALFONSO ROJAS JIUÉMAPE
> 1 111❖@41
(13) Un corredor de bolsa compra 500 acciones a ♦ Si el kg de cada uno cuesta SI. 1,20; SI. 1,40;
SI. 2,75 cada una. Al día siguiente de la compra SI. 2,30 respectivamente y la empleada gastó

estas acciones subieron SI. 0,75. En los 2 días en total SI. 29,40; ¿cuántos kg de cada clase
siguientes bajaron a SI. 0,30 por día; el si- ♦ compró?
guiente día subieron hasta SI. 6,25 cada una.
♦ (23) Un Sr. pesa 72,5 kg. Él y su esposa pesan juntos
¿Cuál fue el cambio neto de estas acciones?
♦ 124 kg y junto con su hijo, los 3 pesan 156,5 kg
(14) AmaldoledebeaMatíasSl.13,80ydiariamente ¿Cuánto pesan la esposa y el hijo?
le paga SI. 2,30. ¿Cuántos días transcurrieron ♦
hasta que, sin darse cuenta ambos, Matías le ♦ (24) Un librero compró 6 docenas de libros a
debe ahora a Amaldo SI. 6,90? SI. 3,50 cada uno. Los vende recibiendo
♦ SI. 192. ¿Cuánto ganó en cada libro?
(15) El Sr. Pérez tiene en su cuenta, un saldo de
♦ (25) Entre Julio y César tienen juntos SI. 36,80. Si
SI. 237,50. Durante el fin de semana hace
gastos con su tarjeta de crédito. El lunes el ♦ Julio tiene Sl.4,20 más que César, ¿cuánto tiene
Banco le avisa que se ha sobregirado en cada uno?

SI. 135,20. ¿Cuáles han sido los gastos del Sr.
Pérez el fin de semana? ♦ (26) Lucho le debe SI. 42,20 a Carlos y Carlos le
debe SI. 16,70 a Lucho. ¿Cuánto le debe real-
(16) En la alcancía de un niño habían 12 monedas de ♦ mente Lucho a Carlos?
SI. 0,20 y algunas de SI. 0,50. Si en total el ♦
niño contó SI. 18, ¿cuántas monedas eran de (27) Un alumno rinde un examen de 20 preguntas;
SI. 0,50? ♦ por cada respuesta correcta obtiene I punto y

por cada respuesta equivocada pierde 0,25
(17) Dos amigos juegan a los dados con la condición puntos. Si contestó correctamente 12 preguntas
que el ganador de cada juego pagará SI. 4,50 al ♦ y las otras respuestas fueron equivocadas,¿ qué
otro. Si el primero ganó 6 veces y el segundo
ganó 4 veces.¿ Cuánto más ha ganado el primero
que el segundo?
• puntaje obtuvo?

♦ (28) Si un palmo equivale a 22,3 cm, ¿cuál será el


perímetro de un cuadrado en cm, si el lado

(18) Un globo aerostático asciende 125,30 m; luego mide 18 palmos?
desciende 2 veces 32,20 m y luego vuelve a ♦
ascender 52, 1Om. ¿A qué altura se encuentra? (29) En un depósito hay 22,4 t de aceite y en otro
♦ hay la mitad del primero. ¿Cuántos litros se
(19) Un niño mide 1,25 m y una niña mide O, 13 m ♦ deben sacar del segundo, para que quede la
menos. Si cada año ambos crecen 0,02 m, cuarta parte del primero?
¿cuánto medirá cada uno luego de 3 años? ♦
(30) En un teatro hay tantos hombres como mujeres,

(20) Hugo, Paco y Luis van de pesca y convienen y tantas mujeres como la mitad del número de
que Luis, por ser dueño del bote, recibiría la ♦ niños. Si cada niño paga SI. 8,50 y cada adulto
mitad de la pesca total. Hugo y Paco se repar- ♦ SI. 12,50 y además se sabe que asistieron 42
tirían por igual el resto. Si pescaron en total niños, ¿cuál fue la recaudación total'!
134,8 kg, ¿cuánto correspondió a cada uno? ♦
(31) Jorge pesa 76,5 kg y Rosa pesa 52,5 kg. Raúl

(21) Un agricultor cosecha 45,2 kg de tubérculos, pesa tanto como la mitad de lo que pesan Jorge
por cada m2 de su chacra. Si su chacra tiene un ♦ y Rosa, menos 1,5 kg. ¿Cuánto pesa Raúl?
área de 165,5 m 2, ¿cuántos kg de tubérculos
♦ (32) El mayor de 4 hermanos pesa 50,2 kg y 2,5 kg
cosechará?
♦ más que el siguiente, que a su vez pesa 1,5 kg
(22) Una empleada ha comprado en una tienda, el más que el siguiente. Si los 4 juntos pesan
mismo número de kg de arroz, azúcar y frijoles . ♦ 184.8 kg; ¿cuánto pesa el menor?
1 MATEMATICA

BLOQUEil
O ¡MUY IMPORTANTE/
(1) ¿Cuántos números de la fonna O,ab
cumplen con la siguiente condición: 27 = 10x2+ 7
58 = 10x5+8
o,ati' + o,@ = o,'3' ? 76 = 10x7+6
a) 6 b)4 c) 3 d) 5 e) 1

Solución:
ab = 10a + b
Halla111os ¡,nmero todas La.1· genera/1 ice., . L11q,:o

l
reclucimo.r las e/os fraccionn ele la izquierda a una Desco:pos,c.Jn
sola, c'.1·c1 ihienclo el 111í111ero ah co1110 JOa + h. polmom1ca del
1c::'o númeroab
S1111pl1/1cwulo halla111os 1111 valor pwa a+ h. La tJrra sobre ab
ind..;a que se trata
de un número de 2
cifras.
• Escribiendo las generatrices ab ba 3
-+-=- ó
de todos los decimales: 99 99 9
ab+ba 3
---
99 9
• Descomponiendo los números lOa+b+IOb+a 3 ó l la+ l lb = 3
con barra: 99 9 99 9
11( a+ b) 3 a+ b 3
--'----'-=- ó - - = - ➔ a+b=3
99 9 9 9

• Como a y b representan una Si: a= O, b "' es 0,03


3 luego O,ab "'
sola cifra cada una, entonces "'
Si: a= 1 ' b = 2 luego O,ab es 0,12
,....__
,....__ re---,
tenemos las siguientes Si: a= 2, b = 1 luego O,ab es 0,21
posibilidades: ,....__ ,....__
Si: a= 3, b = o luego O,ab es 0,30

Respuesta.-
,....__
Los números 0,03; O, ,....__
12; 0,2re---, ,....__
l; 0,30 cumplen con la cond.1c10n 'b'
. ' d ad a. Rpta.\!!./

(2) Calcular el valor de E si:

E= [ ✓0,3211 l... - ✓0,24999 ... r


a) 2 b)l7 c)lS d) 1/15 e) I
,,0. 1 ALFONSO ROJAS PUÉMAPE 1
Solución:
S1 nu JJodemos extraer la rafz uwd1acla de ntíme,ol O ES MUY IMPORTANTE...
decimale.~ periodicus mixtos. entonces hallamos sm recordar lo siguiente.
generatrices, luegu e).{raemos las ,afees ele é.1U1s \' las
restamos. Finalmente i11ve1 timos el re.111lraclo ub1e11ido
al intt'I iordel corchete, \'ll que esta elemdoal exJJonellfe
-1 . e:' O
ª = ( l)
"EL EXPONENTE NE-
GATIVO INVIERTE A LA
• Reemplazamos los de-
BASE"
cimales por sus genera-
trices: E= [ ✓ 321-32 - ✓ 249-24
900 900
]-i
E=[ {289 - {ns ]-1
l900 ~wo
• Extraemos raíz cua-
drada: E= [.!2_-~]-1
30 30

• Restando el interior del 1 1


2 ]- [ l ]-
corchete: E= [ 30 ó E= 15

• Invertimos la base de -1: 1E = 151 Rpta.@


Qo
(3) Calcular a+ b si se sabe que: ♦ (6) Calcular el valor de p si se cumple que:
12

n p
O,ab = - 0,5m =
25 9

a) 10 b) 8 c) 12 a) 1 b) 2 e) 3
♦ d)4 e) 5
d) 9 e) 10

(4) ¿Cuál es el valor de b - a si se cumple que: (7) Hallar d si se sabe que:

n 1 n d
O,ab = - ? ♦ 0,2c =
15 11

a) 2 b) 5 c)3 a) 1 b) 5 e) 9
d) 6 e)4 ♦ d) 3 e)7
♦ n
(5) Hallar a sabiendo que: (8) Hallar el número O,ab sabiendo que a excede
♦ en 6 a b y que además:
n 9 2 n n n
a,8 a=---
♦ O,ab + O,ba "' 0,8
2 3 n n n
♦ a) 0,71 b) 0,17 c) 0,77
a) l b) 3 c) 2 n n
d) 5 e) 4 ♦ d) 0,7 e) 0,07
1 !\I.\TEMATICA
1 ,t,
(9) Hallar la fracción generatriz equivalente a: ♦ (1 7) Efectuar:

a)l4l/19
º· 15 + 2,33333 ...
b) 143/60 c) 149/60
♦ [✓1.36111... + ✓0.69444... r
♦ a) 1 b) 4 c) 8
d)lll/17 e)N.A.
♦ d) 16 e) 81
(10) ¿A qué es igual: (5,777 ...)- (0,777 ... )? ♦ (18) ¿Qué fracción deberíamos aumentara0,0733 3...
a} 6 b}7 c) 5 ♦ para que sea igual a la unidad?
d) 2 e) 9

(11) ¿Cuál es la fracción generatriz equivalente a ♦


• a) 119/150 b) 139/150
d) 17/150
c) 101/150
e) 13/150

º· 12 + 0,333 ... + 0,58222... ? ♦ (19) ¿Cuántodeberíamosdisminuira ✓ 2.24999 ...


a) 113/225 b) 233/225 c) 17/250
e) 1/900 ♦ para obtener la unidad?
d) 1/990
♦ a) 1/7 b) 0,5 c) 2/3
(12) Efectuar: d) 0.2 e) 1/3

(15, 15626262...) -(O, 15626262...)
♦ (20) ¿En cuántos ochenta-avos es mayor 0,32 que
a) 13 b) 10 c) 5 0,1325?
d) 15 e) O ♦
a) 12 b) 11 c) 18
♦ d) 80 e) 15
(13) Calcular la raíz cuadrada de E si:

E = (99,777 ...) + (0,2222... ) (21) ¿Cuántas veces 7 es mayor que 0,002?

a) 3499 b) 3 500 c) 3 702
a)7 b) 8 c) 6
♦ d) 3 498 e) 3 501
d) 9 e) 10


r
(22) Calcular el valor de:
( 14) Efectuar: r-, r-, r-,
♦ (0,2 + 0,3 + ... + 0,7] : 0,16 '
[ 5.5121212... -0.5121212 ... ♦ a) 15 b) 1 c) 6
98,222 ... +I, 777 ... d) 18 e) 8

a) 20 b) 25 c) 30 ♦ (23) ¿Cuántas cifras tiene el período de 7/11 ?


d) 50 e) 1/20 ♦ a) 2 b) 3 c) 4
d) 5 e) 6
(15) Efectuar: ♦

♦ (24) ¿Cuál es el número decimal que dividido entre


924,3555 ... -24,3555...
su inversa da como resultado 0,4011 ...?
97,666... +2,333 ... ♦
a)0,5666 ... b) 5 c) 1
♦ d) 0,2333 ... e) 0,6333 ...
a) 6 b) 5 c) 4
d) 10 e) 3 ♦
(25) Efectuar:

( 16) Calcular: 0,88888 0,66666
✓o.28444 ... •

0,22222 0, 22222
a) 1,6777 ... b) 3,3555 ... c) 2,3555 ... a) o,í' b) 1 c) 3
d) 1,5333 ... e) 0,5333 ... ♦ d) 2 e) 5
1 ALFONSO ROJAS PUEMAPE 1
(26) Efectuar: ♦
demás tenga que abonarun adicional de SI. 9,6.
¿Cuál era la deuda total a pagar?
666,66 5,5555 ♦
---+--- a) SI. 512 b) SI. 480 c) SI. 532
888,88 4,4444 ♦ d) SI. 534 e) S/. 460
0,44444 0,22222

0,33333 0,66666 ♦ (6) Dieciséis personas tienen que pagar por partes
iguales SI. 7 500;comoalgunasdeellasnopue-
a) 5 b) 3 c) 6
♦ den hacerlo, cada una de las restantes tienen
,d) 2 e) l
que poner S/. 281,25 más, para cancelar la deu-
♦ da. ¿Cuántas personas no pagaron'!
♦ (EXAMENDEADMISIÓN-UNIVERSIDAD
BLOQUEIII
♦ NACIONAL DE INGENIERÍA)
(l) Calcular la expresión equivalente a:

[
n n 3 n 0,3
1,6
0,16-0,6+----¡:,:¡ ----¡:,:¡-~
][
n] ♦


a) 8
d) 3
b) 4
e) 6
c) 9

2.6 0,3 1,6 Un librero adquirió 780 libros a S/. 4,48 cada
♦ (7)
a) l b) 5 c) 2 uno, recibiendo además en calidad de obsequio,
♦ un libro por cada docena que compró.
d) 3 e) 4
♦ Determinar el precio al que debe vender cada
(2) Una señora ha comprado en la tienda igual ♦ ejemplar para obtener una utilidad de
número de kilogramos de azúcar a S/. 1,2; de S/. I 170,40 después de obsequiar una docena
frijoles a S/. 2,4 y de arroz a S/. 1,6. Si pagó un ♦ de libros.
total de SI. 124,8, ¿cuántos kilogramos compró ♦ a) SI. 5,4 b) SI. 4,8 c) SI. 5,6
de cada producto? d) SI. 6,2 e) S/. 7,3

a)20kg b)21kg c)24kg
d)25kg e)23kg ♦ (8) Se paga S/. 102 por la compra de 2,5 docenas de
tarros de una conserva. ¿Cuánto deberíamos

(3) A las 8 a.m. sale un auto de A a 58,5 km/h y va pagar por un ciento?
al encuentro de otro que sale de B a 62,3 km/h ♦ a) S/. 320 b) S/. 310 c) SI. 360
a la misma hora. Sabiendo que se encuentran a d) SI. 340 e) S/. 380
las 12 m., ¿cuál es la distancia entre A y B?

a) 512,5 km b) 480 km c) 234 km (9) Un lechero vende el Ji tro de leche pura a S/. 3,5
d) 483,2 km e) 249,2 km ♦ y agrega agua hasta que 117 litros de la mezcla

tengan un valor de SI. 350. ¿Qué cantidad de
(4) Parte de A, un auto a las 7 a.m. a 45,8 km/h al agua agregó?
encuentro de otro que sale de B (en el sentido ♦
a) ¡5¿ b) 18[ c) 22t
opuesto) a las 9h 30 min a 51,25 km/h. Si se ♦
encuentran en C a las 11 a.m., ¿qué distancia d)34l e)l7l
hay entre A y B? ♦
♦ (10) En la misma obra trabajan dos obreros. El
a) 117,25 km b) 210,5 km c) 245,25 km primero gana S/. 0,8 más que el segundo por día
d) 250 km e) 260,075 km de trabajo, y recibe por 35 días SI. 48,8 menos

que lo que recibe el segundo por 37. Calcular la
(5) 16 personas acordaron pagar en partes iguales ♦ suma de los jornales de ambos.
una deuda; pero resulta que 6 de ellos sólo ♦
pueden pagar la mitad de lo que les corresponde, a) SI. 57,5 b) SI. 77,6 c) SI. 39,2
obligando de este modo a que cada uno de los ♦ d) SI. 38,4 e) SI. 52,6
1 l\lATEI\L\TICA
1
(11) Efectuar: ♦ (17) Si al peso de un objeto se le multiplica por 3: al
r'\ r'\
resultado se le aumenta 14,94 kg y luego se
3,6-0,3 ♦ extrae laraízcuadradaaese resultado, se obtiene
0,3 ♦ 7,2. ¿Cuál es el peso de dicho objeto?
♦ a) 10,6 kg. b) 9,6 kg. c) 8,7 kg.
3 10 1 d) 12,3 kg. e) 14,5 kg.
a) - b) - c) -

10 3 3
♦ (18) En cierto lugar 5 lopis equivalen a 3,2 soros:
1 2 2,4 soros equivalen a 15,2 antos y 3,8 antos
d) - e) -
6 3 ♦ equivalen a¿cuántos lopis?
♦ a) 0,9357 b) 0,8736 c) 0,9264
(12) Un ferrocarril conduce 150 pasajeros en primern
y segunda clase. Los primeros pagan 1,5 dólares
y los últimos I dólar. Si la recaudación total fue


d) 0,9372 e) 0,9375

(19) Para pagar el consumo de agua de un edificio,


187 dólares, ¿cuántos viajaron en segunda? el administrador pensó cobrar a cada inqui lino
♦ SI. 15,20 pero le faltaría SI. 31,10. Entonces
a) 48 b) 76 c) 64
d) 102 e) Ninguna ♦ decidió cobrar SI. 17, 15 a cada uno y de esta
manera le sobra SI. 2,05; ¿cuántos son los

(13) Un alumno obtuvo una nota de 71,25 puntos en inquilinos?
un examen. Si contestó 140 preguntas y el ♦ a) 15 b) 16 c) 17
puntaje por respuesta correcta es I punto y por
respuesta equivocada es -0,25 puntos, ¿cuán-
ta~ respuestas correctas obtuvo?

♦ (20) Efectuar:
d) 18 e) 19

a) 95
d) 73
b) 75
e) 60
c) 85


✓ ,. . r\ r'\
O,l+0,2+0,3+ .... +0,8
n

1
(14) Un comerciante compró 2 tipos de tela, por ♦ a) 4 b) 2 c) -
SI. 1 260. Vendió 18 m por SI. 251.10. ga- 2
nando SI. 1,95 por metro. Calcular la longitud ♦ d)OJ e) o/4'
de cada pieza, si una de ellas tiene el doble de ♦
longitud de la otra CL.\YE DE RESPUEST.\ S

a)70y60 b}40y80 c) lOy 100 RAZO:\F:\IOS
d)35y70 e)20y90 ♦

♦ BLOQUEII
(15) ¿Cuántas cifras tiene el período de la frac-
(1) b (7) d (12) d (17) d (22)d

1 (2)c (8) a (13)e (18) b (23)a
ción - ? (14) a
♦ (3)c (9) e (19) b (24)e
11
(4) a (10) e (15)e (20)e (25)b
♦ (21) a
a) 1 b) 2 c) 3 (5) b (11) b (16)e (26)d
d) 4 e) 5 ♦ (6)e

♦ BLOQUE III
(16) Una frutera razonaba de la siguiente manera: si
vendo a SI O, 70 cada manz,rna, gano ♦ (l)d (5) a (9)e (13) e (17)d
SI. 1.20; pero si las vendo a SI. 0,50 perdería ♦ (2)c (6)e (10) b (14) d (18)e
SI. 0,60. ¿Cuántas manzanas tiene la frutera? (3)d (?)e (11) b (15) b (19)c
♦ (4) e (8) d (12)e (16) a (20)b
,l) 9 b) 8 c) lO
d) 7 e) 6 ♦
1 ALFONSO ROJAS PUEMAPE 1

LAS l\IATEI\IÁTICAS DE TODOS LOS DÍAS


(l) Una frutera compra la docena de manzanas en SI. 10,20 y vende cada una en SI. 1, 10.
¿Cuántas deberá vender si quiere una ganancia de SI. 16,50? ¿Cuánto gana en cada
manzana?

Solución:

Respuestas

a)I
1
b)I
1
(2) En un salón de clase, se pregunta por la cantidad de dinero que tenía cada alumno, en
ese momento. Cinco de ellos tenían SI. 3,50; doce tenían SI. 2,20; tres tenían menos de
SI. 1,50 y dos tenían más de SI. 5. Se quiere saber:
a) ¿Cuántos tenían más de SI. 3?
b) ¿Cuántos tenían menos de SI. 2,20?
c) Si 4 de ellos tienen diferentes sumas de dinero y en total tienen SI. 12,70; ¿cuánto
tendrán 2 de ellos, si se sabe que son números enteros?

Solución:

Respuestas

a)I
1
b) 1
1
c) 1
1
1 MATEMÁTICA
1
(3) En un ómnibus interprovincial viajan 54 pasajeros, desde Lima aArequipa unos, y hasta
Tac na otros. El pasaje hasta Tacna costó S/. 28,50 y hasta Arequipa S/. 19,50. Si en total
se recaudó S/. 1 368; se quiere saber:
a) ¿Cuántos viajaron hasta Arequipa?
b) ¿Cuántos viajaron hasta Tacna?

Soluci6n:

Respuestas

(4) En un colegio de la capital, los alumnos de 1°"D" quieren hacerle un regalo al profesor
del curso de Matemática, por el "Día del Maestro". Si cada uno da S/. 1,80 faltarían
SI. 14,40 para comprar el regalo, motivo por el cual acuerdan dar S/. 2,40 y de esta
manera se compra el regalo y todavía les queda S/. 4,80.
a) ¿Cuántos alumnos eran?
b) ¿Cuánto costaba el regalo?

Solución:

Respuestas
ALFO'.\SO ROJAS PUEMAPE

RAZONAMIENTO MATEMÁTICO

(1) Si se sabe que O.~ + O.~ = 1,Í: (8) ¿Cuántas veces 1 es mayor que 0,001?
Hallar el valor de a + b.
a) 10 b)99 c)999
a) 5 b) 11 c) 21 d) 100 e) 1 000
d)l9 e) 18
(9) ¿Cuántos cuadriláteros hay en la siguiente
(2) Siendo ~ una fracción irreductible, si se figura?
n
le disminuye en sus 1 resulta .!. :
. 7 7
¿Cuál es el valor den - m?
a) 1 b) 6 c) 7
d)3 e) 10

(3) Un número decimal disminuido en 0,7 da


O, 7. Calcular la suma de las cifras de su a) 19 b) 15 c) 7
parte decimal. d)4 e) 8
a) 5 b) 6 c)4
d) 8 e) 3
(10) ¿Cuántos triángulos hay en la siguiente
figura?
(4) Para que el número decimal 0,565656....
sea igual a 2,333 . ... le falta:
a) 1,777 ... . b) 1,666... .
c) 1,676767... . d) 1,333 ... .
e) 1,767676.. . .

(5) ¿Cuánto le falta a 5n para ser igual al


cociente de dividir 5n entre 7/8? a) 17 b) 23 • c) 21
d)29 e) 6
a) 1/49 b) 2/49
c)NADA d) 5/49
(11) ¿Qué número continúa en la siguiente
e) Es mayor que dicho cociente
serie:
(6) Efectuar:
0,15 ; 3/10; 0,45 ; _ ?

[..Jo. 69444. .. + v-o-.2-11-1-1.-.. ] 2


a) 0,06
d) 1/6
b) 6/100 c) 0,6
e) No se sabe
a) 1,71 b) 1,86 c) 1,69
(12) ¿Cuál es el número por el que hay que
d) 1,96 e) 0,69
multiplicar la fracción 0,75/0,05 para que
el producto sea igual a 1?
(7) ¿En cuántos veinticincoavosesmayor0,356
que0,076'!
a) 6 b) 0,06'
c)8 d)15
a) 8 b) 7 c) 5
e) NO EXISTE TAL NÚMERO
d)4 e) 6
MATEMATICA

CAPÍTULO 6

ECUACIONES E
di
INECUACIONES 1

t
1
l 1

DE PRIMER GRADO
)

OBJETIVOS:

• Efectuar una introducción al estudio de las ecuaciones e


inecuaciones de primergrado con una incógnita con coeficientes en
rD, aprovechando el reciente estudio del Conjunto de Números
Racionales
1 AlFOi'\SO ROJAS PUEMAPE 1

ESTRUCTURA DEL CAPÍTULO:

I. INTRODUCCION AL ÁLGEBRA
1.1 Expresión Algebraica
1.2 Término Algebraico
1.3 Elementos de un Término Algebraico
1.4 Términos Semejantes
1.5 Reducción de Términos Semejantes. Supresión de signos de colección.
1.6 Polinomio en <D.. Notación.
SKANIPRÁCI1CA 1

2. ECUACIONES DE PRIMER GRADO CON UNA INCOGNITA


2.1 ¿Qué es una Ecuación?
2.2 Ecuaciones Equivalentes
2.3 Propiedades de las ecuaciones
2.4 Procedimiento práctico de resolución de una ecuación.
SKANIPRÁCI1CA 2

3. SISTEMAS DE ECUACIONES
3.1 Método de Reducción.
SKANIPRÁCTICA 3

4. INECUACIONES DE PRIMER GRADO CON UNA INCÓGNITA


4.1 Desigualdad
4.2 Tipos de desigualdad. Inecuación.
4.3 Propiedades de la desigualdad
4.4 Inecuaciones de ter grado y su resolución
SKANIPRÁCI1CA 4
RAZONAMIENTO MATEMÁTICO
1 MATEMATICA
t
( 1. INTRODUCCIÓN AL ÁLGEBRA )

Bueno eso es verdad, ya


que el que no sabe es
como el que no ve . ¡Se
1 • • • •• •

Empezaremos diciendo que:


ÁLGEBRA.es una rama de la matemática.que estudia a
la cantidad del modo más general posible y a las
operaciones que con ella se realiza en los diferentes O j IMPORTANTE '
conjuntos numéricos. La expresión
Al estudiar a la cantidad del modo más general posible, 3(3)2 +2 = 29
el ÁLGEBRA emplea CONSTANTES y VARIABLES.
puede ser expresada de modo
CONSTANTE.- Símbolo que admite un solo valor más general.así:
conocido o ya definido, por ejemplo 3;
3Jf+2=y
-7
de manera que:
VARIABLE.- Símbolo que admite cualquier valor,
dependiendo de la expresión de la que Si x = 3 entonces y= 29
forme parte. Estos símbolos son por lo Si x = 4 entonces y= 50
Si x = 5 entonces y= 77
general las últimas letras del alfabeto
( X, y, Z, .... etc.) QQ
1 ALFO:\"SO ROJAS PUÉMAPE 1 11FJ@a;fl
1. 1 EXPRESIÓ'.111 ALGEBRAICA (E.A.)
Es un conjunto finito de constantes y variables (números y letras) con exponentes racionales
y fijos, relacionados por las operaciones de adición, sustracción, multiplicación, división,
potenciación y radicación.

* Es un conjunto finito porque las constantes y variables se


pueden enumerar hasta la última. O RECUERDA QUE:
Ejemplos: Si un número puede ser escrito en
la forma de una fracctón,entonces
(1) 4x - 3x y +Vx Si es E.A.
4 2
tal numero es racional.
(2) 1 + 2x + 3x3 + ... No es E.A.
Ejemplos:
* Los exponentes en una E.A. deben ser números racionales.
c::;>0 2 . - 5 ; 213 ; 0,5 etc

Ejemplos: S1 no es así.tal numero es trracional


Ejemplos·
(1) 2x 8 - 3x 112 - 1 Si es E.A
(2) x-.Í2. + 5y.Ji No es E.A
'Y2•'Y3;Y 5:V7:etc.
* Los exponentes deben ser fijos. No pueden ser variables
Ejemplos:
(]) 6x 4 - 5y6 Si es E.A.
(2) 2• + 3' No es E.A.

1.2 TÉ R\111';0 \LGEUR \ ICO


Es una expresión algebraica cuyas bases NO están
relacionadas por las operaciones de Adición o Sustracción
Ejemplos:
(1) 6x4y 5 , las bases no están relacionadas por adición o
sustracción alguna, entonces se trata de un término
00 ¡ REGLA PRACTICA !
algebraico.
Los signos + ó - separan un
(2) [ 2x8 + y5 ] , las bases x e y están relacionadas por la
término de otro.
adición, entonces toda la expresión dentro del corchete
no es un solo término algebraico sino dos. e:;> 00 Así:
x2 ➔ 1 termino
1.3 ELEMENTOS DE UN T ÉRMINO ALGEURAICO x1 + y ➔ 2 termmos
x2 + y + z' ➔ 3 terminos
Todo Término tiene: un Coeficiente y una Parte Literal x2 + y + z' + 1 -➔ 4 terminos
7
Así: 12x
...... y9
........,.....,
COEFICIENTEJ L PARTE LITERAL
Observa que la PARTE LITERAL está formada por:
* bases ( x e y )
* exponentes ( 7 y 9 )

1.4 TÉRMii\OS SE:\IEJANTES

Si dos o más términos tienen la misma PARTE LITERAL,


entonces son TÉRMINOS SEMEJANTES.
Ejemplos:
O ¡CUIDADO !
l 8
(1) - 5x 8 ; 0,2 x 8 ; - x
2 Al sumar o restar los
(2) 5m ; 2m ; - 4m ; IOm coeficientes de los
términos semejantes,
1.5 RFDUCCIÓN DE Tf~RMINOS SEl\lE.JANTES recurrimos ala REGLA DE
SIGNOS para la suma y
Si descubrimos que dos o más términos son semejantes, resta de números enteros.
éstos pueden ser reducidos a uno solo.sumando o restando
los coeficientes y escribiendo la misma parte literal.
Ejemplos:
( l) Reducir
8x2 + 5x2 = ( 8 + 5 ) x2 = 13x2
(2) Reducir:
5y4 - 3y4 - 6y4 = ( 5 - 3 - 6 ) y4 = - 4y4 Q 0 00 RECUERDA QUE:
(3) Reducir: Los pnncipales SIGNOS DE
- 6a4 + 3b2 - 6b2 + 4a4 = ( - 6 + 4 ) a4 + ( 3 - 6 ) b 2 COLECCIÓN son:
= - 2a 3b
4
-
2
Paréntesis ( )
OBSERVACIÓN Corchete~ [ J
Llaves { }
Para suprimir signos de colección Q 00 , se procede del
Si estos signos de colección
siguiente modo:
se encuentran unos dentro
(a) Si delante de un signo de colección aparece el signo +, de otros se van eliminaado
eliminamos tal signo de colección y los signos + 6 - «desde adentro hacia
de los términos interiores no cambian. afuera».

Ejemplo:
+ ( 2x3 - 5x2 - x + 2 ) = 2x3 - 5x2 - x + 2
(b) Si delante de un signo de colección aparece el signo
(-), eliminamos tal signo de colección y los signos + 6
- de los términos interiores cambian.
Ejemplo:
- ( 5a4 - 3x2 - 6x + 1 ) = - 5a4 + 3x2 + 6x - l
1 AltO!',SO ROJAS PUÉMAPE 1
1.6 POLIN0!\110 EN (D.

Le llamamos polinomio a una expresión algebraica cuyos exponentes de sus variables son
números naturales.
Si los coeficientes de los términos de un polinomio son números racionales, entonces se trata
de un Polinomio en <D.
Ejemplo: O IMPORTANTE
10 8 2
3x - 6x + 3x - 4
Este mayor exponente recibe el
nombre de GRADO DEL
* NOTACIÓN POLINÓMICA POLINOMIO
Un polinomio cuya única variable es x, puede ser represen- Ejemplos:
tado así: P ( x ) x + 5 es polinomio de 1º grado
Lo cual se lee: « P de x» o «P en x» 5x1 + 7 es polinomio de 2º grado
2Jf - 8 es polinomio de 3º grado
y significa: Polinomio cuya única variable es x.
En general, un polinomio de ( n + 1 ) términos puede ser
expresado así:

donde:
* x es la variable cuyo mayor exponente es n e;> 0
* a0 , a 1 , ~ , ••• , ª• son los coeficientes de P ( x ).
* Si todos los coeficientes EXCEPTO UNO son NULOS
(iguales a cero), tal expresión P ( x ) recibe el nombre
00 ¡ATENCIÓN !
especial de MONOMIO e;> 00
Ejemplo: 2x4 es un monomio Si todos los coeficientes
* Todo monomio es un término algebraico pero no todo EXCEPTO DOS de un
término algebraico es un monomio. polinomio son nulos, tal
Ejemplos: polinomio recibe el nombre de
El monomio 5x 4 cumple con la definición de término BINOMIO
algebraico. Ejemplos:
El término Sx-4 no cumple con la definición de monomio
3+x
(polinomio de un término) porque el exponente de x no
x2+1
es número natural. 5x4+8
OBSERVACIÓN.-
En situaciones como 3 ( 5x + 2 ), procedemos a aplicar la Propiedad Distributiva de la
Multiplicación respecto a la adición en los números racionales, es decir:
a(b+c) = axb+axc
·En nuestro ejemplo: 3 ( 5x + 2) = 3x 5x + 3 x 2 = 15x + 6
1 MATEMÁTICA
1

____ ! I
__._
SKANIPRÁCTICA © ------------------
1

l. En el siguiente «laberinto de términos algebraicos» unir con una línea contínua aquéllos
que sean términos semejantes y escribir en el recuadro correspondiente aquél que resulta
de reducir tales términos.

3x4 - 5x2 8x3 - 3x 2x5 - 6x3 ➔


D
- 2x2 - x4 3x - 4x 3x• x4 ➔
D
10x - 6x 6x2 3x3 -2x - 5x ➔
D
- x3 5x4 -4x3 4x2
- 2x3 x6 ➔
D
7x6 6x3 9x5
- 4x6 - 6x2 xs ➔
D
3x6 - 8x4 - 7x5 xs 4x2 ➔
D
11. Eliminar los signos de colección en las siguientes expresiones:

(1) -(2x2 -x+1) (6) -(x-x4-1) (11) ( 0,2 m6 - m2 + 2m• -1 )


(2) +(5x-x4+2) (7) +(2m•-m+2) (12) - ( m5 - 1 - m3 )
(3) (2x-x6+ 7) (8) - ( 0,5 m2 - m + 6 ) (13) - ( a2 - a - 1 )
(4) -(a-b+c) (9) - (x4 + X2 - X- 1) (14) - ( - 2a - a2 - 7 )
(5) (2a-b-c) (10)(2x6+x-x2 +1) (15) + ( 2x5 - 6x2 - 0,5 )

111. Eliminar los signos de colección aplicando la Propiedad Distributiva de la Multiplicación


respecto a la adición:

(1) 3( a+b) (6) - 5 ( 6x4- 3x2 ) (11) - 6 ( 2x2 + 5xy- 3x)

(2) 5(2x+3y) (7) - 2 ( 4X2 + X- 1 ) (12) - 2 ( 4ab - 7b2 + 6c)


1 1
4 ( 16x - 24y + 8)
2 2
(3) (6m+8n) (8) 5(a+2b+3c) (13) -
2
(4) -2(3x2 +5x) (9) -2(5a2 -b2 +3) (14) 0,5 ( 24a - 12b2 + 3b)
1
(5) - 3 (-9a+27b) (10) - 4 ( 2x2- 6x + 1 ) (15) 3 ( X2 + 2xy - 6)

IV. Eliminar los signos de colección y reducir términos semejantes en:

(1) (x+y)-(x-y) (6) ( a - b ) + 3 ( 2a - 5b)


(2) ( 2x - y- z) - ( 3x + y - z) (7) [ - ( x - y) - ( 2x - 3y)] + 2 ( 2x - 5y)
(3) - ( a + b ) + ( 3a - 2b ) (8) - 3 ( a2 - 4a + 1 ) + [ - ( 2a2 - a ) - 5a2]
(4) (5x2 -x+1 )-(4x2 -x+2) (9) 4 ( x' - 3x2 - 6 ) - 3 ( x' - 5x2 + 1 )
(5) - ( a• - b - 3c) - ( - a' + 5c) (10) 7 [ (X - 1 ) - ( 1 - X) ) + 2 [ 3 (X+ 3) - ( X+ 1 ) )

J
1 ALFO:\"SO ROJAS Pl1EMAPE 1 lfr.\s@Vi
Aquí mis operaciones de SKANIPRÁCTICA (D
1 MATEMATICA
L
2. ECUACIONES DE JER GRADO CON UNA
INCÓGNITA

2.1 ¿QUE ES UNA ECUACIÓN?


O ¡ ATENCIÓN !
Ecuación es una igualdad de dos expresiones
algebraicas que se verifica o satisface sólo para Los valores de x que
determinados valores de sus incógnitas. satisfacen a una ecuación
reciben el nombre de
SOLUCIONES o RAÍCES,
Ejemplos: éstas se agrupan en un
con1unto al que llamamos
(l) 6x + I = 13 se satisface sólo para x =2 con1unto solucion o C.S.
(2) x - 5 = 6 se satisface sólo para x = 11
S1 la ecuación es de grado
* OBSERVACIONES: n • entonces tendra n
soluciones o ra1ces.
• El signo IGUAL separa a una ECUACIÓN en Primer Miembro
(el de la izquierda) y Segundo Miembro (el de la derecha).
• Cada miembro está formado poruno o más términos algebraicos.
• RESOLVER una ecuación significa, hallar los valores de x que
la satisfacen. e:;> O

2.2 ECU \CIONFS EQUI\ ALENTES

Son aquellas que tienen las mismas raíces o soluciones.


Ejemplo:
00 IMPORTANTE
En una ecuación como 5x - 7 = 8, la raíz es 3
Despejar una Incógnita
es decir: tal ecuación se satisface sólo para x = 3
significa dar todos los
. , como x + 1 = 8 , l a ra1z
En otra ecuac10n , tamb"1en
, es pasos necesarios para
9 5 15 que aparezca en uno de
3 es decir: tal ecuación también se satisface sólo para x =3
los dos miembros SÓLO
luego ambas ecuaciones son Equivalentes. LA INCÓGNITA.

2.3 PROPffDADES DE LAS ECUACIONES En nuestro e1emplo,


hemos despejado x
1° .- Si sumamos o restamos a los dos miembros de una aphcando la primera
ecuación una cantidad constante, la ecuación que propiedad.
obtenemos es EQUIVALENTE a la primera.
Ejemplo:
En la ecuación x + 4 = 5 podemos verificar que su Raíz es I; si restamos 4 a ambos
miembros,la ecuación obtenida es equivalente a la primera, es decir:
x+4-4 =5-4 Ó X= 5-4 QOO
1 ALFONSO ROJAS PUEMAPE 1
REGLA PRÁCTICA
Podemos trasladar un término de un miembro al otro (TRANSPOSICIÓN DE TÉRMINOS) con
el solo cambio del signo de su coeficiente.
1 x+b = a ➔ x = a - b \ 1 x es la incógnita
2º Si multiplicamos o dividimos a los dos miembros de
una ecuación por una cantidad constante diferente ( O ¡ ATENCIÓN !
de cero, la ecuación que obtenemos es
EQUIVALENTE a la primera. S1 a ..e O y b ..e Oexiste raíz o
solución racional
Ejemplo:
S1a = 0yb..e0
En la ecuación 3x = 6 podemos verificarque su Raíz
No existe solución.
es 2, si dividimos entre 3 a ambos miembros la ecuación
obtenida es equivalente a la primera, es decir: Sia ..e 0yb=0
tendremos x = O
~=~ Ó X=~ Ó X=2
3 3 3 Sia=0yb=0
REGLA PRÁCTICA Tendremos solución INDETER-
MINADA O NO DEFINIDA
* Dada ax = b entonces x = -b
a / x es la incógnita
* Dada ..!. = b entonces x = ab
a

Una ECUACIÓN DE PRIMER GRADO CON UNA INCÓGNITA tiene la siguiente


FORMA GENERAL: ~---~ x : incógnita
1 ax + b = O I a y b :coeficientes

Despejamos x así:
a,b E <D.
00 ¡ CUIDADO !
• Aplicamos la primera propiedad de ecuaciones: ax = - b
l
Si una ecuación tiene sus
• Aplicamos la segunda propiedad de ecuaciones: términos con denominadores
numéricos, multiplicamos a
x= - b Q0 ambos miembros por mcm de
a
tales denominadores. con lo
2.4 PROCEDIMIENTO PRÁCTICO DE RESOLUCIÓN cual transformaremos a la
DE UNA ECUACIÓN. ecuación inicial en otra equi-
valente pero sin denomina-
1º .- Suprimimos signos de colección o agrupación dores donde ya podemos
2º .- Efectuamos reducción de términos semejantes en cada aplicar el procedimiento
miembro. descrito aqu1.
3º.- Hacemos transposición de términos, escribiendo los )
que son independientes en uno de los miembros y los
que no lo son en el otro miembro de la ecuación. 6x _ ( 4x + 2 ) = ( x _ 1 ) + 4 e:;> 00
4°.- Volvemos a reducir términos semejantes. 6x _ 4x _ 2= x _ 1 + 4
5°.- Despejamos la incógnita. 2x _ 2= x + 3
Ejemplo: 2x - x= 3 + 2
Resolver: - - - - - - - - ~ x= 5
IB'@ 3¡\J 1 MATEMATICA
1 ~tr
®
'

SKANIPRACTICA

l. Resuelve las siguientes ecuaciones, mostrando el procedimiento:

(1) 4x -1 = X - 4 (27) , 3 ( X - 4) + 6 = 5 ( X+ 1 ) - 13
(2) 3x-2 = x+6 (28) 7(x-2)+3 = 4(2x-6)=-2
(3) 7-5x=3x-1 (29). 5 ( X - 4) = 3 (X+ 6 )
(4) 12x-12 = 16x+8 (30) 3(x+ 1)+4(2x-1) = 5(x+5)-2(x-3)
(5) 7x + 5 - 3x = 4x + 3 - 2x (31) 15(2x+1 )-2 = 3(-2x+8)-4
(6) 16x-21 = 20x+3
(32) 1+..!=J..+x
(7) 19- 3x + 5x = 15- 4x 2 2

(8) 20x+7-2-= 15x+3 °(33) .I 5 -1 = ~ +6


1
(9) 16- 4x + 6x = 12x + 8 2 X' 1
(34) -sx-3= 4 +
3
(10) 13x - 2,4 = 6,2 + 11x
X 1 X
(ll) 4x-10x+15 = Sx-13 (35) · "'2" + 4 = 4 - 4

(12) 7x-6x-4 = 15x+3-6x 1 2 3 X


(36) -¡+5x=-¡+""'2"
(13) 0,5 + 10x = - 8,3 + 2x
(37) ..!.(x-1)-1.=
2 3
1+[x _ ..!.1
2
(14) 20x+6-4x = 12,3+6x
(15) -2x+7x-3 = 3x-x+6 (38) [-½ (X - 2 )] + 2 = [ ½(x - 2 )] +~
' tl6) 15x-6 = 13x+30 X-2 X-2
(39) - 3 - +2 =-5- +6
(17) 14+6x = 11x+4
(40) [½(x - 1)]+2 = ~(2x - 1) - 2---....
(18) 2x + 8- 3x = 4x + 15- 2x

~~9) 2,4 - 16x = - 14,2 + 4x (41) 1(x+2)=2{4(x- 2)]


(20) 25x - 64 = - 40x + 21

(71) 17x - 4 + 13x = 12x- 5 + 8x


(42) [-f (x - 1)] + 2 =~(x + 2)]- 6

(22) 14x-15+2x = 2x+40+3x (43) [-¾(x · 1)] +x =~(x · 3)]+ 5

(23~ 40x + 97 = 120x - 63 (44) [1-(x • 4)]+ 5x =-É.x +


3 7
1!...:.Jl
3
+ 30
(24) 2(x+3) =4x-8
(45) -½[{(x + 5) + {( x + 6)} x + 4
(25l 5(2x-4) = 2(3x+4)
(26) -13-[3(x+2)+4)=11-[6(-2x-2)+ 1) (46) .!(x
6
+1) + 4 x = ~ + 1
4
1 ALFONSO ROJAS PUÉMAPE 1
Aquí mis operaciones de SKANIPRÁCTICA @
1 MATEMATICA
1
SISTEMAS DE ECUACIONES)

El siguiente es un SISTEMA DE ECUACIONES:


2x+y = I .......... (D
5x-y = 13 ........ @
Este sistema está conformado por dos ecuaciones con dos incógnitas.
Resolver un sistema significa encontrar valores de las incógnitas
que las satisfagan simultáneamente. O IMPORTANTE
En nuestro ejemplo al resolver el sistema, tales valores de las
incógnitas son: Este metodo permite
eltminar una de las
X = 2 e y = -3
incógnitas del sistema
¿Podemos y quedamos sólo con
comprobar? Claro que sí; reemplacemos estos valores una ecuacion de una
en cada u nade las ecuaciones del sistema: mcógnita cuya solu-
ción ya es de nuestro
En(D:2(2)+(-3) =I c:;) I = 1 dominio¿ veraad?
EnQ):5(2)-(-3) =13 e:;> 13 = 13 ¡Comprobado!
Al conjunto de valores de las incógnitas que satisfacen a las ecuaciones del sistema. se le llama
SOLUCIÓN COMÚN o CONJUNTO SOLUCIÓN.
Pero ... ¿cómo hallamos esta solución común?, es decir. .. ¿cómo resolvemos este sistema de
ecuaciones?, ... emplearemos para esto un método muy sencillo que estudiaremos a continuación.

3.1 l\IÉTODO DE REI>l "CTIÓN Qo


00 ¡ATENCIÓN!
Procedimiento a seguir:
Puedes notar que en el
l. Preparamos las ecuaciones del sistema, eliminando signos de
segundo miembro de
colección, reduciendo términos semejantes, suprimiendo cada ec¡,ac,vil deben
denominadores y transponiendo términos, hasta que el sistema aparecer se s ter-
tenga la siguiente forma: mmos independien-
ax+by = c ......... Q) tes, es dear aquéllos
que so/o constan de un
dx+ey = f .......... @
número.
donde x e y son las dos únicas incógnitas y a, b, e, d, e y f
son los coeficientes. e:;> 00 _
2. Aplicando las propiedades de ecuaciones, hacemos que los coeficientes de la incógnita que
se desea eliminar, sean números opuestos en ambas ecuaciones. Por ejemplo, luego de aplicar
las propiedades de ecuaciones el sistema debe quedar así:
3x+4y = 18
5x + 4y = -2
donde los coeficientes de y son 4 y-4 respectivamente.
1 ALFONSO ROJAS PUE.\IAPE 1
3. Enseguida sumamos miembro a miembro ambas ecuaciones, eliminándose los términos con
incógnita y.
4. La ecuación que resulta sólo tiene a x como incógnita, la cual procedemos a despejar.
5. El valor de x hallado en el paso anterior se reemplaza
en cualquiera de las ecuaciones del sistema, de donde O RECUERDA QUE:
despejamos ahora y.
Si multiplicamos o dividimos
Ejemplos: a los dos miembros de una
ecuación por una cantidad
(l) Resolver el sistema: x + 2y = 17 ··········· (D
constante diferente de cero,
x-y = -1 ............ (v
la ecuación que obtenemos
es EQUIVALENTE a la
Solución: pnmera.

• Si multiplicamos (í) por 2, tendremos


los términos en y con coeficientes opuestos: 2x - 2y = -2 ..... G)C::;> O
• Sumamos miembro a miembro (M.A.M.) x + '/19 = 17
las ecuaciones Q) y @: 2x - 2s, = -2
3x+0 = 15
• Despejamos x de la nueva ecuación: l.__x_=_51
• Reemplazamos el valor de x obtenido,
en cualquiera de las ecuaciones del sistema: x+2y = 17
.,¡,
5 +2y = 17
- despejamos y : 1y= 6 1
RESPUESTA.- La solución común que satisface al sistema es x = 5 y y = 6.

(2) Resolver el sistema: i-y = f-13 ...... CD 00 ¡CUIDADO!

2x + 18 = 3y ........ (v Observa que en este


caso la intención es
Solución: eliminar la mcogmta
x, para lo cual, los
• Preparamos la ecuación CD: términos en x deben
- Para suprimir denominadores
multiplicamos "todo" por 3 :
3 ( i) -3 (y)= 3 ( ½)- 3(13) tener como coefi-
cientes dos números
X - 3y = y- 39 opuestos.
- Transponemos términos: x - 3y - y = -39
- Reducimos términos semejantes: x - 4y = -39 ...... Q)
• Preparamos la ecuación a) : 2x - 3y = -18 ...... @
• MultiplicamosQ) x (-2): - - - - - -2x + 8y = 78 ....... G)
• Sumamos@+G): 0+5y = 60 c::::>oo
• Despejamos y: Y ! = 121
• Reemplazamos y = 12 en(Z): 2x + 18 = 3(12) 01 x = 9 I
1 MATEMÁTICA
1

SKANIPRÁCTICA @
l. Comprueba que la SOLUCIÓN COMÚN dada, corresponde al sistema de ecuaciones
propuesto.

Nº SISTEMA SOLUCIÓN Nº SISTEMA SOLUCIÓN


COMÚN COMÚN
1 x+y = 6 X =4 5 2(a - b) + 5(a + b) = 13 a = l
0

X-y = 2 y = 2 7a + 2 - b = 2a + b + 3 b = 2
2 2x - y = O X =I 6 ~-y= 2'.. +x-8 X = 6
3x + y = 5 y = 2 3 3
2x = y - X+ 15 y = 3
3 5m-t = 16 m = 3 7 (x + l) + (y - 2) = 29 X = 10
2m- 3t= 9 t = -1 xy = 200 y = 20

4 2x + 1 = -3y X = -2 8 F+t + -.JY+I = 7 X = 15


X = 7y- 9 y =l (y+ 22)/x = o,5· 1 y = 8

II. Resuelve los siguientes sistemas sumando miembro a miembro las respectivas
ecuaciones:

Nº SISTEMA SOLUCIÓN Nº SISTEMA SOLUCIÓN


COMÚN COMÚN
l X+ y = 18 X= 10 4 3x +5y = 8 X= 6
X-y = 2 y = 8 -3x-4y=-10 y = -2
2 3x - 2y = 8 X= 6 5 2x+9y = -38 X = -1
X+ 2y = 16 y = 5 x-9y = 35 y = -4
3 a+7b = 15 a = I 6 5a- 3b = 7 a =2
3a- 7b = -11 b = 2 7a+3b = 17 b =l
111. Resuelve los siguientes sistemas de ecuaciones empleando el Método de Reducción:
X y
(l) X+ 2y = 15 (6) (x + 2y) - (2x - y) = 8 (11) - -y=--x+18
5 5
X- 2y = -5 X - 1 - [y - 2x] = -1
2x-y =40
(2) X -y = 4 (7) 5(x - y)+ 3(y - x) = 32
3x +4y = 68 (x + y)/3 = -4/3 (12) 4(x + 1) - 5(y + 2) = -12
(3) a = 14- 5b (8) 3(2x+l)-2(3y+l) = 13 5(x-1)+4(y-2) = -4
2a = 3b- 11 x-y = 2
(4) 7m- 2n+ 34 = O (9) (7y - X)+ 2(x - 1) = -25 (13) 7 - [(2y - 3) + 4(x - I)] = 22
5m + 3n + 11 = O (2y-x)+7(y- l) = -32 [5(x + 2) - 3(y - 2)] - 8 =X
(5) X = 5 + 3y (10) 3x - 2[(x - 1) - (y - 1)] = 18 (14) 3[x -4y] + 7[2x - yJ = -10,5
7x - 39 = 9y x+y = 10 14x - 3y = 4
1 ALFONSO ROJAS PUÉMAPE 1

Aquí mis operaciones de SKANIPRÁCTICA G)


1 MATDL\TICA
1
4. INECUACIONES DE PRIMER GRADO CON
UNA INCÓGNITA

-U DESIG UALDAD

Sean dos números a y b e <D. tal que a ~ b.


Desigualdad, es una relación entre a y b que se representa así:
a > b 1 «a es mayor que b», si ( a - b ) es positiva
a < b 1 «a es menor que b», si ( a - b) es negativa
Ejemplos: O I CUIDADO/

(1) 7 > 4 es correcto porque 7 - 4 = 3 Qo En toda DESIGUAWAD


la expresión escrita a la
(2) 5 > - 3 es correcto porque 5-(-3) = 8
izquierda del simbo/o >
(3) - 6 < O es correcto porque -6-O= -6
ó < es el Primer Miem-
(4) - 3 < 3 es correcto porque -3- 3 = -6 bro y la expresión de la
(5) O> - 4 es correcto porque O- ( - 4 ) = 4 derecha es el Segundo
Miembro de la desi-
4.2 TIPOS DE DESIGl'.\LDAD gualdad.

1° DESIGUALDADES ABSOLUTAS
Son aquellas que son indiscutiblemente ciertas.
Ejemplos:
(1) 10>0 (2) -8< I
o también:
00 ¡ CUIDADO ! )
Son aquellas que se verifican para cualquier número racional
que le asignemos a sus variables. En esta inecuación: 1
¿Cuántos valores rac,o- 1
Ejemplos: na/es puede admitir x?
(1) x2 2 O (2) (x + l )2 + 5 ? 0 ¿Cuálsena el valor más
pequeño?
¿ Cuál serla el valor más
2º DESIGUALDADES RELATIVAS grande?
Son aquellas que se verifican o satisfacen sólo para ciertos
valores de sus variables. Estas reciben también el nombre de
INECUACIONES.
Ej emplos:
( I) x + 3 > 7 Si x recibe el valor 2 tendríamos 2 + 3 > 7 ó 5 > 7 lo cual no es cierto.
En este caso x puede admitir sólo valores mayores que 4. Entonces
x + 3 > 7 es una inecuación cuya solución es x > 4 e:;> 00
1 ALFO'\SO ROJAS PUÉMAPE 1
4.3 PROPIEDADES DE LA DESIGUALDAD

1ª Siendo una cantidad, mayor que otra y esta mayor que una tercera, entonces la primera
cantidad será mayor que la tercera (PRINCIPIO DE TRANSITIVIDAD)
Es decir: Si a > b y b > c entonces a > c
Ejemplo: O ¡ ATENCI0N /
Si 15>6 y 6>2entonces 15>2 Si a > b , ó b < a en la
recta numérica tendre-
2º Si una cantidad es MAYOR que otra, entonces esta será mos·
MENOR que la primera e:;> O b a
Es decir: si a > b entonces b < a
donde a esta ala derecha 1

Ejemplos: de b.
(1) Si 18 > 10 entonces 10 < 18 As1: los numerospositivos
m se expresan como
(2) Si 2 < x entonces x > 2
m>O
3° Si a ambos miembros de una desigualdad se le suma o resta Losnúmerosnegat,vos m
una misma cantidad, el sentido de la desigualdad NO SE se expresan como m < O
ALTERA.
Si a> b y m E (l),
entonces a + m > b + m
Ejemplos:
( 1) Dada la desigualdad: 6 > 2
Adicionemos 5 a cada miembro: 6+5>2+5
11 > 7 i cierto !
(2) Dada la desigualdad: 3 > -9
Restemos 4 a cada miembro: 3-4 > -9-4
- 1 > - 13¡ cierto !
00¡ IMPORTANTE /
(3) Dada la desigualdad: x+3<10
Restemos 3 a cada miembro: x+3-3 < 10-3 Como en ECUACIO-
X < 10 - 3 e:;> 00 NES... En el ejemplo (3)
X < 7 podemos notar que:
Al transponer un término
4º Si multiplicamos a ambos miembros de una desigualdad por de un miembro al otro.el
una misma cantidad positiva.el sentido de la desigualdad NO signo de su coeficiente
SE ALTERA cambia

Es decir: Si a > b y m>O


entonces am > bm
1 MATEMATICA 1
Ejemplos:
(1 ) Dada la desigualdad: 5 > 3 y además m = 8
entonces: 5 x 8 > 3 x 8
40 > 24 ¡ Verdadero !
( O ¡ IMPORTANTE 1 )

(2) Dada la desigualdad: ~ > 3 y además m = 5 Como en ECUACIO-


NES.. en el e¡emplo (2)
entonces: ~x-5' > 3 x 5 de la 4 propiedad
x>3x5 c::;>o podemos notar que ·s1
un termino está dividien-
do, pasa al otro miembro
(3) Dada la desigualdad: ~ + 2 > 5 multiplicando' Sólo hay
que tener cwdado al
Vamos a combinar la 3° y 4° propiedad: determmar si el sentido >
ó <. cambia o no
* Restamos 2 a ambos miembros: ~+2-2>5-2
5

* Multiplicamos por 5 a ambos miembros: ~ x5 > 3 x 5


X> ]5

5º Si multiplicamos a ambos miembros de una desigualdad por


una misma cantidad negativa, el sentido de la desigualdad SE
ALTERA.
Es decir: Si a > b y m < O
00 ¡ CUIDADO 1
entonces: am < bm
Dada la mecuaetón.
Ejemplos: _!._>2
5
( 1) Dada la desigualdad: 7 > 2 y m =- 4
Hemos obtenido
entonces : 7 ( - 4 ) < 2 ( - 4 ) ¡ Se invierte
- 28 < - 8 el sentido ! X < • 10
X Lo cual significa que
(2) Dada la desigualdad: 5 > 2
todos los números
racionales menores que
multipliquemos a ambos - 10 satisfacen a .1a me-

miembros por (- 5) - 5X x (-5)<2 x (-5)


x < - 10 c::;>oo
6° Si dividimos a ambos miembros de una desigualdad por una n1.isma cantidad m positiva
el sentido de la desigualdad NO SE ALTERA. Si dicha cantidad m es negativa, el sentido
1 ALFONSO ROJAS PUÉMAPE 1
de la desigualdad ¡ SE ALTERA !
Es decir: Si a > b y m >O
entonces: .l!.. > .E...
m m
Ejemplo:
O RECUERDA QUE:
Si 30 > 18 y m = 6
Los números positivos son
entonces 30 > 1ª. ó 5>3 siempre mayores que
6 6 CERO
Además: Si a > b y m<O
m>O
entonces: ..!.. <
m
.E...
m Los números negativos
son siempre menores que
Ejemplo: CERO
Si 12 > 6 y m = - 2 ~O
12 6
entonces _ < _ ó -6<-3
2 2

4.4 INF<.:UACIONES DE PRI!\IER GRADO Y Sl' RESOLUCION

Formas generales: 1 ax+ b > O 11 ax+ b:;?0

1 ax+ b < O 11 ax+ bs O


Para resolver inecuaciones de primer grado con una incógnita seguimos los siguientes pasos:
1º Suprimimos signos de colección.
2º Reducimos términos semejantes. 00 ¡ ATENCION I
3° Hacemos transposición de términos empleando las El hecho de aplicar
propiedades de las DESIGUALDADES. ~ 00 estas propiedades,
4° Volvemos a reducir los términos semejantes y despejamos la permite que proceda-
incógnita. mos en forma similar a
la resolución de ecua-
Ejemplo: ciones de primer
Resolver: 3x+12<x-[5x+2] grado.

Solución Ejemplo:
ax+b>O
1º Suprimimos signos de colección: 3x + 12 < x - 5x - 2
ax> -b
2º Reducimos términos semejantes: 3x + 12 < - 4x - 2
b
3º Transponemos términos : 4x + 3x < - 2 - 12 X>. T
4° Reducimos términos semejantes: 7x < - 14 pero ,cuidado con el
Despejamos x dividiendo a sentido de la desigual-
ambos miembros entre 7: b_< __!i~ X<· - 2 dad!
7 7
Respuesta.
Los valores de x que satisfacen a la inecuación dada son todos aquellos que sean menores
que - 2 6 x < - 2.
jj¡f¡\@Y¡fj 1 MATEMÁTICA 1-=-.0•

1l
SKANIPRÁCTICA @I
l. Resuelve las siguientes inecuaciones mostrando el procedimiento:

(1) 3x-2<x+6 (25) 4(x-5)+3<7

(2) .. 5x - 9 :S 2x + 15 (26) 5(x+2)-4~-9

(3) 9x+12>2x-2 (27) 6(x-7) +8:S20


(4) 12x-3~4x+21 (28) x+2 +!.> 2
7 5
(5) 3x + 2 + 2x < 7x 2x - 1 <
(29) -3-+x - 3
(6) 4x - 5 + x :S 5x - 4 + x 5 -X 6- X
(30) --+-->3-
2 5 -
(7) 21x - 20 > 20x - 21
(31) 3x + 1 + !. <2
4 5
(8) 123 - 321x2'.122 - 320x
x+2 x-3>
(32) 3 + 2 - 5
(9) 9 - 5x + 10 < 7 - 3x + 6
(33) x+3 2 + 2 -
x+1< ~
(10) 8+9x+ 10< 11 + 12x+ 13
X+4
(11) -2-3x-4 :::: -5-6x-7 (34) - - +2 >X
3
(35) 3x + 1_ 3<X
(12) 5x + 4x + 3x < 108 2
(36) - 2x -++1 2> _x
(13) 7x + 5x + 3x > 225
3
(14) 2x - 3x + 4x :S 21 (37) 5x+1 + 1 :Sx
6
(15) 9x - 7x + 5x ~56 5X + 1 + X+ 5 > 5
(38)
2 3
(16) 7x + 2x - 1 < 7 + 2 - x 3x - 1 x- 3
(39) - - 2
+--<O
2 -
(17) 3x+5+x ~ 3+5x+1 2x + 1 x - 3 1
(40) , - - + - - > -
(18) 3x - 4x + 5x - 6x < - 200 9 4 2
(41) 3-[2x+(x+2)]<2
(19) 8x + 7x-6x- 5x~28
(42) 4+3(x+1 )>5+4(x-1)
(20) 3x+4x+5x+6x:S36
1 1
(21) 1 - 2x + 3 - 4x > 2x - 28 (43)
3( X+ 2) + 4 ( X+ 3) ~ 2
(44) 2 X -1) + -(X
-( 3 - 2) < 6
(22) 2x + 3 + 4x + 5 :S 68 5 4
(23) 5x + 2x + x < 7x + 1 (45) 4( ¡ + 1) + 3( ½+ 2) < 20 + X
(24) 3(x+2)+1>22 ✓ (46) 3(x + ½)+4(x + ½)> 5(x + ¡)
,_ j ~ . ---~ J ,.... ....
·-:..:-
-
1 ALFONSO ROJAS PUÉMAPE 1

Aquí mis operaciones de SKANJPRÁ_CTICA ©


1 MATEMÁTICA 1

RAZONAMIENTO MATEMÁTICO

(1) Herbert quiere saber cuántas cifras se ( 6) En una bolsa con caramelos hay 3 sabores
emplearon en la numeración de las páginas diferentes: fresa, limón y menta.¿ Cuántos
de un libro de 234 hojas. Ud. contestaría: caramelos se deben probar, como mínimo,
tomándolos al azar, para tener la seguridad
a) 468 b) 1 404 c) 1 269 de haber probado 2 caramelos del mismo
d) 1 296 e) 1 926 sabor?

(2) Un reloj se adelanta 3 seg. cada 4 horas. a) 2 b) 3 c) 4


¿Cuántos minutos se adelantó en 3 días? d) 5 e) 6

9 mm
. . b) 1 mm.
. 3 . (7) Hallar el número que sigue en la siguiente
a ) To c) mm .
2 5 sucesión:
1 . 7 .
d) To mm . e)
5 mm. 12
-6;-4;0;
2 ; ...
(3) Rubén demora 6 días en hacer un trabajo.
a) 12 b)14 c)l6
¿Qué tiempo emplea en hacer otro trabajo
d) 8 e) 10
doblemente difícil?

a) 3 b) 9 c) 12 (8) La diferencia entre el mayor número de 3


d) 18 e) 24 cifras diferentes y el menor número de 3
cifras diferentes es:
(4) ¿De cuántas maneras se podrá viajar desde
la ciudad A hasta la ciudad F. si en cada a) 800 b) 864 c) 872
viaje no se pasa 2 veces por una misma d) 899 e) 885
ciudad?
(9) ¿Cuántos números enteros hay, cuyos
cuadrados sean menores que 25?

a) 9 b) 5 c) 4
d) 6 e) 7

a) 5 b) 6 c) 7 (10) Sequierenempacar 144latasdeconservas;


d) 8 e) 9 se podría hacer en:
(5) Se vendió una bicicleta en SI. 1 200,
a) 18 cajas de 6 latas cada una
ganando la quinta parte de la venta. ¿Cuál
b) 24 cajas de 8 latas cada una
fue el costo de la bicicleta?
e) 9 cajas de 15 latas cada una
a) SI. 980 b) SI. 960 c) SI. 975 d) 12 cajas de 14 latas cada una
d) SI. 920 e) SI. 950 e) 36 cajas de 4 latas cada una.
~t. 1 AI.F01'SO ROJAS PUÉMAPE 1

Aquímis operaciones de RAZONAMIENTO MATEMÁTICO


1 MATEMATICA
1

CAPÍTULO 7

SISTEMA
INTERNACIONAL
DE UNIDADES (S.I.)

OBJETIVOS:

• Conocer el moderno SISTEMA INTERNACIONAL DE


UNIDADES cuya aplicación va de acuerdo a los cambios en la
ciencia, técnica, industria y el comercio en el mundo
1 AlFOXSO ROJAS PUEM .\PE 1 11©@Ui

ESTRUCTURA DEL CAPÍTULO:

1. Revisemos algo de historia.


2. El SISTEMA INTERNACIONAL S.1.- Sus Unidades

3. La UNIDAD de LONGITUD.

4. La UNIDAD de MASA.

5. La UNIDAD de TIEMPO.

SKANTPRÁCTICA G)
,
6. La UNIDAD de AREA

7. La UNIDAD de VOLUMEN

SKANIPRÁCTICA @
RAZONAMIENTO MATEMÁTICO
1 MATE.M;\TICA
1
( 1. REVISEMOS ~ DE IDSTORIA)
Desde que al hombre le es necesario MEDIR, crea mecanis-
mos de medición, que al correr del tiempo y las culturas se
han mulliplicado, de un modo tal que llegó el momento de O ¡CUIDADO!
reunir a los entendidos para intentar unifom1izar dichos me-
canismos, para lo cual la Asamblea Nacional de Francia SI no es una abreviatura n,
una sigla.
pidió a la Academia de Ciencias de ese país.la creación de un
sistema de medición que lo puedan usar todos los países;
Sólo es un sfmbolo.
resultando de esto el SISTEMA MÉTRICO DECIMAL
(basado en el metro, kilogramo y segundo) que en 1 862 se
empezó a empícar en el Perú.

Posteriormente, en 1 948 la CONFERENCIA GENERAL


DE PESAS Y MEDIDAS delegó al COMITÉ INTERNA- 00 ¡ATENCIÓN!
CIONAL DE PESAS Y MEDIDAS la revisión de las unida- Las Unidades DERIVADAS
des de medición con el fin de logmr un sistema de medidas del SI, deben su nombre a
más sencillo, moderno y práctico acorde con el vertiginoso que se der,van de las Unida-
avance de la ciencia, siendo que después de varias reuniones, des de BASE. Aquf mostra-
en el año 1 960 se crea el SISTEMA INTERNACIONAL DE mos solo algunas ya que
UNIDADES (SJ) basado en fenómenos naturales que casi no son alrededor de 900.
ofrecen variación alguna que impida uniformizar internacio-
nalmente las unidades de medida. En el Perú su uso es oficial
a panir del año 1 982. O 0

2. EL SISTEMA INTERNACIONAL (S.I.) .-


SUS UNIDADES

En el cuadro siguiente, presentarnos a las UNIDADES DEL


SI agrupadas como Unidades de Base, Unidades S uplcmen-
tarias y Unidades Derivadas. O 00

UNIDA DES DE BASE

MAGNITUD FÍSICA UNIDAD SÍMBOLO


-
Longitud Metro m
Masa Kilogramo kg
Tiempo Segundo s
Intensidad de Corriente Eléctrica Ampere A
Temperatura Termodinámica Kelvin K
Intensidad Luminosa Candela cd
Cantidad de sustancia Mol mol
1 AlfO:'<iiSO ROJAS PUEJ\I\PE ~

UNIDADES SUPLEMENTARIAS O ¡IMPORTANTE!


MAGNITUD . UNIDAD SIMBOLO En cuanto a las unidades de volumen. en t m"
FÍSICA hay t 000 litros.
El litro, es una unidad muy antigua que aun se
~Í,gulo plano radián rad sigue usando. En el Si aparece como t dm3
Angulo sólido estereorradián sr t litro = t dm"

---
UNIDADES DERIVADAS

MAGNITUD UNIDAD SÍMBOLO 00 ;CUIDADO/


FiSICA Se sabe que. "a1 =a- n

Area metro cuadrado m2 Entonces:


Volumen metro cúbico m 3
O O 0, 1 • un décimo
Densidad kilogramo por metro cúbico kg/m3 = - 1- = _1_ = 10 1
Velocidad metro por segundo mis 10 1
10
Fuerza (peso) newton N
Presión pascal Pa 0,01 • un centésimo
= _1_ = _1_ = 10· 2
En este texto sólo estudiaremos las tres primeras Unidades 100 102
Básicas y las dos primeras Unidades Derivadas. t
0,001 • un milésimo

e
3. LA UNIDAD DE WNGITUD )
= -1-= _1_= 10- 3
1000
n lug•rN
,..............
0,00 •. 01 =
103

-
1
n =10
- n

Es el metro, (m). En el siguiente cuadro, mostramos los 10


MÚLTIPLOS y SUBMÚLTIPLOS del metro, sus sím-
bolos y equivalencias en metros.
UNIDAD SÍMBOLO EQUIVALENCIA EN m EN POTENCIAS
DE DIEZ

exametro Em 1 000 000 000 000 000 000 m 10' 1 m


petametro Pm 1 000 000 000 000 000 m 10' ' m
lera.metro Tm 1 000 000 000 000 m 1o·•m
gigámetro Gm 1 000 000 000 m 10'm
MÚLTIPLOS mcgámetro Mm 1 000000m 10' m
kilómetro
hectómetro
Km
hm
1 000m
100m
103 m
10'm
o
decámetro dam 10m 1o·m 00

UNIDAD DE BASE metro m 1m 10' m

decímetro dm 0,1 m 10 ' m


centímetro cm 0,01 m 10' m
milímetro mm 0,001 m 10 3 m
micrómetro um 0,000 001 m 10• m
sus- nanómetro nm 0,000 000 001 m 10 1 m
MÚLTIPLOS picómetro pm 0,000 000 000 001 m 10 ''m
fentómetro fm 0,000 000 000 000 001 m 10 '' m
attómetro am 0.000 000 000 000 000 001 m 10'1 m
1 MATEMATICA
1
Podemos convertir cm a m; o km a m; o dm a micrómetros.
Veamos algunos ejemplos:
(1) Escribir 7 gigámetros en hectómetros.
Soluci{m

Como nuestra tabla de múltiplos y submúltiplos expresa equiva-


lencias en metros, usaremos el metro como unidad-puente para
empicar el método de FACTOR de CONVERSIÓN.
O ¡IMPORTANTE!
Este método consiste en multiplicar por 1 a la expresión que se r-:
Este FACTOR DE CON-
desea trnnsformar. Como es 16gico, al dar este paso la expresión
VERSIÓN podemos em-
dada no se altera. Si este 1 lo reemplazamos por 1Km/1 Km ó
plearlo vanas veces en un
ldm/ldm ó 1mm/1mm, la expresión sigue sin alterarse. e1ercicio según sea con-
veniente, por e1emplo:
Supongamos que deseamos expresar A metros en kilómetros:
Am x1x1x1 • Am
Am x 1 ·= Am Am x1x1x1x1 • Am
Am x
_l_km_ ,_ Am
1 km

ó Am x 1 km := Am
, 1000 m
1 FACTOR DE CONVERSIÓN: QO
Veamos nuestro ejercicio escribiendo primero 7 Gm en metros:
9
10 m 9
7 Gm x
1
Gm =7 x 10 m e:;> 00
00 ¡CUIDADO!
Ahora, escribimos 7 x 109m en hectómetros: El FACTOR de CONVER-
9 Ihm 7 X 10
9 SIÓN en este caso es :
g
7Gm=7x10mx IOOm= 100 hm 10 m
2 7
7 x 10 x 10 hm 1Gm
2
que es eqwvalente a 1ya que
10
7 1 Gm= 1Cf m.
= 7x 10 hm Observa que hemos escrito 1
RESPUESTA.- Gm en el denominador con el
fin de cancelarlo con el Gm del
7 gigámetros se escribe también como 7 x 107 hm.
numerador y quedarnos sólo
conm.
OTRA FORMA.-
Podemos hacer esta conversión en una sola línea, así:
9 9
7
_ '>.-. 10 TI\.
7 G m - 7 '-'ll:!. x G x
1hm _ lQ_ h ➔ =7 x 10 hm.
2 - 7 x 2 m
1
~ lO)ll_ 10
1 ALFONSO ROJAS PUÉMAPE 1
(2) Escribir 1 500 000 cm en km.
O ¡IMPORTANTE!
Solución
50 = 5 X 101
• Escribimos primero el número dado en la forma de un factor 500 = 5 X 102
multiplicado por una potencia de 10: O O 5 000 = 5 X 103
5 000 000 = 5 X 106
I 500 000 = I 5 x 105 0,5 • 5 X 10·1
• Aplicamos en una sola línea el FACTOR DE CONVER- 0,05 a 5 X 10·2
SIÓN por medio del cual transformamos primero a metros 0,005 • 5 X 1Q"3
y luego a kilómetros: 0,0005"' 5 X 10" 4
5
s 1m 1 km 15 >< h(
1500000cm= 15 x 10 cm>< - 2- >< - 3- = 2 3 km
10 cm 10 m )<t >< )Q
= lSKm
RESPUESTA.- 00 ¡CUIDADO!
1 500 000 cm equivale a 15 km. S1 "pesamos· un paquete en una balanza oref1-
naria y leemos 50 kg, la forma correcta efe
;i ~:~~:;!~' situación es: "el paquete tiene50 kg

( 4. IA UNIDAD DE MASA ) /·· Lo s,gu,ente es mcorrecto:


i_li_!. ·e/ paquete bene 50 kg efe peso· o 'el paquete
pess50 kg~
f
Es el kilogramo (kg). \\rt~~.-.,.···-·"->"'
0 000
• ·-·""'
·"'" -·<0·""
~-""' 0-~-""=
~ =-""=···=-="" 000
,:-:•=""·=·=·,""-,-=;""•=-·-""·
• , ""·=· -0•;:,..,
;:""" ::-,·=·.,,..=
c--·""-·-·0·""
0 0
,=· ,"'"··
-·-·,""'="'""···,.-""
·

En el siguiente cuadro mostramos los MÚLTIPLOS y SUBMÚLTI-


PLOS del kilogramo, sus símbolos y equivalencias en kilogramos.

UNIDAD SÍMBOLO EQUIVALENCIA EN kg EN POTENCIAS


DE DIEZ

exagramo Eg 1 000 000 000 000 000 kg 1015 kg


petagramo Pg 1 000 000 000 000 kg 1012 kg
teragramo Tg 1 000 000 000 kg 10g kg
MÚLTIPLOS gigagramo Gg 1 000000kg 106 kg
megagramo Mg 1 000 kg 103 kg

UNIDAD DE BASE Kilogramo kg 1kg O 00 100 kg

gramo cg 0,001 kg 1Q-3 kg


miligramo mg 0,000001 kg 10-5 kg
microgramo µg 0,000 000 001 kg 10-g kg
SUB- nanogramo ng 0,000 000 000 001 kg 10-12 kg
MÚLTIPLOS picogramo pg 0,000 000 000 000 001 kg m1s kg
lentogramo lg 0,000 000 000 000 000 001 kg 10-1skg
attogramo • ag 0,000 000 000 000 000 000 001 kg 10-21 kg
1 MATEMATICA
1
Veamos algunos ejemplos de conversión:
f
(1) Expresar 7 500 000 miligramos en kilogramos. O ¡ATENCIÓN!
Solución A esta forma de escribir
• Escribimos primero el número dado en la forma de un
!} unnúmeroempleandopo-
t lencias de to también se
factor multiplicado por una potencia de 10: f le llama NOTACIÓN
7 5()(} 000 = 75 X 1(}5 o 0 i CIENTIFICA.

• Aplicarnos el método del FACTOR de CONVERSIÓN:


-6
s 10 kg
7 500000mg =75" 10 mj" 1 m% e:;> 00
5 - 6 Í'. 00 RECUERDA QUE:
= 75 >< 10 >< 10 kg e:;> 000 ~-e"
,,_ De la tabla anterior:
= 75 >< 10
- 1
= 7, 5kg {
s_ 1mg-10 6 kg

(2) Escribir 0,025 gigagrarnos en micrograrnos. Por esta razón el factor de


conversión -6
10 kg
Solución
es igual a 1. 1mg
• Escribimos primero el número dado en NOTACIÓN
CIENTÍFICA. O O
0,025 = 25 X ] 0·3

• Aplicamos el método del FACTOR DE CONVERSIÓN


en una sola línea transformando primero a kilogramos y
~·:·
luego a micrograrnos: J:' 000 RECUERDA QUE:
6
- 3 · 10 kg lµg
O, 025Gg =25" 10 Gg "---.-ag"
9 _
10 kg Por lo tanto:
- 3 6
105 X 10·6 z 105· 6
= 25 )( 10 "_910 µg = 25 )( 1012µg 105 X 10·6 • ,O•I
10 -:::
-====t-~::.:..:.-:::::-:;.;..;::.,; ·:¡;::.:-::::::=f:·.- ~=:-=:::...:-::: :-:-::::.:-:-.-:::·::: :-._:_.·
RESPUESTA.-
0,025 gigagrarnos es equivalente a escribir 25 x 1012 µg o también:
0,025 Gg = 25 000 000 000 000 µg

(3) Escribir 4 320 000 nanogramos en gramos.

Solución
Por NOTACIÓN CIENTÍFICA: 4 320 000 = 432 X 1()4
• Para efectuar la conversión aplicarnos el FACTOR DE CONVERSIÓN:
-12.
" 10 kg lcg
4 320000ng - 432 >< 10 ng >< ng " _,
1 10 kg
~o, 1 ALFONSO ROJAS PUÉMAPE 1
104 xw-12
= 432 X 10--1
cg
~
= 432 X 10-5 cg O ¡CUIDADO!
Sólo para fmes comercia-
RESPUESTA.-
les y contables, se consi-
4 320 000 nanogramos pueden ser esenios como 432 x I o-s dera el año de 360 días.
gramos ó 4 320 000 nanogramos = 0,00432 gramos.

(5. lA UNIDAD DE TIEMPO) 00 ¡IMPORTANTE!

Es el segundo (s). Una vuelta completa


Presentamos aquí, algunas equivalencias no ~swblccidas en alrededor del sol le
el S.I y que son de uso general todavía: demora a la Tierra 1
año ó 365 d1as más un
cuarto de día. Por esta
l minuto (min) =60s
razón, cada 4 años se
l hora (h) = 60 min ó 3 600 s completa un d,a mas
l día (d) = 24 h ó 86 400 s apareciendo entonces
laño c::::>o (a) = 365 d ó 3 l 356 000 s un año de 366 días al
que se le llama AÑO
Ejemplo: BISIESTO.
Escribir 5 días 8 horas en minutos.
Solución
. • 24h 60 min
• Transfonnando los días a minu- 5 d1as= 5 d1as>< - - , " - - -
tospor FACTOR DE CONVER- ld1a lh
SIÓN: 5 24 60
5 días= " " min 6 5 días= 7 200 min
1" l
• Transfonnando las horas a mi-
nutos por FACTOR DE CON-
G5
VERSIÓN: 8 horas= 8h >< 60 min = 480 min .... .
lh
@
• Sumando CD + a) obtenemos 5 días 8 horas en minutos:
5 días 8 horas = 7 200 + 480 = 7 680 min.
RESPUESTA.-
5 días 8 horas equivale a escribir 7 680 min.
OBSERVACIO:\ES.-

• Al tratar de escrihir la llamada "hora exacta", el SI recomienda el siguiente orden: hora. minuto y segundo. Así:
6 horas, 7 minutos, 18 segundos det>cn ser escri1os así: 06h 07 min 18s ó 6h 7min 18
5 minutos 36 segundos deben ser escritos así. 00h 05 m in 36s ó 00h 05 min 36
2 horas, 45 segundos dehcn ser escritos así: 02h 00 min 45
• La Tierra gira alrededor de su eje (movimiento de rol.ación) de Oeste a Este demorando 1 día en dar una vuelta
completa.
• La Tierra gira alrededor del sol y demora I año en dar una vuelta completa O 00
1 MATEMÁTICA
1
SKA~IPRÁCTICA CD
l. Escribir:

(1) 7 200 decímetros en metros (10) 2,5 terámetros a hectómetros


(2) 85 000 milimetros en metros (11) 3 200 decámetros a kilómetros
;3) 166 000 000 micrómetros en decímetros (12) 8 620 decímetros a hectómetros
(4) 1,72 gigámetros en hectómetros (13) 0,02 petámetros a kilómetros
(5) 15.25 kilómetros en metros (14) 0.16 kilómetros a centímetros
(6) 76 000 000 000 nanómetros en centimetros (15) 0,004 megámetros a decímetros
(7) 2,5 hectómetros en decímetros (16) 0,015 exámetros a hectómetros
(8 ) 7 800 000 dec1metros en hectómetros (17) 12 450 000 micrómetros a centímetros
(9) 6 580 000 centímetros en decámetros (18) 18 300 decímetros a decámetros

11. Escribir:

(1 ) 2.75 gigagramos a gramos (11) 325 miligramos a gramos


(2) 1 800 000 microgramos a kilogramos (12) 42 155 000 microgramos a kilogramos
(3) 0.45 megagramos a miligramos (13) 165 000 000 picogramos a miligramos
(4) 1.25 teragramos a kilogramos (14) 72 x 1025 attogramos a microgramos
(5) 0,00052 petagramos a megagramos (15) 48 x 1028 lentagramos a megagramos
(6) 0.000 000 12 exagramos a gigagramos (16) 1.5 teragramos a gramos
(7) 0,75 gigagramos a miligramos (17) 6.025 megagramos a miligramos
(8) 0,004 gigagramos a microgramos (18) 2 x 102 ª picogramos a microgramos
(9) 62 000 000 nanogramos a miligramos (19) 5,3 gigagramos a exagramos
(10) 125 000 000 lentogramos a microgramos (20) 0,025 petagramos a megagramos

III. Escribir:

(1) 6 días 3 horas en minutos (9) 9 de la mañana con 5 minutos 3 segundos


(2) 72 días en minutos (10) 6 de la tarde con 16 minutos 13 segundos
(3) 3 días 6 horas en segundos (11) 11 am con 5 minutos 1Osegundos
(4) 8 d1as 5 horas 27 segundos en segundos (12) 3 pm con 6 segundos
(5) 16 días 3 horas 30 minutos en horas (13) 6 pm con 10 minutos y 15 segundos
(6) 7 d1as 17 horas en horas (14) 6 am con 10 minutos y 15 segundos
(7) 7 días 17 horas en minutos (15) 2 am con 2 minutos y 2 segundos
(8) 7 días 17 horas en segundos (16) 12 pm con 9 minutos, 8 segundos

·_;:
·•
1 ALFO:\'SO ROJAS PUEMAPE
i#i@Ui
Aquí mis operaciones de SKANIPRÁCTICA 1

:-.: ·-.-:-.·-:-:-.--::- ..-.•-·· ··-


&-" -:;:; -·-· · ·-:-:(·$•·:,¡-:;:::::: >~:=:;-;;:;-~:-:-::z;::::•::::: :-~•=·•::.;;y=•;,.-:-:-;-:::::,·-····v:•: -•= ~- :,:•<··-f
I..____MA:_T_E_MA_T_ICA
___ ]

( 6. LA VMDAD DE ÁREA )
O ¡ATENCIÓN!

Es el metro cuadrado (m2) Un metro cuadrado:


Con esta unidad se miden las superficies planas. Un metro

' 1m 0 1m
1m
cuadrado representa una superficie cuadrada de un metro por
lado.O O
En el siguiente cuadro mostramos los MúLTIPl OS y SUB-
MúLTIPLOS del metro cuadrado. sus símbolÓS y equiva- 1m
lencias en m2•

UNIDAD SÍMBOLO EQUIVALENCIA

exámetro cuadrado Em2 1016m2


petámetro cuadrado Pm 2 101° m2
lerámetro cuadrado Tm2 1024 m2
MÚLTIPLOS gigámetro cuadrado Gm2 101am2
megámetro cuadrado Mm2 1012 m2
kilómetro cuadrado km2 106 m2
hectómetro cuadrado hm2 104m2 c:;>00
decámetro cuadrado dam2 102 m2

UNIDAD BASE METRO CUADRADO m2 100 m2

decímetro cuadrado dm 2 10.2m2


centímetro cuadrado cm2 10·•m2
milímetro cuadrado mm2 1Q·6m2
SUB- micrómetro cuadrado µm2 10-12m2
MÚLTIPLOS nanómetro cuadrado nm2 10.1a m2
picómetro cuadrado pm2 10-2•m2
fentómetro cuadrado fm2 10.aom2
Allómetro cuadrado am2 10.a5m2

Ejemplo de conversión:
Escribir 28 000 000 cm 2 en decámetros cuadrados.
00 ¡ CUIDADO !
Solución
El hectómetro cuadrado
• Escribimos el número dado en NOTACIÓN CIENTÍFICA: eqwvale a una hectárea
28 000 000 = 28 X 106 que es el nombre mas
• Aplicamos FACTOR DE CONVERSIÓN: ! popular de la unidad con

= 28 x 10 6cf
4 que se miden terrenos de
28 000 000 cm 2
c 2
x
l0 -
-~ 2
!1!: x ---;y
ldam 2 cultivo
2
lyu 10 'f
28 000 000 cm 2 = 28dam 2
RESPUESTA.-
28 millones de cm 2 pueden ser escritos como 28 decámetros cuadrados.
'º :1 ALFONSO ROJAS PUEMAPE 1
e7. LA UNIDAD DE VOLUMEN) o 0
f
t.:
t
O IMPORTANTE
Es el metro cúbico (m 3). ( El volumen y la capaci-
Si tuviéramos un cubo (una caja de laaos cuadrados) cuya f dad son una misma mag-
arista mide I m y lo llenamos con agua, tendríamos 1 m 3de "' nitud.
líquido vital.
En el siguiente cuadro mostramos los MúL TIPLOS y SUB-
MúLTIPLOS del metroeúbico, sus símbolos y equivalencias
enm\
t:"_:00,::::,
UNIDAD SÍMBOLO EQUIVALENCIA

exámetro cúbico Em 3 1054 m3


petámetro cúbico Pm 3 1045 m3
terámetro cúbico Tm 3 103 6 m3
MÚLTIPLOS gigámetro cúbico Gm 3 1021 mJ
megámetro cúbico Mm3 1018 m3
kilómetro cúbico km 3 109 ml
6 3
hectómetro cúbico hm3 10 m
decámetro cúbico dam3 103 m3

UNIDAD BASE METRO CÚBICO mJ 10° m3

decímetro cúbico dm3 rn-J mJ


centímetro cúbico cm3 1Q-6 mJ
milímetro cúbico mm 3 rn-g mJ
SUB- micrómetro cúbico µmJ 10.,smJ
MÚLTIPLOS nanómetro cúbico nm3 10-21 mJ
picómetro cúbico pmJ 10-36 mJ
lentómetro cúbico lm3 10-•5 mJ

attómetro cúbico am3 10-5• mJ

Algunasunidadesdecapacidadnoestablccidasencl S.I y que


todavía se siguen usando:
00 ¡ CUIDADO !
1 mt= 1 cm 3
1 t = 1 dm 3
O 00 El litro no aparece en el S.I;
ahora sólo es el nombre
comercial de 1 dm3
Ejemplo de conversión:
Escribir 2 metros cúbicos en litros.
1 litro = 1 de~1metro
cubico
Soluci,ín
• En lugar de empicar LITROS, empicaremos dm 3
• Por FACTOR DE CONVERSIÓN:
2
2m1 = 2~ x dm 3 - -dm 3 = _3_dm 3 = 2 000 dm 3 ó 2 000 litros.
10 3{i3 10 J _1_
103
RESPUESTA.-
2 metros cúbicos equivale a 2 000 litros, o mejor aún: 2 000 dm 3
1 MATEMÁTICA
1
SKANIPRÁCTICA @
-
l. Escribir:

(1) 17 km 2 en m2 (I I) 16 m2 cn mm2

(2) 19 hm2 en m2 (12) 28 km2 en dm2

(3) 72 000 mm 2 en m 2 (13) 6 248 000 mm 2 en cm 2

(4) 48 000 cm 2 en dm2 (14) 76 x 1012 fm 2 en dm2


.
. (5) 75 x 1018 µm 2 en m2 (15) 2,5 km 2 en m 2

(6) 62 x 104º pm 2 en dm2 (16) 6,8 x 108 µm 2 en mm2

(7) 517 x 1022 fm 2 en cm 2 (17) 6km 2 en cm 2

(8) 68 x 1010 µm 2 en dm2 (18) 4 616 µm 2 en dm2

(9) 168 m2 en cm 2 (19) 16,8 km 2 en m 2

(IO) 8 x I<f mm 2 en dm 2 (20) 18 x 1018 am 2 en cm 2

11. Escribir:

(1) 8 m 3 en litros (I 1) 45 cm 3 en liu-os

(2) 8 m 3 en dm 3 (12) 100 mm3 en µm 3

(3) 75 m 3 en litros (13) 125 000 000 µm 3 en dm3

(4) 75 m3 en dm3 (14) 640 000 000 mm 3 en litros

(5) 25 cm 3 en µm 3 (15) 640 000 000 mm 3 en dm3

(6) 7 000 000 fm 3 en mm 3 (16) 725 000 cm 3 en litros

(7) 16 x wx am 3 en nm 3 (17) 725 000 cm 3 en dm 3

(8) 12 m3 en cm 3 (18) 452 litros en cm 3

(9) 18 dm 3 en um3 (19) 16 m 3 en litros

(10) 6 215 000 mm 3 en dm 3 (20) 144 000 000 000 am 3 en liu-os


.... ... .·.··. .. . .

-. ·-=:::¼:.·==-x~--~-.-~:-~=-t-;¡;_~;;~=:l~-= ~;;.~:-:-=~·=:~!-=: ,:-.-·-: :«::= ;. .


,$:· 1 ALFONSO ROJAS PUÉMAPE 1
Aquí mis operaciones de SKANIPRÁCTICA 0

mszzt ww wu-ww? ·ct1: et«h"?itc-t


1 MATEMÁTICA
1
RAZONAMIENTO MATEMÁTICO

(1) Un señor tiene S/.180, pierde los 3/5 y luego a)450m b)350m c)400m
presta los 5/6 de lo que quedaba ¿Cuámo d)380m e)480m
tiene finalmente?
(7) Para cortar una varilla en "n" partes iguales,
a) SI. 20 b) SI. 28 c) S/. 12 hay que hacer:
d) S/. 26 e) S/. 32
a) n cortes b)(n - 1) cortes
(2) Un caño demora 8 horas en llenar un e) (n + 1) cortes d) (n - 1)/2 cortes
depósito, en tanto que el desagüe demora 12 • e) (n + 1)/2 cortes
horas en vaciarlo. Si estando lleno el depósito
(8) Una operación representada por O se define
en sus 3/4 se abren el caño y el depósito: ¿En
así:
qué tiempo se llenará completamente el
depósito.
a)6 horas b) 12 horas c) 18 horas
0 = 2x, si x es par.
d) 24 horas e) 4 horas
0= x, si x es impar.
(3) Si cada vez que juego pierdo la mitad de lo Hallar el valor tic:
que tenía anteriormente; ¿dentro de cuántos
juegos perderé todo si comencé con 100?.
a)9 b)12 c) 5 a) 5
m + 7 - BI- 0
b) 6 c)9
d) 8 e) No se puede d)lO e) 12

(4) Lasumadetresnúmeroses210.Sicadacual (9) ¿Cuál de las siguientes palabras se leería


es la mitad del otro.¿cuál es el mayor? correctamente al leerlas mirando un espejo?
a) SOMOS b) ADA c) ANA
a) 80 b)l20 c)92 d) MOTA e) Ninguna
d)64 e) 150
(5) Si trabaja los lunes inclusive el señor Pérez,
cconomiLa S/.400 semanales, en cambio, la (1 O) Un hombre recorre 3 metros en 2 segundos y
un muchacho 100 metros en un minuto.
semana que no trabaja el día lunes tiene que
¿Qué ventaja sacará el muchacho al hombre
retirar S/.200 de sus ahorros. Si durante I O
semanas logra economizar S/.2 200,¿cuán- en 2 minutos y medio de recorrido?
tos lunes dejó de trabajar en estas 10 sema- a)30m b)45m c)32m
nas? d)64m e)25m
a)2 b)3 c) 6 (11) ¿Entre qué número hay que dividir 40 para
d)4 e) 5 obtener 3/5?
(6) Un policía persigue a un ladrón que escapa a) 66 b) 65 c) 66 2/3
con una velocidad de 7 m/seg. Si el policía d) 67 e) 66 1/3
corre con una velocidaa mayor en 2 m/s
respecto a la del ladrón y sabiendo que ori- (12) Si A• B = A~ B :
ginalmente estaban separados por una dis-
tancia de 100 m., ¿qué distancia recorrerá el Calcular (2 • 3) + (3 • 2).
policía hasta alcanzar al ladrón? a) 7/5 b) 3/5 . e) 1
d) 1/5 e)2/5
ALFONSO ROJAS PUÉMAPE

Aquí mis opcmcioncs de RAZONAMIENTO MATEMÁTICO

1
f
i,XW,41 MATEMÁTICA

CLAVES DE RESPUESTAS DE LAS SERIES DE


RAZONAMIENTO l\1ATEl\1ATICO

CAPITULO l. NUMEROS NATURALES

(1) b (3) d (5) d (7) e (9) b (11) a


(2) e (4) e (6) d (8) e (10) a

CAPITULO 2. NUMEROS ENTEROS

(1) e (3) e (5) b (7) e (9) b


(2) d (4) a (6) e (8) d ( 10) a

CAPITULO 3. PROPIEDADES DE LOS NUMEROS

(1) d (3) d (5) e (7) b (9) e


(2) a (4) a (6) e (8) a (10) b

CAPITULO 4. NUMEROS RACIONALES

(1) e (3) e (5) d (7) b (9) e (llJ d


(2) a (4) b (6) d (8) e (10) e

CAPITULO 5. REPRESENTACION DECIMAL DE LOS NUMEROS RACIONALES

(1) b (3) e (5) d (7) b (9) a (11) e


(2) d (4) e (6) e (8) e (JO) b (12) b

CAPITULO 6. ECUACIONES E INECUACIONES DE PRIMER GRADO

(1) d (3) e (5) b (7) b (9) a


(2) a (4) d (6) e (8) e (10) e

CAPITULO 7. SISTEMA INTERNACIONAL DE UNIDADES (SI)

( 1) e (3) e (5) b (7) b (9) d (11) e


(2) a (4) b (6) a (8) a (JO) e (12) e
~,__AL
_ FO~OJAS PUEMAPE

BIBLIOGRAFÍA

PROGRAMA CURRICULAR DE MATEMÁTICA. Primer grado de secundaria.

MATEMÁTICA PROGRESIVA 1 Y 2. Nelson Londoño y Hernando Bedoya. Editorial Norma.


Colombia.

MATEMÁTICAS BÁSICAS. Taylor y Wade. Editorial Limusa. México.

TEORÍA DE LA ARITMÉTICA. Paterson y Hashisaki. Editorial Limusa. México.

MATEMÁTICA MODERNA 1 Y 2. Dolciani, Wooton y Beckenbach. Publicaciones Cultural S.A.

ARITMÉTICA TEÓRICA. V. Ampuero. E.S.M.

LECCIONES DE ARITMÉTICA. Juan A. Viedma. Editorial Norma. Colombia

ÁLGEBRA MODERNA. l. Kruger Sarapura - Fabio Contreras. Edca Editores. Perú.

ÁLGEBRA ELEMENTAL. Estructura y aplicaciones. Barnett - Nolasco Me Graw Hill. México.

SERIE MATEMÁTICA MODERNA I, 11, III y V. Editorial Norma. Colombia.

ÁLGEBRA MODERNA. Nichols, Heimer Garland. Compañía Editorial Continental S.A. México.
~~~~~~~~~~~~~~~~=~~~~==~~~.~

OBRAS DEL AUTOR


COLECCIÓN SKANNERS

PARA LA EDUCACIÓN SECUNDARIA

MATEMÁTICA PRIMER GRADO


MATEMÁTICA SEGUNDO GRADO
MATEMÁTICA TERCER GRADO
MATEMÁTICA CUARTO GRADO
MATEMÁTICA QUINTO GRADO

RAZONAMIENTO MATEMÁTICO PRIMER GRADO


RAZONAMIENTO MATEMÁTICO SEGUNDO GRADO
RAZONAMIENTO MATEMÁTICO TERCER GRADO
RAZONAMIENTO MATEMÁTICO CUARTO GRADO
RAZONAMIENTO MATEMÁTICO QUINTO GRADO

PARA LA EDUCACIÓN PRIMARIA:

SKÁNIMAT@

RAZONAMIENTO LÓGICO - MATEMÁTICO

SKÁNIMAT@

RAZONAMIENTO LÓGICO-MATEMÁTICO

·(~ ow,w.-.~E=COS"
Jr. Natalio Sánchez 220 - Of. 303 - 602
Alt. Cuadra S de Av. Arenales Telf.: 33-3077
ALF01'SO ROJM Pt.:DIAPE

PROTOCOLO PERUANO-ECUATORIANO DE PAZ, AMISTAD V LÍMITES


Río de Janeiro - 1 942

L01 Gob1em01 del Perú y del Ecuador, deseando dar aoluclón a 2" Rlo Zarumllla y Quebrada Balsarnal o Lajas;
la cue,tlOn de limites que por largo tiempo los separa. y teniendo en
3• Rlo Puyango o Tumbes. hasta la Quebrada de Cazaderos:
consldereclOn el ofrecimiento que les hicieron 101 Gobiernos de 101
Estad01 Unld01 de América. de la República de Argentina, de los 4" Cazaderos:
Estados Unidos del Bra.sll y de Chile, de sus servIcI01 amlstos01 para
5º Quebrada de PIiares y del Alamor hasta el Rlo Chlra;
procurar una pronta y honrosa solución del problema, y movidos por
el esplrltu americanista que prevalece en la III Reunión de Consulta de e• Rlo Chlra, aguas arriba:
Ministros de Relaciones Exteriores de las Repúblicas Americanas,
7° Rlos Macaré, Calvas y Esplndola, aguas arriba, hasta los
han reauelto celebrar un Protocolo da Paz, Amistad y Limites an
orígenes da este último en el Nudo de Sabanlllas;
presancla de 101 RapreS1ntentes de esos cuatro Gobiernos amigos.
e• Del Nudo de Sabenlllas, hasta el Rfo Canchls;
Para este !In Intervienen 101 algulentes PlenIpotenclarl01:
9° F!lo Canchl1, en todo su curso. aguas abajo;
Por la República del Perú, el seno, doctor Allredo Sol! y Muro, 10• Rlo Chlnchlpe, aguas abajo. hasta el punto en qua recibe
Ministro de Relacionas Exteriores; y el Rlo San Francisco;

Por la República dal Ecuador, el seno, doctor Julio Tobar b) En el Orienta:


Don010, Ministro de Relaciones Exteriores; 101 cuales de1pué1 de
exhibidos los plen01 y respectivos poderes de las Partes y habiéndolos 1• De la Quebrada de San Francisco el "dlvortlum aquarurn",
encontrado en buena y debida forma, acordaron la suscripción del entre el Río Zamora y el Rlo Santiago, hasta la confluencia
slgulenta Protocolo: del Rlo Santiago con el Yaupl;
2• Una linea hasta la boca del Bobonaza en eI Pastaza.
ARTÍCULO PRIMERO.- L01 Gobiernos del Ecuador y del Perú, Confluencia del Rlo Cunambo con el Plntoyacu en el Rlo
afirman solamnemente su decidido propOs,to de mantener entre l01 Tigre;
dos puebl01 relac,onee de paz y amlatad, de comprensión y de buena
voluntad, y de abstenerse. el uno respecto del olro. de cualquier acto 3° Boca del Cononaco en el Cureray. aguas abajo has te
capaz de perturbar esas relaciones. BallavI11e;
4" Una linea hasta Ia boca del Yasunl en eI Rlo Napo. Por el
ARTÍCULO SEGUNDO.- El Gol>lemo del Parú retiraré, dentro Napo, aguas abajo. haste la boca del Aguarlco;
del plazo de 15 d,as, a contar de esta !echa. sus fuerzas m,Htares a la
línea que se halla descrita en el Articulo VII de este Protocolo. 5" Por ésta, aguas arriba, hasta la confluencia dal Rlo
Lagartocoeha o Zancudo con el Aguarlco:
ARTÍCULO TERCERO.- Estados Unidos de América. Argentl· 6° El Rlo Lagartococha o Zancudo, aguas arriba, hasta sus
na. Brasil y Chile cooperarán, por medio de observadores militares, a orlgenes. y de alll una recia que vaya a encontrar el Rlo
!In de ajuster a las clrcunstencles la desocupación y al retiro de tropas Güep,, y poré1ta hasta su desembocadura en el Putumayo,
en los términos del Articulo anterior y por el Putumayo arribe hasta I0s limites del Ecuador y
Colombia
ARTICULO CUARTO.- Las fuerzas mlllteres de los dos palses,
quedaran en sus nuevas posiciones hasta la damarcac10n definitiva de ARTICULO NOVENO.- Queda entendido que la linea anterior-
la linea fronteriza Haste entonces, el Ecuador, tendré solamente mente descrita, seré aceptada por el Perú y el Ecuador para la hjec,ón,
Jurisdicción civil en las zonas que desocuparé el Perú, que quedan en por los técnicos. en el terreno. de la frontera entre los dos pafses. Las
las mismas condiciones en que ha estado la zona desmllltarizada del partes podrán, sin embargo. al procederse a su trazado sobre el
Acta de Talara. terreno, otorgarse las concesiones reciprocas que consideren conve-
nientes a fin de ajustar la referida linea a la realidad geogré!1ca Dichas
ARTICULO QUINTO.- La gestión de Estados Unidos, Argentina, rect1t1caclones se efectuarén con la colaboración de Represententes
Brasil y Chile continuará hasta le demarcac16n def1mt1va de las de los Estados Unidos de América, República Argentina, Brasil y Chde.
fronteras entre el Perú y el Ecuador. quedando este Protocolo y su
eJecuc16n bajo la garant1a de los cuatro países mencionados el Los Gobiernos del Perú y el Ecuador someterán el presente
comenzar este articulo. Protocolo a 'SUS respectivos Congresos. debiendo obtenerse la apro-
bación correspondiente en un plazo no mayor de 30 dlas.
ARTÍCULO SEXTO.- El Ecuador gozaré para la navegac,ón en
el Amazonas, y sus afluentes septentrionales, de las mismas conce- En fe de lo cual I0s Plenipotenc1anos arnba mencionados firman
siones de que gozan el Brasil y Colombia, más aquellas que fueran y sellan, en dos ejemplares en Castellano. en la c,uded del Rlo de
convenidas en un Tratado de Comercio y Navegación destinado a Janeiro, a la 1 hora del d(a veintinueve de enero del ario mil novecien-
facd1tar la navegación hbre y gratuita en los referidos rios. tos cuarenta y dos. el presente Protocolo, bajo los auspicios de Su
Excelencia al Senor Presidente del Brasil y en presencia de los
ARTÍCULO SÉPTIMO.- Cualquier duda o desacuerdo que sen ores Mrn,stros de Relaciones Exteriores de le República Argentina,
surgiere sobre la ejecución de este Protocolo será resuelto por les Brasil y Chlla y del Subsecretano de Estado de los Estados Unidos de
Partes con el concurso de los Representantes de Estados Unidos, la América.
Argenlma. Brasil y Chile. dentro del plazo més breve que sea posible
Firmado: Alfredo Sol!y Muro. -J. Tobar Oonoso. • E. Ruiz Gu1ñazú. -
ARTÍCULO OCTAVO.- La linea de frontera seré re!enda a los Oswaldo Aranha.- Juan B Rossett1.- Summer Welles. '
siguientes puntos·
El Congreso del Perú aprobo el Protocolo por ResoIuc1ón Legis-
lativa N" 9574, del 26 de febrero de 1 942 y lo propio hizo el Congreso
a) En el Occidente:
ecuatoriano en la misma fecha El car11e de ratd1cacIones se realizó
1• Boca de Capones en el Océano solemnemente en Petr6polls el 31 de marzo de 1 942.
1 MATEMÁTICA
1
Impreso en los talleres gráficos de
EDITORIAL MONTERRICO S.A.
Los Tapiceros 280, Urb. El Artesano - ATE
Telfs.: 436-5782- 436-5783 Fax 475-7011
Lima-Perú

También podría gustarte